OBGYN uwise -_-

Pataasin ang iyong marka sa homework at exams ngayon gamit ang Quizwiz!

A 27-year-old G0 woman comes to the clinic because she has been unable to conceive for the last year. She is in good health and has not used any hormonal contraception in the past. She had normal cycles in the past every 28 days until about six months ago. At that time, she began to have irregular menses every two to three months, with some spotting in between. She is not taking any medications. She has no history of abnormal Pap smears or sexually transmitted infections. Her physical examination is normal. Laboratory tests show: Results Normal Values Results Normal Values TSH 10 mIU/ml 0.5-4.0 mIU/ml Free T4 0.2 ng/dl 0.8-1.8 ng/dl Prolactin 40 ng/ml <20 ng/ml FSH 6 mIU/ml 5-25 mIU/ml LH 4 mIU/ml 5-25 mIU/ml What is the most appropriate step in the management of this patient? A. Begin levothyroxine B. Begin bromocriptine C. Order a clomiphene citrate ovulation challenge test D. Obtain a brain MRI E. Order a thyroid ultrasound

A

A 29-year-old G1P0 woman at 41 weeks gestation presents in early labor. The prenatal course was uncomplicated. Ultrasound at 21 weeks was consistent with gestational age. Her vitals reveal a blood pressure of 128/76; pulse 74; and she is afebrile. Fundal height is 36 cm with estimated fetal weight of 2700 gm. Cervix is dilated to 1 cm, 50% effaced and the fetal vertex is at -2 station. The nurse calls you to evaluate the fetal tracing. Which statement best describes the tracing seen below? (baseline 140s, variability 5-20, no accels no decels. CTX odd pattern) A. Normal fetal heart rate with good variability and regular contractions B. Fetal tachycardia with good variability and regular contractions C. Normal fetal heart rate with poor variability and regular contractions D. Fetal tachycardia with poor variability and irregular contractions E. Normal fetal heart rate with poor variability and irregular contractions

A

A 17-year-old female comes to your office for her first gynecologic visit. She has been sexually active for the last year and always uses condoms. What is the most appropriate management regarding Pap smear screening for this patient? A. Pap smear at age 21 B. Pap smear at this visit and then anually C. Pap smear now and then every 3 years E. Pap smear at age 18 D. Pap smear now and then every other year

A. Regardless of cloitarche, Pap smears start at 21

A 25-year-old G2P1 woman presents at 26 weeks gestation with preterm labor. She is currently receiving tocolytic therapy with magnesium sulfate. The patient's nurse is concerned the patient may have magnesium sulfate toxicity. The patient is alert and has no complaints. Her contractions have stopped and her vital signs are stable. Which of the following findings associated with magnesium sulfate treatment would this patient experience before she develops respiratory depression? A. Areflexia B. Hyperreflexia C. Tachycardia D. Hypertension E. Oligouria

A. ! High levels of magnesium sulfate may cause respiratory depression (12-15 mg/dl) or cardiac depression (>15 mg/dl). Prior to developing respiratory depression the patient should have diminished or absent deep tendon reflexes (areflexia). Magnesium sulfate does not cause hypertension.

A 28-year-old G1P1 previously healthy woman is brought into the emergency department by her husband following a seizure at home. She had been complaining of a severe headache for two days prior to this, and had been feeling more and more fatigued and short of breath since the delivery of their child three months ago. She has been breastfeeding, and began to have vaginal spotting one month ago. Her neurologic and physical examinations are unremarkable and her pelvic exam reveals a normal uterus with no adnexal masses. Her work-up reveals multiple nodules on chest X-ray and within the brain and liver, suspicious for metastasis. Choriocarcinoma is highly suspected. Which of the following tests will confirm the diagnosis in this case? A. Quantitative Beta-hCG B. Serum CA-125 C. Transvaginal ultrasound D. Fine needle aspiration of the liver lesions E. Biopsy of a chest nodule

A. A diagnosis of choriocarcinoma is made once the presence of Beta-hCG is confirmed. Certainly, intrauterine pregnancy and ectopic pregnancy must be excluded, but this can easily be done depending on the quantitative level. In the presence of metastatic disease of unclear primary, the diagnosis of GTD (choriocarcinoma) must be considered. Ultrasound is useful in ruling out an intrauterine or ectopic pregnancy, but provides no information if the Beta-hCG is negative or below the discriminatory zone. Serum CA-125 is a tumor marker for most epithelial ovarian cancers, but, because it is non-specific, its possible elevation in this case is not diagnostic. Because metastatic choriocarcinoma is quite vascular, suspicious lesions should never be biopsied. Tissue diagnosis is the standard in establishing a diagnosis of almost all malignancies, with the exception of choriocarcinoma. Only a positive Beta-hCG in a reproductive-aged woman who has a history of a recent pregnancy (term, miscarriage, termination, mole) is necessary to establish the diagnosis.

A 72-year-old G3P1 woman has progressive ovarian cancer. She and her husband have already completed a medical power of attorney form. However, the patient did not complete a living will or any other documents expressing her wishes for the initiation of mechanical ventilation or cardioversion in the event of a respiratory or cardiac arrest. Unfortunately, the patient is brought into the hospital after suffering an incapacitating seizure. She is not arousable when she reaches the oncology unit. Her husband Jim is present and willing to act as Mary's surrogate decision-maker. When he decides on the proper course of care, the husband should make decisions based primarily on which of the following? A. What Mary would have chosen B. Mary's best interest C. Hospital Ethics Committee's recommendation D. The family's wishes E. His own wishes

A. A person who has power of attorney should make decisions based on what the patient would have wanted for herself, regardless of what they think her best interests might be.

A 26-year-old G0 woman presents with hirsutism and irregular menses. Her mother, who is diabetic, had similar complaints prior to menopause. On physical exam, this patient is noted to have terminal hair on her chin and a gray-brown velvety discoloration on the back of her neck. This lesion is acanthosis nigricans. Which of the following is the most appropriate first test to order for this patient? A. Fasting insulin B. TSH C. 17-hydroxyprogesterone level D. Cortisol level E. Pelvic ultrasound

A. Acanthosis nigricans is associated with elevated androgen levels and hyperinsulinemia. Since this woman has a family history of diabetes and also has acanthosis nigricans, the most appropriate test of those listed would be the fasting insulin. The other tests would also be reasonable, but hyperinsulinemia is most likely in this patient.

A 37-year-old G2P1 woman with poorly controlled chronic hypertension presents in labor at term. Her prenatal course was uncomplicated. She delivers a 3500 gram infant spontaneously after oxytocin augmentation of labor. Immediately postpartum, she experiences excessive bleeding. Her blood pressure is 130/90; pulse 84; and she is afebrile. On examination, uterine fundus is firm and the placenta is intact. Which of the following is the most appropriate next step in the management of this patient? A. Exploration for lacerations B. Methylergonovine C. B Lynch suture D. IV push of oxytocin E. Uterine artery embolization

A. After ensuring appropriate backup, establishing intravenous access and stabilizing a patient as needed, the first steps in the management of postpartum hemorrhage are to make sure the uterus is well-contracted, there is no retained placental tissue and to look for lacerations. This patient has a firm fundus, which indicates a contracted uterus. Her placenta is complete, which typically rules out retained placental tissue, so it is important to rule out lacerations, which can lead to hemorrhage. Methylergonovine, prostaglandins and oxytocin are all uterotonics and used to increase uterine contractions and decrease uterine bleeding. Methylergonovine is an ergot alkaloid, which is a potent smooth muscle constrictor. It is also a vasoconstrictive agent and should be withheld from women with hypertension and/or preeclampsia. B Lynch suture is used at time of laparotomy for uterine atony. Oxytocin should not be given as an IV push. Uterine artery embolization can be considered after other sources of bleeding such as lacerations are ruled out.

A 33-year-old G2P1 woman at 29 weeks gestation presents with confirmed preterm premature rupture of membranes. She denies labor. She takes prenatal vitamins and iron. She denies substance abuse, smoking or alcohol use. Her prior pregnancy was delivered vaginally at 41 weeks after spontaneous rupture of membranes. Her blood pressure is 110/70; pulse 84; temperature 98.6°F (37.0°C). Which of the following is the next best step in the management of this patient? A. Ampicillin and erythromycin B. Clindamycin and gentamicin C. Nifedipine D. Terbutaline E. Magnesium sulfate

A. Antibiotic therapy with ampicillin and erythromycin given to patients with preterm premature rupture of the membranes has been found to prolong the latency period by 5-7 days, as well as reduce the incidence of maternal amnionitis and neonatal sepsis. Clindamycin and gentamicin are not indicated for the management of PPROM. Tocolytics may also prolong the pregnancy for various lengths of time, but generally not seven days.

19-year-old G1 woman at 36 weeks gestation presents for her first prenatal visit, stating she was recently diagnosed with HIV after her former partner tested positive. The HIV Western Blot is positive. The CD4 count is 612 cells/µl. The viral load is 9,873 viral particles per ml of patient serum. Which of the management options would best decrease the risk for perinatal transmission of HIV? A. Treatment with intravenous zidovudine at the time of delivery B. Treatment of the newborn with oral zidovudine only if HIV-positive C. One week maternal treatment with zidovudine now D. Cesarean section in second stage of labor E. Single drug therapy to minimize drug resistance

A. Antiretroviral therapy should be offered to all HIV-infected pregnant women to begin maternal treatment as well as to reduce the risk of perinatal transmission regardless of CD4+ T-cell count or HIV RNA level. The baseline transmission rate of HIV to newborns can be reduced from about 25% to 2% with the HAART (highly active antiretroviral therapy) protocol antepartum and continuing through delivery with intravenous zidovudine in labor and zidovudine treatment for the neonate. Cesarean section prior to labor can reduce this rate to 2% (although the benefit is less clear in women with viral loads).

A 35-year-old woman presents to the emergency department with heavy vaginal bleeding at seven weeks gestation. On examination, she has a dilated cervix with blood and tissue present at the cervical os. Which of the following is the most likely chromosomal abnormality to be found in the karyotypic evaluation of the products of conception? A. Autosomal trisomy B. Triploidy C. Tetraploidy D. Monosomy X (45X,0) E. Fragile X mutation

A. Autosomal trisomy is the most common abnormal karyotype encountered in spontaneous abortuses, accounting for approximately 40-50% of cases. Triploidy accounts for approximately 15%, tetraploidy 5% of cases, and Monosomy X (45X, 0) identified in 15-25% of losses. The Fragile X mutation involves an expanded number of trinucleotide repeats in the CGG (cytosine-guanine) sequence.

A 34-year-old G2P1 woman presents at 17 weeks gestation. She did not seek preconception counseling and is worried about delivering a child with Down syndrome, given her maternal age. She has no significant medical, surgical, family or social history. Which of the following tests is most effective in screening for Down syndrome in this patient? A. Cell-free DNA screen B. Triple screen C. Quadruple screen D. Maternal serum alpha fetoprotein level E. Nuchal translucency measurement with serum PAPP-A (pregnancy associated plasma protein-A) and free Beta-hCG level

A. Cell-free DNA screening is the most effective screening test for Down syndrome. The test may be performed as early as 9 weeks gestation and until delivery. The test detects over 99% of cases of Down syndrome. The quadruple test (maternal serum alpha fetoprotein, unconjugated estriol, human chorionic gonadotropin, and inhibin A) may be used to screen for Down syndrome in the second trimester. Down syndrome occurs in about 1 in 800 births in the absence of prenatal intervention. The efficacy of screening for Down syndrome is improved when additional components are added to the maternal serum alpha fetoprotein screening. The addition of unconjugated estriol and human chronic gonadotropin (the Triple Screen) results in a 69% detection rate for Down syndrome. Adding inhibin A to produce a quadruple screen achieves a detection rate of 80-85%. Nuchal translucency measurement with maternal serum PAPP-A and free Beta-hCG (known as the combined test) is a first trimester screen for Down syndrome. It detects approximately 85% of cases of Down syndrome at a 5% false positive rate.

A 67-year-old G3P3 woman presents with severe pelvic protrusion several years following an abdominal hysterectomy. She denies any incontinence. She failed conservative management with a pessary. As a result, she underwent a vaginal surgical repair where the pubocervical fascia was plicated in the midline, as well as laterally to the arcus tendineus fascia (white line). What defect was repaired in this patient? A. Cystocele B. Rectocele C. Uterine prolapse D. Enterocele E. Urethral diverticulum

A. Central and lateral cystoceles are repaired by fixing defects in the pubocervical fascia or reattaching it to the sidewall, if separated from the white line. Defects in the rectovaginal fascia are repaired in rectoceles. Uterine prolapse is surgically treated by a vaginal hysterectomy, but this patient already had a hysterectomy. Enteroceles are repaired by either vaginal or abdominal enterocele repairs. Vaginal vault prolapse is treated either by supporting the vaginal cuff to the uterosacral or sacrospinous ligaments, or by sacrocolpopexy. Urethral diverticulum does not present with severe pelvic protrusion.

A 27-year-old G1P0 woman at 12 weeks gestation presents for first prenatal care visit. She is previously healthy and takes no medications. An ultrasound is performed and a viable pregnancy is confirmed. At the end of the visit, the patient discusses with you her desire to have a Cesarean section for delivery, as she does not wish to go through the pain of labor. Her husband, an orthopedic surgeon, expresses concerns as they desire to have at least three children and he is worried about potential complications with repeated Cesarean sections. What is the most appropriate next step in the counseling of this patient? A. Agree with her decision after proper counseling and perform a Cesarean section at 39 weeks gestation B. Agree with her decision after proper counseling and perform a Cesarean section at 41 weeks gestation if she has not gone into labor by then C. Advise her that it is not possible to plan a Cesarean section for delivery D. Advise her to listen to her husband and plan for a vaginal birth E. Refer her to psychiatric counseling

A. Elective Cesarean section on demand has been getting more popular among women for a variety of reasons. Although, it might sound unreasonable to undergo a Cesarean section for being afraid of pain, the patient has the right to request it and the physician's duty is to make sure she understands all the risks and potential complications associated with such a decision. Elective delivery should not be scheduled prior to 39 weeks due to risks associated with prematurity. Her husband is appropriately concerned, but it is up to her to make the decision regarding an elective procedure.

A 72-year-old G3P3 presents to the emergency room complaining of abnormal vaginal discharge for the past two months. She has had two episodes of vaginal bleeding over the last year. She used combination hormone replacement therapy for 10 years when she went through menopause at age 58, but stopped once the Women's Health Initiative report came out. Her last gynecologic exam and Pap smear were two years ago and were normal. She has tried several over-the-counter antifungal creams for what she presumed was a yeast infection; however, she reports no change in the nature of the discharge. She does note that she has some mild lower abdominal discomfort. The only significant finding on exam is a mucopurulent discharge from a multiparous cervical os. She has a 10-week sized globular uterus. Which of the following findings is most concerning for presence of endometrial cancer? A. Vaginal bleeding B. Late menopause C. Abnormal vaginal discharge D. Enlarged uterus E. Hormone replacement therapy

A. Endometrial cancer is a disease that typically presents with symptoms and clinical findings that lead to an early diagnosis. The most common symptom is abnormal postmenopausal bleeding. However, other symptoms or clinical findings that may be seen include abnormal vaginal discharge and lower abdominal discomfort. Endometrial cancer can increase the size of the uterus as it grows, but is usually not the most common finding given the early diagnosis of this cancer. Unopposed estrogen replacement therapy does increase the risk, but not when taken in combination. Early menarche and late menopause are additional risk factors that may be related to endometrial cancer development.

A 37-year-old G0 woman complains that she experiences mood swings, irritability, bloating and headaches monthly for two to three days prior to her menstrual cycle. Her medical history is unremarkable and physical examination is normal. The physician advises her to keep a calendar of her symptoms. He also recommends a balanced diet, avoidance of caffeine and alcohol, and daily regular exercise. The patient has never exercised regularly and wonders how this will help her mood swings and bloating. Which of the following would provide the best explanation for the benefits of exercise on her PMS symptoms? A. Endorphins B. Cortisol C. Progesterone D. Estrogen E. Androgen

A. Exercise increases circulating endorphins in the brain which are "feel good" hormones and act similar to serotonin. Therefore, in addition to being a benefit to cardiovascular health, regular exercise can significantly decrease symptoms of PMS.

A 51-year-old G1P1 woman presents for annual examination. She notes vaginal dryness, some hot flashes, and fatigue. She reports that her last menstrual period was 14 months ago. She and her husband use lubrication for intercourse, and she denies any significant pain. Her past medical history is significant for hypertension, which she controls with diet and regular exercise. She is concerned that she should begin hormone replacement, because her mother started HRT around the same age. Which of the following is the most appropriate next step in her management? A. Expectant management B. FSH level C. LH level D. Estrogen level E. Initiate combination HRT

A. Expectant management is reasonable in this patient, as she notes minimal menopausal symptoms. Her vaginal dryness is not interfering with her ability to enjoy intercourse, and she has only occasional hot flashes. An FSH level is not indicated as by definition she is menopausal given amenorrhea for greater than 12 months. Estrogen and LH levels are not indicated in the diagnosis of menopause. While HRT is appropriate for patients with significant menopausal symptoms, and should be used at the lowest effective dose for the shortest amount of time, this patient at this time is not experiencing significant symptoms and therefore HRT should not be initiated at this time.

A 24-year-old G0 woman is considering pregnancy in the next year. Her medical history and physical examination are normal. She is unaware of any significant family history. She is vegetarian, exercises regularly and does not smoke or drink alcohol. Which of the following is the next best step in the management of this patient? A. Folate supplementation is recommended for this patient B. Most patients get an adequate intake by diet alone C. She is not currently planning a pregnancy and does not need to worry about folate D. Current grain supplementation adequately prevents neural tube defects E. Folate does not need to be started until after a pregnancy is documented

A. Folate lowers homocysteine levels. The Nurses Health Study showed fewer nonfatal MIs and fatal coronary events in women with adequate intake doses of folate and vitamin B6. Folate can also help prevent neural tube defects. Studies have shown that diet alone is not effective in achieving adequate levels, and routine folate supplementation is therefore recommended. Women of reproductive age should take a daily 400-microgram supplement. Adequate levels are especially important prior to pregnancy and during the first four weeks of fetal development. Folic acid levels may be used to diagnose B12 or folate deficiency and are not routinely check to guide folic acid supplementation prior to pregnancy unless a patient is suspected to have a deficiency.

A 48-year-old G3P3 woman presents to the office for a health maintenance examination. Her past medical history is negative. Her family history is significant for hypertension in her father and diabetes mellitus in her mother. Her grandfather passed away from colon cancer at the age of 82. She is worried about getting colon cancer and desires to undergo screening. Her body mass index (BMI) is 23 and her physical exam is normal. What is the most appropriate next step in the management of this patient? A. Recommend a colonoscopy at age 50 and, if normal, repeat every 10 years B. Recommend a colonoscopy at age 50 and then every two years C. Order a colonoscopy now and, if normal, repeat in five years D. Order a sigmoidoscopy now and, if normal, repeat in five years E. Recommend a sigmoidoscopy at age 50 and then every two years

A. For patients with average risk for colon cancer, the recommended screening is to begin colonoscopy at age 50 and then every 10 years, if normal. Despite having a grandfather who passed away from colon cancer, this patient is not necessarily at increased risk and does not need to be screened at different intervals than the general population. If there is a history of a first degree relative with colon cancer before age 60, then begin screening with colonoscopy at age 40, or 10 years before the youngest relative diagnosis, and repeat every five years. Although a sigmoidoscopy can be an acceptable screening procedure, it would still begin at age 50 and repeat every five years, if normal.

A 60-year-old G4P4 woman presents with a two-year history of urine leakage with activity such as coughing, sneezing and lifting. Her past medical history is significant for vaginal deliveries of infants over 9 pounds. She had a previous abdominal hysterectomy and bilateral salpingo-oophorectomy for uterine fibroids. She is on vaginal estrogen for atrophic vaginitis. Physical examination reveals no anterior, apical or posterior wall vaginal prolapse. The vagina is well-estrogenized. Post-void residual is normal. Q-tip test shows a straining angle of 60 degrees from the horizontal. Cough stress test shows leakage of urine synchronous with the cough. Cystometrogram reveals the absence of detrusor instability. The patient failed pelvic muscle exercises and is not interested in an incontinence pessary. Which of the following is the best surgical option for this patient? A. Retropubic urethropexy B. Needle suspension C. Anterior repair D. Urethral bulking procedure E. Colpocleisis

A. Genuine stress incontinence (GSI) is the loss of urine due to increased intra-abdominal pressure in the absence of a detrusor contraction. The majority of GSI is due to urethral hypermobility (straining Q-tip angle >30 degrees from horizon). Some (<10%) of GSI is due to intrinsic sphincteric deficiency (ISD) of the urethra. Patients can have both hypermobility and ISD. Retropubic urethropexy such as tension-free vaginal tape and other sling procedures have the best five-year success rates for patients with GSI due to hypermobility. Needle suspensions and anterior repairs have lower five-year success rates for GSI. Urethral bulking procedures are best for patients with ISD, but with little to no mobility of the urethra. Colpocleisis is one option to treat uterine prolapse, and is not indicated for urinary incontinence.

A 62-year-old G5P5 woman presents with a seven-month history of pelvic pain and pressure, as well as abdominal distention and bloating. She experiences occasional constipation, but no melena or hematochezia. She also has mild to moderate urinary frequency without dysuria, hematuria or flank pain. Her medical history is significant for hypertension and obesity. She went through menopause 12 years ago and has never been on hormone therapy. She reports one episode of light vaginal bleeding several months ago. Her family history is significant for postmenopausal ovarian cancer in her mother and maternal aunt, but is otherwise negative for breast, endometrial or colon cancer. Pelvic examination is remarkable for vaginal atrophy, cervical stenosis and difficult uterine and adnexal assessment due to her body habitus. What is the most appropriate next step in the management of this patient? A. Transvaginal ultrasound B. CT scan of the abdomen and pelvis C. Colonoscopy D. Hysteroscopy E. Diagnostic laparoscopy

A. Given the patient's age, nonspecific abdomino-pelvic symptoms, recent postmenopausal bleeding episode and family history of ovarian cancer, a transvaginal ultrasound is the next best step as it is more sensitive than CT for evaluation of the uterus and adnexa. Colonoscopy is useful for colorectal cancer screening, as well as evaluation of the patient's gastrointestinal symptoms, but would not provide information regarding pelvic anatomy. Diagnostic laparoscopy would be a more invasive procedure that could be performed as indicated, after these other diagnostic studies. Hysteroscopy might be useful based on the ultrasound results, since it might be difficult to perform an endometrial biopsy in the office.

A 22-year-old G0 woman presents with worsening pelvic pain. She previously underwent a laparoscopic ablation of endometriosis followed by continuous oral contraceptive pills. She had short-term relief from this approach, but now has failed this treatment and is seeking additional medical management. Which of the following mechanisms best explains how a gonadotropin releasing hormone (GnRH) agonist would help alleviate her symptoms? A. Down-regulation of the hypothalamic-pituitary gland production B. Up-regulation of the hypothalamic-pituitary gland production C. Suppression of both LH and FSH mid-cycle surges D. Induction of a pseudopregnancy state E. Competitive inhibitor for estrogen receptors

A. Gonadotropin-releasing hormone (GnRH) agonists are analogues of naturally occurring gonadotropin-releasing hormones that down-regulate hypothalamic-pituitary gland production and the release of luteinizing hormone and follicle-stimulating hormone leading to dramatic reductions in estradiol level. Numerous clinical trials show GnRH agonists are more effective than placebo and as effective as Danazol in relieving endometriosis-associated pelvic pain. Danazol, a 17-alpha-ethinyl testosterone derivative, suppresses the mid-cycle surges of LH and FSH. Combined estrogen and progestin therapy in oral contraceptives produces the pseudopregnancy state.

A 40-year-old G1P0 woman at 34 3/7 weeks gestation was found on the floor at work having a grand mal seizure. Her airway was secured. Blood pressure in the ambulance was 140/90. What is the initial treatment for her condition? A. Magnesium sulfate B. Valium C. Hydantoin D. Phenobarbital E. Naloxone

A. Magnesium sulfate is the treatment of choice for eclampsia, which is her most likely diagnosis. Valium, hydantoin, tiagabine, and barbiturates can also be used to treat seizures, but are not first-line therapy for eclampsia. They can be added as second agents, or used if magnesium is contraindicated. Naloxone (Narcan) is a drug used to counter the effects of opioid overdose, for example heroin or morphine overdose, and is specifically used to counteract life-threatening depression of the central nervous system.

A 35-year-old G2P2 woman comes to the office due to heavy menstrual periods. The heavy periods started three years ago and have gradually worsened in amount of flow and duration. The periods are now interfering with her daily activities. The patient had two spontaneous vaginal deliveries. She smokes one pack of cigarettes per day. On pelvic examination, the cervix appears normal and the uterus is normal in size, without adnexal masses or tenderness. A urine pregnancy test is negative. TSH and prolactin levels are normal. Hemoglobin is 12.5 mg/dl. On pelvic ultrasound, a 2 cm submucosal leiomyoma is noted. An endometrial biopsy is consistent with a secretory endometrium; no neoplasia is found. Which of the following would be the best therapeutic option for this patient if she desires to have another child? A. Hysteroscopy with myoma resection B. Laparoscopic myomectomy C. Endometrial ablation D. Oral contraceptives E. Dilation and curettage

A. Hysteroscopic myomectomy preserves the uterus, while removing the pathology causing the patient's symptoms. A laparoscopic approach is not indicated as the myoma is submucosal and not accessible using a laparoscopic approach. Endometrial ablation destroys the endometrium and can create Asherman's syndrome, thus it is reserved for patients who have completed childbearing. Dilation and curettage is unlikely to remove the myoma and is a blind procedure (carried out without direct visualization). Oral contraceptives would typically help with heavy menses, but are contraindicated in this patient, who is over 35 and smokes.

A 20-year-old G0 woman presents requesting birth control pills. She received the HPV vaccine series last year, and had her first sexual encounter last month. Otherwise, she is in good health and is a non-smoker. Her pelvic examination reveals normal external genitalia, and a nulliparous cervix without discharge or mucosal lesions. A urine pregnancy test is negative. Which of the following is the appropriate screening recommendation for this patient? A. Return next year for a Pap test B. Return in three years for a Pap test C. Perform a Pap test now D. Perform HPV testing E. Perform colposcopy

A. In 2013, the American Congress of Obstetrics and Gynecology (ACOG) updated the following recommendations for cervical cancer screening: Cervical cancer screening should start at age 21 years. Women aged 21-29 years should have a Pap test every three years. Women aged 30-65 years should have a Pap test and an HPV test (co-testing) every five years (preferred). It is acceptable to have a Pap test alone every three years. Women should stop having cervical cancer screening after age 65 years if they do not have a history of moderate or severe dysplasia or cancer and they have had either three negative Pap test results in a row, or two negative co-test results in a row within the past 10 years, with the most recent test performed within the past five years. Women who have a history of cervical cancer, are infected with HIV, have a weakened immune system, or who were exposed to DES before birth should not follow these routine guidelines. ACOG recommends that women who have been vaccinated against HPV should follow the same screening guidelines as unvaccinated women.

A 28-year-old G1 at approximately 40 weeks gestation presents to triage with mild contractions. You measure her fundal height at 34 cm. You are concerned about intrauterine growth restriction and you want to confirm her dates. In reviewing her records, she reports first feeling fetal movements at 18 weeks gestation. The crown-rump length measurements determined at eight weeks and femur length at 20 weeks are consistent with 40 weeks gestation. Today's assessment reveals biometrics consistent with 34 weeks, amniotic fluid index of 1, and placental calcifications. Which of the following is considered the most reliable method of determining the gestational age in this patient? A. Crown-rump length measurement B. Second trimester ultrasound C. Quickening date D. Third trimester composite biometry E. Placental calcifications

A. In order to accurately confirm gestational age at term, one of the following criteria should be met: Fetal heart tones have been documented for 20 weeks by a non-electronic fetoscope or for 30 weeks by Doppler; it has been 36 weeks since a positive serum or urine HCG pregnancy test was performed by a reliable laboratory; an ultrasound measurement of the crown-rump length, obtained at six to twelve weeks, supports a gestational age of at least 39 weeks; and an ultrasound obtained at 13-20 weeks confirms the gestational age of at least 39 weeks, determined by clinical history and physical examination. The crown-rump length can reliably date a pregnancy within five to seven days.

A 29-year-old G1P0 woman at 42 weeks gestation presents to labor and delivery because of intermittent contractions. She denies ruptured membranes. Her prenatal course was uncomplicated. Her vital signs are: blood pressure 140/96; pulse 72; afebrile; fundal height 32 cm; and estimated fetal weight of 2900 gm. Cervix is closed, 25% effaced, -2 station. The fetal heart rate tracing shows occasional late decelerations. Of the following, what is the next best step in management? A. Maternal left lateral position B. Intrauterine resuscitation with terbutaline Start an amnioinfusion Begin magnesium sulfate Augment labor with oxytocin

A. Initial measures to evaluate and treat fetal hypoperfusion include a change in maternal position to left lateral position which increases perfusion to the uterus, maternal supplemental oxygenation, treatment of maternal hypotension, discontinue oxytocin, consider intrauterine resuscitation with tocolytics and intravenous fluids, fetal acid-base assessment with fetal scalp capillary blood gas or pH measurement. An amnioinfusion may be used to treat patients with variable decelerations. Measures to improve uteroplacental blood flow should be attempted prior to proceeding with Cesarean delivery. Magnesium sulfate is not yet indicated in this patient with one slightly elevated blood pressure. Augmentation of labor may accentuate the late decelerations.

A 16-year-old girl is brought by her mother to her primary care physician requesting that her daughter have a gynecological examination. She is fearful that her daughter is sexually active. A private conversation with the girl reveals that she is sexually active and is on oral contraceptives prescribed by a community clinic. Having seen what her older sister went through when she became sexually active, she prefers not to tell her mother about her sexual activity for fear of severe punishment. The pelvic examination is normal. Which is the most appropriate action at this time? A. Tell the mother that her daughter is healthy B. Insist to the daughter that she disclose her sexual activity to her mother C. Reassure the mother that her daughter's sexual activity is a normal activity for most girls her age D. Speak to the mother openly about her daughter's condition E. Disclosure of any medical information is forbidden and a major HIPPA violation

A. It is not appropriate to discuss the daughter's sexuality with her mother, although her mother has the right to know about her daughter's overall health as her legal guardian. The daughter should be encouraged to have an open conversation with her mother, but that this is entirely up to her discretion and that she will be supported in whatever decision she ultimately makes about disclosing her sexual activity.

A 29-year-old G1P0 woman at 42 weeks gestation presents in labor. She denies ruptured membranes. Her prenatal course was complicated by chronic hypertension. Her vital signs are: blood pressure 130/80; pulse 72; afebrile; fundal height 38 cm; and estimated fetal weight of 3000 gm. Cervix is dilated to 4 cm, 100% effaced, -1 station, and bulging bag of water. The fetal heart rate tracing reveals five contractions in 10 minutes and repetitive late decelerations. What is the most likely cause of her late decelerations? A. Uteroplacental insufficiency B. Umbilical cord compression C. Uterine hyperstimulation D. Intrauterine growth restriction E. Fetal head compression

A. Late decelerations when viewed as repetitive and/or with decreased variability are an ominous sign. The can be associated with uteroplacental insufficiency as a result of decreased uterine perfusion or placental function, thus leading to fetal hypoxia and acidemia. Common causes include chronic hypertension and postdate pregnancies. Variable decelerations are associated with cord compression. Uterine hyperstimulation may cause prolonged bradycardia. While fetuses with intrauterine growth restriction may have late decelerations, the estimated fetal weight of 3000 g is a normal birth weight for this gestational age. Fetal head compression may be associated with early decelerations.

A 28-year-old G2P0020 experienced her second miscarriage within 14 months. A recent ultrasound was notable for two uterine fibroids. The patient is worried that the fibroids may have caused her early pregnancy losses. She is otherwise healthy and has no previous surgeries. She presents to you for further consultation. Which type of fibroid is the most likely explanation of her miscarriages? A. Submucosal B. Intramural C. Subserosal D. Pedunculated E. Cervical

A. Leiomyomas are an infrequent cause of miscarriages and subfertility either by mechanical obstruction or distortion (and interference with implantation). When a mechanical obstruction of fallopian tubes, cervical canal or endometrial cavity is present and no other cause of infertility or recurrent miscarriage can be identified, myomectomy is usually followed by a prompt achievement of pregnancy. Submucosal or intracavitary myomas are most likely to cause lower pregnancy and implantation rates. Presumed mechanisms include: 1) focal endometrial vascular disturbance; 2) endometrial inflammation, and; 3) secretion of vasoactive substances. Submucosal fibroids are best treated by hysteroscopic resection.

A 30-year-old G1P1 woman presents with a history of chronic vulvar pruritus. The itching is so severe that she scratches constantly and is unable to sleep at night. She reports no significant vaginal discharge or dyspareunia. She does not take antibiotics. Her medical history is unremarkable. Pelvic examination reveals normal external genitalia with marked lichenification (increased skin markings) and diffuse vulvar edema and erythema as shown in picture below. Saline microscopy is negative. Potassium hydroxide testing is negative. Vaginal pH is 4.0. The vaginal mucosa is normal. Which of the following is the most likely diagnosis in this patient? A. Lichen simplex chronicus B. Lichen sclerosus C. Lichen planus D. Candidiasis E. Vulvar cancer

A. Lichen simplex chronicus, a common vulvar non-neoplastic disorder, results from chronic scratching and rubbing, which damages the skin and leads to loss of its protective barrier. Over time, a perpetual itch-scratch-itch cycle develops, and the result is susceptibility to infection, ease of irritation and more itching. Symptoms consist of severe vulvar pruritus, which can be worse at night. Clinical findings include thick, lichenified, enlarged and rugose labia, with or without edema. The skin changes can be localized or generalized. Diagnosis is based on clinical history and findings, as well as vulvar biopsy. Treatment involves a short-course of high-potency topical corticosteroids and antihistamines to control pruritus.

A 20-year-old G2P2 healthy woman presents for her post-partum check six weeks after a full term normal spontaneous delivery. She has a 13 month old in addition to the six-week newborn, and is already feeling overwhelmed. She desires a reliable form of contraception. On exam, her vital signs are normal. BMI is 27. The remainder of the exam is unremarkable. Of the following, what is the most effective and appropriate form of contraception for this patient? A. Intrauterine device B. Tubal ligation C. Depo-Provera® D. Oral contraceptive pills E. Essure®

A. Long-acting reversible contraceptives (LARC) methods such as contraceptive implants and intrauterine devices are a good option for this patient. Despite high up-front costs and the need for office visits for insertion and removal, LARC methods provide many distinct advantages over other contraceptive methods as Depo-Provera® and oral contraceptives. While Depo-Provera is an effective form of contraception, it may not be the best choice in this woman with a high BMI. For this young mother who desires a reversible, but reliable form of contraception, the high effectiveness, continuation rate and user satisfaction of LARC methods would be of most benefit. Emerging evidence indicates that increasing the use of LARC methods also could reduce repeat pregnancy among adolescent mothers and repeat abortions among women seeking induced abortion. ("Increasing Use of Contraceptive Implants and Intrauterine Devices To Reduce Unintended Pregnancy," ACOG Committee Opinion, No. 450, 2009). Tubal ligation and Essure® are permanent and are not appropriate for this patient.

A 24-year-old G1P0 woman at 22 weeks gestation with systemic lupus erythematosus (SLE) presents complaining of malaise, joint aches, and fever. Physical examination reveals the following: pulse 88; temperature 98.6°F (37.0°C); respiratory rate 22; and BP 150/110 (baseline is 100/70). Laboratory analysis reveals 1 + proteinuria, AST 35, and ALT 28. Which of the following is the most appropriate initial therapy for the treatment of this patient? A. Steroids B. Nonsteroidal anti-inflammatory drugs (NSAIDs) C. Azathioprine D. Cyclophosphamide E. Magnesium sulfate

A. Lupus is notoriously variable in its presentation, course, and outcome. Clinical manifestations include malaise, fever, arthritis, rash, pleuro-pericarditis, photosensitivity, anemia, and cognitive dysfunction. A significant number of patients have renal involvement. There is no cure and complete remissions are rare. Mild disease may be disabling because of pain and fatigue. Nonsteroidal anti-inflammatory drugs are used to treat arthralgia and serositis. Severe disease is best treated with corticosteroids. Hydroxychloroquine is used to help control skin manifestations and may be associated with lupus flares if discontinued. Azathiopine and cyclophosphamide are not indicated in this patient.

A 35-year-old G0 woman comes to the office because of six months of spotting between her periods and a desire for a pregnancy. She reports using 30 pads/cycle the last two months and has blood clots and cramping pain. Prior menses were light and required 15 pads/cycle. She has been trying to conceive for six months. Her work-up included a transvaginal ultrasound which revealed a 2 cm endometrial polyp. What is the next best step in the management of this patient? A. Hysteroscopic polypectomy B. Observation C. Combination birth control pills D. Endometrial ablation E. In-vitro fertilization

A. Management of an endometrial polyp includes the following: observation, medical management with progestin, curettage, surgical removal (polypectomy) via hysteroscopy, and hysterectomy. Observation is not recommended if the polyp is > 1.5 cm. In women with infertility polypectomy is the treatment of choice. While her inability to get pregnant may be more complicated than just her polyp, removal of the polyp should occur prior to infertility treatments.

A 24-year-old G1P0 woman at 32 weeks gestation presents with leaking watery fluid from the vagina. On evaluation, preterm premature rupture of membranes is confirmed. She has occasional Braxton Hicks contractions associated with fetal heart rate accelerations. She does not have vaginal bleeding and vaginal fluid phosphatidylglycerol is absent. Her blood pressure is 110/70; pulse 90; temperature 98.6°F (37.0°C). Fundal height is 30 cm and her fundus is tender. Amniotic fluid index (AFI) is 4. Which of the following findings is an indication for delivery in this patient? A. Tender uterine fundus B. Size less than dates C. Fetal heart rate accelerations D. Amniotic fluid index of less than 5 E. Absence of vaginal fluid phosphatidylglycerol

A. Maternal signs of chorioamnionitis or other evidence of intra-amniotic infection are indications for delivery. This patient has ruptured membranes and a tender fundus, which indicate chorioamnionitis. Labor at 32 weeks would be allowed to progress and prolonged non-reassuring fetal testing would prompt delivery. There are no criteria for amniotic fluid index or degree of oligohydramnios as an indication for delivery. Most authors agree that the achievement of fetal lung maturity (i.e. positive phosphatidylglycerol or 34 weeks gestational age) is the threshold at which the risk of morbidity and mortality of maintaining the pregnancy in utero outweighs the benefits of prolonging the pregnancy.

A 27-year-old G1P1 woman with no significant prior medical history reports three months of low energy, lack of enjoyment with her daily activities, early morning awakening, and trouble concentrating. What is the next best step in the assessment of this patient? A. Risk of suicide B. Willingness to accept medical treatment C. Family history of mental illness D. Good support system E. Current medication profile

A. Most depressed patients who are suicidal are relieved to be asked about it. Although all the items listed are components of a complete history, the most important topic is assessment of suicide risk.

A 25-year-old G1P1 woman who is breastfeeding her 2 ½-week-old comes to the office with left breast pain and fever. The symptoms began earlier today and are not relieved by acetaminophen. She has no known drug allergies. Her vital signs are: blood pressure 120/60; pulse 64; temperature 99.9° F (37.7° C). On exam, there is erythema on the upper outer quadrant of the left breast, which is tender to touch; there are no palpable masses. What is the most appropriate antibiotic therapy for this patient? A. Dicloxacillin B. Erythromycin C. Doxycycline D. Gentamicin E. Cefotetan

A. Most postpartum mastitis is caused by staphylococcus aureus, so a penicillin-type drug is the first line of treatment. Dicloxacillin is used due to the large prevalence of penicillin resistant staphylococci. Erythromycin may be used in penicillin allergic patients. Doxycycline, gentamicin, and cefotetan are not appropriate antibiotics for treatment of mastitis.

A 26-year-old G2P0 woman presents for counseling following manual vacuum aspiration of an eight-week missed abortion. The patient asks whether an uncomplicated first trimester pregnancy termination three years ago might have predisposed her to the subsequent spontaneous abortion. What are the patient's risks associated with the prior surgical abortion in the first trimester? A. Does not predispose the patient to subsequent spontaneous abortion B. Increases the risk of spontaneous abortion two-fold C. Predisposes the patient to primary infertility D. Increases the likelihood of subsequent pregnancy loss in both the first and second trimesters E. Increases the likelihood of spontaneous abortion and future delivery complications

A. Neither controlled trials nor surveillance data support the contention that a single, prior first trimester surgical abortion increases the risk of subsequent first trimester pregnancy loss. Indeed, first trimester surgical abortion confers no subsequent obstetric disadvantage, particularly when compared with an appropriate control population. The clinician should reassure this patient that first trimester spontaneous abortion is a common occurrence and that she has not caused this missed abortion.

A 22-year-old G1P0 woman at 38 weeks gestation has been pushing for four hours. You recommend an operative vaginal delivery. In obtaining informed consent, which of the following is less likely to occur during a vacuum delivery vs. forceps assisted delivery? A. Maternal lacerations B. Fetal cephalohematoma C. Neonatal lateral rectus paralysis D. Neonatal hyperbilirubinemia E. Neonatal retinal hemorrhage

A. Newer forms of vacuum extractors cause less maternal discomfort as they are applied to the vertex of the fetal head and do not take up additional space in the maternal pelvis. If properly applied, this leads to a decreased rate of maternal lacerations. Fetal and neonatal complications related to vacuum use include lacerations at the edges of the vacuum cup, particularly if torsion is applied. Torsion may also lead to separation of the fetal scalp from the underlying structures can cause a cephalohematoma and places the fetus at risk of jaundice. Transient neonatal lateral rectus paralysis has been found to occur more frequently in vacuum-assisted deliveries, but, because the paralysis resolves spontaneously, it is unlikely to be of clinical importance.

A 36-year-old female G1 presents for her prenatal care visit at 35 weeks gestation. She has good dating criteria that were confirmed by a first trimester ultrasound. Her previous medical history is positive for hypertension and type 2 diabetes. You have been following fetal growth with serial ultrasounds. At this visit, ultrasound reveals limited fetal growth over the past three weeks. Biometry is consistent with 32-5/7, EFW 2175 g, <10th percentile. What is the most appropriate next test indicated in the management of this patient? A. Amniotic fluid volume, umbilical artery Doppler systolic: diastolic ratio, non-stress test B. Daily fetal kick counts with follow up ultrasound to reassess fetal growth in one week C. Amniocentesis for fetal lung maturity D. Twice daily fetal kick counts with delivery at 37 weeks gestation E. None, delivery is indicated

A. When a pregnancy is complicated by fetal growth restriction, various fetal physiologic parameters require assessment. In growth-restricted pregnancies, oligohydramnios is frequently found. This finding is presumably due to reduced fetal blood volume, renal blood flow and urinary output. Chronic hypoxia is responsible for diverting blood flow from the kidney to organs that are more critical during fetal life. The significance of the amniotic fluid volume with respect to fetal outcome has been well documented. Ninety percent of patients with oligohydramnios delivered growth restricted infants. These infants experienced a high rate of fetal compromise. The systolic/diastolic (S/D) ratio of the umbilical artery is determined by Doppler ultrasound. An increase in the S/D ratio reflects increased vascular resistance. It is a common finding in IUGR fetuses. A normal S/D ratio indicates fetal well-being. As vascular resistance increases, the S/D ratio increases. With severe resistance, there is absence and ultimately reversal of end-diastolic flow. These findings are associated with an increased rate of perinatal morbidity and mortality, and a higher likelihood of a long-term poor neurologic outcome. Options for antenatal testing include the non-stress test, contraction stress test, and the biophysical profile. Any of these may be used in a growth-restricted fetus as a means of detecting possible or probable fetal asphyxia. While fetal kick counts may be of value, additional fetal testing such as twice weekly NST with AFI and weekly umbilical artery Doppler studies is indicated in monitoring fetuses with IUGR.

A 26-year-old G2P1 woman at 33 weeks gestation presents in preterm labor. She has a history of a prior preterm birth at 32 weeks gestation. She has insulin dependent diabetes and has a history of myasthenia gravis. She has regular contractions every three minutes and fetal heart tones are reassuring. Cervix is 3 cm dilated and 0 station. Her blood pressure is 140/90. Which of the following is the most appropriate tocolytic agent to use in this patient? A. Nifedipine B. Terbutaline C. Magnesium sulfate D. Indomethacin E. Ritodrine

A. Nifedipine, a calcium channel blocker, is the best option for her as she has contraindications to the other agents listed. Terbutaline and ritodrine are contraindicated in diabetic patients and the FDA made a formal announcement in 2011 warning against using terbutaline to stop preterm labor, stating that terbutaline is both ineffective and dangerous if used for longer than 48 hours. Magnesium sulfate is contraindicated in myasthenia gravis. Indomethacin is contraindicated at 33 weeks due to risk of premature ductus arteriosus closure.

A 40-year-old G1 woman comes in for her first prenatal visit. This is an unplanned pregnancy and she had a positive urine pregnancy test a week ago. She is 16 weeks gestation based on her last menstrual period. She elects to have screening for aneuploidy and open neural tube defects. Her cell-free DNA test returns screen negative. Her maternal serum alpha-fetoprotein (MSAFP) is increased (2.6 MoM). What is the most likely explanation for the elevated MSAFP in this patient? A. Under-estimation of gestational age B. Over-estimation of gestational age C. Twins D. Neural tube defect E. Fetal demise

A. Ninety to ninety-five percent of cases of elevated MSAFP are caused by conditions other than neural tube defects including under-estimation of gestational age, fetal demise, multiple gestation, ventral wall defects and a tumor or liver disease in the patient. Incorrect dating, specifically under-estimation of gestational age, is the most common explanation for an elevated MSAFP. The next appropriate step in the management of this patient is to obtain an ultrasound to assess the gestational age, viability, rule out multiple gestation as well as a fetal structural abnormality.

A 27-year-old G1P0 woman complains of mood swings and fatigue in the week prior to her menstrual period. These symptoms have worsened over the past six months. Some months the symptoms are so severe she misses several days of work. Her medical history is otherwise unremarkable and a physical examination is normal. Which of the following is the most appropriate next step in this patient's management? A. Symptom diary for two months B. Dietary changes C. Anxiolytic agent D. Psychiatric consultation E. Pelvic ultrasound

A. Obtaining further history with a menstrual calendar determines the cyclic nature of the PMS or PMDD symptoms and helps guide appropriate therapy. While dietary changes may help, it is first important to establish the diagnosis. An anxiolytic agent or psychiatric consultation is not indicated.

A 32-year-old G2P1 woman is at 41 weeks gestation. Her cervix is long and closed. She does not report contractions and states there is active fetal movement. The patient strongly desires to avoid an induction of labor. She would like to wait until she goes into labor spontaneously. Which of the following management options is optimal at this time? A. Perform a non-stress test (NST) and amniotic fluid index (AFI) twice a week with induction of labor for a nonreactive non-stress test or oligohydramnios B. Patient should perform daily fetal movement counts and proceed with induction for decreased fetal movement C. Perform daily biophysical profiles and deliver if 4 or less D. Immediate induction of labor E. Immediate Cesarean section

A. Optimal management for the patient with an unfavorable cervix at 42 weeks gestation is controversial. Induction of labor in a patient with a reactive tracing and an unfavorable cervix will minimize any risk of antepartum fetal demise; however, the risk of Cesarean section is significantly increased compared to a patient who goes into spontaneous labor. It is reasonable to follow a patient who is 41 weeks with antepartum fetal testing, such as twice weekly NSTs with amniotic fluid assessment. The risk of fetal death is 1-2/1,000 high-risk pregnancies with a reassuring non-stress test, contraction stress test or biophysical profile. The addition of amniotic fluid assessment may improve the predictive value of a reactive NST and reduce the risk of antepartum fetal demise to even lower levels.

A 19-year-old G1P0 woman presents in labor at term. Her prenatal course was uncomplicated. She delivers a 3500 gram infant spontaneously after oxytocin augmentation of labor. Postpartum, she experiences excessive bleeding. Which of the following defines postpartum hemorrhage in this patient? A. Greater than 500 cc B. Greater than 750 cc C. Greater than 1000 cc D. Greater than 1500 cc E. Any amount of bleeding that leads to hypovolemia

A. Postpartum hemorrhage is defined as bleeding in excess of 500 cc after a vaginal delivery or in excess of 1000 cc after a Cesarean delivery.

A 29-year-old G1P0 woman is at 42 weeks gestation based on her last menstrual period and a first trimester ultrasound. Of the following, what factor is most likely to be associated with postterm pregnancy? A. Placental sulfatase deficiency B. Fetal adrenal hyperplasia C. Fetal alpha-fetoprotein deficiency D. Fetal renal anomalies E. Fetal chromosomal abnormalities

A. Postterm pregnancies are associated with placental sulfatase deficiency, fetal adrenal hypoplasia, anencephaly, inaccurate or unknown dates and extrauterine pregnancy. Postterm pregnancies are not associated with fetal adrenal hyperplasia, alpha-fetoprotein deficiency, renal anomalies or chromosomal abnormalities.

A 36-year-old G0 woman who is a health worker presents for a health maintenance examination. She is sexually active and not using contraception. She doesn't think she is pregnant, but would be happy if she were. As part of her general preventive care, you discuss immunizations. Which vaccination is contraindicated if this patient gets pregnant now? A. Measles-Mumps-Rubella (MMR) B. Pneumococcus C. Hepatitis B D. Polio E. Influenza

A. Pregnancy, or the possibility of pregnancy, within four weeks is a contraindication to the MMR and varicella vaccinations. Tetanus, Hepatitis B, Polio and Pneumococcal vaccinations would not be contraindicated.

A 25-year-old G2P1 woman at 20 weeks gestation is diagnosed with preterm premature rupture of the membranes. She denies labor. She takes prenatal vitamins and iron. She denies substance abuse, smoking or alcohol use. Her prior pregnancy was delivered vaginally at 36 weeks after preterm rupture of membranes. Her blood pressure is 110/70; pulse 84; temperature 98.6°F (37.0°C). Her amniotic fluid index is 2. The patient's fetus is greatest risk for which of the following? A. Pulmonary hypoplasia B. Cardiac anamolies C. Urinary tract anamolies D. Microcephaly E. Compression fractures

A. Preterm premature rupture of the membranes that occurs before viability is associated with significant risk of poor outcome. Neonatal survival when rupture occurs between 20 and 23 weeks is approximately 25%. Complications that may be found in the developing fetus include structural abnormalities that are primarily deformations (abnormalities that occur due to an insult after a structure has already formed) rather than malformations (abnormal development of the structure itself). Pulmonary hypoplasia is seen when rupture of membranes occurs before 25 weeks gestation because the lack of amniotic fluid interferes with the normal intrauterine breathing process. The result is failure of normal development and growth of the respiratory tree.

A 27-year-old G0 woman presents to the clinic because of concerns that she has not been able to get pregnant for the last three months. She married a year ago and was using condoms for contraception, which she stopped three months ago when she decided to start a family. She is in good health and her only medication is a prenatal vitamin. Her periods are regular every 28 days with normal flow; her last period was two weeks ago. She has no history of sexually transmitted infections and no abnormal Pap smears. Her husband is also healthy with no medical problems. She is 5 feet 4 inches tall and weighs 130 pounds. Her examination, including a pelvic exam, is completely normal. What is the most appropriate next step in the management of this patient? A. Reassurance and observation B. Hysterosalpingogram C. Transvaginal pelvic ultrasound D. Semen analysis E. Mid-cycle blood LH and FSH levels

A. Reassurance and observation is most appropriate as the patient has only been trying to conceive for three months. After one month, 20% of couples will conceive; after three months, 50%; after six months, 75%; and after 12 months, 90% will conceive. Primary infertility is defined as the inability to conceive for one year without contraception. The patient is young and healthy with no obvious reasons for infertility, so at this point reassurance and observation is the proper management. There is no reason to order any studies now, especially since she has normal cycles.

A 17-year-old female is brought to the physician because she has never had a menstrual cycle. She has normal breast and pubic hair development. Physical examination reveals a small vaginal opening with a blind pouch. Pelvic ultrasound reveals normal ovaries, but absence of uterus and cervix. Which of the following is the most appropriate next study in this patient? A. Renal ultrasound B. Brain MRI C. FSH and LH determination D. Cortisol level E. Testosterone level

A. Renal anomalies occur in 25-35% of females with Mullerian agenesis. The uterus and cervix are absent, but the ovaries function normally and, therefore, secondary sexual characteristics are present. You would expect the karyotype in this patient to be 46,XX and testosterone levels in the female range.

A 17-year-old G0 female presents with hirsutism, irregular menses and obesity. Her mother is moderately obese with mild hirsutism. Recently, the patient's hirsutism has worsened and she has been depressed. She has also gained 20 pounds in the past two months and has noticed stretch marks on her abdomen. At the time of your examination, you note that she has terminal hair growth on her chin and hair growth on the back of her hands. Her cheeks appear flushed. Her stretch marks are purplish in color. The rest of her exam is normal. Which of the following is the most appropriate first test to order for this patient? A. Overnight dexamethasone suppression test B. 17-hydroxyprogesterone C. Fasting insulin D. TSH E. Pelvic ultrasound

A. Since Cushing's syndrome is suspected, either a dexamethasone suppression test or a 24-hour urinary measurement for cortisol can be performed. Elevated cortisol would be indicative of Cushing's syndrome. The other tests listed would be reasonable, but only after Cushing's syndrome had been excluded.

A 32-year-old G1 presents at 35 weeks gestation with decreased fetal movement. Her prenatal course has been complicated by size less than dates. Serial ultrasounds show a decrease of the estimated fetal weight from 60th to 20th percentile. The non-stress test is reactive and the amniotic fluid index is 10. What is your next step in management? A. Continue with weekly non-stress tests B. Obtain umbilical artery systolic: diastolic ratio C. Admission for daily fetal surveillance D. Induction of labor today E. Cesarean section today

A. Since the patient reported decreased fetal movement, a non-stress was performed and was reassuring. The NST is based on the principle that when the fetus moves, its heartbeat normally accelerates. The NST assesses fetal health through monitoring accelerations of the heart rate in response to the baby's own movements. Amniotic fluid volume is important because a decreased amount raises the possibility that the fetus may be under stress. Since the fetus does not show growth restriction and fetal status was reassuring, there are no indications for Doppler studies or delivery. In light of the dramatic decrease in growth, it is reasonable to follow this patient with weekly non-stress tests.

A 21-year-old G0 woman comes to the office because of acne, irregular menses and hirsutism. She initially was evaluated six months ago, at which time she was diagnosed with idiopathic hirsutism. She was started on oral contraceptive pills to improve her symptoms. Menstrual periods now occur every month, but her hirsutism has not significantly improved. In addition to the oral contraceptives, which of the following would be an appropriate treatment for hirsutism? A. Spironolactone B. Lupron C. Danazol D. Depo-Provera E. Steroids

A. Spironolactone, an aldosterone antagonist diuretic, can also be used in addition to the oral contraceptives for hirsutism. Danazol is primarily used for the treatment of endometriosis and may actually worsen hirsutism and acne. Lupron and Depo-Provera are also reasonable as second-line treatments of hirsutism, had the patient not already been on oral contraceptives. Steroids will not help.

A 28-year-old G0 woman presents for preoperative counseling. She has a suspected diagnosis of severe endometriosis and has failed conservative medical management with OCPs. Her symptoms include severe pelvic pain especially prior to and during menstruation, and deep dyspareunia. On pelvic examination, she has uterosacral nodularity and tenderness throughout both adnexa. Ultrasound reveals normal ovaries bilaterally. Which of the following procedure is the best option for this patient? A. Laser ablation B. Ovarian cystectomy C. Salpingectomy D. Bilateral salpingo-oophorectomy E. Total laparoscopic hysterectomy and bilateral salpingo-oophorectomy

A. Surgery is the gold standard in the diagnosis of endometriosis, but often is not the initial treatment as suspected endometriosis is often managed medically. The role of surgery is often to manage the symptoms of endometriosis, often pelvic pain. As such, surgery may be conservative (laparoscopic ablation or excision of implants, excision of endometriomas) or definitive (total hysterectomy/BSO). In this young nulliparous patient, definitive surgery is not indicated, and only laser ablation would be recommended. As she has no evidence of adnexal masses or enlarged ovaries on ultrasound, it is unlikely that ovarian cystectomy would be needed. There is no indication for salpingectomy.

A 54-year-old G2P2 woman presents to your office for a health maintenance examination. Her last menstrual period was eight months ago. She complains of severe vasomotor symptoms, vaginal dryness, and dyspareunia, and desires treatment for her symptoms. She has otherwise been in good health and has no significant past medical or surgical history. Her family history is significant for a mother who has severe osteoporosis at the age of 75, a grandmother who died of breast cancer at the age of 79, and a father who died at age 77 from a myocardial infarction. She denies smoking, alcohol or drug use. On physical exam her BP is 130/78, pulse is 84, and BMI is 26. The remainder of her exam is within normal limits except for severe vaginal atrophy noted on the pelvic examination. The best recommendation for this patient would include which of the following? A. Lowest effective dose of combination hormone replacement therapy for the shortest duration possible B. Long term hormone replacement therapy to treat her vasomotor symptoms and prevention of osteoporosis C. Testosterone cream D. Progesterone cream E. Biosphophonates

A. The American College of Obstetricians and Gynecologists (ACOG) recommendations on hormone replacement therapy considers hormone replacement therapy (HRT) the most effective treatment for severe menopausal symptoms that include hot flashes, night sweats and vaginal dryness. The physician should counsel the woman about the risks and benefits before initiating treatment. ACOG recommends "the smallest effective dose for the shortest possible time and annual reviews of the decision to take hormones." HRT should not be used to prevent cardiovascular disease due to the slight increase in risk of breast cancer, myocardial infarction, cerebrovascular accident, and thromoboembolic events. A woman with an intact uterus should not use estrogen-only therapy because of the increased risk of endometrial cancer. In addition to the same risks as FDA approved treatments, bioidentical hormones such as testosterone and progesterone cream may have additional associated risks. While her family history is significant for one second degree relative with breast cancer, this is not an absolute contraindication for short-term HRT.

A 28-year-old G0 woman has a pap test which is reported as high-grade squamous intraepithelial lesion (HSIL). She is currently sexually active. She has had six sexual partners and has been in a monogamous relationship with her fiancé for the last year. What is the next most appropriate next step in the management of this patient? A. Colposcopy B. Cryotherapy C. Reflex HPV testing D. Repeat Pap test in one month E. Repeat Pap test in six months

A. The American Society for Colposcopy and Cervical Pathology guidelines recommend immediate LEEP or colposcopy for women with HSIL cytology test results. A diagnostic excisional procedure is recommended for women with HSIL cytology test results when the colposcopic examination is inadequate. Unlike a LEEP, cryotherapy is inadequate as this procedure does not provide a tissue specimen. Repeat cytology testing alone or reflex HPV testing is unacceptable. For women not managed with immediate excision, colposcopy is recommended regardless of HPV result obtained at co-testing.

A 58-year-old G3P3 woman who has been menopausal since age 50 comes to you for a health maintenance examination. She is in good health, eats a balanced diet, exercises regularly, and has an unremarkable physical exam. Her bone mineral density as determined by central dual energy X-ray absorptiometry is -1.7. She wants to discuss treatment for her osteopenia. What is the next step in the management of this patient? A. Evaluate her risk factors for fracture B. Determine her frequency of exercise C. Assess her exogenous dietary intake of estrogen D. Assess her exogenous dietary intake of progesterone E. Repeat DEXA scan in one year

A. The World Health Organization (WHO) defines osteopenia (low bone mass) as -1 to -2.5. The American College of Obstetricians and Gynecologists (ACOG) Committee Opinion recommends that physicians interpret T scores between −1.0 and −2.5 in combination with the patient's risk factors for fracture. The authors state: "Clinicians must be careful because the diagnosis of osteopenia often is interpreted as indicating a pathologic skeletal condition or significant bone loss, neither of which is necessarily true. Until better models of absolute fracture risk exist, postmenopausal women in their 50s with T scores in the osteopenia range and without risk factors may well benefit from counseling on calcium and vitamin D intake and risk factor reduction to delay initiation of pharmacologic intervention." Some of the risk factors for fracture include prior fracture, family history of osteoporosis, race, dementia, history of falls, poor nutrition, smoking, low body mass index, estrogen deficiency, alcoholism, and insufficient physical activity.

A 24-year-old G1P0 woman at 12 weeks gestation presents for prenatal care. She is 5 feet 4 inches tall and weighs 220 pounds (BMI: 37.8 kg/m2). She wants to know if there is an increased risk on her pregnancy because of her size. Which of the following is the most common complication in this patient? A. Hypertension B. Preterm labor C. Post-term pregnancy D. Small for gestational age E. Shoulder dystocia

A. The body mass index (BMI) is equal to a person's weight in kg divided by their height in meters squared. The National Heart, Lung, and Blood Institute identify a normal BMI as 18.5 to 24.9 kg/m2; overweight as a BMI of 25 to 29.9 kg/m2; and obesity as a BMI of 30 kg/m2 or greater. Obesity is further categorized as class I (BMI: 30 to 34.9 kg/m2), class II (BMI: 35 to 39.9 kg/m2), and class III (BMI: > or = 40 kg/m2). Increased maternal morbidity results from obesity and includes chronic hypertension, gestational diabetes, preeclampsia, fetal macrosomia, as well as higher rates of Cesarean delivery and postpartum complications. This patient's BMI is approximately 38 so she is a class II and has over a 7-fold increase risk for preeclampsia and a 3-fold risk for hypertension.

A 39-year-old G0 woman presents to the clinic reporting non-tender spots on her vulva for about a week. No pruritus or pain is present. She also notes a brownish rash on the palms of her hands. She admits to IV drug abuse. She was diagnosed as HIV-positive two years ago, but has not been compliant with suggested treatment. On examination, three elevated plaques with rolled edges are noted on the vulva. They are non-tender. A brown macular rash is noted on the palms of her hands and the soles of her feet. What is the most appropriate next step in the management of this patient? A. Obtain a treponemal-specific test B. Biopsy of the lesion C. Colposcopic evaluation of the vulvar lesions D. Culture the base of the lesion E. Initiate empiric treatment with doxycycline and ceftriaxone

A. The diagnosis of syphilis is often established by serologic testing. Non-treponemal tests (VDRL or RPR) are non-specific. In this patient with high suspicion for syphilis, specific testing with treponemal antibody can confirm infection. The classic coiled spirochete is easily seen with dark-field microscopy but availability is limited. A characteristic finding is a macular rash on the palms and soles that are often described as copper penny lesions. Colposcopy would not be diagnostic, but certainly is helpful to evaluate for any vulvar lesions thought to be dysplastic. Biopsies can be stained for spirochetes and may show a necrotizing vasculitis, but certainly would not be the most expedient way to make the diagnosis. Penicillin G is the preferred drug for treating all stages of syphilis.

A 46-year-old female is scheduled to undergo a total laparoscopic hysterectomy for menorrhagia. She comes into your office to discuss the effects a hysterectomy will have on her sex life. She is concerned about how a hysterectomy will affect her "womanhood" and read in a woman's magazine that the cervix, as well as the uterus, is necessary for orgasm. Which of the following organs is most responsible for sexual arousal? A. Vulva B. Cervix C. Uterus D. Ovary E. All of the above

A. The external genitalia, specifically the clitoris, are essential organs involved in the arousal component of the sexual response cycle, which consists of desire, arousal, orgasm, and resolution. Hormones such as androgen and estrogen (produced by the ovaries) are key to desire, while genital mechanisms such as clitoral, labial, and vaginal engorgement are key to arousal. With arousal and adequate sensory stimulation, orgasm ultimately may occur consisting of repeated motor contractions of the pelvic floor including uterine and vaginal smooth muscle contractions.

A 38-year-old G1P0 woman presents to the hospital at 39 weeks in early labor. She has had routine prenatal care and no antepartum complications to date. She reports good fetal movement and denies vaginal bleeding and leakage of fluid. What is the next best step in the initial assessment of this patient? A. Physical examination B. Nitrazine test C. Fetal ultrasound D. Biophysical profile E. Contraction stress test

A. The initial evaluation of patients presenting to the hospital for labor includes a review of the prenatal records with special focus on the antenatal complications and dating criteria, a focused history and a targeted physical examination to include maternal vital signs and fetal heart rate, and abdominal and pelvic examination. A speculum exam with a nitrazine test to confirm rupture of membranes is indicated if the patient's history suggests this, or if a patient is uncertain as to whether she has experienced leakage of amniotic fluid. Performing a fetal ultrasound is not a routine part of an assessment in a patient who may be in early labor. A prenatal ultrasound may be used in cases to determine fetal presentation, estimated fetal weight, placental location or amniotic fluid volume.

A 24-year-old G1P1 woman presents with a complaint of decreasing breast size and hirsutism noted over the last three months. She also notes her skin feels oily and her husband has mentioned her voice seems to be getting deeper. She has no medical or surgical problems and takes no medications. Physical examination reveals oily skin, upper lip and chin terminal hair, and normal appearing breasts. Pelvic examination reveals her clitoris to be 2 cm in length and 1 cm wide. Which of the following is the most likely cause of her symptom constellation? A. Steroid cell tumor B. Hypothyroidism C. Polycystic ovary syndrome D. Congenital adrenal hyperplasia (early onset) E. Anabolic steroids

A. The likely cause of this patient's sudden onset of symptom is an increase in androgens due to a tumor. Hirsutism is often the result of a benign condition, however, may be a sign of significant disease if sudden in onset and coupled with virilization. Virilization in the female may be manifested by frontal hair thinning, oily skin or acne, deepening of the voice, clitoral enlargement, menstrual irregularities, and increased muscle strength. Possible causes of virilization include PCOS, hypothyroidism, androgen producing tumors (ovarian, adrenal, or pituitary), and anabolic steroid use. A rare cause may be late onset congenital adrenal hyperplasia.

A 34-year-old G2P2 woman presents with concerns of hormonal changes. She is worried about facial hair growth, worsening acne, and deepening of her voice. She also realized that she has missed her period for two months, and has been sexually active and had tubal ligation. On examination, she is moderately obese and noted to have severe acne, upper lip and chin terminal hair. Her abdomen is obese with moderate hair growth. Pelvic examination is most notable for an enlarged clitoris, and pelvic exam reveals an enlarged right-sided adnexal mass. Which of the following is the most likely diagnosis in this patient? A. Sertoli-Leydig cell tumors B. Granulosa cell tumor C. Benign cystic teratoma D. Thecoma E. Cystadenoma

A. The most likely diagnosis in this patient is a testosterone-secreting ovarian tumor. Sertoli-Leydig cell tumors are commonly diagnosed in women between the ages of 20-40, and are most often unilateral. Rapid onset of hirsutism and virilizing signs are hallmarks of this disease, and include many of the findings in this patient including acne, hirsutism, amenorrhea, clitoral hypertrophy, and deepening of the voice. Abnormal laboratory findings include suppression of FSH and LH, marked elevation of testosterone, and presence of an ovarian mass. The constellation of findings is most consistent with a testosterone-secreting tumor, and a pelvic ultrasound will confirm the presence of an ovarian mass. The other tumors do not cause virilization. Granulosa cell tumors and thecomas are estrogen-secreting tumors.

A 23-year-old G1P0 woman presents with cramping, vaginal bleeding and right lower quadrant pain. Her last normal menstrual period occurred seven weeks ago. On physical exam, vital signs are: blood pressure 110/74; pulse 82; respirations 18; and temperature 99.4°F (37.4°C). On abdominal exam, she has very mild right lower quadrant tenderness. On pelvic exam, she has scant old blood in the vagina and a normal appearing cervix. Her uterus is normal size and slightly tender. On bimanual exam, there is no cervical motion tenderness, and she has slight tenderness in right lower quadrant. Quantitative Beta-hCG is 2500 mIU/ml; progesterone 6.2 ng/ml; hematocrit 34%. The transvaginal ultrasound shows an empty uterus with endometrial thickening, a mass in the right ovary measuring 3 x 2 cm and a small amount of free fluid in the pelvis. Which of the following is the most appropriate next step in the management of this patient? A. Methotrexate B. Antibiotics C. Observation D. Dilation and curettage E. Culdocentesis

A. The next best step in management is methotrexate administration. Certain conditions must be met prior to initiating methotrexate therapy for treatment of an ectopic pregnancy. These include: hemodynamic stability; non-ruptured ectopic pregnancy; size of ectopic mass <4 cm without a fetal heart rate or <3.5 cm in the presence of a fetal heart rate; normal liver enzymes and renal function; normal white cell count; and the ability of the patient to follow up rapidly (reliable transportation, etc.) if her condition changes. There is no indication for antibiotics in this scenario. Offering observation delays treatment and pain control would not address the underlying cause of the patient's problem. Culdocentesis is not indicated and would not change the management of this patient.

A 26-year-old G2P2 woman presents with a new onset of vulvar burning and irritation. She is sexually active with a new male partner. She is using oral contraception for birth control and did not use a condom. She thought she had a cold about 10 days ago. Which of the following is the most likely diagnosis in this patient? A. Herpes simplex virus B. Primary syphilis C. Secondary syphilis D. Human immunodeficiency virus E. Trichomonas

A. The patient is most likely infected with herpes. Herpes simplex virus is a highly contagious DNA virus. Initial infection is characterized by viral-like symptoms preceding the appearance of vesicular genital lesions. A prodrome of burning or irritation may occur before the lesions appear. With primary infection, dysuria due to vulvar lesions can cause significant urinary retention requiring catheter drainage. Pain can be a very significant finding as well. Treatment is centered on care of the local lesions and the symptoms. Sitz baths, perineal care and topical Xylocaine jellies or creams may be helpful. Anti-viral medications, such as acyclovir, can decrease viral shedding and shorten the course of the outbreak somewhat. These medications can be administered topically or orally. Syphilis is a chronic infection caused by the Treponema pallidum bacterium. Transmission is usually by direct contact with an infectious lesion. Early syphilis includes the primary, secondary, and early latent stages during the first year after infection, while latent syphilis occurs after that and the patient usually has a normal physical exam with positive serology. In primary syphilis, a painless papule usually appears at the site of inoculation. This then ulcerates and forms the chancre, which is a classic sign of the disease. Left untreated, 25% of patients will develop the systemic symptoms of secondary syphilis, which include low-grade fever, malaise, headache, generalized lymphadenopathy, rash, anorexia, weight loss, and myalgias. This patient's symptoms are less consistent with syphilis, but she should still be tested for it. Human immunodeficiency virus is an RNA retrovirus transmitted via sexual contact or sharing intravenous needles. Vulvar burning, irritation or lesions are not typically noted with this disease, although generalized malaise can be. HIV can present with many different signs and symptoms, therefore risk factors should be considered, and testing offered. Trichomonas is a protozoan and is transmitted via sexual contact. It typically presents with a non-specific vaginal discharge. It does not have a systemic manifestation.

A 36-year-old G2P2 woman presents with irregular vaginal bleeding. Six weeks ago, she had her first Depo-Provera® injection and now she has unpredictable bleeding. She is concerned by these symptoms. She has a history of hypertension but is currently on no medications. Vital signs reveal: blood pressure 130/90; weight 188 pounds; height 5 feet 5 inches; BMI 31.4kg/m2. Which of the following is the most appropriate next step in the management of this patient? A. Reassurance B. Begin oral contraceptives C. Begin estrogen D. Insert etonogestrel implant (Implanon) E. Perform an endometrial biopsy

A. The patient should be reassured since initially after Depo-Provera injection there may be unpredictable bleeding. This usually resolves in 2-3 months. In general, after one year of using Depo-Provera, nearly 50% of users have amenorrhea.

A 28-year-old G0 woman presents to your office for preconception counseling. She has a history of type 1 diabetes, diagnosed at age six, and uses an insulin pump for glycemic control. She has a history of proliferative retinopathy treated with laser. Her last ophthalmologic examination was three months ago. Her last hemoglobin A1C (glycosylated hemoglobin level) six months ago was 9.2%. Which of the following complications is of most concern for her planned pregnancy? A. Fetal growth restriction B. Fetal cardiac arrhythmia C. Twins D. Oligohydramnios E. Macrosomia

A. The patient with type 1 diabetes is at risk for many pregnancy complications. In women with insulin-dependent diabetes, the rates of spontaneous abortion and major congenital malformations are both increased. The risk appears related to the degree of metabolic control in the first trimester. Overt diabetic patients are also at an increased risk for fetal growth restriction, although fetal macrosomia may also occur. The former becomes a greater concern as in this patient, with longer-term diabetes and vascular complications, such as retinopathy. Diabetics also have increased risk for polyhydramnios, congenital malformations (cardiovascular, neural tube defects, and caudal regression syndrome), preterm birth and hypertensive complications. Her diabetes does place her at an increased for twins.

A 53-year-old G2P2 woman comes to your office complaining of six months of worsening hot flashes, vaginal dryness, night sweats, and sleep disturbances. Her last normal menstrual period was six months ago and she has been experiencing intermittent small amounts of vaginal bleeding. Her medical history is significant for hypertension, which is well-controlled by a calcium-channel blocker, type 2 diabetes, for which she takes metformin, and hyperthyroidism, for which she takes propylthiouracil. The patient is 5 feet 7 inches tall and weighs 140 pounds. Blood pressure is 120/70. Physical examination is unremarkable. Which of the following medical conditions in this patient is a contraindication to treatment of menopausal symptoms with hormone therapy? A. Vaginal bleeding B. Hypertension C. Diabetes D. Osteoporosis E. Hyperthyroidism

A. The principal symptom of endometrial cancer is abnormal vaginal bleeding. Although the patient's worsening symptoms make treatment an important consideration, the specific organic cause(s) of abnormal bleeding must be ruled out prior to initiating therapy. A tissue diagnosis consistent with normal endometrium or a pelvic ultrasound with an endometrial stripe of <4 mm ought to be documented. In addition, risks and benefits of hormone replacement therapy must be discussed with this patient at length prior to beginning treatment.

A 15-year-old G0 female presents with severe menstrual pain for the past 12 months. The pain is severe enough for her to miss school. The pain is not relieved with ibuprofen 600 mg every four hours. She is not sexually active and the workup reveals no pathology. Which mechanism of action best explains why oral contraceptives would be the most appropriate treatment for this patient? A. Inducing endometrial atrophy B. Decreasing inflammation C. Increasing prolactin levels D. Decreasing inhibin levels E. Thickening cervical mucous

A. The progestin in oral contraceptives causes endometrial atrophy. Since prostaglandins are produced in the endometrium, there would be less produced. Dysmenorrhea should be improved.

You are asked to give a lecture on a new chemotherapy drug that has demonstrated a reasonable efficacy in women with advanced cervical cancer. The day before giving the lecture, you realize that you own stock in the company that makes the drug. Which of the following statements about conflict of interest is true? A. Pharmaceutical companies can support the costs of medical conferences in which physicians receive continuing medical education credits B. b. Physicians should engage in agreements in which companies make a substantial donation to an educational activity, when the donation is contingent on the physician's use or advocacy of a product C. c. The hospital may not interfere with a physician's decision to use a new surgical device D. d. An investigator may not own stock in a company if he/she does research for that company E. e. Physicians are not required to disclose any potential conflict of interest before speaking in a national forum

A. The relationship of physicians and hospitals with pharmaceutical companies is a sensitive one, as there is potential for conflict of interest. It is acceptable for pharmaceutical companies to support conferences in which physicians receive CME credit. Physician participation in those activities should not be contingent upon physician use or advocacy of the product. An investigator may own stock in a company if he/she does research for that company, as long as he/she declares the conflict of interest and the conflict of interest is addressed.

A 28-year-old G1P0 internal medicine resident at 34 weeks gestation had pulmonary function tests performed two days ago because she was feeling slightly short of breath. She is a non-smoker, and has no personal or family history of cardiac or respiratory disease. Vital signs are: respiratory rate 16; pulse 90; blood pressure 112/70; temperature 98.6°F (37°C); oxygen saturation is 99% on room air. On physical examination: lungs are clear; abdomen non-tender; fundal height is 34 cm. The results of the pulmonary function tests are: inspiratory capacity (IC) increased; tidal volume (TV) increased; minute ventilation increased; functional reserve capacity (FRC) decreased; expiratory reserve capacity (ERC) decreased; residual volume (RV) decreased. What is the next best step in the evaluation of this patient? A. Routine antenatal care B. Chest x-ray C. Arterial blood gas D. Spiral CT of the lungs E. Echocardiogram

A. The results of her PFT are consistent with normal physiologic changes in pregnancy. Inspiratory capacity increases by 15% during the third trimester because of increases in tidal volume and inspiratory reserve volume. The respiratory rate does not change during pregnancy, but the TV is increased which increases the minute ventilation, which is responsible for the respiratory alkalosis in pregnancy. Functional residual capacity is reduced to 80% of the non-pregnant volume by term. These combined lead to subjective shortness of breath during pregnancy.

A 24-year-old Rh-negative G1P1 woman just delivered a healthy term infant who is Rh-positive. You recommend RhoGAM administration but she declines because she does not desire any blood products. What is her approximate risk of isoimmunization if she does not receive the RhoGAM? A. Less than 20% B. 40% C. 60% D. 80% E. 100%

A. The risk of isoimmunization is 2% antepartum, 7% after full term delivery, and 7% with subsequent pregnancy so less than 20% total. While 75% of all gravidas have evidence of transplacental hemorrhage during pregnancy or immediately after delivery, 60% of these patients have <0.1 cc of fetal blood in the maternal circulation, which is enough to sensitize a patient. The incidence and size of transplacental hemorrhage increases as pregnancy advances. During the second month of gestation, 5-15% of women will have evidence of feto-maternal hemorrhage. By the third trimester, this number increases to 45% of patients.

A 24-year-old G1P0 woman has just delivered 37 week male twins. On your initial assessment, you notice twin A is large and plethoric, and twin B is small and pale. A complete blood count (CBC) is obtained on both twins. What is the most likely potential neonatal risk in this case? A. Twin A is at high risk for polycythemia B. Twin A is at high risk for thrombocytopenia C. Twin B is at high risk for thrombocytopenia D. Twin B is at high risk for tachycardia E. Twin B is at high risk for hyperbilirubinemia

A. This case is suggestive of twin-twin transfusion syndrome (TTTS). Polycythemia is a common complication for the plethoric twin. TTTS is a complication of monochorionic pregnancies. It is characterized by an imbalance in the blood flow through communicating vessels across a shared placenta leading to under perfusion of the donor twin, which becomes anemic and over perfusion of the recipient, which becomes polycythemic. The donor twin often develops IUGR and oligohydramnios, and the recipient experiences volume overload and polyhydramnios that may lead to heart failure and hydrops.

A 42-year-old G4P4 woman presents with a history of progressively worsening severe menstrual pain. Menses are regular, but she complains of very heavy flow requiring both a menstrual pad and tampon and often bleeds through both. She takes oxycodone that her husband used for back pain to relieve her dysmenorrhea. She had a tubal ligation four years ago. Pelvic examination shows an enlarged, soft, boggy uterus. No masses are palpated. Pregnancy test is negative, hemoglobin 9.8 and hematocrit 28.3%. What is the most likely diagnosis? A. Adenomyosis B. Endometrial carcinoma C. Endometriosis D. Primary dysmenorrhea E. Endometrial hyperplasia

A. This is a typical presentation of adenomyosis (presence of endometrial glands and supporting tissues in the muscle of the uterus). The gland tissue grows during the menstrual cycle and, at menses, tries to slough, but cannot escape the uterine muscle and flow out of the cervix as part of normal menses. This trapping of the blood and tissue causes uterine pain in the form of monthly menstrual cramps. Endometrial hyperplasia and carcinoma are less likely in a woman with regular menses and no inter-menstrual spotting. Endometriosis would most likely have presented earlier in life and would not explain the enlarged uterus.

A 16-year-old female goes to the doctor to discuss why she has not had a menstrual cycle. She is healthy and plays weekend volleyball. She studies hard and gets good grades in school. She has a good relationship with her parents. On examination she is 5 feet 1 inch tall and weighs 80 pounds. Breast and pubic hair growth are at a Tanner stage II. External genital examination is normal. What is the most likely reason this patient has not had any menses? A. Inadequate body weight B. Poor nutrition C. Inadequate sleep D. Excessive exercise E. Familial reasons

A. This patient does not weigh enough since a body weight of 85 to 106 pounds is needed before menses begins. There are two other critical elements for secondary sexual characteristics: sleep and optic exposure to sunlight. These factors especially can delay the onset of menarche. Psychosocial causes of delayed puberty include eating disorders, excessive exercise (playing volleyball on the weekend is not considered excessive exercise), and stress or depression.

A 27-year-old G1P0 woman at 14 weeks gestation presents with a two-month history of insomnia, feeling depressed, and unintentional weight loss. Symptoms began after the unexpected death of her father. She is not excited about this pregnancy and reports no suicidal ideation. Physical examination reveals a woman of stated age with a flat affect. Which of the following therapies is contraindicated in this patient? A. Paroxetine (Paxil®) B. Sertraline (Zoloft®) C. Fluoxetine (Prozac®) D. Citalopram (Celexa®) E. Bupropion (Wellbutrin®)

A. This patient has classic depression. The most commonly used antidepressants are the selective serotonin reuptake inhibitors (SSRIs). One SSRI, paroxetine (Paxil) has been changed to a category D drug because of the increased risk of fetal cardiac malformations and persistent pulmonary hypertension. Other SSRI compounds, fluoxetine, sertraline, and citalopram have not been reported to cause early pregnancy loss or birth defects in animals or in humans. Because these agents have few side effects compared with other antidepressants, they are a good choice for pregnant women. Bupropion is not an MAO inhibitor, nor is it an SSRI and a report by the Bupropion Pregnancy Registry reports no unusual effects in 90 exposed pregnancies.

A 27-year-old G1P0 woman at 34 weeks gestation presents with increased swelling in her face and hands over the last two days. Blood pressure is 155/99. A 24-hour urine sample for protein is 440 mg/dL. BMI is 27. Repeat blood pressure two days later is 150/92. Which of the following is the most likely diagnosis in this patient? A. Preeclampsia B. Preeclampsia with severe features C. Eclampsia Syndrome D. Gestational hypertension E. Chronic hypertension

A. This patient has met criteria for the diagnosis of mild preeclampsia based on her persistent elevation of blood pressure and 24-hour urine results. The amount of protein excreted in the urine varies throughout the day, therefore a sample is collected over a 24-hour time period. Twenty-four hour urine protein values greater that 300 mg are required for the diagnosis of mild preeclampsia. Values greater than 5000 mg (or 5 g) are required for the diagnosis of severe preeclampsia (assuming no other defining criteria are present such as SBP >160 or DBP >110). This patient has not had a seizure which is the hallmark of eclampsia syndrome.

A 34-year-old G4P3 woman at 19 weeks gestation presents to the emergency department with chest pain, palpitations and sweating, which began 2-3 hours ago. On further questioning, she states that she has been very anxious lately and is not sleeping well, which she attributes to the pregnancy. She reports that she has lost 40 pounds in the last year without trying. She denies significant medical problems. On exam, the patient appears diaphoretic and anxious, her eyes are wide open, prominent, with easily visible sclera surrounding the pupil. Vital signs are: temperature 100.2°F (37.9°C); pulse 132; and blood pressure 162/84. Her height is 5 feet 10 inches and weight is 128 pounds. Her thyroid is palpably enlarged, with an audible bruit. Electrocardiogram shows sinus tachycardia. Remaining labs are pending. Which of the following therapies is contraindicated at this time? A. Radioactive iodine (I-131) B. Propylthiouracil (PTU) C. Propranolol D. Inorganic iodide E. Intravenous fluid replacement

A. This patient is in thyroid storm, an acute, life-threatening, hypermetabolic state. Radioactive iodine (I-131) concentrates in the fetal thyroid and may cause congenital hypothyroidism, so it should not be intentionally used in pregnancy. Acute treatment of thyroid storm may include thioamides (i.e. PTU), propranolol, sodium iodide and dexamethasone. Oxygen, digitalis, antipyretics and fluid replacement may also be indicated. Maternal mortality with thyroid storm exceeds 25%.

A 21-year-old G1P0 woman presents to labor and delivery at 39 weeks gestation with a chief complaint of decreased fetal movement over the last two days. An ultrasound shows a fetus with biometry consistent with 34 weeks gestation with no cardiac activity. The head circumference and biparietal diameter are consistent with 37 weeks and the abdominal circumference, femur and humerus lengths are all lagging by approximately five weeks. The amniotic fluid volume is slightly decreased. No other abnormalities are identified. The patient's medical history is notable for a deep venous thrombosis which she had three years ago while she was using oral contraceptives. She had a reassuring quad screen. She denies any history of fever or viral illnesses during the pregnancy. She works as a preschool teacher. The patient had a fetal ultrasound at 20 weeks gestation. At that time all of the fetal anatomy was well-visualized and no abnormalities were identified. Which of the following is the most likely explanation for the fetal demise in this case? A. Trisomy 21 B. Trisomy 18 C. Poorly controlled undiagnosed diabetes mellitus D. Fetal parovirus infection E. Factor V Leiden mutation

A. This patient is most likely to have the autosomal dominant Factor V Leiden (FVL) mutation based on her history. FVL is the most common inherited thrombophilic disorder affecting approximately 5% of Caucasian women in the United States. It is a point mutation which alters factor V making it resistant to inactivation by protein C. The thrombophilic effect of a FVL mutation has been clearly established. Heterozygosity for FVL is associated with a five- to ten-fold increased risk of thrombosis, while homozygosity is associated with an 80-fold increased risk. The FVL mutation is associated with obstetric complications including stillbirth, preeclampsia, placental abruption and IUGR. Fetuses with Trisomy 18 are likely to have congenital anomalies that are detectable on prenatal ultrasound. Over 90% of cases of trisomy 21 and 18 may be detected with the quad screen. A congenital parvovirus infection associated with a fetal demise would likely cause hydrops in the fetus which would be identified on ultrasound. Although poorly controlled diabetes mellitus are associated with fetal demise, they are not the most likely etiologies in this patient whose presentation is classic for the FVL mutation.

A 17-year-old G0 female is brought in by her mother because she has not yet had any menses. She is otherwise in good health, but recently has been experiencing cyclical lower abdominal cramping. She has never had sexual intercourse. She is 5 feet 6 inches tall and weighs 120 pounds. On examination, her breasts are Tanner Stage IV. She has some suprapubic tenderness on abdominal exam. Her pelvic exam reveals normal external genitalia, but there was difficulty inserting a speculum due to patient's discomfort. Beta-hCG < 5 mIU/mL. What is the most likely diagnosis in this patient? A. Genital tract outflow obstruction B. Müllerian agenesis C. Hypothalamic-pituitary dysfunction D. Psychogenic amenorrhea E. Constitutional delay in menarche

A. This patient's primary amenorrhea, with normal secondary sexual characteristics, development and cyclical abdominal pain, points to an anatomical cause of amenorrhea, which is preventing menstrual bleeding. An imperforate hymen commonly causes this and the treatment is surgical. In Mϋllerian agenesis, or Mayer-Rokitansky-Kϋster-Hauser syndrome, there is congential absence of the vagina and usually an absence of the uterus and fallopian tubes. Ovarian function is normal and all the secondary sexual characteristics of puberty occur at the appropriate time.

A 22-year-old G0 woman college student returns for follow-up of mood swings and difficulty concentrating on her schoolwork the week before her menses for the past 12 months. Her past medical history is unremarkable and physical examination is normal. Which of the following would be an appropriate treatment option for this patient? A. Oral contraceptive pills B. Reassurance and observation C. Methylphenidate (Ritalin) D. Gabapentin E. Ginkgo

A. This woman has premenstrual syndrome (PMS) with symptoms that warrant treatment. Patients with PMS and premenstrual dysphoric disorder (PMDD) experience adverse physical, psychological and behavioral symptoms during the luteal phase of the menstrual cycle. PMS is characterized by mild to moderate symptoms, while PMDD is associated with severe symptoms that seriously impair usual daily functioning and personal relationships. Mild symptoms of PMS often improve by suppressing the hypothalamic-pituitary-ovarian axis with oral contraceptive pills. Ritalin and Ginkgo are not effective treatments for PMS. Gabapentin is used for neuropathic pain and will not help alleviate her symptoms.

A 34-year-old G4P2 woman at 18 weeks gestation presents with fatigue and occasional headache. She has a sister with Grave's disease. On physical exam, vital signs are normal. BMI is 27. Thyroid is difficult to palpate due to her body habitus. The remainder of her exam is unremarkable. Thyroid function studies show: Results Reference Range TSH 1.8 mU/L 0.30 - 5.5 mU/L Free T4 1.22 ng/dL 0.76 - 1.70 ng/dL Total T4 14.2 ng/dL 4.9 - 12.0 ng/dL Free T3 3.4 ng/dL 2.8 - 4.2 ng/dL Total T3 200 ng/dL 80 - 175 nd/dL What is the next best step in the management of this patient? A. Continue routine prenatal care B. Check anti-thyroid antibody levels C. Obtain a thyroid ultrasound D. Initiate propylthiouracil E. Initiate methimazole

A. Thyroid binding globulin (TBG) is increased due to increased circulating estrogens with a concomitant increase in the total thyroxine. Free thyroxine (T4) remains relatively constant. Total triiodothyroxine (T3) levels also increase in pregnancy while free T3 levels do not change. In a pregnant patient without iodine deficiency, the thyroid gland may increase in size up to 10%. This patient's thyroid function is normal for pregnancy, and her symptoms of fatigue can be explained by other physiologic changes in pregnancy, including anemia, difficulty with sleep, and increased metabolic demand.

A 27-year-old G1P0 woman presents to labor and delivery and is found to have a fetal demise at 34 weeks gestation. She did not have access to prenatal care during the pregnancy. Her vital signs are normal and she is not in labor. Her uterus is non-tender and she does not have any vaginal bleeding or ruptured membranes on exam. Which untreated condition is the most likely cause? A. Diabetes B. Anemia C. Hypothyroidism D. Herpes E. Rh-isoimmunization

A. Uncontrolled glucose is associated with adverse fetal outcome. A patient with type 1 diabetes is at risk for many pregnancy complications, including fetal death and fetal macrosomia, although fetal growth restriction may also occur. Diabetics also have increased risk for polyhydramnios, congenital malformations (cardiovascular, neural tube defects, and caudal regression syndrome), preterm birth, and hypertensive complications. The anemia most often seen in pregnancy is mild and would not be the most likely cause. It is unlikely that she has an abruption causing the anemia, but this should be considered. Hypothyroidism is usually associated with menstrual irregularities and infertility, and is a less likely cause. Rh sensitization is unlikely since this is her first pregnancy and she did not have any bleeding or procedures during the pregnancy.

A 26-year-old lesbian has chronic herpetic lesions on her lip. She is concerned about the affect this will have on her partner when she performs cunnilingus on her. Physical examination is normal except for a herpetic lesion on the lip, which has been diagnosed as Herpes type 1 in the past. Acyclovir is prescribed. Which of the following is the most appropriate advice to this patient? A. Use a dental dam when having oral sex B. Provide a prescription for acylovir for her partner to take before and after oral sex C. Apply a thin layer of petrolatum jelly over her lips while having oral sex D. Inform her that the Herpes virus type 1 is not contagious to the genitalia E. Discontinue oral sex

A. Use of a dental dam or a latex condom cut down the middle is effective in avoiding infection. Type 1 virus can cause ulcers on the vulva, as it is contagious. Prophylaxis of the partner with acyclovir is not a recommended strategy to prevent transmission, and petrolatum jelly is not an effective barrier.

A 30-year-old G2P1 woman has an ultrasound at 42 weeks for size greater than dates. The fetus had an isolated enlarged head measurement with a BPD of 11 cm, but otherwise appeared to have normal femur length and abdominal circumference. Polyhydramnios is noted. The estimated fetal weight is 3900 g. There is a 10 cm lower uterine segment fibroid protruding into the uterine cavity. The fetus is in the vertex presentation and the fetal head is above the level of the uterine fibroid. Which of the following is an indication for primary Cesarean section in this patient? A. Uterine fibroid B. Fetal hydrocephalus C. Polyhydramnios D. Macrosomia E. 42 weeks gestation

A. Uterine fibroids located in the lower uterine segment may obstruct labor by preventing the fetal head from entering the pelvis. A fetal head with measurements greater than 12 cm could benefit from delivery by Cesarean section. The fetus in the case presented does not necessarily have hydrocephalus. The fetus does not have macrosomia which may be defined as an estimated fetal weight greater than 4000 grams in a diabetic and greater than 4500 grams in a non-diabetic patient. Macrosomia defined as greater than 4000 grams, 42 weeks gestation, and polyhydramnios are not indications for primary Cesarean section.

A 19-year-old G1P0 woman at 39 weeks gestation presents in labor. She denies ruptured membranes. Her prenatal course was uncomplicated and ultrasound at 18 weeks revealed no fetal abnormalities. Her vital signs are: blood pressure 120/70; pulse 72; temperature 101.0° F (38.3° C); fundal height 36 cm; and estimated fetal weight of 2900 gm. Cervix is dilated to 4 cm, 100% effaced and at +1 station. She receives 10 mg of morphine intramuscularly for pain and soon after has spontaneous rupture of the membranes. Light meconium-stained fluid was noted and, five minutes later, the fetal heart rate tracing revealed variable decelerations with moderate variability. What is the most likely cause for the variable decelerations? A. Umbilical cord compression B. Meconium C. Maternal fever D. Uteroplacental insufficiency E. Morphine administration

A. Variable decelerations are reflex mediated usually associated with umbilical cord compression as a result of cord wrapped around fetal parts, fetal anomalies or oligohydramnios. The presence of light meconium-stained fluid is not associated with a specific fetal heart rate tracing. Uteroplacental insufficiency is associated with late decelerations. Maternal drugs may cause loss of variability.

A 19-year-old G1P0 woman at 39 weeks gestation presents in labor. She denies ruptured membranes. Her prenatal course was uncomplicated and ultrasound at 18 weeks revealed no fetal abnormalities. Her vital signs are: blood pressure 120/70; pulse 72; temperature 101.0° F (38.3° C); fundal height 36 cm; and estimated fetal weight of 2900 gm. Cervix is dilated to 4 cm, 100% effaced and at +1 station. She receives 10 mg of morphine intramuscularly for pain and soon after has spontaneous rupture of the membranes. Light meconium-stained fluid was noted and, five minutes later, the fetal heart rate tracing revealed variable decelerations with good variability. What is the most likely cause for the variable decelerations? A. Umbilical cord compression B. Meconium C. Maternal fever D. Uteroplacental insufficiency E. Umbilical cord prolapse

A. Variable decelerations are typically caused by cord compression and are the most common decelerations seen in labor. Placental insufficiency is usually associated with late decelerations. Head compression typically causes early decelerations. Oligohydramnios can increase a patient's risk of having umbilical cord compression; however, it does not directly cause variable decelerations. Umbilical cord prolapse occurs in 0.2 to 0.6% of births. Sustained fetal bradycardia is usually observed.

A 33-year-old G2P1 woman at 39 weeks gestation presents with painful contractions. Her membranes ruptured two hours prior to presentation. Her pregnancy has been uncomplicated and she has a history of a Cesarean section for breech presentation. She highly desires a vaginal birth. On admission, she is having contractions every four minutes and fetal heart tracing is Category I. On cervical exam, she is 5 cm dilated, 80% effaced, and the fetal vertex is at -1 station. Four hours later, she continues to contract every four minutes with reassuring fetal status and her cervical exam is unchanged. What is the next best step in the management of this patient? A. Place an intrauterine pressure catheter B. Administer vaginal prostaglandin E1 C. Administer vaginal prostaglandin E2 D. Perform a vacuum assisted vaginal delivery E. Perform a Cesarean section

A. While the patient is contracting every four minutes, it is not clear if her contractions are adequate. An intrauterine pressure catheter (IUPC) will help determine if her contractions are adequate and if oxytocin augmentation is appropriate. Prostaglandins are used for cervical ripening and are contraindicated in patients with history of previous Cesarean section. While a vacuum assisted delivery is not contraindicated in a patient with prior history of C-section, it should not be performed in a patient who is not completely dilated. A C-section is not indicated yet because it is unclear if the patient is having adequate strength contractions.

A 32-year-old G2P2 woman has just had a spontaneous vaginal delivery. She is concerned that no breast milk is yet being produced when she tries to feed her baby. You reassure her that colostrum is rich in protein and nutrients, and that her breast milk will come in 2-3 days when which of the following hormones have been cleared? A. Estrogen and progesterone B. Estrogen and oxytocin C. Human placental lactogen and prolactin D. Progesterone and prolactin E. Growth hormone and GnRH

A. With delivery, there is a rapid and profound decrease in the levels of progesterone and estrogen, which removes the inhibitory influence of progesterone on the production of alpha-lactalbumin by the rough endoplasmic reticulum. The increased alpha-lactalbumin serves to stimulate lactose synthase and ultimately to increase milk lactose. Progesterone withdrawal allows prolactin to act unopposed in its stimulation of alpha-lactalbumin production. This may take up to two days.

A 30-year-old G1P0 woman with type 1 diabetes mellitus presents at 10 weeks gestation for a routine visit. She smokes a half a pack of cigarettes per day. Her hemoglobin A1C level is 9.7. What structural anomaly is the fetus at highest risk of developing? A. Cardiac anomalies B. Caudal regression malformation C. Hydrocephalus D. Microcephaly E. Limb reductions

A. Women with poorly controlled diabetes immediately prior to conception and during organogenesis have a four- to eight-fold risk of having a fetus with a structural anomaly. The majority of lesions involve the central nervous system (neural tube defects) and the cardiovascular system. Genitourinary and limb defects have also been reported. Although caudal regression malformation occurs at an increased incidence in individuals with diabetes, this condition is very rare.

A 25-year-old G6P2 woman in active labor is treated with mepiridine (Demerol). The patient reports the use of marijuana to control nausea during her pregnancy. She quickly progresses from 4 cm to fully dilated in 1 hour and is now pushing. A limp unresponsive infant is delivered. Heart rate is greater than 90 beats/minute. The infant has no respiratory effort. Which of the following is the most appropriate next step in the management of the neonate? A. Give positive pressure ventilation and prepare to intubate B. Give positive pressure ventilation and prepare to give naloxone C. Give stimulation only and continue to monitor heart rate D. Suction thoroughly and check heart rate E. Suction thoroughly and give naloxone

A. You should give positive pressure ventilation and prepare to intubate the infant, if necessary. Any history of substance abuse may be a relative contraindication to the use of naloxone (Narcan) because the mother may have used narcotics during the pregnancy and administration of naloxone to the infant can cause life-threatening withdrawal. Stimulation may not be sufficient for this infant. Suction will not necessarily stimulate a respiratory effort.

A 26-year-old G1P0 woman with last menstrual period 13 weeks ago presents to your office for her first prenatal visit. She reports vaginal spotting for the last two days. You perform an ultrasound that shows an intrauterine pregnancy consistent with 11 weeks gestation with no cardiac activity. She denies cramping or abdominal pain. What is the most important laboratory test to check for this patient? A. Quantitative Beta-hCG B. Maternal blood type C. Hemoglobin and hematocrit D. Platelet count E. Progesterone

B. A maternal blood type should be checked on all women with vaginal bleeding during pregnancy, unless it was documented earlier in the pregnancy. If the patient's blood type is Rh-negative, RhoGAM would be indicated to prevent Rh sensitization. Serial quantitative Beta-hCGs can be useful in confirming an ongoing pregnancy before fetal heart rate activity can be noted, but in this case there is a non-viable intrauterine pregnancy. The patient is unlikely to have had significant blood loss making a blood or platelet count unlikely to be necessary at this time. Although progesterone levels can be useful in determining if a pregnancy is failing, the diagnosis is already clear in this case.

A 24-year-old G1P1 woman just delivered a healthy infant at term. She has a history of a psychiatric disorder and was treated for depression while in college. Which of the following is she at most increased risk for in the postpartum period? A. Postpartum blues B. Postpartum depression C. Postpartum psychosis D. Postpartum anxiety E. She is not at increased risk for a psychiatric disorder

B. A patient's history of a psychiatric illness is a risk factor for the development of a postpartum depression. Patients with a prior history of depression, either situational or spontaneous, are at very high risk for postpartum depression. In fact, one-third of patients with a postpartum psychiatric problem report a prior history. These patients need careful follow up after delivery, which should include an early appointment for a postpartum visit. Questions at this time should be directed to her moods and thoughts.

A 48-year-old G0 comes to the office for a health maintenance examination. She is healthy and not taking any medications. She has no history of abnormal Pap tests or sexually transmitted infections. She is not currently sexually active. Her menstrual cycles are normal and her last cycle was three weeks ago. She smokes one pack of cigarettes per day. Her mother was diagnosed with endometriosis and had a hysterectomy and removal of the ovaries at age 38. She is 5 feet 4 inches tall and weighs 130 pounds. On pelvic examination, the patient has a palpable left adnexal mass. An ultrasound was obtained, which showed a 4 cm complex left ovarian cyst and a 2 cm simple cyst on the right ovary. What is the most appropriate next step in the management of this patient? A. Oral contraceptives B. Repeat ultrasound in two months C. CT scan of the abdomen and pelvis D. Needle aspiration of the cyst E. Abdominal hysterectomy and bilateral salpingo-oophorectomy (TAH/BSO)

B. A repeat ultrasound is the most appropriate next step, as this is most likely a hemorrhagic cyst which will resolve on its own. Oral contraceptives are contraindicated in this patient, as she is older than 35 and smokes. A CT scan of the pelvis will not add any more information. Needle aspiration is not the standard of care in this asymptomatic premenopausal patient. There is no indication to proceed with a TAH/BSO.

A 32-year-old G3P1 woman at 37 weeks gestation is admitted to labor and delivery for a scheduled repeat Cesarean delivery. Maternal labs show: HIV positive; blood type B+; RPR non-reactive; HBsAg negative; GBS negative; PPD positive; CXR negative. She received adequate antiretroviral therapy prior to and during the pregnancy. Her viral load was undetectable throughout the second and third trimester. A live male infant is delivered with Apgar scores of 9 and 9 at 1 and 5 minutes, respectively. Which of the following is the most appropriate next step in the management of the newborn? A. Order HIV testing on the infant immediately on admission to the nursery B. Treat the infant with zidovudine (AZT) immediately after delivery C. Encourage breastfeeding D. Start zidovudine at 24 hours of life E. Isolate the infant from the other infants in the nursery

B. A usual protocol is to start AZT immediately after delivery. HIV testing begins at 24 hours. There is no reason to isolate the infant even though the mother is PPD positive, because her CXR is negative. Breastfeeding would not be encouraged in a mother with HIV.

A 32-year-old G2P1 woman at 36 weeks gestation presents with preterm premature rupture of the membranes that occurred 36 hours ago. She denies labor. She takes prenatal vitamins and iron. She denies substance abuse, smoking or alcohol use. Her prior pregnancy delivered vaginally at 34 weeks after spontaneous rupture of membranes. Her blood pressure is 110/70; pulse 84; temperature 98.6°F (37.0°C). The estimated fetal weight is 2700 grams. She is having one contraction per hour and fetal heart tracing is Category I. Which of the following is the most appropriate next step in the management of this patient? A. Observation until spontaneous onset of labor B. Augmentation of labor C. Magnesium sulfate D. Nifedipine E. Corticosteroids

B. In this patient, the benefits for delivery outweigh the risk of expectant management, so the patient should undergo augmentation of labor. Expectant management at 36 weeks poses a large risk to the development of chorioamnionitis. The role of tocolytics in the setting of preterm premature rupture of membranes is controversial and is contraindicated at 36 weeks gestation. Steroid administration after 32 weeks is controversial.

A 40-year-old G2P2 woman presents for her first health maintenance examination. She denies any new complaints or symptoms. She has no history of any gynecologic problems. Family history is significant for a father with hypertension and a mother, deceased, with breast cancer diagnosed at age 56. A paternal aunt has ovarian cancer which was diagnosed at age 83. A physical exam is unremarkable. What screening test should be offered to this patient next? A. Breast MRI B. Mammogram C. Transvaginal pelvic ultrasound D. Breast ultrasound E. BRCA-1/BRCA-2 testing

B. ACOG recommends that women aged 40 years and older be offered screening mammography annually. Ultrasonography is an established adjunct to mammography. It is useful in evaluating inconclusive mammographic findings, in evaluating young patients and other women with dense breast tissue, and in differentiating a cyst from a solid mass. Breast ultrasound is not recommended as a primary screening modality for women at average risk of developing breast cancer. A combination of first and second-degree relatives on the same side of the family diagnosed with breast and ovarian cancer (one cancer type per person) increases the risk of BRCA mutation. Based on the limited history provided, this patient does not meet the criteria published by ACOG for genetic cancer risk assessment. A more detailed family history regarding risk factors should be obtained to determine whether the patient should be referred for genetic counseling.

A 25-year-old G0 woman presents to the clinic for follow-up after having a first trimester spontaneous abortion. She wants to discuss the cause of this event. Which of the following etiologic categories accounts for the majority of first trimester spontaneous abortions? A. Immunologic abnormalities B. Conceptus genetic anomalies C. Maternal genetic anomalies D. Structural/uterine anomalies E. Uterine infections

B. Although investigators have implicated all of the listed categories as possible causes of spontaneous abortion, genetic abnormalities involving the conceptus account for the majority. In fact, approximately 50 to 60 percent of embryos and early fetuses that are spontaneously aborted contain form of chromosomal abnormalities.

A 29-year-old G1P1 woman had an uncomplicated vaginal delivery and breastfed immediately postpartum. She has a significant amount of abdominal soreness secondary to a tubal ligation performed on postpartum day two. She is breastfeeding on her side with the baby lying on her side, well away from the abdomen to prevent pain at the incision site. She developed bleeding and cracked nipples. Which of the following is the most likely cause? A. Feedings not frequent enough B. Poor positioning of infant C. Feedings too frequent D. Not enough milk production E. Irritation from the bra

B. Although the side lying position is a good one for breastfeeding, it is important for mother and baby to be belly-to-belly in order for the infant to be in a good position to latch on appropriately, taking a large part of the areola into its mouth. The pain experienced by the patient from her tubal may be interfering with appropriate position and she should be counseled about a different, more comfortable position.

A 24-year-old G4P0 woman presents to your office at seven weeks gestation after two days of bleeding and cramping. She thinks that she miscarried at home and brought in the tissue for pathologic evaluation. What is the karyotype most likely to be found on chromosomal analysis? A. Turner Syndrome (45, X) B. Autosomal Trisomy C. Monoploidy D. Triploidy E. Tetraploidy

B. Autosomal trisomy is the most common abnormal karyotype encountered in spontaneous abortuses, accounting for approximately 40-50% of cases. The most common chromosomal aneuploidy noted in abortuses is Trisomy 16. Triploidy accounts for approximately 15%, and tetraploidy for 5% of cases. Monosomy X (45X, 0) is seen in 15-25% of losses.

A 36-year-old woman requests prenatal diagnosis. She is healthy and excited about finally getting pregnant. She is interested in genetic counseling and asks about the advantages of chorionic villus sampling versus amniocentesis. Which of the following is true when chorionic villus sampling is compared to amniocentesis? A. Reduced post-procedure loss rate B. Performed earlier C. More likely to obtain an adequate sample D. Lower rate of procedure related birth defects E. Less risk of alloimmunization

B. Chorionic villus sampling (CVS) is a prenatal test that can detect genetic and chromosomal abnormalities of a fetus. The loss rate with amniocentesis is quoted as 0.5% vs. ~1 to 3% for chorionic villus sampling. CVS is performed between 10 and 12 weeks gestation, while amniocentesis is performed after 15 weeks. Early CVS (<10 weeks gestation) is associated with an increase in rare limb abnormalities. It is more likely that a CVS will involve multiple attempts - a failure to obtain an adequate sample of cells and the woman requiring a repeat test later on - when compared with amniocentesis. Pregnancies complicated by isoimmunization can be followed by serial assessment of the amniotic fluid for bilirubin.

A 4-year-old female is being evaluated for premature hair growth in the pubic area. She has no breast development and has not had any menstrual bleeding. Laboratory evaluation revealed high DHEA and DHEAS levels and low levels of LH and FSH. Which of the following is the most likely cause of this patient's premature adrenarche? A. Idiopathic isosexual precocious puberty B. Congenital adrenal hyperplasia C. Hypothalamic dysfunction D. Pituitary adenoma E. Polycystic ovarian syndrome

B. Congenital adrenal hyperplasia of the 21-hydroxylase type results in the adrenal being unable to produce adequate cortisol as a result of a partial block in the conversion of 17-hydroxyprogesterone to desoxycorticosterone, with the accumulation of adrenal androgens. This leads to precocious adrenarche. Treatment includes steroid replacement. Idiopathic isosexual precocious puberty is GnRH dependent and leads to an appropriate (although early) order of pubertal events. Some girls with premature adrenarche develop polycystic ovarian syndrome in adolescence, but not at this age.

A 34-year-old G1P0 woman is diagnosed with a fetal demise at 37 weeks. She and her husband seem stunned. They cannot believe the news. What is the next psychological response you would expect from this couple after the initial denial? A. Depression B. Anger C. Assessment D. Bargaining E. Acceptance

B. Couples who are presented with the news of a fetal birth defect or loss progress through a series of coping responses. The response to "bad news" varies with the severity, treatability and the coping level of the couple. As an individual starts to understand the situation, frustration or anger may be self-directed or directed to the spouse, the affected child or the caregiver without a rational basis. This is important to recognize to help the couple through these stages: Denial, Anger, Bargaining, Depression, Acceptance.

Thirty-six hours ago a 23-year-old G1P1 woman delivered vaginally and sustained a 2nd-degree laceration. She had a prolonged first stage of labor, ruptured membranes for 26 hours and received penicillin for group B Strep prophylaxis. She now complains of increasing abdominal pain, cramping and heavy, foul-smelling lochia. Her vital signs reveal a temperature of 100.0° F (37.8° C); pulse 80; blood pressure 120/60; and respirations 18. She has a tender uterine fundus that measures at the umbilicus. Her extremities reveal mild bilateral edema; no erythema or tenderness. Blood work reveals a white count of 12.2; hematocrit of 34%; and normal chemistries. Her urinalysis is positive for blood and negative for WBCs, leukocyte esterase and nitrites. In addition to ampicillin, which of the following would be the best antibiotic choice? A. Erythromycin B. Gentamicin C. Doxycycline D. Vancomycin E. Ciprofloxacin

B. Endomyometritis is a common complication of prolonged labor, prolonged rupture of membranes and multiple vaginal examinations. The infection is polymicrobial, mostly anaerobic and requires broad-spectrum antibiotics for treatment until the patient is afebrile for 24 hours. By adding Gentamicin, you are covering the spectrum of gram-negative organisms. Erythromycin provides good coverage for upper respiratory infections. Vancomycin provides good coverage for S. aureus and penicillin-resistant gram-positive bacteria. Ciprofloxacin provides excellent coverage for gram-negative pathogens, including Pseudomonas.

A 49-year-old G2P2 woman status post hysterectomy at age 45 for fibroids presents to your office complaining of severe vasomotor symptoms for three months. Hot flashes are affecting her quality of life and she would like to discuss options for treatment. What treatment option for hot flashes associated with menopause do you recommend as the most effective? A. Lifestyle modifications such as dressing in layers B. Estrogen C. Selective estrogen receptor modulator (SERMs) D. Selective serotonin reuptake inhibitors (SSRIs) E. Treatment with phytoestrogen (soy)

B. Except for estrogen receptor modulator therapy, all of the above treatment options will improve hot flash symptoms. Treatment with estrogen is most effective, and the current recommendation is for the lowest dose for the shortest duration of time. Hot flashes will resolve completely in 90% of patients receiving this therapy. Raloxifene, a selective estrogen receptor modulator, may actually cause hot flashes to worsen in a patient who has not stopped having these symptoms completely. SSRI antidepressants, some anti-seizure medications and alternative treatments, such as soy products and herbs, have not been shown to be as effective as estrogen.

A 37-year-old G3P0 woman at 29 weeks gestation presents with uterine contractions every five minutes. Her cervix is 1 cm dilated and 50% effaced. Fetal fibronectin test is negative. The patient stops having contractions after bedrest and hydration. What is the strength of using a fetal fibronectin test in patients with preterm contractions? A. Positive predictive value B. Negative predictive value C. High sensitivity D. Low false positive rate E. High false positive rate

B. Fibronectin is an extracellular matrix protein that is thought to act as an adhesive between the fetal membranes and underlying decidua. It is normally found in cervical secretions in the first half of pregnancy. Its presence in the cervical mucus between 22 and 34 weeks is thought to indicate a disruption or injury to the maternal-fetal interface. Fetal fibronectin is FDA approved for use in women with symptoms of preterm labor from 24 to 35 weeks and during routine screening of asymptomatic patients from 22 to 30 weeks gestation. Fetal fibronectin has a negative predictive value of 99.2% in symptomatic women — 99 out of every 100 patients with a single negative test result will not deliver in the next 14 days. The positive predictive value in symptomatic women is 16.7% — 17 out of 100 women with a positive test will deliver within 14 days. In asymptomatic women, a negative fetal fibronectin test has a negative predictive value of 96.7% for delivery before 35 weeks.

A 25-year-old G1P1 woman comes in for her annual health maintenance examination. She has intermittent left lower quadrant discomfort. She has regular menses every 30 days and uses a diaphragm for birth control. Her last menstrual period was approximately three weeks ago. Her physical examination is notable for a 3 x 5 cm left adnexal mass. Ultrasound shows a unilocular simple cyst. Which of the following is the most likely diagnosis in this patient? A. Endometrioma B. Functional ovarian cyst C. Mucinous cystadenoma D. Serous cystadenoma E. Dermoid

B. Functional ovarian cysts are a result of normal ovulation. They may present as an asymptomatic adnexal mass or become symptomatic. Ultrasound characteristics include a unilocular simple cyst without evidence of blood, soft tissue elements or excrescences. An endometrioma is an isolated collection of endometriosis involving an ovary. This would not classically appear as a simple cyst on ultrasound. Serous cystadenomas are generally larger than functional cysts and patients may present with increasing abdominal girth. Mucinous cystadenomas tend to be multilocular and quite large. Dermoid tumors usually have solid components or appear echogenic on ultrasound, as they may contain teeth, cartilage, bone, fat and hair.

A 72-year-old woman presents to the office reporting a history of vulvar itching that has been worsening over the last six months. She has a long history of lichen sclerosus, for which she has not been receiving treatment. On exam, you find an irregular-shaped lesion which measures 3.5 cm in greatest dimension, suspicious for malignancy. You perform a punch biopsy at the edge of the lesion and send it for pathologic evaluation. The pathologist reports an invasive moderately differentiated squamous cell carcinoma. Which of the following is the most appropriate treatment for this patient? A. Treatment with a topical steroid B. Radical vulvectomy and groin node dissection C. Excisional biopsy D. Laser vaporization of the lesion E. Cryotherapy

B. Given the findings of obvious, moderately differentiated carcinoma, definitive treatment can be recommended with radical vulvectomy and groin node dissection. Only microinvasive squamous cell carcinoma of the vulva can be treated by wide local excision, but it is a diagnosis that is only made after pathology evaluation of a small (<2 cm), well-differentiated lesion, with invasion <1.0 mm. Excisional biopsy is not indicated given the larger lesion and confirmed finding of cancer. It would be inappropriate to laser a malignant lesion. Squamous cell carcinoma is the most common vulvar malignancy and may arise in the setting of chronic irritation from lichen sclerosus. Steroids would treat the lichen sclerosus, but would only result in needless delay in treatment of the malignancy. Cryotherapy is not an acceptable treatment for squamous cancer of the vulva.

A 25-year-old G1P0 woman presents to labor and delivery with contractions. She is at 40 weeks gestation. Her cervix is 6 cm dilated and 100% effaced. The fetus is in the occiput anterior presentation at +1 station. Fetal heart tones are reassuring with a baseline in the 140s, multiple accelerations and no decelerations. The patient had a fetal ultrasound three days ago which reported an EFW of 2900 grams. The patient's older sister had a forceps assisted vaginal delivery and has anal incontinence. The patient would like to avoid having this same complication. Which of the following management plans is most appropriate for this patient? A. Cesarean delivery B. Vaginal delivery with no episiotomy C. Vaginal delivery with a small, controlled midline episiotomy D. Forceps assisted delivery with no episiotomy E. Vacuum assisted delivery with no episiotomy

B. Historically, the purpose of performing an episiotomy was to facilitate completion of the second stage of labor to improve both maternal and neonatal outcomes. Maternal benefits were thought to include a reduced risk of perineal trauma, subsequent pelvic floor dysfunction and prolapse, urinary incontinence, fecal incontinence, and sexual dysfunction. Current data does not demonstrate these theoretical maternal and fetal benefits and there are insufficient objective evidence-based criteria to recommend episiotomy, and especially routine use of episiotomy. The risk of incontinence increases with increasing degrees of pelvic trauma. One study of extended episiotomies demonstrated that the occurrence of a fourth-degree extension was more highly associated with anal incontinence. Performance of a median episiotomy is the single greatest risk factor for third- or fourth-degree lacerations. Avoiding the use of episiotomies may be the best way to minimize the risk of subsequent extensive damage to the perineum. This patient is in active labor and has a high chance of having a vaginal delivery. A cesarean delivery is not indicated. There is no indication to perform a forceps or vacuum assisted vaginal delivery in this patient at this time.

A 17-year-old G1P1 female delivered a term infant two days ago. She is not interested in breastfeeding and she asks for something to suppress lactation. Which of the following is the safest method of lactation suppression in this patient? A. Bromocriptine B. Breast binding, ice packs and analgesics C. Medroxyprogesterone acetate D. Oral contraceptives E. Manual milk expression

B. Hormonal interventions for preventing lactation appear to predispose to thromboembolic events, as well as a significant risk of rebound engorgement. Bromocriptine, in particular, is associated with hypertension, stroke and seizures. The safest method to suppress lactation is breast binding, ice packs and analgesics. The patient should avoid breast stimulation or other means of milk expression, so that the natural inhibition of prolactin secretion will result in breast involution.

A 25-year-old G2P1 woman at 38 weeks gestation presents to labor and delivery with spontaneous onset of labor and spontaneous rupture of membranes. Her cervical exam was 5 cm dilated, 90 percent effaced and 0 station at presentation two hours ago. Presently, the patient is uncomfortable and notes strong contractions. The cervical examination is unchanged from admission. You decide to place an intrauterine pressure catheter (IUPC). On placement, approximately 300 cc of frank blood and amniotic fluid flow out of the vagina. What is the most appropriate next step in the management of this patient? A. Emergent Cesarean delivery B. Withdraw the IUPC, monitor fetus and then replace if tracing reassuring C. Begin amnioinfusion D. Begin Pitocin augmentation E. Keep IUPC in position and connect to tocometer

B. If an intrauterine pressure catheter is placed, and a significant amount of vaginal bleeding is noted, the possibility of placenta separation or uterine perforation should be considered. In this case, withdrawing the catheter, monitoring the fetus and observing for any signs of fetal compromise would be the most appropriate management. If the fetal status is found to be reassuring, then another attempt at placing the catheter may be undertaken.

A 16-year-old G1P0 woman at 39 weeks gestation presents to labor and delivery reporting a gush of blood-tinged fluid approximately five hours ago and the onset of uterine contractions shortly thereafter. She reports contractions have become stronger and closer together over the past hour. The fetal heart rate is 140 to 150 with accelerations and no decelerations. Uterine contractions are recorded every 2-3 minutes. A pelvic exam reveals that the cervix is 4 cm dilated and 100 percent effaced. Fetal station is 0. After walking around for 30 minutes the patient is put back in bed after complaining of further discomfort. She requests an epidural. However, obtaining the fetal heart rate externally has become difficult because the patient cannot lie still. What is the most appropriate next step in the management of this patient? A. Place the epidural B. Apply a fetal scalp electrode C. Perform a fetal ultrasound to assess the fetal heart rate D. Place an intrauterine pressure catheter (IUPC) E. Recommend a Cesarean delivery

B. If the fetal heart rate cannot be confirmed using external methods, then the most reliable way to document fetal well-being is to apply a fetal scalp electrode. Putting in an epidural without confirming fetal status might be dangerous. Although ultrasound will provide information regarding the fetal heart rate, it is not practical to use this to monitor the fetus continuously while the epidural is placed. An intrauterine pressure catheter will provide information about the strength and frequency of the patient's contractions, but will not provide information regarding the fetal status. Closer fetal monitoring via a fetal scalp electrode should be performed.

A 44-year-old G0 woman returns to the office for a post operative check following tumor debulking for stage IIIB endometrioid adenocarcinoma of the ovary. Her medical history is significant for diabetes, hypertension, obesity, hypercholesterolemia and major depression. Which of the following is the most appropriate next step in the management of this patient? A. Hospice B. Chemotherapy C. Surveillance D. Pelvic radiation E. Second look laparotomy

B. In all patients with advanced ovarian cancer, post-operative chemotherapy with a combination of a taxane and platinum adjunct is considered standard of care in the United States. Women who undergo surgical cytoreduction, followed by chemotherapy, have a better overall survival rate than those who undergo surgery alone. The overall response rate in women with advanced ovarian cancer following surgery and 4-6 cycles of combination chemotherapy with a taxane and platinum adjunct is 60-80%. The overall five-year survival for women with stage III and IV disease is approximately 30%. Second look laparotomy is no longer considered standard of care.

A 33-year-old G2P1 woman returns at 16 weeks gestation for a follow-up prenatal visit. Her first pregnancy was uncomplicated and delivered at term. Fundal height is 22 cm. Maternal serum alpha-fetoprotein is 3.0 MoMs (multiples of the median). She has not felt fetal movement. What is the next best step in the management of this pregnancy? A. Repeat the maternal serum AFP B. Fetal sonogram C. Biophysical profile D. Fetal Doppler studies E. Amniocentesis

B. In pregnancies with size greater than dates and an elevated maternal serum AFP, you should consider multiple gestation as the etiology. Repeating the maternal serum AFP would only delay further workup. A biophysical profile or fetal Doppler studies are not indicated or performed at this gestational age. Amniocentesis is invasive and would not be utilized prior to performing a fetal survey by ultrasound.

A 27-year-old G1P0 woman at 34 weeks gestation is brought in by ambulance after a motor vehicle accident. Although restrained in the car with a safety belt, she suffers a significant head laceration. When she arrives in the emergency department, her initial trauma survey is completed. On her secondary survey, there is bright red blood coming from the vagina. Her abdomen is noted to be tense. Subsequent documentation of the fetal heart tones reveals fetal tachycardia. Abruption is suspected and the patient is rushed to the operating room for an emergent Cesarean section. After delivery, the nurse notes that an informed surgical consent was never signed. Which of the following is true? A. Informed consent is valid if the doctor-patient discussion occurred soon after the patient received intravenous morphine for pain relief B. Informed consent is unnecessary in an emergency situation if a delay in treatment would risk the patient's health/life C. Informed consent is only required for invasive procedures D. Informed consent would not have been valid anyway because the patient sustained a head laceration E. In an emergency situation, informed consent documents can be signed after the procedure is over and the patient is stable

B. Informed consent needs to be obtained for all procedures while patient is fully alert and has not received any narcotics or other medications that may affect her decision-making. The only exception is in true emergency situations that would risk the patient's life. Obtaining informed consent does not necessarily protect the provider from lawsuits and should never be signed after a procedure is already completed.

A 24-year-old G0 woman presents with a one-year history of introital and deep thrust dyspareunia. She has a two-year history of severe dysmenorrhea, despite the use of oral contraceptives. She also reports significant urinary frequency, urgency, and nocturia. A recent urine culture was negative. She underwent a diagnostic laparoscopy six months ago that showed minimal endometriosis with small implants in the posterior cul de sac only, which were ablated with a CO2 laser. What is the most likely diagnosis in this patient? A. Acute cystitis B. Interstitial cystitis C. Acute urethral syndrome D. Acute urethritis E. Salpingitis

B. Interstitial cystitis (IC) is a chronic inflammatory condition of the bladder, which is clinically characterized by recurrent irritative voiding symptoms of urgency and frequency, in the absence of objective evidence of another disease that could cause the symptoms. Pelvic pain is reported by up to 70% of women with IC and, occasionally, it is the presenting symptom or chief complaint. Women may also experience dyspareunia. The specific etiology is unknown, but IC may have an autoimmune and even hereditary component.

A 16-year-old female comes to the doctor to discuss contraception. She recently became sexually active and states she has never had a menstrual cycle. She regularly attends school and participates in the band. On physical examination, she is 5 feet 3 inches tall and weighs 130 pounds. She has no secondary sexual characteristics with normal appearing external genitalia. The physician suspects Kallmann syndrome. Which of the following diagnostic tests will help confirm the diagnosis? A. MRI of the pituitary B. Olfactory challenge C. Measurement of testosterone levels D. Pelvic ultrasound E. Cortisol levels

B. Kallmann syndrome is characterized by olfactory tract hypoplasia and the arcuate nucleus does not secrete GnRH. Therefore, these females have no sense of smell and do not develop secondary sexual characteristics. The diagnosis is often one of exclusion found during the workup of delayed puberty. The presence of anosmia with delayed puberty should suggest Kallmann syndrome. Treatment is pulsatile GnRH therapy. Testosterone levels would be needed if the patient had symptoms of male hormone (androgen) production such as excess hair growth, male pattern baldness, or clitoris enlargement. Cortisol levels are obtained if you suspect adrenal or pituitary gland problems.

A 19-year-old G0 woman presents with severe menstrual pain that causes her to miss school. She takes 600 mg of ibuprofen every four to six hours to control the pain, but this does not relieve the discomfort. You started oral contraceptives, but her symptoms persisted. She also tried Depo-Provera for three months without much improvement. She still has menstrual pain and continues to miss some classes. What is the most appropriate next step in the management? A. Transdermal narcotic for pain relief B. Diagnostic laparoscopy C. Presacral neurectomy D. Prescribe a selective serotonin reuptake inhibitor E. Prescribe GnRH agonist with estrogen add-back

B. Laparoscopy is recommended to confirm the diagnosis of endometriosis and exclude other causes of secondary dysmenorrhea. SSRIs are not used to treat dysmenorrhea, rather they are a good treatment for PMS. Some authors suggest that a course of GnRH agonists are appropriate, with laparoscopy reserved for those women who have pain during or after completion of a three-month course, but using estrogen add-back would remove the diagnostic sensitivity of the GnRH agonist.

A 64-year-old G2P2 woman presents with a 12-month history of severe vulvar pruritus. She has applied multiple over-the-counter topical therapies without improvement. She has no significant vaginal discharge. She has severe dyspareunia at the introitus and has stopped having intercourse because of the pain. Her past medical history is significant for allergic rhinitis and hypertension. On pelvic examination the external genitalia show loss of the labia minora with resorption of the clitoris (phimosis). The vulvar skin appears thin and pale and involves the perianal area as in the picture below. No ulcerations are present. The vagina is mildly atrophic, but appears uninvolved. Which of the following is the most likely diagnosis in this patient? A. Squamous cell hyperplasia B. Lichen sclerosus C. Lichen planus D. Candidiasis E. Vulvar cancer

B. Lichen sclerosus is a chronic inflammatory skin condition that most commonly affects Caucasian premenarchal girls and postmenopausal women. The exact etiology is unknown, but is most likely multifactorial. Patients typically present with extreme vulvar pruritus and may also present with vulvar burning, pain and introital dyspareunia. Early skin changes include polygonal ivory papules involving the vulva and perianal areas, waxy sheen on the labia minora and clitoris, and hypopigmentation. The vagina is not involved. More advanced skin changes may include fissures and erosions due to a chronic itch-scratch-itch cycle, mucosal edema and surface vascular changes. Ultimately, scarring with loss of normal architecture, such as introital stenosis and resorption of the clitoris (phimosis) and labia minora, may occur. Treatment involves use of high-potency topical steroids. There is less than a 5% risk of developing squamous cell cancer within a field of lichen sclerosus.

A 19-year-old G2P1 woman at 28 weeks gestation has been diagnosed with preterm labor. Her physician has chosen to treat her with magnesium sulfate. By what mechanism of action does magnesium sulfate work as a tocolytic? A. Decreases prostaglandin (PG) production B. Competes with calcium for entry into cells C. Increases cAMP in the cell D. Blocks calcium entry into muscle cells E. Inhibits calcium transport

B. Magnesium sulfate works by competing with calcium entry into cells. Beta-adrenergic agents work by increasing cAMP in the cell, thereby decreasing free calcium. Prostaglandin synthetase inhibitors, such as Indomethacin, work by decreasing prostaglandin (PG) production by blocking conversion of free arachidonic acid to PG. Calcium channel blockers prevent calcium entry into muscle cells by inhibiting calcium transport.

A 32-year old G1P0 woman at 29 weeks gestation presents with preterm labor. She is started on indomethacin. What is a possible adverse fetal effect associated with indomethacin treatment? A. Polyhydramnios B. Premature constriction of the ductus arteriosus C. Fetal growth restriction D. Hypoxia E. Chorioamnionitis

B. Maternal indomethacin exposure can result in premature constriction of the ductus arteriosus, especially if used after 32 weeks gestation. Polyhydramnios is not associated with indomethacin. In fact, indomethacin is associated with oligohydramnios. Fetal hypoxia and decreased uteroplacental blood flow have been associated with the use of calcium channel blockers, such as Nifedipine. Indomethacin should not cause chorioamnionitis.

A 44-year-old G1P1 woman was placed on three months of a GnRH agonist in order to diminish the size of a 5 cm submucosal myoma and allow it to be accessible to a hysteroscopic removal. About two weeks prior to surgery, she was no longer having severe menorrhagia although the drug side effects were becoming incapacitating especially the hot flashes. She decides to cancel the surgery and she stops the GnRH agonist. Which of the following is most likely to happen to the myoma? A. Continues to regress B. Resumes former growth potential C. Grows but to half of its original size D. Grows at a more rapid rate E. Becomes hemorrhagic

B. Maximal response is usually achieved by three months of GnRH agonist treatment. The reduction in size correlates with the estradiol level and with body weight. Hot flashes are experienced by >75% of patients, usually in three to four weeks after start of treatment, although they should not persist for longer than one to two months from end of treatment. After cessation of treatment, menses return in four to ten weeks, and myoma and uterine size return to pretreatment levels in three to four months. The regrowth is consistent with the fact that reduction in size is not due to a cytotoxic effect. However, it is not true that secondary to the GnRH agonist withdrawal they will grow at a more rapid rate.

A 36-year-old G1P0 woman presents in active labor. Her past medical history and prenatal course were complicated by chronic hypertension and superimposed preeclampsia. She received magnesium sulfate for seizure prophylaxis and oxytocin augmentation. She undergoes an uneventful spontaneous vaginal delivery. Postpartum, she has a 1000 ml hemorrhage due to uterine atony. Her blood pressure is 130/80; pulse 96; and she is afebrile. Which of the following uterotonic agents is contraindicated in this patient? A. Oxytocin B. Methylergonovine C. Prostaglandin F2-alpha D. Prostaglandin E2 E. Misoprostol

B. Methergine, prostaglandins, misoprostol, and oxytocin are all uterotonics and used to increase uterine contractions and decrease uterine bleeding. Methylergonovine is an ergot alkaloid, which is a potent smooth muscle constrictor. It is also a vasoconstrictive agent and should be withheld from women with hypertension and/or preeclampsia. Misoprostol, prostaglandin E1, used for cervical ripening and labor induction, is a uterotonic agent frequently used for uterine atony, although not FDA approved for this use.

A 29-year-old G1P0 woman at 31 weeks gestation presents with watery discharge from the vagina commencing several hours ago. Her prenatal course has been uncomplicated and she takes prenatal vitamins and iron. She denies substance abuse, smoking or alcohol use. On examination, her blood pressure is 110/70; pulse 84; temperature 98.6°F (37.0°C). Which of the following is the most appropriate next step in the management of this patient? A. Nitrazine testing of mucus swabbed from cervix B. Examination of vaginal fluid for ferning C. Digital examination of cervix D. Determination of amniotic fluid index (AFI) E. Non-stress test

B. Methods to confirm rupture of membranes include testing the vaginal fluid for ferning and nitrazine testing. It is important to test the fluid from the vagina and not to test cervical mucus because of false positive ferning patterns. A digital exam should be avoided in a patient you suspect might have preterm rupture of membranes because of the risk of introducing bacteria into the uterine cavity and increasing risk for chorioamnionitis. Determination of AFI with ultrasound may reveal oligohydramnios and support the diagnosis of rupture of membranes, but does not confirm this diagnosis. Similarly, a non-stress test may reveal variable decelerations, which may be present in the setting of rupture of membranes.

A 22-year-old G1P0 woman who underwent dilation and curettage for a presumed missed abortion has a pathology report revealing a partial molar pregnancy. Compared to a complete mole, which of the following is true about a partial mole? A. Karyotype 69XXY, fetus present, higher risk of developing post-molar GTD B. Karyotype 69XXY, fetus present, lower risk of developing post-molar GTD C. Karyotype 46XX, fetus present, higher risk of developing post-molar GTD D. Karyotype 46XX, fetus present, lower risk of developing post-molar GTD E. Karyotype 46XX, fetus absent, lower risk of developing post-molar GTD

B. Molar pregnancies are classified as either complete or partial, depending on several histologic, pathologic and genetic characteristics. Partial moles may contain fetus/fetal parts, placenta/cord; complete moles do not. Partial moles are triploid karyotype (usually 69XXY, 69XXX, or 69XYY) resulting from fertilization of egg by dispermy; complete moles are diploid resulting from fertilization of "empty egg" by single sperm (46XX, 90%) or by two sperm (X & Y = 46XY 6-10%). Partial moles show marked villi swelling; complete moles show trophoblastic proliferation with hydropic degeneration. Clinically, partial moles present with lower Beta-hCG levels, affect older patients, have longer gestations, and are often diagnosed as missed or incomplete abortions. Complete moles usually present with larger uteri, preeclampsia and higher likelihood of developing into post-molar GTD.

A 20-year-old G0 college student presents with a one-month history of profuse vaginal discharge and mid-cycle vaginal spotting. She uses oral contraceptives and she thinks her irregular bleeding is due to the pill. She is sexually active and has had a new partner within the past three months. She reports no fevers or lower abdominal pain. She has otherwise been healthy. On pelvic examination, a thick yellow endocervical discharge is noted. Saline microscopy reveals multiple white blood cells, but no clue cells or trichomonads. Potassium hydroxide testing is negative. Vaginal pH is 4.0. No cervical motion tenderness or uterine/adnexal tenderness is present. Testing for gonorrhea and chlamydia is performed, but those results will not be available for several days and the student will be leaving for Europe tomorrow. Which of the following is the most appropriate treatment for this patient? A. Metronidazole and erythromycin B. Ceftriaxone and azithromycin C. Ampicillin and doxycycline D. Azithromycin and doxycycline E. No treatment is necessary until all tests results are known

B. Mucopurulent cervicitis (MPC) is characterized by a mucopurulent exudate visible in the endocervical canal or in an endocervical swab specimen. MPC is typically asymptomatic, but some women have an abnormal discharge or abnormal vaginal bleeding. MPC can be caused by Chlamydia trachomatis or Neisseria gonorrhoeae; however, in most cases neither organism can be isolated. Patients with MPC should be tested for both of these organisms. The results of sensitive tests for C. trachomatis or N. gonorrhoeae (e.g. culture or nucleic acid amplification tests) should determine the need for treatment, unless the likelihood of infection with either organism is high or the patient is unlikely to return for treatment. Antimicrobial therapy should include coverage for both organisms, such as azithromycin or doxycycline for chlamydia and a cephalosporin or quinolone for gonorrhea. Uncomplicated cervicitis, as in this patient, would require only 125 mg of Ceftriaxone in a single dose. Ceftriaxone 250 mg is necessary for the treatment of upper genital tract infection or pelvic inflammatory disease (PID).

A 45-year-old G2P2 woman underwent an abdominal hysterectomy for a large fibroid uterus via a low transverse skin incision. Her postoperative course was significant for new onset right lower quadrant pain and numbness, radiating into the right inguinal area and medial thigh. Her pain was exacerbated by adduction of her right thigh. On abdominal examination, there is a well-healed low transverse incision. Her pain is reproduced with adduction of the right thigh. There is decreased sensation to light touch and pinprick over the right inguinal area and right medial thigh. Patellar reflexes are 2+ and symmetric. Entrapment of which of the following nerves is the most likely cause of her pain? A. Obturator nerve B. Ilioinguinal nerve C. Lateral femoral cutaneous nerve D. Femoral nerve E. Iliohypogastric nerve

B. Nerve entrapment syndrome is a commonly misdiagnosed neuropathy that can complicate pelvic surgical procedures performed through a low transverse incision. The nerves at risk are the iliohypogastric nerve (T-12, L-1) and the ilioinguinal (T-12, L-1) nerve. These two nerves exit the spinal column at the 12th vertebral body and pass laterally through the psoas muscle before piercing the transversus abdominus muscle to the anterior abdominal wall. Once at the anterior superior iliac spine, the iliohypogastric nerve courses medially between the internal and external oblique muscles, becoming cutaneous 1 cm superior to the superficial inguinal ring. The iliohypogastric nerve provides cutaneous sensation to the groin and the skin overlying the pubis. The ilioinguinal nerve follows a similar, although slightly lower, course as the iliohypogastric nerve where it provides cutaneous sensation to the groin, symphysis, labium and upper inner thigh. These nerves may become susceptible to injury when a low transverse incision is extended beyond the lateral border of the rectus abdominus muscle, into the internal oblique muscle. Symptoms are attributed to suture incorporation of the nerve during fascial closure, direct nerve trauma with subsequent neuroma formation, or neural constriction due to normal scarring and healing. Damage to the obturator nerve, which can occur during lymph node dissection would result in the inability of the patient to adduct the thigh.

A 62-year-old G0 postmenopausal woman is being referred to your gynecologic oncology colleague after an office endometrial sample demonstrated a FIGO grade 1 endometrioid adenocarcinoma. The patient has no significant medical, surgical or other gynecologic history. She does not smoke and drinks only occasionally at social events. She takes a multivitamin. Her physical exam is unremarkable. Which of the following additional tests is indicated for this patient? A. Pelvic ultrasound B. Chest x-ray C. Pelvic MRI D. CA-125 E. Serum estrogen level

B. Once a pathologic diagnosis is confirmed by biopsy, a basic clinical assessment should ensue in all patients to help define the extent of the disease. If a careful history and clinical gynecologic exam suggests that the carcinoma is likely of an early stage, minimal pre-treatment evaluation is necessary. Routine evaluation in this setting should include a chest x-ray as the lungs are the most common site of distant spread. A pelvic ultrasound is not indicated once a pathologic diagnosis has been established, although one may have been obtained as part of the initial evaluation of postmenopausal bleeding. When there is a low suspicion for advanced disease, a CT scan, MRI, PET scan, and other invasive and costly tests are not indicated. A CA-125 may be helpful in predicting those patients that may have extrauterine spread, but is not absolutely necessary.

A 32-year-old G2P2 woman is concerned about symptoms associated with her menstrual cycle. During the second half of her cycle, she feels anxious, sad and has difficulty sleeping. She has done research on the Internet and believes she suffers from premenstrual dysphoric disorder (PMDD). Which of the following symptoms of the patient is most consistent with this diagnosis? A. Cyclic constellation of symptoms during the follicular phase B. Cyclic occurrence of a minimum of described symptoms and interference in social functioning C. Chronic, mild depressive symptoms that have been present for many years D. Depressed mood or the loss of interest or pleasure in activities E. Anxiety/nervousness interfering in social functioning

B. PMDD is a psychiatric diagnosis, describing a severe form of premenstrual syndrome in which the diagnostic criteria include five out of 11 clearly defined symptoms, functional impairment and prospective charting of symptoms present during the last week of the luteal phase that begin to resolve with the beginning of the follicular phase. All three areas of symptoms need to be represented for the diagnosis of PMDD.

A 32-year-old G0 woman presents with irregular menses occurring every six to eight weeks for the past eight months. The bleeding alternates between light and heavy. Her irregular menses were treated successfully with medroxyprogesterone acetate (MPA), 10 mg every day, taken for 10 days each month. By which mechanism does the MPA control her periods? A.Stimulates rapid endometrial growth and regeneration of glandular stumps B. Converts endometrium from proliferative to secretory C. Promotes release of Prostaglandin F2α D. Regenerates functional layer of the endometrium E. Decreases luteal phase inhibin production

B. Patients with anovulatory bleeding have predominantly proliferative endometrium from unopposed stimulation by estrogen. Progestins inhibit further endometrial growth, converting the proliferative to secretory endometrium. Withdrawal of the progestin then mimics the effect of the involution of the corpus luteum, creating a normal sloughing of the endometrium. Stimulation of rapid endometrial growth, conversion of proliferative to secretory endometrium, and regeneration of the functional layer describe effects of estrogen on the endometrium. Inhibin is increased in the luteal phase.

A 38-year-old G5P4 woman with a history of four Cesarean deliveries is at 36 weeks gestation with a singleton pregnancy. She presents to labor and delivery with complaints of vaginal bleeding for the last hour. Prenatal care has been unremarkable except for a second trimester ultrasound discovering an anterior placenta, which partially covers the cervical os. Follow up ultrasound exams have confirmed these findings. The patient denies uterine contractions and abdominal pain. She feels the baby moving. Her blood pressure is 110/60; pulse 110; and she is afebrile. Her abdomen and uterus are non-tender and soft. Fetal heart tones have a baseline of 140 and are reassuring. This patient is at greatest risk for which of the following complications? A. Vasa previa B. Placenta accreta C. Placental abruption D. Uterine rupture E. Preterm labor

B. Placenta accreta occurs when the placenta grows into the myometrium. This patient is at risk for this condition due to her history of four previous Cesarean deliveries, and the low anterior placenta. The scar tissue from the previous surgery prevents proper implantation of the placenta and it subsequently grows into the muscle. Vasa previa is a rare condition where the umbilical cord inserts into the membranes. Placental abruption is the premature separation of the normally implanted placenta. Risk of uterine rupture could be as high as 5% in this case, and the risk of placenta accreta with four prior Cesarean deliveries approaches 50%. The patient is not experiencing contractions at the present time, so preterm labor is unlikely.

An 81-year-old G3P3 presents to your office with a history of light vaginal spotting. She states this has occurred recently and in association with a thin yellow discharge. She never experienced any vaginal bleeding since menopause at the age of 52, and denies ever having been on hormone replacement therapy. She is otherwise reasonably healthy, except for osteoporosis, well-controlled hypertension, and diabetes. She is physically active and still drives to all her appointments. She is no longer sexually active since the death of her husband two years ago. On examination, she is noted to have severe atrophic changes affecting her vulva and vagina. A small Pederson speculum allows for visualization of a normal multiparous cervix, and the bimanual examination is notable for a small, mobile uterus. Rectovaginal exam confirms no suspicious adnexal masses or nodularity. Which of the following is the most appropriate management for this patient? A. Pelvic transvaginal ultrasound B. Office endometrial biopsy C. Reassurance and observation for further bleeding D. Vaginal estrogen therapy E. Clindamycin vaginal cream

B. Postmenopausal bleeding or discharge accounts for the presenting symptom in 80-90% of women with endometrial cancer. However, the most common causes of postmenopausal bleeding are atrophy of the endometrium (60-80%), hormone replacement therapy (15-25%), endometrial cancer (10-15%), polyps (2-12%), and hyperplasia (5-10%). Any history of vaginal bleeding requires a thorough history, physical/pelvic examination, and assessment of the endometrium. This is ideally done via office endometrial sampling as part of the initial work-up. The use of pelvic transvaginal ultrasound can provide useful information as to the presence of any structural changes (polyps, myomas, endometrial thickening), and for which a diagnosis of endometrial cancer would be less likely if the endometrial thickness is < 5 mm. Although this patient is likely to have atrophy as the cause of her spotting, a thin endometrial stripe does not exclude the possibility of a non-estrogen dependent carcinoma of the atrophic endometrium. Vaginal estrogen or clindamycin are not indicated.

A 29-year-old G1P0 woman at 41 weeks gestation presents for a prenatal visit. Her prenatal course is complicated by tobacco abuse and intermittent prenatal care. Her last visit was at 35 weeks. Prenatal labs are unremarkable except cervical DNA probe positive for Chlamydia, which was treated, and a Pap smear with low-grade squamous intraepithelial lesion. Ultrasound at 21 weeks was consistent with gestational age based on her certain regular LMP. Her vitals reveal a blood pressure of 128/76; pulse 74; and temperature 98° F (36.7° C). Fundal height is 39 cm with estimated fetal weight of 2700 gm. Cervix is dilated to 1 cm, 50% effaced, -2 station. What is the next best step in the management of this patient? A. Return visit in one week B. Non-stress test and assessment of amniotic fluid volume C. Ultrasound to assess fetal growth D. Oxytocin challenge test E. Cesarean section

B. Postterm pregnancies should be followed with antepartum fetal surveillance because perinatal morbidity and mortality increases beginning at 41 weeks of gestation. Many practitioners use twice-weekly testing with some evaluation of amniotic fluid volume beginning at 41 weeks of gestation. A non-stress test and amniotic fluid volume assessment (a modified BPP) should be adequate. The non-stress test is an assessment of fetal well-being that measures the fetal heart rate response to fetal movement. The normal or reactive non-stress test occurs when there are two fetal heart rate accelerations of 15 beats/minute for 15 seconds within 20 minutes. Contraction stress test assesses uteroplacental insufficiency and looks for persistent late decelerations after contractions (3/10 minutes); however, it is not necessary to perform, as the non-stress test will assess fetal well-being, as well. An ultrasound to assess fetal growth is not indicated as the patient's fundal height is appropriate for her gestational age and she does not have any other indication to assess fetal growth such as a history of chronic hypertension or diabetes. Observation alone would not be proper care as the patient is postterm. Delivery is indicated if there is evidence of fetal compromise or oligohydramnios.

A 22-year-old G1P0 woman at 39-weeks gestation presents in active labor. Her pregnancy is complicated by diet-controlled gestational diabetes. She has a history of uterine fibroids. On examination, she is found to be 4 cm dilated in breech presentation. An ultrasound confirms the breech presentation, amniotic fluid index is 5, and the estimated fetal weight is 3900 g. Which of the following is the most likely cause of the breech presentation in this patient? A. Gestational diabetes B. Uterine fibroids C. Oligohydramnios D. Macrosomia E. Gravidity

B. Prematurity, multiple gestation, genetic disorders, polyhydramnios, hydrocephaly, anencephaly, placenta previa, uterine anomalies and uterine fibroids are all associated with breech presentation.

A 28-year-old Rh negative G1P0 woman at eight weeks gestation presents to the clinic for a first prenatal visit. Which of the following is the current recommendation for RhoGAM administration to prevent Rh isoimmunization? A. Routine administration for every Rh-sensitized woman at term B. Administration for Rh-negative patients with no Rh antibodies at 28 weeks C. Administration for every Rh-negative woman who delivers an Rh-negative infant D. Routine administration for all Rh-negative patients during first trimester E. Routine administration for all Rh-negative patients during each trimester

B. RhoGAM (Anti-D-immunoglobulin) is administered to Rh-negative women to prevent isoimmunization. Each dose provides 300 micrograms of D-antibody and is given to the D-negative non-sensitized mother to prevent sensitization after any pregnancy-related events that could result in fetal-maternal hemorrhage. Up to 2 percent of women with a spontaneous abortion and 5 percent of those undergoing elective termination may become isoimmunized without D-immunoglobulin. The current recommendations for Rh-negative women without evidence of Rh immunization is prophylactically at 28-weeks gestation (after an indirect Coombs' test), and within 72 hours of delivering an Rh-positive baby, following spontaneous or induced abortion, following antepartum hemorrhage and following amniocentesis or chorionic villus sampling. If the father of the fetus is known to be Rh-negative, RhoGAM is not necessary since the fetus will be Rh-negative and not at risk for hemolytic disease.

An 18-year-old G0 woman comes in for a health maintenance examination with her mother. The mother had severe PMS symptoms in her twenties and thirties and would like to know if her daughter would inherit this as well. Which of the following has the strongest association with premenstrual syndrome? A. Obesity B. Positive family history C. History of early menarche D. Insulin dependent diabetes mellitus E. Vitamin K deficiency

B. Risk factors for PMS include a family history of premenstrual syndrome (PMS) and Vitamin B6, calcium, or magnesium deficiency. PMS becomes increasingly common as women age through their thirties, and symptoms sometimes get worse over time. Previous anxiety, depression or other mental health problems are significant risk factors for developing premenstrual dysphoric disorder (PMDD). There is no known association between premenstrual syndrome and obesity or insulin dependent diabetes mellitus.

A 48-year-old G0 woman presents to the office for preoperative counseling. She has severe endometriosis that has failed medical management, and she is planning to undergo a robotic total hysterectomy and salpingo-oophorectomy. She is concerned about developing a "dropped bladder" following her surgery, since both her mother and aunt have undergone surgery for this condition. She reports no urinary incontinence or other urinary or bowel symptoms. She is in good health and exercises with running and weight lifting. Pelvic examination reveals a well-estrogenized vagina, a normal nulliparous cervix, anteverted uterus, and mildly tender adnexa without masses. Which of the following is likely to increase her risk of subsequent development of pelvic organ prolapse? A. Age B. Family History C. Endometriosis D. Exercising E. Hysterectomy

B. Risk factors for the development of pelvic organ prolapse are increasing parity, increasing age, obesity, some connective tissue disorders (Ehlers-Danlos syndrome), and chronic constipation. Vaginal delivery is associated with a higher risk of POP than Cesarean delivery. It is unclear whether occupations that require heavy lifting increase the risk of POP. Women with a family history of POP have up to a 2.5 fold increase in prolapse. Although hysterectomy is associated with an increased risk of apical prolapse, studies show mixed results on the role of hysterectomy in the development of prolapse. The risk of future prolapse may be highest when hysterectomy is performed in women with existing prolapse, while the risk in women with normal pelvic support is less clear.

A 68-year-old G5P5 woman presents for counseling following the diagnosis of cervical cancer. She has not seen a physician in 30 years since the birth of her last child. She has been widowed for three years and has only had sexual activity with her husband. The patient questions how she contracted this cancer as she practices excellent perineal hygiene including the use of talc. Which of the following most likely explains the pathogenesis of cervical cancer in this woman? A. New onset of sexual activity with another partner B. Past exposure to high risk HPV C. De novo cervical cancer development D. Genetic inheritance E. Exposure to talc

B. Sexual transmission of HPV is known to be a necessary event for the pathogenesis of cervical neoplasia (dysplasia and cancer). However, exposure to HPV is not sufficient, as other factors come into play in the ultimate development of cervical disease including smoking and immunologic factors. In this patient, exposure to HPV likely occurred earlier in her life, and because she never underwent screening, persistence of HPV infection ultimately resulted in the development of cervical cancer. This patient reports only one sexual partner, and although the risk of HPV correlates with the lifetime number of sexual partners, the risk is still relatively high even in those with one partner (up to 20%). At least 75 to 80% of sexually active women will have acquired a genital HPV infection by age 50. Cervical cancer is not genetically inherited. The use of talc does not increase risk of cervical cancer. Although nothing can be done to reverse the events that led to the development of this cancer in this individual, patient education in prevention and screening for cervical neoplasia should be practiced among all patients.

A 26-year-old African-American G1P0 woman presents to your office at seven weeks gestation with her husband, who is also African-American. The patient's brother has sickle cell anemia, and has been hospitalized on numerous occasions with painful crises requiring narcotic pain medication and blood transfusions. What are the odds that this couple will have a child with sickle cell anemia, if the carrier rate for sickle cell disease in the African American population is 1/10? A. 1 in 15 B. 1 in 60 C. 1 in 100 D. 1 in 160 E. 1 in 400

B. Sickle cell anemia is an autosomal recessive condition that occurs in 1/500 births in the African-American population. The carrier state, or sickle-cell trait, is found in approximately 1/10 African-Americans. Since the patient's brother is affected, both of their parents have to be carriers. Each time two carrier parents for an autosomal recessive condition conceive there is a 1/4 chance of having either an affected or an unaffected child and a 1/2 chance of having a child who is a carrier. Since the patient is unaffected, she has a 1/3 chance of not being a carrier and a 2/3 chance of being a carrier. The patient's husband has a 1/10 chance of being a carrier (the general population risk for African-Americans). Thus, the chance that this couple will have a child with sickle cell anemia is: 2/3 X 1/10 X 1/4 = 1/60.

A 30-year-old G2P0 woman at 38 weeks gestation has just delivered a male infant. She has a history of type 1 diabetes since age 11. Maternal labs show: blood type B+; RPR non-reactive; HBsAg negative; HIV negative; and GBS negative. She had moderate control of blood sugar during her pregnancy. Which of the following would be the most likely finding in the newborn? A. Large and hypoglycemic B. Small and hypoglycemic C. Large and hyperglycemic D. Small and hyperglycemic E. Normal size and euglycemic

B. Small babies are more common with type 1 diabetes than with gestational diabetes, and the blood sugar level of all newborns of diabetic mothers should be monitored closely after delivery, as they are at increased risk for developing hypoglycemia. Macrosomic (large) infants are typically associated with gestational diabetes.

A 9-year-old female goes to the doctor's office for a regular check-up. She is healthy, active in school sports and gets good grades. On examination she is 4 feet 8 inches tall and weighs 80 pounds. She is concerned about when she might expect to have her first menstrual period since her friends have been talking about it. On physical examinations, she has Tanner stage 1 breasts and no pubic hair. You explain to her that she can expect to experience which of the following first sexual developments? A. Adrenarche B. Thelarche C. Growth spurt D. Menarche E. Pubarche

B. The normal and predictable sequence of sexual maturation proceeds with breast budding, then adrenarche (hair growth), a growth spurt and then menarche. In a minority of cases, pubarche (pubic hair growth) can occur before thelarche (breast/areolar development). Breast development begins around the age of 10 and average age of menarche is 12.7 years for Caucasian girls and 12.1 for Black girls. Menarche also occurs earlier for heavier girls and later for thinner, physically active girls.

An 88-year-old woman is brought in for evaluation of blood found in her diapers. She is a nursing home resident and has a history of lichen sclerosus of the vulva, which was diagnosed fifteen years ago. She quit smoking in her fifties. Examination of the external genitalia reveals an elevated, firm, erythematous, ulcerated lesion arising from the left labia, measuring 2.5 cm in greatest dimension. What is the most likely diagnosis in this patient? A. Malignant melanoma B. Squamous cell carcinoma C. Lichen sclerosus D. Paget's disease E. Verrucous carcinoma

B. Squamous cell carcinoma accounts for approximately 90% of vulvar cancers. Patients commonly present with a lump and they commonly have a long-standing history of pruritus. The chronic itch-scratch cycle of untreated lichen sclerosus, or any other chronic pruritic vulvar disease, is thought to stimulate the development of squamous carcinoma. The mean age of squamous cell carcinoma is 65 years and smoking is known to increase the risk of development of vulvar cancer, especially in the setting of HPV infection. With lichen sclerosus, the skin appears thin, inelastic and white, with a "crinkled tissue paper" appearance. Paget's disease of the vulva is associated with white plaque-like lesions and poorly demarcated erythema, not a discrete mass. Verrucous carcinoma has cauliflower-like lesions. Melanoma typically presents as a pigmented lesion.

A 42-year-old G5P4 woman at eight weeks gestation presents for her first prenatal appointment. She has glycosuria noted on urine dipstick in the office. She has a history of four prior vaginal deliveries at full-term with birth weights ranging from 9 to 10.5 pounds. Family history is positive for type 2 diabetes in her mother and two siblings. Weight is 265 pounds and height is 5 feet 4 inches (BMI is 45.5 kg/m2). Which of the following recommendations concerning weight gain during this pregnancy is most appropriate? A. Maintain current weight B. Gain 11 - 20 pounds C. Gain 15 - 25 pounds D. Gain 25 - 35 pounds E. Gain 28 - 40 pounds

B. The Institute of Medicine (IOM) has developed guidelines (2009) on weight gain in pregnancy. Historical data show that women who gained within the IOM guidelines experienced better outcomes of pregnancy than those who did not. The recommendations are: underweight (BMI < 18.5 kg/m2) total weight gain 28 - 40 pounds; normal weight (BMI 18.5 - 24.9 kg/m2) total weight gain 25 - 35 pounds; overweight (BMI 25 - 29.9 kg/m2) total weight gain 15 - 25 pounds; and obese (BMI > 30 kg/m2) total weight gain 11 - 20 pounds.

A 20-year-old G0 woman presents with severe menstrual pain. She takes 600 mg of ibuprofen every four to six hours to control the pain, but this does not relieve the discomfort. She is sexually active with one partner and has four lifetime partners. She uses condoms for contraception. Past medical history is unremarkable, except for breast cancer in her father's sister. Examination is normal. Which of the following is the most appropriate test for this patient? A. HPV DNA typing B. Chlamydia testing C. Pap smear D. Hysterosalpingogram E. Diagnostic laparoscopy

B. The US Preventive Services Task Force recommends chlamydia and gonorrhea screening for all sexually active patients, age 25 and younger. Since pelvic inflammatory disease is a cause of secondary dysmenorrhea, it needs to be evaluated as a potential cause of her symptoms. Although HPV screening is common, it can be used as an adjunct to cytology in primary screening in women 30 years or older, and is not indicated in a 20 year old. Pap smears are not indicated in women under 21 years of age regardless of sexual history. A hysterosalpingogram is used for infertility work-up and will not necessarily help determine the cause of her pain. A diagnostic laparoscopy would be premature at this point.

A 41-year-old G3P2 woman presents with cramping, vaginal bleeding and right lower quadrant pain for five days which has progressively worsened. Her last normal menstrual period occurred seven weeks ago. Her surgical history is notable for a bilateral tubal ligation following her last delivery. On physical exam, vital signs are: blood pressure 110/74; pulse 82; respirations 18; temperature 98.6°F (37.0°C). On abdominal exam, she has right lower quadrant tenderness, with rebound and bilateral guarding in the lower quadrants. On pelvic exam, she has scant old blood in the vagina and a normal appearing cervix. Her uterus is normal size and slightly tender. She has cervical motion tenderness on bimanual examination, and marked tenderness on rectal examination. Her quantitative Beta-hCG is 4000 mIU/ml; progesterone 6.2 ng/ml; hematocrit 34%; and WBC 15,400/mcL, with 88% segmented neutrophils and no bands. The transvaginal ultrasound shows an empty uterus with endometrial thickening, a mass in right ovary measuring 3.8 x 2 cm, and a small amount of free fluid in the pelvis. What is the most likely diagnosis in this patient? A. Pelvic inflammatory disease B. Ectopic pregnancy C. Heterotopic pregnancy D. Missed abortion E. Ruptured corpus luteum cyst

B. The diagnosis of ectopic pregnancy is made when either: 1) a fetal pole is visualized outside the uterus on ultrasound; 2) the patient has a Beta-hCG level over the discriminatory zone (the level at which an intrauterine pregnancy should be seen on ultrasound, usually 2000 mIU/ml) and there is no intrauterine pregnancy (IUP) seen on ultrasound; or 3) the patient has inappropriately rising Beta-hCG level (less than 50% increase in 48 hours) and has levels which do not fall following diagnostic dilation and curettage. This patient meets criteria #2, as her Beta-hCG is >2000 mIU/ml with no intrauterine pregnancy seen on ultrasound. The history, physical exam and lab data are not consistent with pelvic inflammatory disease, ovarian torsion, appendicitis or a ruptured corpus luteum cyst. With a heterotopic pregnancy, there should be a visible pregnancy in the uterus. With a missed abortion there should also be some visible tissue or a fetal pole within the uterus.

A 35-year-old African-American G0 woman has a family history of ovarian cancer. Her mother was diagnosed with ovarian cancer at age 50 and is in remission. The patient had onset of menarche at age 14. She has used oral contraceptives for a total of 10 years. She smokes one to two packs of cigarettes per week. The patient had a LEEP for treatment of cervical dysplasia. Which of the following places the patient at greatest risk for developing ovarian cancer? A. African American race B. Family history of ovarian cancer C. Gravidity D. Late age at menarche E. Smoking

B. The events leading to the development of ovarian cancer are unknown. Epidemiologic studies, however, have identified endocrine, environmental and genetic factors as important in the carcinogenesis of ovarian cancer. The established risk factors include nulliparity, family history, early menarche and late menopause, white race, increasing age and residence in North America and Northern Europe. Smoking has not been demonstrated to be associated with an increased risk of ovarian cancer.

A 54-year-old woman presents with a breast mass she noticed two months ago. She has no family history of breast cancer. On exam, there is a 2 cm mass palpable in the upper outer quadrant of the left breast. There are no other masses noted and no palpable lymphadenopathy. A fine needle aspiration returns bloody fluid and reduces the size of the mass to 1 cm. In addition to obtaining a mammogram, what is the most appropriate next step in the management of this patient? A. Repeat exam in two months B. Excisional biopsy of the mass C. Obtain a breast MRI D. Perform a lumpectomy and lymph node dissection E. Follow-up in one year if mammogram is normal

B. The first noticeable symptom of breast cancer is typically a lump that feels different from the rest of the breast tissue. More breast cancer cases are discovered when the woman feels a lump. Breast cancer can also present with a spontaneous bloody nipple discharge. Even though the mass decreased in size after aspiration, the bloody discharge obtained obligates an excisional biopsy be performed to rule out breast cancer. If clear discharge is obtained on aspiration and the mass resolves, reexamination in two months is appropriate to check that the cyst has not recurred. An MRI is not the appropriate next step and lumpectomy with lymph node dissection is not yet indicated in this case. A normal mammogram does not rule out breast cancer, especially in the presence of bloody discharge.

A 61-year-old G3P3 woman is diagnosed with stage IIIA papillary serous adenocarcinoma of the ovary. She is concerned about her long-term prognosis. Which of the following factors would be most helpful in determining this patient's prognosis? A. Volume of residual disease B. Tumor stage C. Presence of non-malignant ascites D. Tumor grade E. Ovarian tumor diameter

B. The five-year survival of patients with epithelial ovarian cancer is directly correlated with the tumor stage. The volume of residual disease following cytoreductive surgery is also directly correlated with survival. Patients who have been optimally debulked (generally <2 cm or <1 cm maximal residual tumor diameter) have a significant improvement in median survival. Histologic grade of tumor is important. Women with poorly differentiated tumors or clear-cell carcinomas typically have a worse survival than those with well to moderately differentiated tumors. This is especially important in early-stage disease. Tumor size, bilaterality and ascites without cytologically positive cells, are not considered to be of prognostic importance.

A 39-year-old G1P1 woman comes to see you because of increased bleeding due to her known uterine fibroids, especially during her menses. She reports that her bleeding is so heavy that she has to miss two days of work every month. She has been using oral contraceptives and NSAIDs. Her most recent hematocrit was 27%. She is undecided about having more children. You discuss with her short and long-term options to decrease her bleeding. What is the next best step in the management of this patient? A. Blood transfusion B. Gonadotropin-releasing hormone agonists C. Endometrial ablation D. Uterine artery embolization E. Hysterectomy

B. The goals of medical therapy are to temporarily reduce symptoms and to reduce myoma size. The therapy of choice is treatment with a GnRH agonist. The mean uterine size decreases 30-64% after three to six months of GnRH agonist treatment. Unfortunately, GnRH agonist therapy is recommended for only a short period of time (3-6 months) typically before a surgical procedure, or to bridge a woman who is close to menopause. In this case, it is the best short-term option. Even though she is anemic, she is asymptomatic and able to work so a blood transfusion will not be indicated. Although uterine artery embolization and endometrial ablation effectively reduce bleeding, pain and fibroid size, they are contraindicated in a patient who desires future fertility. The failure rate is about 10-15%. A hysterectomy would obviously take care of her bleeding but would not be performed if she desires future fertility.

A 30-year-old G2P1 woman with last menstrual period 10 weeks ago presents for her first prenatal care visit. She is healthy and takes no medications. Her previous pregnancy was an uncomplicated vaginal delivery at 39 weeks. On examination, her vital signs are normal. Her exam is notable for a uterus measuring 14 weeks gestation. Ultrasound shows a diamniotic monochorionic twin gestation at 10 weeks. Which of the following obstetrical complications is more likely in this pregnancy compared to her previous pregnancy? A. Low maternal weight gain B. Congenital anomalies C. Induction after 40 weeks D. Macrosomia E. Rh isoimmunization

B. The incidence of congenital anomalies is increased in twins, particularly monozygotic twins, compared to singletons. The majority of twin pairs in which an anomaly is present will be discordant for the anomaly. Twin gestations tend to deliver earlier than singleton gestations, with the average length of twin gestation being 35-37 weeks. The optimal length of twin gestation is a matter of some controversy. An observational study comparing perinatal mortality among twin and singleton gestations showed that perinatal mortality reached a nadir at 37-38 weeks in twins and then increased. There have been no prospective studies to demonstrate that induction of labor after 38 weeks in twin gestations improves perinatal outcome. Twins typically weigh less than singletons of the same gestational age, but their weights usually remain within the normal range. Macrosomia is, therefore, uncommon. Rh isoimmunization is not increased in twin gestations.

A 22-year-old G1P1 woman with no prenatal care delivered a 2100 g female infant. The infant had a spontaneous cry upon delivery. The baby's father is concerned because his new daughter's skin appears to be peeling and has a green/yellow hue, and her fingernails are very long. Overall both parents are concerned, as the baby appears to be thin and fragile. Which of the following diagnoses best explains these findings? A. Acute fetal hypoxia B. Fetal dysmaturity C. Trisomy 18 D. Trisomy 21 E. Fragile X syndrome

B. The incidence of infants with dysmaturity approaches 10% when the gestational age exceeds 43 weeks. Infants are described as withered, meconium stained, long-nailed, fragile and have an associated small placenta. These infants are at great risk for stillbirth. The diagnosis of dysmaturity is more common in women with unknown last menstrual periods and unsure dating. While low birth weight is a common finding in infants with Trisomy 18, overlapping fingers, micronathia, and cardiac defects are the most common findings. Trisomy 21 can be associated with low birth weight, but the syndrome is characterized by a constellation of facial findings (low set ears, flattened bridge of the nose, and almond shaped eyes) and nearly 50% are associated with cardiac defects. Acute fetal hypoxia can be associated with meconium stained amniotic fluid but not the other findings noted in this patient. The hallmark of Fragile X syndrome (more common in males than females) is developmental delay not apparent at birth.

A 24-year-old G4P2 woman at 34 weeks gestation complains of a cough and whitish sputum for the last three days. She reports that everyone in the family has been sick. She reports a high fever last night up to 102°F (38.9°C). She denies chest pain. She smokes a half-pack of cigarettes per day. She has a history of asthma with no previous intubations. She uses an albuterol inhaler, although she has not used it this week. Vital signs are: temperature 98.6°F (37°C); respiratory rate 16; pulse 94; blood pressure 114/78; peak expiratory flow rate 430 L/min (baseline documented in the outpatient chart = 425 L/min). On physical examination, pharyngeal mucosa is erythematous and injected. Lungs are clear to auscultation. White blood cell count 8,700; arterial blood gases on room air (normal ranges in parentheses): pH 7.44 (7.36-7.44); PO2 103 mm Hg (>100), PCO2 26 mm Hg (28-32), HCO3 19 mm Hg (22-26). Chest x-ray is normal. What is the correct interpretation of this arterial blood gas? A. Acute metabolic acidosis B. Compensated respiratory alkalosis C. Compensated metabolic alkalosis D. Hypoventilation E. Hyperventilation

B. The increased minute ventilation during pregnancy causes a compensated respiratory alkalosis. Hypoventilation results in increased PCO2 and the PO2 would be decreased if she was hypoxic. A metabolic acidosis would have a decreased pH and a low HCO3. The patient's symptoms are most consistent with a viral upper respiratory infection.

A 24-year old G1P1 woman had an uncomplicated Cesarean section 20 hours ago under general anesthesia secondary to an umbilical cord prolapse. You are called to evaluate her because her temperature is 102.0° F (38.9° C). The patient does not have any specific complaints. She has experienced intermittent chills. Her exam is non-focal. There is no uterine tenderness. A urine analysis shows no WBCS and is nitrate and leukocyte estrase negative. What is the next appropriate step in the management of this patient? A. Endometrial cultures B. Chest X-ray C. Treatment with intravenous broad spectrum antibiotics D. Pelvic ultrasound E. Culture of the incision

B. The lungs are the most common source of fever on the first postpartum day, particularly if the patient had general anesthesia. Atelectasis may be associated with a postpartum fever. Aspiration pneumonia should be considered in patients who had general anesthesia. Ideally antibiotic treatment should be targeted at a specific source of infection. Other sources of infection to consider in a febrile postpartum patient include endometritis, mastitis, the urinary tract, the wound, and the extremities (thrombophlebitis). Endometrial and incision cultures are not indicated since her uterus is non-tender and the incision appears normal.

A 50-year-old G2P2 woman has a history of menorrhagia, pelvic pain, dyspareunia, dysmenorrhea, constipation and occasional spotting in between periods. She has a three-year history of urinary urgency and frequency. The patient is concerned that she has fibroids, as her close friend was recently diagnosed with fibroids. What is the symptom most commonly associated with leiomyomas? A. Intermenstrual spotting B. Menorrhagia C. Dyspareunia D. Dysmenorrhea E. Urinary symptoms

B. The major symptom associated with myomas is menorrhagia, thought to be secondary to: 1) an increase in the uterine cavity size that leads to greater surface area for endometrial sloughing; and/or 2) an obstructive effect on uterine vasculature that leads to endometrial venule ectasia and proximal congestion in the myometrium/endometrium resulting in hypermenorrhea. Other relatively frequent symptoms include pain and pressure symptoms related to the size of the tumors filling the pelvic cavity, as well as causing pressure against the bladder, bowel and pelvic floor.

A 50-year-old G3P3 woman complains of menorrhagia. Physical examination is notable for a 14-week size irregularly shaped uterus. Her hematocrit is 35%. Which of the following is the next most appropriate step in this patient's management? A. Hysteroscopy B. Endometrial sampling C. Treatment with GnRH analogue D. Hysterectomy E. Myomectomy

B. The majority of patients with uterine fibroids do not require surgical treatment. If patients present with menstrual abnormalities, the endometrial cavity should be sampled to rule out endometrial hyperplasia or cancer. This is most important in patients in their late reproductive years or postmenopausal years. If the patient's bleeding is not heavy enough to cause iron deficiency anemia, reassurance and observation may be all that are necessary. Treatment with GnRH analogues to inhibit estrogen secretion may be used as a temporizing measure. This is helpful in premenopausal women who are likely to be anovulatory with relatively more endogenous estrogen. Treatment with GnRH analogues can be used for three to six months prior to a hysterectomy to decrease the uterine size and increase a patient's hematocrit. This may also lead to technically easier surgery and decreased intraoperative blood loss. Treatment with GnRH analogue can also be used in perimenopausal women as a temporary medical therapy until natural menopause occurs. Myomectomy may be an appropriate treatment for a younger patient who desires future fertility. Hysteroscopy is not indicated at this point prior to endometrial sampling. Hysterectomy is a definitive treatment for women who have completed childbearing. Particularly in a perimenopausal woman, it is important to first rule out an underlying endometrial malignancy with endometrial sampling.

A 40-year-old G1P0 woman at 22 weeks gestation presents to the office with a complaint of pelvic pressure. She reports that she had intercourse the night prior to presentation and noted some mucous mixed with blood this morning. Her history is significant for type 1 diabetes and she is on an insulin pump. Her surgical history is significant for a history of cone biopsy for treatment of abnormal Pap smear three years ago. On examination, her BMI is 26. She is noted to have a 2 cm dilated cervix with bulging membranes that rupture upon placing the speculum. Fetal parts are noted in the vagina. What is the most likely cause of this finding? A. Uncontrolled diabetes B. Cervical incompetence/insufficiency C. Preterm labor D. Advanced maternal age E. Infection

B. The most likely cause of painless cervical dilation which leads to pelvic pressure, bulging membranes and fetal loss is cervical incompetence or insufficiency. This patient has a history of cone biopsy which can lead to cervical incompetence. Preterm labor by definition does not occur until 24 weeks gestation. Although uncontrolled diabetes can lead to fetal malformations and early miscarriage, it is not typically a cause of fetal loss in the second trimester. Advanced maternal age is associated with an increased risk of stillbirth, preeclampsia, gestational diabetes and intrauterine growth restriction.

A 56-year-old G2P2 post-menopausal woman presents with abnormal vaginal bleeding of four months duration. The bleeding initially was only light spotting, but has become heavier, longer and now with bleeding occurring almost daily. She also notes abdominal bloating. She is otherwise healthy and takes no medications. On physical examination, she has a mildly distended abdomen with fullness in the lower region. On pelvic examination, she has a multiparous cervix with dark blood in the vaginal vault. Bimanual and recto-vaginal examination confirms an eight-week uterus with a separate 10 cm right adnexal mobile mass. Endometrial biopsy confirms complex hyperplasia without atypia, and pelvic ultrasound reveals a 10 cm complex heterogenous mass with solid components and no ascites. What is the most likely etiology of the adnexal mass that would also explain the findings on endometrial biopsy? A. Dysgerminoma (Germ cell tumor) B. Granulosa cell tumor (Sex-cord stromal tumor) C. Papillary serous carcinoma (Epithelial ovarian tumor) D. Krukenberg tumor (Metastatic carcinoma) E. Endometrioma

B. The most likely diagnosis of the adnexal mass that would also explain the finding of endometrial hyperplasia would be a granulosa cell tumor (sex-cord stromal tumor). GCT are functional tumors that secrete high levels of estrogen, which can ultimately stimulate the endometrium to undergo hyperplastic changes and even lead to endometrial cancer. Approximately 25-50% of women with GCT will have endometrial hyperplasia on biopsy, and 5-10% will have endometrial cancer. Granulosa cell tumors represent 70% of sex-cord stromal tumors and typically affect women in their 50's (most common type is the adult GCT - 95%; the juvenile type affects females before puberty). The three main histologic sub-types of ovarian cancer include germ cell tumors (5%), sex-cord stromal tumors (1-2%), and epithelial tumors (90%). Germ cell tumors typically affect women of younger age groups (ages 10-30), comprise 20-25% of ovarian neoplasms overall (benign and malignant) and account for 70% of tumors in this age group. Epithelial ovarian tumors are the most common and can affect women of all ages, but typically the malignant types occur in women in their sixth decade of life.

A 19-year-old G1P0 woman notes vaginal spotting. Her last normal menstrual period occurred six weeks ago. She began having spotting early this morning and it has increased only slightly. She has no pain and denies other symptoms. Her medical history is noncontributory. Vital signs are: blood pressure 120/68; pulse 68; respirations 20; and temperature 98.6°F (37.0°C). On pelvic exam, her cervix is normal; uterus is small and nontender; and no masses are palpable. Initial labs show quantitative Beta-hCG 2000 mIU/ml and hematocrit 38%. A repeat Beta-hCG level 48 hours later is 2100 mIU/ml. A transvaginal ultrasound shows an empty uterus with a thin endometrial stripe and no adnexal masses. What is the next best step in the management of this patient? A. Dilation and curettage B. Treat with methotrexate C. Exploratory laparotomy D. Repeat Beta-hCG level in 48 hours E. . Repeat ultrasound in 24 hours

B. The patient clearly has an abnormal pregnancy, as demonstrated by the slowly increasing Beta-hCG levels. Since the Beta-hCG level is above 2000 mIU/ml, and she has a thin endometrial stripe, this excludes an intrauterine pregnancy and the diagnosis is an ectopic pregnancy. She is a good candidate for medical treatment with methotrexate. Criteria to consider for medical treatment include hemodynamic stability, non-ruptured ectopic pregnancy, and size of ectopic mass.

A 60-year-old G2P2 woman presents with complaints of urinary frequency and urge incontinence. Past medical history is unremarkable. She is on no medications. Pelvic examination reveals no evidence of pelvic relaxation. Post void residual is normal. Urine analysis is negative. A cystometrogram reveals uninhibited detrusor contractions upon filling. Which of the following is the best treatment for this patient? A. Amitriptyline B. Oxybutynin C. Topical (vaginal) estrogen D. Pseudoephedrine E. Kegel exercises

B. The patient has the diagnosis of detrusor instability. The parasympathetic system is involved in bladder emptying and acetylcholine is the transmitter that stimulates the bladder to contract through muscarinic receptors. Thus, anticholinergics are the mainstay of pharmacologic treatment. Oxybutynin is one example. Although the tricyclic antidepressant, amitriptyline, has anticholinergic properties, its side effects do not make it an ideal choice. Vaginal estrogen has been shown to help with urgency, but not urge incontinence. Pseudoephedrine has been shown to have alpha-adrenergic properties and may improve urethral tone in the treatment of stress incontinence. Kegel exercises or pelvic muscle training are used to strengthen the pelvic floor and decrease urethral hypermobility for the treatment of stress urinary incontinence.

A 28-year-old G1P1 woman delivered four days ago. She tearfully reports that she had trouble sleeping, felt anxious and has been irritable for the last two days. She feels somewhat better today, but is still concerned. What is the most likely diagnosis? A. Hypothyroidism B. Postpartum blues C. Depression D. Normal postpartum state E. Anxiety

B. The patient is describing symptoms of postpartum blues that affects 40-80% women within two to three days postpartum and resolve within two weeks. Symptoms include insomnia, easy crying, depression, poor concentration, irritability or labile affect and anxiety. Symptoms often last a few hours per day and are mild and transient. While hyperthyroidism and anxiety may cause insomnia, this is not the most likely explanation in this patient. Postpartum depression symptoms, such as mood changes, insomnia, phobias and irritability are more pronounced than with the blues and last longer than two weeks.

A 34-year-old G2P1 woman at 40 weeks gestation with a history of one prior vaginal delivery strongly desires an induction of labor, as she is unable to sleep secondary to severe back pain. Her cervical exam is closed, 20% effaced and -2 station. The cervix is firm and posterior. Which of the following is the most appropriate next step in the management of this patient? A. Wait until 42 weeks for induction B. Administer cytotec C. Insert a foley bulb in the cervix D. Perform artificial rupture of membranes E. Perform a Cesarean delivery

B. The patient is multiparous at term and waiting until she reaches 42 weeks may increase the risk of perinatal mortality. Since she is uncomfortable with back pain, it is reasonable to induce labor. Her cervix is unfavorable; therefore, cytotec administration is appropriate prior to pitocin induction. A foley bulb or artificial rupture of membranes cannot be achieved in a patient with a closed cervix. At this time, there are no indications to perform a Cesarean delivery in this patient.

A 57-year-old G2P2 is seen for a routine visit. She states she and her 75-year-old husband stopped having sexual intercourse three years ago when he had an operation for prostate cancer. Menopause occurred at age 50 and she denies taking hormones. Her husband now wishes to resume intercourse and is able to get an erection with sildenafil (Viagra). Attempts at intercourse have been unsuccessful due to the pain she experiences when insertion is attempted. Examination is normal except for a narrowed vagina with atrophic mucosa. Which of the following is the most appropriate recommendation at this time? A. Progesterone cream B. Estrogen cream C. Oral combined hormone therapy D. Petroleum jelly E. Vaginal dilators

B. The patient is not on hormone replacement therapy. With decreased estrogen production after menopause, the vaginal mucosa and other estrogen-dependent tissues can become atrophic. Topical estrogen therapy can restore the integrity of the vaginal epithelium, as well as the support tissues around the vagina. Long-term use may require addition of a progestin due to potential systemic absorption and effect on the endometrium. Dyspareunia (pain with intercourse) can often be improved with the use of estrogen cream used vaginally. Water based lubricants may be helpful, but petroleum jelly is contraindicated because it can cause irritation of the vaginal mucosa. This patient does not have any other menopausal symptoms that would be treated by oral combination HRT. Oral progesterone may help to decrease hot flushes, but progesterone cream would not alleviate vaginal atrophy.

A 28-year-old G0 woman presents to the office for a health maintenance examination. She is currently on oral contraceptive pills, but reports a history of irregular menses prior to starting them. Her past medical history is otherwise non-contributory. On physical examination, she is 5 feet 2 inches tall and weighs 180 pounds. She has an area of velvety, hyperpigmented skin on the back of her neck and under her arms. What is the next best step in the management of this patient? A. Complete blood count B. Diabetes screen C. Biopsy pigmented area D. Pelvic ultrasound E. Lipid profile

B. The patient is obese, with a BMI of 33. In addition, the skin changes are consistent with acanthosis nigricans, which is closely associated with insulin resistance. Given these risk factors, she should be tested for diabetes.

A 32-year-old G1P0 woman comes to your office for her first prenatal care visit. She has recently read an article about the rising Cesarean section rate in the United States and asks you about the rate in your hospital. What do you explain as the major cause of higher Cesarean delivery rates? A. The rate of abnormal presentations has increased B. Less women are having vaginal births after Cesarean C. Obstetricians' reluctance to perform forceps delivery D. Increased rate of fetal macrosomia due to uncontrolled gestational diabetes E. Rate of twins has increased

B. The rate of vaginal birth after Cesarean (VBAC) has decreased in recent years due to studies that showed an increased risk of complications, especially uterine rupture. This is one factor that has led to the increased Cesarean section rate. In addition, although the rate of breech and other abnormal presentation is stable, there are significantly fewer obstetricians who are willing to perform vaginal breech deliveries (due in part to a lack of training in vaginal breech deliveries). Many obstetricians do not perform instrumental vaginal deliveries, such as forceps and vacuum extractions, further contributing to the rising rate. Gestational diabetes is a well-known pregnancy complication with clear clinical guidelines.

A 30-year-old G3P1 woman undergoes an uncomplicated dilation and curettage for a first-trimester miscarriage. Pathology reveals a complete mole. The patient's medical history is significant for chronic hypertension. She has a history of a previous uncomplicated term pregnancy and a termination of a pregnancy at 16 weeks gestation with trisomy 18. What is the risk of her developing a recurrent molar pregnancy? A. No increase risk over general population B. Less than 2% C. 5% D. 10% E. More than 20%

B. The recurrent risk for molar pregnancies ranges from 1 to 2%, which is a 20-fold increase from background risk. The risk of recurrence after two molar pregnancies is 10%.

A 34-year-old G1P0 woman is in a motor vehicle accident. While in the emergency department, the doctors order multiple x-rays to evaluate her injuries. At what gestational age would the fetus be most susceptible to developing intellectual disability with sufficient doses of radiation? A. 0-7 weeks B. 8-15 weeks C. 16-25 weeks D. 26-30 weeks E. 31-35 weeks

B. The risk of developing microcephaly and severe intellectual disability is greatest between eight and 15 weeks gestation. In 1990, the Committee on Biological Effects reported that no risk of mental retardation (now referred to as intellectual disability) has been documented with doses even exceeding 50 rad at less than eight weeks or greater than 25 weeks gestation.

A 35-year-old G3P0020 woman presents at 11 weeks gestation for chorionic villus sampling (CVS). She has had two prior first trimester losses. What is the risk of miscarriage associated with CVS in this patient? A. 0.1% B. 1% D. 10% E. 15% C. 5%

B. The risk of fetal loss associated with CVS is approximately 1% and is not related to her prior miscarriage history.

A 33-year-old G2P1 woman delivered a male infant after a precipitous second stage. On initial assessment, the infant has no respiratory effort. You decide to proceed with positive pressure ventilation. Which of the following techniques will impede positive pressure ventilation on this newborn? A. Adjusting head position to modified flex position B. Adjusting the head to sniffing position C. Securing mask for a good seal D. Compressing the bag just until chest rise is seen E. Having the oxygen flow at minimum 10 L/minute

B. The sniffing position (tilting the neonate's head back and lifting the chin) is the correct position for application of positive pressure ventilation in a newborn infant. It is important to also secure the mask to the infant's face and to observe an initial chest rise. A recommended rate of oxygen flow is 10 L/minute.

A 29-year-old G0 woman presents to your office for a routine visit. She has been trying to conceive for the last six months unsuccessfully. She requests fertility medications and hopes to get pregnant with twins. What counseling do you tell her regarding the risks of multifetal gestation? A. The morbidity with twin gestations is similar to triplet pregnancies B. The twin infant death rate is five times higher than that of singletons C. The rate of cerebral palsy is double in twin infants D. The incidence of abnormal fetal growth is similar to singleton pregnancies E. The incidence of prematurity is similar to singleton pregnancies

B. The twin infant death rate is five times higher than that of singletons. The epidemic of multiple gestations resulting from assisted reproductive techniques is of great significance to individual parturients and to society because of the major morbidities associated with twinning as well as with triplets and higher order multiples. The risk for development of cerebral palsy in twin infants is five to six times higher than that of singletons. One study, with dichorionic twins, monochorionic twins and singletons, showed that twins had a higher incidence of IUGR (intrauterine growth restriction) than singletons. Fifty-eight percent of twins deliver prematurely, with an average gestational age at delivery of 35 weeks. Twelve percent of twins deliver very prematurely.

An 18-year-old G1P0 Asian woman is seen in the clinic for a routine prenatal visit at 28 weeks gestation. Her prenatal course has been unremarkable but she has been reporting increased fatigue. She has not been taking prenatal vitamins. Her pre-pregnancy weight was 120 pounds. Initial hemoglobin at the first visit at eight weeks gestation was 12.3 g/dL. Current weight is 138 pounds. After performing a screening complete blood count (CBC), the results are notable for a white blood cell count 9,700/mL; hemoglobin 10.6 g/dL; mean corpuscular volume 88.2 fL (80.8 - 96.4); and platelet count 215,000/mcL. The patient denies vaginal or rectal bleeding. Which of the following is the best explanation for this patient's anemia? A. Folate deficiency B. Relative hemodilution of pregnancy C. Iron deficiency D. Beta thalassemia trait E. Alpha thalassemia trait

B. There is normally a 36% increase in maternal blood volume; the maximum is reached around 34 weeks. The plasma volume increases 47% and the RBC mass increases only 17%. This relative dilutional effect lowers the hemoglobin, but causes no change in the MCV. Folate deficiency results in a macrocytic anemia. Iron deficiency and thalassemias are associated with microcytic anemia.

A 44-year-old G2P2 woman presents with six months of intermittent vulvar itching. She denies any bleeding, but does have a whitish discharge. She has not felt any obvious lumps or sores. She was diagnosed with lupus over 10 years ago and is on prednisone, mycophenolate mofetil (CellCept), and hydroxychloroquine (Plaquenil). Her periods are irregular and her Pap smears have been normal, and her last one was 5 years ago. She reports treatment in the past for warts when she was first diagnosed with lupus, but denies any other sexually transmitted infections. Examination of the vulva is notable for diffuse, erythematous labia, with a thin white filmy discharge. There appears to be subtle but multi-focal, flat, whitish lesions measuring 0.5 - 1 cm on the labia bilaterally. Her remaining pelvic exam is unremarkable. A saline and potassium hydroxide wet prep is performed and is negative. What is the most appropriate next step in the management of this patient? A. Treatment with oral fluconazole and a topical imidazole B. Biopsy of the vulvar lesions C. Treatment of the lesions with topical trichloroacetic acid D. Wide local excision of the vulva E. Adjustment of her immunosuppressive therapies

B. These lesions most likely represent an HPV-related condition such as condyloma or vulvar dysplasia. Women who are on immunosuppressive therapy are at higher risk of such conditions and require close surveillance. Although, her history of prior treatment for warts suggests these could be condyloma again, their flat, subtle appearance raises a concern that they may be dysplastic lesions, and biopsy is indicated. Treatment for presumed yeast infection is reasonable, given her susceptibility, but would not be the sole treatment in this setting of new clinically evident lesions especially in light of a negative wet prep. Wide excision is not indicated at this time without a diagnosis.

A 24-year-old G3P0 woman at 26 weeks gestation was brought to the hospital by paramedics. Her husband found her shivering and barely responsive. Two days prior, the patient noted that she was feeling sick, with a slight cough. She was having back pain at the time, but thought it was probably normal for pregnancy. Her pregnancy has been uncomplicated except for the recent diagnosis of gestational diabetes. On exam, vital signs are: temperature 100.2°F (37.9°C); pulse 160; and blood pressure 68/32; respiratory rate 32. Oxygen saturation is 82% on room air. There is no apparent fundal tenderness, although the patient exhibits pain with percussion of the right back. Fetal heart tones are not audible. There is no evidence of vaginal bleeding. Extremities are cool to touch. White blood cell count 24,000; hemoglobin 9.5; hematocrit 27%. Urine microscopic analysis shows many white blood cells. What is the most likely etiology for this patient's disease? A. Abruptio placentae B. Pyelonephritis C. Diabetic ketoacidosis D. Chorioamnionitis E. Pneumonia

B. This is a patient in septic shock. The most common cause of sepsis in pregnancy is acute pyelonephritis. Given the absence of bleeding, the clinical picture is not suggestive of placental abruption. Diabetic ketoacidosis is unusual in gestational diabetic patients. Chorioamnionitis and pneumonia may both lead to sepsis, but are not suggested by the clinical picture.

A 23-year-old nulliparous woman presents with a painful nodule in her axilla for three days. She is healthy and has no personal or family history of breast disease. On exam, no abnormalities are seen on inspection and no breast mass is palpated. In the axillary area, shaved skin is noted and an erythematous raised 1 cm lesion is palpated and is slightly tender to touch. What is the most likely diagnosis? A. Paget's disease B. Folliculitis C. Fibroadenoma D. Supernumerary nipple E. Duct obstruction

B. This is a typical presentation for folliculitis which can occur with shaving the axillary hair. Paget's disease is a malignant condition of the breast that has the appearance of eczema and does not typically present in the axillary area. Fibroadenomas are common and are usually firm, painless and freely movable. A supernumerary nipple is a congenital variation and is typically located in the nipple line and not tender. A clogged milk duct can be present in the axillary region, but it is typically present in a woman who is breastfeeding.

A 32-year-old G0 woman comes to the office due to the inability to conceive for the last two years. She reports having been on oral contraceptives for eight years prior. She had menarche at age 14 and had irregular cycles about every three months until she started birth control pills, which made her cycles regular. In the last year, she has had about five cycles in total; her last menstrual period was six weeks ago. She is otherwise in good health and has not had any surgeries. She has no history of abnormal Pap smears or sexually transmitted infections. Her husband of four years is 35-years-old, and has a 10-year-old son from a previous marriage. She is 5 feet 4 inches tall and weighs 165 pounds. On general appearance, she is hirsute on the face and the abdomen. The rest of her exam is otherwise normal. In addition to weight loss and starting metformin, what is the most appropriate treatment for this patient's infertility problem? A. Laparoscopy B. Ovulation induction agents C. Intrauterine insemination D. In vitro fertilization E. Restart oral contraceptives

B. This patient has PCOS (polycystic ovarian syndrome) based on her history, signs and symptoms. It is most important for her to try to lose weight. Metformin and ovulation induction agents are the first-line of treatment for ovulatory dysfunction in PCOS patients. Since there is known ovulatory dysfunction and there is no reason to believe there are problems with semen analysis, IVF is not justified. There is no role for the laparoscopy in this patient. Although oral contraceptives would regulate her cycles, it is not indicated because she is trying to get pregnant.

A 25-year-old G1P1 woman comes to the office due to left breast pain and fever. She is breastfeeding her 2 ½-week-old infant. The symptoms began earlier in the day and are not relieved by acetaminophen. Her vital signs are: blood pressure 120/60; pulse 64; temperature 99.9° F (37.7° C). On exam, she has erythema on the upper outer quadrant of the left breast which is tender to touch. There are no palpable masses. In addition to starting oral antibiotics, what is the most appropriate next step in the management of this patient? A. Discontinue breastfeeding B. Begin ibuprofen C. Obtain a breast ultrasound D. Use a topical antifungal E. Apply breast binders

B. This patient has puerperal mastitis which most often occurs during the second to fourth week after delivery and the most appropriate next step in her management is to use ibuprofen in addition to acetaminophen for pain relief. She should be encouraged to continue breastfeeding or expressing their milk during treatment. Mastitis is usually treated as an outpatient. Patients are treated with oral or IV antibiotics, depending on the severity of infection. Cold compresses may reduce inflammation but are not indicated in the management of mastitis. Breast binders may increase breast pain. A breast ultrasound is not indicated if there is no suspicion of a breast abscess.

A 17-year-old G2P0 female has severe right lower quadrant pain. Her last normal menstrual period was seven weeks ago. She notes that last night she began having suprapubic pain that radiated to her right lower quadrant. This morning, the pain awoke her from sleep. She has had no vaginal bleeding, no nausea or vomiting. The patient's history is notable for two first trimester elective abortions and a history of chlamydia treated twice. Vital signs are blood pressure 90/60; pulse 99; respirations 22; and temperature 98.6°F (37°C). On physical exam, the patient is noted to be curled on a stretcher in a fetal position and says she hurts too much to move. She has rebound and voluntary guarding on abdominal examination. She has severe cervical motion tenderness and rectal tenderness. Her Beta-hCG level is 2500 mIU/ml; hematocrit 24%; and urinalysis negative. Ultrasound shows no intrauterine pregnancy, a right adnexal mass that measures 6 x 2 cm, and a moderate amount of free fluid in the cul de sac. Which of the following is the most appropriate next step in the management of this patient? A. Admit for serial examinations B. Exploratory surgery C. Recheck Beta-hCG level in 48 hours D. Administer methotrexate E. Dilation and curettage

B. This patient has a ruptured ectopic pregnancy until proven otherwise. Her vital signs, examination and anemia are consistent with an intra-abdominal bleed. Exploratory laparoscopy/laparotomy is indicated at this point. Conservative management with observation, serial examinations or repeat Beta-hCG testing could be dangerous in a patient suspected of having a ruptured ectopic pregnancy. Medical management (methotrexate) is not used in a patient with an acute surgical abdomen. Dilation and curettage would not be the next step in management and might only be considered in this scenario after the patient's abdomen was explored.

A 29-year-old G3P0 woman presents for prenatal care at eight weeks gestation. Her two prior pregnancies ended in spontaneous losses at 19 and 18 weeks, respectively. Records corroborate the patient's history of an uncomplicated gestation until the evening of the losses, when she experienced a pink-tinged discharge that prompted her to call her obstetrician. In both cases, the obstetrician noted that her cervix had dilated completely with the amnionic sac bulging into the vagina to the level of the introitus. The patient was afebrile without other complaints and there was no uterine tenderness on exam. She spontaneously delivered the fetus and placenta in both cases. A sonohysterogram confirmed normal uterine anatomy several weeks later. What is the most appropriate next step in the management of this patient? A. Begin weekly fetal fibronectin testing B. Placement of a cervical cerclage at approximately 14 weeks gestation C. Immediate placement of a cervical cerclage D. Administer low dose aspirin and heparin E. Administration of prophylactic progesterone

B. This patient has an incompetent cervix and should have a cervical cerclage at 14 weeks. A positive fetal fibronectin does not indicate incompetent cervix and is used later in pregnancy as a negative predictor of preterm delivery. Pregnancy loss in the late second trimester is not usually related to genetic abnormality of the conceptus and most clinicians delay placement of a cerclage until after the first trimester, given the high background prevalence of first trimester pregnancy wastage. Although some clinicians strongly support the existence of an antiphospholipid antibody syndrome, the term most commonly refers to pregnancy loss or demise, rather than the clinical scenario of silent cervical dilation with delivery described. The patient would, therefore, not need aspirin or heparin. Although some clinicians use prophylactic progesterone to prevent recurrent abortion, as well as preterm labor, no controlled trials support the use of prophylactic progesterone in the treatment of cervical incompetence.

A 19-year-old G1P0 woman at 18 weeks gestation presents with a three-month history of palpitations and intermittent chest pain. Physical examination reveals a pulse of 96 and grade II/VI systolic ejection murmur with a click. The ECG shows normal rate and rhythm and an echocardiogram is ordered. Which of the following is the best treatment in the management of this patient? A. Anxiolytics B. β-blockers C. Calcium-channel blockers D. Digitalis E. No treatment needed at this time

B. This patient has classic symptoms of mitral valve prolapse. Most women with mitral valve prolapse are asymptomatic and diagnosed by routine physical examination or as an incidental finding at echocardiography. A small percentage of women with symptoms have anxiety, palpitations, atypical chest pain, and syncope. For women who are symptomatic, b-blocking drugs are given to decrease sympathetic tone, relieve chest pain and palpitations, and reduce the risk of life-threatening arrhythmias. Because she is symptomatic, the option of no treatment is not correct.

A 19-year-old G0 woman presents with lower abdominal cramping. The pain started with her menses and has persisted, despite resolution of the bleeding. She thinks she may have a fever, but has not taken her temperature. No urinary frequency or dysuria are present. Her bowel habits are regular. She denies vomiting, but has mild nausea. A yellow blood-tinged vaginal discharge preceded her menses. No pruritus or odor was noted. She is sexually active, uses oral contraceptives and states that her partner does not like condoms. On examination: temperature is 100.2°F (37.9°C); pulse 90; blood pressure 110/60. She is well-developed and nourished and in mild distress. No flank pain is elicited. Her abdomen has normal bowel sounds, but is very tender with guarding in the lower quadrants. No rebound is present. Pelvic examination reveals a moderate amount of thick yellow discharge. The cervix is friable with yellow mucoid discharge at the os. Cervical motion tenderness is present. Uterus and the adnexa are tender without masses. Urine dip is negative for nitrates. Urine pregnancy test is negative. What is the most likely diagnosis? A. Vulvovaginal candidiasis B. Acute salpingitis C. Trichomonas vaginitis D. Cervicitis E. Bacterial vaginosis

B. This patient has findings suggestive of acute salpingitis (pelvic inflammatory disease) including lower abdominal pain, adnexal tenderness, fever, cervical motion tenderness, and vaginal discharge. Mucopurulent cervicitis with exacerbation in the symptoms during and after menstruation is classically gonorrhea. Chlamydia is frequently associated with gonorrhea and also causes cervicitis and pelvic inflammatory disease. Cervicitis alone would not explain this patient's constellation of findings. Trichomonas may cause a yellow frothy discharge, and Candida may cause a thick white cottage cheese-like discharge, but neither would cause fever and abdominal pain.

A 24-year-old G1P0 woman at 28 weeks gestation reports difficulty breathing, cough and frothy sputum. She was admitted for preterm labor 24 hours ago. She is a non-smoker. She has received 6 liters of Lactated Ringers solution since admission. She is receiving magnesium sulfate and nifedipine. Vital signs are: 100.2°F (37.9°C); respiratory rate 24; heart rate 110; blood pressure 132/85; pulse oximetry is 97% on a non-rebreather mask. She appears in distress. Lungs reveal bibasilar crackles. Uterine contractions are regular every three minutes. The fetal heart rate is 140 beats/minute. Labs show white blood cell count 127,500/mL. Potassium and sodium are normal. Which of the following has most likely contributed to this patient's respiratory symptoms? A. Increased plasma osmolality B. Use of magnesium sulfate and nifedipine C. Chorioamnionitis D. Preterm labor E. Increased systemic vascular resistance

B. This patient has pulmonary edema. Plasma osmolality is decreased during pregnancy which increases the susceptibility to pulmonary edema. Common causes of acute pulmonary edema in pregnancy include tocolytic use, cardiac disease, fluid overload and preeclampsia. Use of multiple tocolytics increases the susceptibility of pulmonary edema, especially with the use of isotonic fluids. Systemic vascular resistance is decreased during pregnancy. Women with chorioamnionitis are also more likely to develop pulmonary edema, but this is not usually the main cause unless the patient is in septic shock and this patient does not have chorioamnionitis.

A 26-year-old G2P1 woman at 26 weeks gestation presents for a routine 50-gram glucose challenge test. After receiving a one-hour blood glucose value of 148 mg/dl, the patient has a follow up 100-gram 3-hour oral glucose tolerance test with the following plasma values: Fasting 102 mg/dl (normal ≤95 mg/dl) 1-hour 181 mg/dl (normal ≤180 mg/dl) 2-hour 162 mg/dl (normal ≤155 mg/dl) 3-hour 139 mg/dl (normal ≤140 mg/dl) What is the most appropriate next step in the management of this patient? A. Repeat the glucose tolerance test at 28 weeks gestation B. Begin a diabetic diet and blood glucose monitoring C. Begin a diabetic diet, an oral hypoglycemic agent, and blood glucose monitoring D. Begin a diabetic diet, insulin, and blood glucose monitoring E. Reassurance and routine prenatal care

B. This patient has three values on the three-hour glucose tolerance test that were abnormal. Initial management should include teaching the patient how to monitor her blood glucose levels at home on a schedule that would include a fasting blood sugar and one- or two-hour post-prandial values after all three meals, daily. Goals for blood sugar management would be to maintain blood sugars when fasting below 90 and one- and two-hour post-meal values below 120. A repeat glucose tolerance test would not add any value, as an abnormal test has already been documented. Oral hypoglycemic agents and insulin are not indicated at this time, as the patient may achieve adequate glucose levels with diet modification alone. Gestational diabetes varies in prevalence. The prevalence rate in the United States has varied from 1.4 to 14% in various studies. Risk factors for gestational diabetes include: a previous large baby (greater than 9 lb), a history of abnormal glucose tolerance, pre-pregnancy weight of 110% or more of ideal body weight, and member of an ethnic group with a higher than normal rate of type 2 diabetes, such as American Indian or Hispanic descent.

A 27-year-old G0 woman presents to the clinic because of concerns that she has not been able to get pregnant for the last year. She has been married for two years and was using oral contraceptives, which she stopped a year ago to start a family. She is in good health and her only medication is a prenatal vitamin. She was hospitalized at age 19 for pelvic inflammatory disease. Her periods are regular, every 28 days with normal flow; her last period was two weeks ago. She has no history of abnormal Pap smears. Her husband is also healthy with no medical problems. She is 5 feet 4 inches tall and weighs 130 pounds. Her examination, including a pelvic exam, is completely normal. Which of the following is the most likely diagnostic test to find out the cause of her infertility? A. Hysteroscopy B. Hysterosalpingogram C. Progesterone level mid-cycle D. Clomiphene citrate challenge test E. Cervical mucous monitoring

B. This patient is having difficulty conceiving after trying for one year. Based on her history, the most likely underlying factor is tubal disease, as she has a history of being hospitalized for a pelvic infection, most likely pelvic inflammatory disease. This can cause adhesions and blockage of the tubes, which is best assessed with a hysterosalpingogram to evaluate the uterine cavity and tubes. After a single episode of salpingitis, 15% of patients experience infertility. Hysteroscopy will assess the uterine cavity and while sometimes used during a work up for infertility, it does not provide sufficient information about tubal patency. Progesterone levels, a Clomiphene challenge test or cervical mucous monitoring are used at times with infertility workups, but, in a young patient of normal BMI and with normal cycles, it is unlikely to find major ovulatory dysfunction.

A 21-year-old G1 woman at 36 weeks gestation presents with sudden onset of abdominal pain and bleeding. She smokes a pack of cigarettes a day, but otherwise her pregnancy has been uncomplicated. She takes no medications other than prenatal vitamins. Her blood pressure is 150/90; pulse 90; and she is afebrile. Her uterus is tense and very tender. Pelvic ultrasound shows the placenta to be posterior and fundal, with a cephalic presentation of the fetus. Cervical examination reveals no lesions, blood coming through the os and is one centimeter dilated. Fetal heart tones have a baseline of 150, with a Category III fetal heart rate tracing. Tocometer reveals contractions every 30-45 seconds. Which of the following is the most appropriate next step in the management of this patient? A. Amniotomy B. Cesarean delivery C. Induction of labor D. Tocolysis E. A double set-up examination

B. This patient is undergoing a placental abruption, with a deteriorating fetal condition. An emergent Cesarean delivery is necessary. The mother risks excessive blood loss, DIC and possible hysterectomy. The fetus risks neurological injury from anoxia or death. Risk factors for abruption include smoking, cocaine use, abdominal trauma, chronic hypertension, multiparity and prolonged premature rupture of membranes. Since immediate delivery is needed, amniotomy, induction, or tocolysis are not appropriate. A double set-up examination (performed in the operating room with a Cesarean section team scrubbed and ready) is not indicated, since the ultrasound determined the location of the placenta to be fundal.

A 36-year-old G0 woman presents due to increasing facial hair growth and irregular menstrual cycles. She has gained 40 pounds over the last three years. Her symptoms began three years ago and have gradually worsened. She has never been pregnant and is not currently on any medications. On physical exam, she is overweight with dark hair growth at the sideburns and upper lip. The pelvic exam is normal. Which of the following would you expect to find in this patient? A. Decreased luteinizing hormone levels B. Elevated free testosterone C. Decreased prolactin level D. Increased ovarian estrogen production E. Elevated 17-hydroxyprogesterone

B. This patient likely has polycystic ovarian syndrome (PCOS). PCOS patients have testosterone levels at the upper limits of normal or slightly increased. Free testosterone (biologically active) is elevated often because sex hormone binding globulin is decreased by elevated androgens. LH is increased in response to increased circulating estrogens fed by an elevation of ovarian androgen production. Insulin resistance and chronic anovulation are hallmarks of PCOS. Prolactin levels may be elevated in amenorrhea but are not elevated in patients with PCOS.

A 32-year-old G0 woman comes to the office due to the inability to conceive for last two years. She reports having been on oral contraceptives for eight years prior. She had menarche at age 14 and has had irregular cycles about every three months until she started oral contraceptives, which regulated her cycles. In the last year, she has had about five cycles in total; her last menstrual period was six weeks ago. She is otherwise in good health and has not had any surgeries. She has no history of abnormal Pap smears or sexually transmitted infections. She is 5 feet 4 inches tall and weighs 165 pounds. On general appearance, she is hirsute on the face and the abdomen. The rest of her exam is otherwise normal. Which of the following is most likely to help identify the underlying cause of this woman's infertility? A. Luteinizing hormone levels B. Testosterone levels C. Follicle stimulating hormone levels D. Thyroid function tests E. Progesterone levels

B. This patient most likely has PCOS (polycystic ovarian syndrome) based on her history of irregular cycles, her body habitus and hirsutism. Having normal cycles on the birth control pills (OCPs) supports the diagnosis as other causes, such as hypothyroidism, will not normalize the cycles on OCPs. Testosterone levels will be helpful to confirm the diagnosis, especially in the presence of hirsutism. Once a diagnosis is established, progesterone levels are helpful during medical treatment to check if the woman is ovulating. An increased LH/FSH ratio is observed to be elevated in PCOS patients but each test separately will not aid in the diagnosis.

A 38-year-old G0 woman comes to the office for a health maintenance examination. She is healthy and not taking any medications. She has no history of abnormal Pap tests or sexually transmitted infections. Her menstrual cycles are normal and her last cycle was three weeks ago. Her mother was diagnosed with endometriosis and had a hysterectomy and removal of the ovaries at age 38. She is 5 feet 4 inches tall and weighs 130 pounds. On pelvic examination, the patient has a palpable left adnexal mass. An ultrasound was obtained, which showed a 4 cm complex left ovarian cyst and a 2 cm simple cyst on the right ovary. What is the most likely diagnosis in this patient? A. Endometrioma B. Hemorrhagic cyst C. Ovarian carcinoma D. Mature teratoma E. Polycystic ovaries

B. This patient most likely has a hemorrhagic cyst, considering her history and where she is in her menstrual cycle. Her mother's history of endometriosis does increase her risk; however, it is unlikely since she has never had any symptoms herself. Ovarian carcinoma would need to be ruled out, but it is unlikely in an otherwise asymptomatic premenopausal patient. A mature teratoma would have more pathognomonic findings on ultrasound. This patient does not have typical symptoms, body habitus or ultrasound findings for patients with polycystic ovaries.

A 42-year-old G0 woman presents to the office for a health maintenance examination. She reports that her menses have been irregular her entire life ever since menarche at age 15, occurring every 20-45 days. She is not sexually active and reports no other medical problems. She smoked for two years during her adolescence. She has a family history of cervical cancer affecting her mother at age 42. On examination, she is 5 feet 4 inches tall and weighs 180 pounds (BMI 31). She has noticeable hair growth on her upper lip and chin. The rest of her examination including a pelvic examination is normal. Compared to the general population, which of the following malignancies is she at increased risk for developing? A. Breast B. Endometrial C. Cervical D. Lung E. Colorectal

B. This patient most likely has polycystic ovarian syndrome (PCOS), with her clinical manifestation of oligo-menorrhea, obesity, and hirsutism. Because of the chronic unopposed estrogen exposure that accompanies women with PCOS, these individuals carry a higher risk of developing endometrial hyperplasia and cancer. Although obesity in postmenopausal women is associated with a higher risk of breast cancer, it does not increase the risk in premenopausal women. PCOS is considered to increase the risk of ovarian cancer. She does not have obvious risk factors for cervical cancer, lung, or colon cancer, but should be screened and counseled accordingly based on usual guidelines.

A 28-year-old G2P1 woman presents at 20 weeks gestation for a routine prenatal care visit. This pregnancy has been complicated by scant vaginal bleeding at seven weeks and an abnormal maternal serum alpha fetoprotein (MSAFP) with increased risk for Down syndrome, but normal amniocentesis: 46, XX. Her previous obstetric history is significant for a Cesarean delivery at 34 weeks due to placental abruption and fetal distress. Prenatal labs at six weeks showed blood type A negative, antibody screen positive: anti-D 1:64. Which of the following is the most likely cause of the Rh sensitization? A. ABO incompatibility B. Placental abruption C. Amniocentesis D. Abnormal maternal serum alpha fetoprotein (MSAFP) E. First trimester bleeding

B. This patient was sensitized during her first pregnancy that was complicated by abruption and required Cesarean delivery. Transplacental hemorrhage of fetal Rh-positive red blood cells into the circulation of the Rh-negative mother may occur following a number of obstetric procedures and complications, such as amniocentesis, chorionic villus sampling, spontaneous/threatened abortion, ectopic pregnancy, dilation and evacuation, placental abruption, antepartum hemorrhage, preeclampsia, Cesarean delivery, manual removal of the placenta and external version. ABO incompatibility is immune system reaction that occurs when blood from two different and incompatible blood types are mixed together.

A 31-year-old G3P0 woman at 27 weeks gestation is being managed expectantly for preeclampsia with severe features remote from term. Her blood pressure is 155/100 on methyldopa (Aldomet) 500 mg three times a day. Her recent 24-hour urine had 6.6 grams of protein. An ultrasound revealed a fetus with adequate growth, having an estimated fetal weight in the 10th percentile. Her labs are normal, except for a uric acid of 8.0 mg/dL; hematocrit 42% (increased from 37%); and platelet count 97,000. Which of these findings necessitates delivery at this time? A. Elevated uric acid B. Thrombocytopenia C. Proteinuria D. Poorly controlled blood pressures E. Hemoconcentration

B. Thrombocytopenia <100,000 is a contraindication to expectant management of severe preeclampsia remote from term (<32 weeks). Other contraindications include: inability to control blood pressure with maximum doses of two antihypertensive medications, non-reassuring fetal surveillance, liver function test elevated more than two times normal, eclampsia, persistent CNS (central nervous system) symptoms and oliguria. Delivery should not be based on the degree of proteinuria. Although elevated, uric acid and hemoconcentration are markers of preeclampsia, they are not part of the diagnostic or management criteria.

A 19-year-old G2P1 woman presents with vaginal spotting and uterine cramping. Her last normal menstrual period was six weeks ago and she began spotting three days ago. She has no history of sexually transmitted infections. Her vital signs are blood pressure 120/70; pulse 78; respirations 20; and temperature 98.6°F (37.0°C). On pelvic examination, she has no cervical motion tenderness, her uterus is normal size and non-tender; no adnexal masses are palpable. Quantitative Beta-hCG 48 hours ago was 1500 mIU/ml. Currently, Beta-hCG is 3100 mIU/ml. Progesterone is 26 ng/ml; hematocrit is 38%; and urinalysis is normal. What is the most likely finding on transvaginal ultrasound? A. Debris in uterus B. Viable intrauterine pregnancy C. Adnexal mass, empty uterus D. No adnexal mass, empty uterus E. Non-viable intrauterine pregnancy

B. Transvaginal ultrasound will most likely show an intrauterine pregnancy. The Beta-hCG level is above the discriminatory zone for ultrasound (2000 mIU/ml), and the level has doubled in 48 hours. Additionally, the progesterone level is within expected range for a normal pregnancy (>25 ng/ml suggests healthy pregnancy) and up to 30% of all normal pregnancies experience first trimester spotting/bleeding. The findings of debris in the uterus, an empty uterus, with or without an adnexal mass, or free fluid (suggesting hemoperitoneum) would not be anticipated.

An 8-year-old female has been diagnosed with precocious puberty due to presence of menarche, Tanner stage III breasts and otherwise normal work-up for brain, adrenal and ovarian abnormalities. What is the most appropriate next step in the management of this patient? A. Depo-Provera B. GnRH agonist C. Danazol D. Estradiol E. Observation

B. True precocious puberty is manifested by premature secretion of GnRH hormone in a pulsatile manner. Once other causes of hormone production are ruled out, treatment would include GnRH agonist to suppress pituitary production of follicular-stimulating hormone and luteinizing hormone. Observation is acceptable if the precocious puberty is within a few months of the routinely expected puberty. The process should be treated if the bone age or puberty is advanced by several years.

A 35-year-old G1P0 woman with a known twin gestation undergoes an ultrasound evaluation at 18 weeks. She would like to know if her twins are identical or fraternal. Which ultrasound marker is suggestive of dizygotic (fraternal) twins? A. Increased amniotic fluid volume in one of the twins B. Two separate placentas (anterior and posterior) C. Dividing membranes less than 1 mm D. Concordant growth of the twins E. Twin-twin transfusion syndrome

B. Ultrasound markers suggestive of dizygotic (non-identical) twins include a dividing membrane thickness greater than 2 mm, twin peak (lambda) sign, different fetal genders and two separate placentas (anterior and posterior). The two different placental types in twin gestation are monochorionic and dichorionic. Dizygotic conceptions always have dichorionic placentas. Monozygotic conceptions may have either monochorionic or dichorionic placentation, depending upon the time of division of the zygote. Diamniotic dichorionic placentation occurs with division prior to the morula state (within three days post fertilization). Diamniotic monochorionic placentation occurs with division between days four and eight post-fertilization. Monoamniotic, monochorionic placentation occurs with division between days eight and 12 post fertilization. Division at or after day 13 results in conjoined twins. The ultrasound markers for determination of chorionicity described above have been used to assess risk for complications of pregnancy, most notably the twin-twin transfusion syndrome.

A 29-year-old G4P2 woman was diagnosed with twin-twin transfusion syndrome when an ultrasound was performed at 24 weeks gestational age. Which of the following is a complication of twin-twin transfusion syndrome? A. Fetal macrosomia in the donor twin B. Neurologic sequelae in the surviving twin C. Tricuspid regurgitation in the donor twin D. Heart failure in the donor twin only E. High perinatal mortality for donor twin only

B. Untreated severe twin-twin transfusion syndrome has a poor prognosis, with perinatal mortality rates of 70-100%. Death in utero of either twin is common. Surviving infants have increased rates of neurological morbidity, including an increased risk of cerebral palsy. Excessive volume can lead to cardiomegaly, tricuspid regurgitation, ventricular hypertrophy and hydrops fetalis for the recipient twin. Although the recipient twin is plethoric, it is not macrosomic. The donor twin becomes anemic and hypovolemic, and growth is retarded. The recipient twin becomes plethoric and hypervolumic. Either twin can develop hydrops fetalis. The donor twin can become hydropic because of anemia and high-output heart failure.

A 42-year-old G2P2 woman presents with a two-week history of a thick, curdish white vaginal discharge and pruritus. She has not tried any over-the-counter medications. She is currently single and not sexually active. Her medical history is remarkable for recent antibiotic use for bronchitis. On pelvic examination, the external genitalia show marked erythema with satellite lesions. The vagina appears erythematous and edematous with a thick white discharge. The cervix appears normal and the remainder of the exam is unremarkable except for mild vaginal wall tenderness. Vaginal pH is 4.0. Saline wet prep reveals multiple white blood cells, but no clue cells or trichomonads. Potassium hydroxide prep shows the organisms. Which of the following is the most appropriate treatment for this patient? A. Clindamycin B. Azole cream C. Metronidazole D. Doxycycline E. Ciprofloxacin

B. Vulvovaginal candidiasis (VVC) usually is caused by C. albicans, but is occasionally caused by other Candida species or yeasts. Typical symptoms include pruritus and vaginal discharge. Other symptoms include vaginal soreness, vulvar burning, dyspareunia and external dysuria. None of these symptoms are specific for VVC. The diagnosis is suggested clinically by vulvovaginal pruritus and erythema with or without associated vaginal discharge. The diagnosis can be made in a woman who has signs and symptoms of vaginitis when either: a) a wet preparation (saline or 10% KOH) or Gram stain of vaginal discharge demonstrates yeasts or pseudohyphae; or b) a vaginal culture or other test yields a positive result for a yeast species. Microscopy may be negative in up to fifty percent of confirmed cases. Treatment for uncomplicated VVC consists of short-course topical Azole formulations (1-3 days), which results in relief of symptoms and negative cultures in 80%-90% of patients who complete therapy.

A 23-year-old G1P0 woman at 24 weeks gestation requires treatment for depression. She has no other pregnancy complications. In addition to counseling, she begins therapy with fluoxetine (Prozac). Which of the following symptoms is the most common side effect of her therapy? A. Fatigue B. Sleep disturbance C. Headache D. Irritability E. Agitation

B. While all the side effects listed are reported in patients on fluoxetine, an SSRI antidepressant medication, the most common side effect is insomnia. Significant insomnia may affect one in five patients taking SSRIs. In addition to sleep disturbances, sexual dysfunction, such as decreased libido and delayed or absent orgasm, are common.

A 20-year-old G1P1 woman delivered her first baby 24 hours ago. Delivery was uncomplicated and she had an epidural placed for analgesia at 5 cm of cervical dilation. Earlier in the afternoon, she was complaining of a headache and was given ibuprofen. Three hours later, she complained of increasing headache, photophobia and nausea. She denies heavy bleeding. Vital signs are pulse 110; respirations 20; temperature 101.5° F (38.6° C); and blood pressure 100/50. Physical examination reveals obvious distress, as she has her eyes covered and pain when she moves her neck. Her lungs are clear and heart has a regular rate. Her abdomen is nontender, and uterine fundus is easily palpable just below the umbilicus and is nontender. Her extremities reveal no erythema, swelling or tenderness. Which of the following would most helpful to establish a diagnosis in this patient? A. Chest x-ray B. Urinalysis C. Lumbar puncture D. CBC with differential E. Pelvic ultrasound

C. Likely meningitis, perhaps from the epidural.

A 23-year-old G1P1 woman develops a fever on the third day after an uncomplicated Cesarean delivery that was performed secondary to arrest of descent. The only significant finding on physical exam is moderate breast engorgement and mild uterine fundal tenderness. What is the most likely diagnosis in this patient? A. Urinary tract infection B. Mastitis C. Endometritis D. Wound cellulitis E. Septic pelvic thrombophlebitis

C. The most common cause of postpartum fever is endometritis. The differential diagnosis includes urinary tract infection, lower genital tract infection, wound infections, pulmonary infections, thrombophlebitis, and mastitis. Endometritis appearing in a postpartum period is most closely related to the mode of delivery and occurs after vaginal delivery in approximately 2 percent of patients and after Cesarean delivery in about 10 to 15 percent. Factors related to increased rates of infection with a vaginal birth include prolonged labor, prolonged rupture of membranes, multiple vaginal examinations, internal fetal monitoring, removal of the placenta manually and low socioeconomic status. Uterine fundal tenderness is commonly observed in patients with endometritis.

A 29-year-old G0 woman presents due to the inability to conceive for the last 18 months. She has a known history of endometriosis, which was diagnosed by laparoscopy three years ago. She has pelvic pain, which is controlled with non-steroidal anti-inflammatory drugs. Her cycles are regular. She is otherwise in good health and has been married for five years. Her husband had a semen analysis, which was normal. She had a hysterosalpingogram, which showed patent tubes bilaterally. She is getting frustrated that she has not yet achieved pregnancy and asks to proceed with fertility treatments. What is the most appropriate next step in the management of this patient? A. Reassurance and return in six months B. Administer a GnRH agonist C. Ovarian stimulation with clomiphene citrate D. Intrauterine insemination E. Proceed with in vitro fertilization

C. A patient with a known history of endometriosis, who is unable to conceive and has an otherwise negative workup for infertility, benefits from ovarian stimulation with clomiphene citrate, with or without intrauterine insemination. Waiting another six months is not appropriate as she has been trying to conceive for 18 months unsuccessfully. A GnRH agonist is used to control pelvic pain in endometriosis patients unresponsive to other hormonal treatments. In vitro fertilization and adoption can be offered if other treatments fail.

A 34-year-old healthy G3P3 woman presents for a health maintenance examination. She has not seen a gynecologist since she had a tubal ligation six years ago. She has been married for 12 years. She has no history of abnormal Pap tests or sexually transmitted infections. The patient's physical examination is normal, except for a small white plaque noted at the 12:00 o'clock position on her external cervical os. In addition to obtaining a Pap test, which of the following is the most appropriate next step in the management of this patient? A. Annual Pap tests B. Pap test in three years C. Biopsy the lesion D. Perform a wet mount E. Perform cervical conization

C. A white plaque found on the cervix is called leukoplakia and should be biopsied directly or under colposcopic guidance as soon as possible, regardless of Pap test outcome. Pap tests have a false-negative rate as high as 20-30%. If there is no evidence of dysplasia and her Pap test is normal, then routine screening can be resumed. A wet mount would be indicated if there was evidence of white discharge. Although cervical conization maybe necessary if high grade dysplasia is diagnosed, this is not the most appropriate step in the management of this patient.

An 18-year-old G0 woman presents to discuss contraception. Her best friend's mother was just diagnosed with ovarian cancer. The patient is healthy and does not have any significant medical history. She does not have a family history of ovarian, breast or any other malignancies. She uses condoms for birth control. She would like to know what she can do to minimize her risk for developing ovarian cancer. Which of the following recommendations is the most appropriate for this patient? A. Begin childbearing now B. Use an intrauterine device C. Use oral contraceptives until she is ready to have children D. Have a risk reducing salpingo-oophorectomy once childbearing is complete E. There are no proven means to reduce the risk of ovarian cancer

C. A woman's risk for development of ovarian cancer during her lifetime is approximately 1%. Factors associated with development of ovarian cancer include low parity and delayed childbearing. Long-term suppression of ovulation appears to be protective against the development of ovarian cancer. Oral contraceptives that cause anovulation appear to provide protection against the development of ovarian cancer. Five years cumulative use decreases the lifetime risk by one-half. Risk reducing salpingo-oophorectomy is not a practical choice for this patient with no family history, even once she completes childbearing. This option might be considered for a woman with a strong family history and the BRCA mutation.

A 45-year-old G2P2 woman comes to the office because of heavy and irregular menstrual periods. The heavy periods started three years ago and have gradually worsened in amount of flow over time. The periods are interfering with her daily activities. The patient has had two spontaneous vaginal deliveries, followed by a tubal ligation three years ago. On pelvic examination, the cervix appears normal and the uterus is normal in size without adnexal masses or tenderness. A urine pregnancy test is negative. TSH and prolactin levels are normal. Hemoglobin is 12.5 mg/dl. On pelvic ultrasound, she has a normal size uterus and a 2 cm simple cyst on the right ovary. Endometrial biopsy is consistent with a secretory endometrium; no neoplasia is found. What is the most likely diagnosis in this patient? A. Polycystic ovarian syndrome B. Mid-cycle bleeding C. Abnormal uterine bleeding D. Benign cystic teratoma E. Ovarian cancer

C. Abnormal uterine bleeding is a term used to describe uterine bleeding abnormalities. This term can encompass both structural causes (polyp, adenomyosis. Leiomyoma, or malignancy [or hyperplasia]) as well as non-structural causes (coagulopathies, ovulatory dysfunction, endometrial, iatrogenic or not classified). The acronym PALM-COEIN is a means for this classification. This patient had a complete workup, including TSH, Prolactin, pelvic ultrasound and endometrial biopsy, which were all normal. Mid-cycle bleeding at the time of ovulation is due to the drop in estrogen. Ovarian teratomas are not associated with abnormal menses. They typically present with abdominal or pelvic pain which may be associated with torsion. The 2 cm cyst is a functional cyst and is a common finding in ovulatory patients.

A 32-year-old G2P2 woman presents for a health maintenance examination. She is in good health and has no concerns. She does not have a history of abnormal Pap test and her last one was three years ago. Her examination is normal including her pelvic exam. A Pap test is performed and returns as normal with HPV negative. What is the most appropriate screening recommendation for cervical cancer in this patient? A. Pap test and HPV testing in one year B. Pap test and HPV testing in three years C. Pap test and HPV testing in five years D. HPV testing alone in one year E. HPV testing alone in three years

C. According to the American Cancer Society (ACS), the American Society for Colposcopy and Cervical Pathology (ASCCP), and the American Society for Clinical Pathology (ASCP) guidelines for the Prevention and Early Detection of Cervical Cancer, women ages 30 to 65 years should be screened with cytology and HPV testing (''co-testing'') every five years (preferred) or cytology alone every three years (acceptable). Screening by HPV testing alone is not recommended for most clinical settings and there is insufficient evidence to change screening intervals in this age group following a history of negative screens.

A 32-year-old G1P1 woman delivered a 9-pound baby and sustained a 4th-degree laceration two days ago. The delivery was complicated by a shoulder dystocia. Her laceration was repaired in layers in the customary fashion. She now complains of increasing pain in her perineal area, fever, chills and weakness. Her vital signs are: blood pressure 90/50; pulse 120; and temperature 102.2° F (39° C). Her abdomen is soft, nontender and her uterine fundus is firm and nontender. Her perineum is erythematous, swollen, but the laceration edges have separated and are grey. The laceration site is nontender and without feeling but there is tenderness of the surrounding tissue. In addition to broad spectrum antibiotics, what is your next step in the management of this patient? A. Sitz baths B. Whirlpool therapy C. Debridement D. Repair of laceration site E. Incision and drainage of perineal laceration

C. Aggressive debridement of the necrotic areas is required to prevent further spread of the infection. Debridement should extend until vital tissue with good blood supply is encountered. Repair of the defect should be delayed until the infection has completely resolved. Sitz baths and whirlpool therapy will provide symptomatic relief for her discomfort, but not adequate treatment. Incision and drainage of perineal laceration is appropriate for an uncomplicated abscess.

A 22-year-old G1P0 woman currently at 38 weeks gestation complains of decreased fetal movement. Ultrasound reveals a 38-week gestation with no cardiac activity. The parents are upset and want to know the next steps. Which of the following would be most helpful to aide the bereavement process? A. Minimize analgesia used during labor B. Plan for postpartum care on the maternity floor C. Allow the parents to decide when to deliver D. Immediately remove the infant from the room after delivery E. Avoid requesting an autopsy and stillbirth evaluation

C. Allowing the parents when to decide to deliver can help the bereavement process. Keeping the patient adequately anesthetized during the labor and delivery as well as letting the parents hold the baby for as long as they desire also helps them grieve. Whether to have care on the maternity floor needs to be the parents' decision as well. Offering an autopsy to determine the cause of death as well as having someone taking pictures and keeping mementos for the parents is helpful.

A 25-year-old G2P1 woman states her gestational age by known LMP is 16 weeks, 3 days. She reports no complaints and is not yet feeling fetal movement. Her fundal height is 22 cm. The MSAFP (maternal serum alpha fetoprotein) result is elevated. Which of the following is the most likely cause for the abnormal MSAFP result? A. Fetal trisomy B. Polyhydramnios C. Twin gestation D. Fetal abdominal wall defect E. Fetal neural tube defects

C. Alpha fetoprotein (AFP) levels in twin gestations are elevated and should be roughly twice that seen in singleton pregnancies. An additional clue to a possible diagnosis of twin gestation is the fundal height exceeding gestational age in weeks. Other causes of elevated maternal serum AFP include neural tube defects, pilonidal cysts, cystic hygroma, sacrococcygeal teratoma, fetal abdominal wall defects, and fetal death. Polyhydramnios is not by itself associated with abnormal MSAFP levels.

A 48-year-old G2P2 woman comes to your office because she has skipped her menstrual period for the past three months. She denies any menopausal symptoms. Review of symptoms and physical exam are unremarkable. What is the most likely diagnosis in this patient? A. Hypothyroidism B. Early pregnancy C. Perimenopause D. Premature ovarian failure E. Autoimmune disorder

C. Although there has been a decline in the average age of menarche with the improvement in health and living conditions, the average age of menopause has remained stable. The Massachusetts Women's Health Study reports that the average age of menopause is 51.3. This patient is most likely perimenopausal and will probably have more menstrual periods in the future. Although it is important to consider pregnancy and hypothyroidism, this patient's presentation is most consistent with perimenopause. Premature ovarian failure occurs before age 35.

A 33-year-old G3P1 woman presents to your office with a positive home pregnancy test. Her last menstrual period was 12 weeks ago. Obstetrical history is notable for a prior full term vaginal delivery and a miscarriage. Ultrasound reveals multiple internal echoes consistent with a "snow storm" appearance within the 20-week sized uterus, as well as bilateral 6 cm multicystic ovaries. Beta-hCG level is >200,000 mIU/mL. Dilation and curettage is performed and final pathology reveals a complete molar pregnancy. What is this patient's risk of developing persistent (post-molar) gestational trophoblastic disease? A. Not at risk B. Lower than that of a partial molar pregnancy C. Higher than that of a partial molar pregnancy D. Same as that of a partial molar pregnancy E. High enough to mandate prophylactic treatment with methotrexate

C. Although very effective in evacuating both complete and partial molar pregnancies, suction curettage provides definitive therapy in the vast majority of partial moles (>95%). For complete molar pregnancies, although Beta-hCG levels initially do drop following dilation and curettage, they can plateau and eventually rise in approximately 20% of cases. The risk following partial moles is much less (5%). The development of this post-molar GTD may be due to persistent (retained or invasive) disease in the uterus or metastatic disease (often to the lungs). The constellation of findings described in this patient (large uterus, theca lutein cysts, high Beta-hCG) increases the risk that this molar pregnancy will persist despite complete evacuation, hence the need for close follow-up with serial Beta-hCG levels. Persistent disease can easily be cured with chemotherapy, if it develops, and is therefore not routinely given prophylactically, except in high-risk situations (e.g. non-compliant patient who will be lost to follow-up).

A 28-year-old G0 woman presents for an annual examination. She is in good health and not taking any medications. She had a history of normal cycles until six months ago, when she stopped having menses after starting an intense exercise regimen. She is 5 feet 6 inches tall and weighs 120 pounds. Her examination is completely normal. Her urine pregnancy test is negative. What is the underlying pathophysiology of the disease process in this patient? A. Psychogenic amenorrhea B. Premature ovarian failure C. Hypothalamic amenorrhea D. Androgen excess E. Hyperthyroidism

C. Amenorrhea associated with exercise falls under the category of hypothalamic amenorrhea, which causes chronic anovulation. Although it may be related to energy requirements, alterations in the hypothalamic-pituitary-ovarian axis have been described in athletic women. The patient's history and physical exam make ovarian failure, androgen excess and hyperthyroidism less likely, although a TSH level would still be recommended.

A 32-year-old G1 woman with an IVF conceived pregnancy at 12 weeks gestation has a slightly increased fetal nuchal translucency (3.0 mm), but her first trimester screen shows no increased risk for Down syndrome or Trisomy 18. Still concerned about the increased nuchal translucency, the patient requests additional testing to exclude chromosomal abnormalities. Which of the following is the next best step to rule out a chromosomal abnormality in this patient? A. Cell-free DNA testing B. Genetic sonogram C. Amniocentesis D. Sequential screen E. Detailed sonogram and fetal echocardiogram at approximately 18 - 20 weeks gestation

C. Amniocentesis is a diagnostic test that may detect Down syndrome as well as other chromosomal abnormalities. Cell-free DNA testing detects over 99% of cases of Down syndrome. The first trimester screen, which consists of a nuchal translucency and maternal serum PAPP-A and beta-hCG, yields an 85% detection rate for Down syndrome. The NT is the measurement of the fluid collection at the back of the fetal neck in the first trimester. A thickened NT may be associated with fetal chromosomal and structural abnormalities as well as a number of genetic syndromes. Patients who desire non-invasive assessment of their risk for aneuploidy can have first trimester screen (a fetal nuchal translucency (NT) measurement and a maternal serum PAPP-A) and a second trimester quadruple screen. The sequential screen which combines the first trimester screen with a quadruple screen yields a 95% detection rate for Down syndrome at a 5% false-positive rate. Since the fetus in this case had a thickened NT, this patient should be scheduled to have a detailed fetal ultrasound and echocardiogram at 18-20 weeks to rule out anomalies. However, it is not possible to diagnose a chromosomal abnormality with an ultrasound. Similarly, although genetic sonograms (targeted sonogram) focus on markers associated with Down syndrome, they are not diagnostic. Approximately 50% of cases of Down syndrome do not have ultrasound findings. Of note, the American Congress of Obstetrics and Gynecology (ACOG) recommends that all patients be offered aneuploidy screening and invasive prenatal diagnosis as indicated.

A 24-year-old G2P1 woman at 42 weeks gestation presents in early labor. At amniotomy, there is thick meconium and variable decelerations are noted. An amnioinfusion is started. Which of the following is most likely to decrease in this patient? A. Admission to the neonatal intensive care unit B. Post maturity syndrome C. Repetitive variable decelerations D. Risk for Cesarean section E. Meconium presence below the vocal cords

C. Amnioinfusion is a procedure where normal saline is infused into the intrauterine cavity. Amnioinfusionremains a reasonable approach in the treatment of repetitive variable decelerations, regardless of amniotic fluid meconium status. Meconium staining of the amniotic fluid is three to four times more common in the postterm pregnancy. This is likely due to two reasons: 1) greater length of time in utero allows for activation of a more mature vagal system; and 2) fetal hypoxia. Routine prophylactic amnioinfusion for thick meconium does not appear to decrease the incidence of meconium aspiration syndrome or have an impact on neonatal outcomes. Based on current literature, routine prophylactic amnioinfusion for meconium-stained amniotic fluid is not recommended.

A 24-year-old G2P1 woman at 18 weeks gestation with a history of asthma presents to the office with worsening symptoms, needing to use her inhaler more frequently. The symptoms began with the pregnancy and have gradually increased. She is using her albuterol inhaler as needed, recently three times a day. She denies any illness or fever. She has had asthma since she was a child. On exam, the patient appears comfortable. Her temperature is 100.2°F (37.9°C) and respiratory rate is 18. Auscultation of the lungs shows good air movement with mild scattered end expiratory wheezes. There are no rales or bronchial breath sounds. Which of the following is the most appropriate next step in the management of this patient? A. Oral theophylline B. Subcutaneous terbutaline C. Inhaled corticosteroids D. Oral zafirlukast (leukotriene inhibitor) E. Antibiotic treatment

C. Asthma generally worsens in 40% of pregnant patients. One of the indications for moving to the next line of treatment includes the need to use beta agonists more than twice a week. The appropriate choice for her treatment would be inhaled corticosteroids or cromolyn sodium. Theophylline would be used in more refractory patients. Subcutaneous terbutaline and systemic corticosteroids would be used in acute cases. Zafirlukast, a leukotriene receptor antagonist, is not effective for acute disease. The safety of zafirlukast in pregnancy is not well established because there is little experience with their use in pregnancy. Antibiotic treatment is only used when a pulmonary infection is diagnosed.

A 22-year-old G1 woman is undergoing treatment with magnesium sulfate for preeclampsia with severe features. She was delivered 10 hours ago via Cesarean section for a non-reassuring fetal heart rate tracing. She has oliguria and appears lethargic. On exam, no deep tendon reflexes can be appreciated. Her magnesium level is 11 mEq/L. Which of the following conditions is most likely to occur in this patient? A. Seizures B. Paralysis C. Respiratory depression D. Pulmonary edema E. Cardiac arrest

C. At a magnesium level of 11 mEq/L, respiratory depression is most likely to occur. The therapeutic magnesium level is between 4-7 mEq/L. Seizures are prevented by the use of magnesium. Loss of deep tendon reflexes occurs at a level of 7-10 mEq/L. Cardiac arrest may occur at a level of 15 mEq/L.

A 25-year-old G0 woman presents for a refill on oral contraceptives. She has a history of recurrent ovarian cysts. She has no significant medical or surgical history. Her grandmother was recently diagnosed with ovarian cancer and her mother is undergoing treatment for metastatic breast cancer. The patient is interested in assessing her risk for ovarian cancer susceptibility. Which of the following is the most appropriate test to offer this patient? A. Annual CA125 levels B. Annual pelvic ultrasound C. Genetic testing of BRCA1 and BRCA2 mutations on the patient's mother D. Genetic testing of BRCA1 and BRCA2 mutations on the patient's grandmother E. Check the patient for p53 mutation

C. BRCA1 and BRCA2 mutations are typically seen in cases of hereditary ovarian cancers. Overall, it has been estimated that inherited BRCA1 and BRCA2 mutations account for 5 to 10 percent of breast cancers and 10 to 15 percent of ovarian cancers among white women in the United States. Given this family history, it is highly likely that a mutation is present, and the affected individual (proband) should be tested if still alive. Because breast cancers are part of the BRCA mutation, the affected mother should be tested. Routine screening for ovarian cancer has not been established.

A 25-year-old G1 woman at 41 weeks gestation presents to labor and delivery with painful contractions every four minutes. Her cervix is 5 cm dilated, 90% effaced. On cervical exam, you are able to feel a fetal body part but it is not the head. Which of the following is the most likely body part you were palplating? A. Foot B. Hand C. Buttocks D. Back E. Shoulder

C. Breech presentation occurs in approximately 3-4% of women in labor overall, and occurs more frequently in preterm deliveries. Frank breech is the most common type, occurring in 48-73% of cases and the buttocks are the presenting part. Complete breech is found in approximately 5-12% of cases and incomplete breech (footling breech) occurs in approximately 12-38% of cases.

A 32-year-old G0 woman with a last menstrual period three weeks ago, presents with a three-month history of a malodorous vaginal discharge. She reports no pruritus or irritation. She has been sexually active with a new partner for the last four months. Her past medical history is unremarkable. Pelvic examination reveals normal external genitalia without rash, ulcerations or lesions. Some discharge is noted on the perineum. The vagina reveals only a thin, gray homogeneous discharge. The vaginal pH is 5.0. A wet prep is shown in the image below. Which of the following is the most appropriate treatment for this patient? A. Ceftriaxone B. Doxycycline C. Metronidazole D. Azithromycin E. Penicillin

C. Bacterial vaginosis is the most common cause of vaginitis. The infection arises from a shift in the vaginal flora from hydrogen peroxide-producing lactobacilli to non-hydrogen peroxide-producing lactobacilli, which allows proliferation of anaerobic bacteria. The majority of women are asymptomatic; however, patients may experience a thin, gray discharge with a characteristic fishy odor that is often worse following menses and intercourse. Modified Amsel criteria for diagnosis include three out of four of the following: 1) thin, gray homogenous vaginal discharge; 2) positive whiff test (addition of potassium hydroxide releases characteristic amine odor); 3) presence of clue cells on saline microscopy; and 4) elevated vaginal pH >4.5. Treatment consists of Metronidazole 500 mg orally BID for seven days, or vaginal Metronidazole 0.75% gel QHS for five days.

A 90-year-old G7P7 woman presents with severe vaginal prolapse. The entire apex, anterior and posterior wall are prolapsed beyond the introitus. She cannot urinate without reduction of the prolapse. Hydronephrosis is noted on ultrasound of the kidneys and thought to be related to the prolapse. She has a long-standing history of diabetes and cardiac disease. She has failed a trial of pessaries. Which of the following is the next best step in the management of this patient? A. Do nothing and observe B. Anterior and posterior repair C. Colpocleisis D. Sacrospinous fixation E. Sacrocolpopexy

C. Because of the hydronephrosis due to obstruction, intervention is required. Colpocleisis is a procedure where the vagina is surgically obliterated and can be performed quickly without the need for general anesthesia. Anterior and posterior repairs provide no apical support of the vagina. She will be at high risk of recurrent prolapse. The sacrospinous fixation (cuff to sacrospinous-coccygeus complex) or sacrocolpopexy (cuff to sacral promontory using interposed mesh) require regional or general anesthesia and is not the best option for this patient with high surgical morbidity.

A 32-year-old G3P3 woman had a Pap test that showed a high-grade squamous intraepithelial lesion (HSIL). She smokes one pack of cigarettes per day. She has a history of three vaginal deliveries and a tubal ligation. On colposcopic examination, at 12:00 there is an acetowhite lesion with punctations that extends into the endocervical canal. Endocervical speculum is unsuccessful at visualizing the entire lesion. Endocervical curettage and biopsy of this area is negative. Which of the following would be the most appropriate next step in the management of this patient? A. Repeat colposcopy in two months B. Cryotherapy ablation of the transformation zone C. Cervical conization D. HPV typing E. Repeat Pap test in six months

C. Because the entire lesion cannot be visualized, this colposcopy is unsatisfactory. Severe dysplasia and even invasive cancer cannot be ruled out. Endocervical curettage has a relatively low sensitivity (i.e. a high amount of false negatives) and, therefore, you cannot rule out endocervical disease. The endocervical canal must be histologically examined. A cervical conization should be performed to obtain a pathologic specimen. Alternatively co-testing can be repeated at 12 and 24 months. Cryotherapy may serve to ablate part of the canal, but will not provide a pathologic sample to assess for dysplasia or to rule out cancer.

A 60-year-old G2P2 postmenopausal woman has an appointment for a health maintenance examination. She wants to discuss bone density screening for osteoporosis. Her medical history is significant for hypertension for the last 10 years that is well controlled with antihypertensive medications. Which of the following is the most appropriate recommendation for this patient regarding initial bone mineral density screening? A. Obtain a DEXA scan now B. Obtain a DEXA scan now only if her blood pressure is not well controlled C. Obtain a DEXA scan at age 65 D. Bone density screening is only necessary if she presents with a fracture E. Recommend bone density screening five years after the onset of menopause

C. Bone density screening is recommended for women beginning at age 65 unless they have pre-existing risk factors which warrant earlier screening. Risk factors for osteoporosis are early menopause, glucocorticoid therapy, sedentary lifestyle, alcohol consumption, hyperthyroidism, hyperparathyroidism, anticonvulsant therapy, vitamin D deficiency, family history of early or severe osteoporosis, or chronic liver or renal disease. These factors would institute early screening in a patient for osteoporosis. A postmenopausal patient presenting with fractures should alert you to suspect osteoporosis.

A 23-year-old G1P0 woman at 38 weeks gestation, with an uncomplicated pregnancy, presents to labor and delivery with the complaint of lower abdominal pain and mild nausea for one day. Fetal kick counts are appropriate. Her review of symptoms is otherwise negative. Vital signs are: temperature 98.6°F (37.0°C); blood pressure 100/60; pulse 79; respiratory rate 14; fetal heart rate 140s, reactive, with no decelerations; tocometer shows irregular contractions every 2-8 minutes; fundal height 36 cm; cervix is firm, long, closed and posterior. A urine dipstick is notable for 1+ glucose with negative ketones. Which of the following is the most likely diagnosis in this patient? A. Appendicitis B. Gestational diabetes C. Braxton-Hicks contractions D. First stage of labor E. Dehydration

C. Braxton Hicks contractions are characterized as short in duration, less intense than true labor, and the discomfort as being in the lower abdomen and groin areas. True labor is defined by strong, regular uterine contractions that result in progressive cervical dilation and effacement. This patient's history does not suggest she is in the first stage of labor. Patients with appendicitis usually present with fever, decreased appetite, nausea and vomiting. Gestational diabetes is diagnosed based on glucose challenge tests. The first test with a 50 gram load is typically performed at 24-28 weeks gestation. It is not abnormal for patients to have glucosuria. This finding is not diagnostic for gestational diabetes. Patients with dehydration frequently present with maternal tachycardia and have ketonuria.

A 37-year-old G3P2 woman presents with her husband at 11 weeks gestation for genetic counseling due to advanced maternal age. The patient and her husband are interested in chorionic villus sampling (CVS). In addition to obtaining a karyotype, which of the following can be detected with this procedure? A. Spina bifida B. Fetal omphalocele C. Cystic fibrosis D. Anencephaly E. Fetal cardiac anomaly

C. CVS is generally performed at 10-12 weeks gestation. The procedure involves sampling of the chorionic frondosum, which contains the most mitotically active villi in the placenta. CVS can be performed using a transabdominal or transcervical approach. The sampled placental tissue may be analyzed for fetal chromosomal abnormalities, biochemical, or DNA-based studies including testing for the mutations associated with cystic fibrosis. CVS cannot be used to detect neural tube defects. Omphaloceles and neural tube defects are generally diagnosed using prenatal ultrasound. Both of these conditions are associated with an increased MSAFP (maternal serum alpha-fetoprotein).

A 17-year-old G1 woman at 24 weeks gestation presents with vaginal bleeding. She denies any pain, cramping or dysuria. She reports last having intercourse three weeks ago. Prenatal care and labs have been unremarkable. Her vital signs are normal and she is afebrile. Pelvic ultrasound reveals a fundal placenta and viable fetus. Abdominal examination is unremarkable. Vaginal examination reveals a uniformly friable cervix with a small amount of blood in the vault. Digital examination reveals a firm, closed cervix. What is the most likely diagnosis that explains the bleeding? A. Trauma B. Cervical cancer C. Cervicitis D. Bloody show E. Threatened abortion

C. Cervicitis caused by chlamydia, gonorrhea, trichomonas or other infections can present with vaginal bleeding. The cervix is much more vascular during pregnancy and inflammation can lead to bleeding. Evaluation for other causes of bleeding must be completed and then treatment for the infection should be initiated. The patient does not give any history of trauma and cancer is unlikely because of her age. She is not in labor, and a bloody show associated with cervical dilatation is not consistent with the history provided. Threatened abortion occurs during the first trimester.

A 17-year-old G0 female presents with a three-year history of severe dysmenorrhea shortly after menarche at age 14. Her menstrual cycles are regular with heavy flow. She has been treated with ibuprofen and oral contraceptives for the last year without significant improvement. She misses 2-3 days of school each month due to her menses. She has never been sexually active. Physical examination is remarkable for Tanner Stage IV breasts and pubic hair. Pelvic examination is normal, as is a pelvic ultrasound. Both the patient and her mother are concerned. What is the next best step in the management of this patient? A. Sonohysterogram B. CT scan of the pelvis C. Diagnostic laparoscopy D. MRI of the pelvis E. Hysterosalpingogram

C. Chronic pelvic pain is the indication for at least 40% of all gynecologic laparoscopies. Endometriosis and adhesions account for more than 90% of the diagnoses in women with discernible laparoscopic abnormalities, and laparoscopy is indicated in women thought to have either of these conditions. Often, adolescents are excluded from laparoscopic evaluation on the basis of their age, but several series show that endometriosis is as common in adolescents with chronic pelvic pain as in the general population. Therefore, laparoscopic evaluation of chronic pelvic pain in adolescents should not be deferred based on age. Laparoscopy can be both diagnostic and therapeutic in this patient in whom you suspect endometriosis. None of the other imaging modalities listed will help in the further workup of this patient.

A 25-year-old G1 woman at term presents in active labor. Her cervix rapidly changes from 7 centimeters to complete dilation in 1 hour. She has been pushing for two hours. The fetal station has changed from -1 to +1. Fetal heart tracing is Category I. The patient is feeling strong contractions every three minutes. Which of the following is the most appropriate next step in the management of this patient? A. Cesarean delivery B. Forceps delivery C. Continued monitoring of labor D. Augmentation with oxytocin E. Ultrasound for estimated fetal weight

C. Continued monitoring of labor is appropriate if clinical evaluation indicates that the fetus is not macrosomic or there is no obvious fetopelvic disproportion. If either were the case, then a Cesarean delivery would be indicated. At this time, there is no fetal or maternal indication to perform a forceps delivery because the station is +1. Augmentation would be indicated if the contractions were inadequate in intensity or frequency. An ultrasound at this stage of labor is inaccurate and one relies on clinical estimates of weight.

A 17-year-old G0 high school student is brought in by her mother for her first gynecologic examination. She began her menses at age 12 and has had regular periods for the past three years. Her last menstrual period was one week ago. For privacy, you ask to examine the patient without her mother. Further history is obtained in the examination room. She admits that she has been sexually active with her boyfriend for the past three years. She uses condoms occasionally and is fearful about possible pregnancy. She requests that her mother not be informed about her sexual activity. On physical examination, she is anxious, but normally developed. Her pelvic examination reveals no vulvar lesions, minimal non-malodorous discharge, and a nulliparous appearing cervix. The bimanual examination reveals a normal size uterus, and her adnexa are non-tender and not enlarged. Urine pregnancy test is negative. In addition to discussing contraception. What is the next best step in the management of this patient? A. Obtain a serum Beta-hCG level B. Obtain a Pap test C. Obtain DNA probes for gonorrhea and chlamydia D. Initiate treatment with doxycycline and ceftriaxone E. Order a pelvic

C. Counseling about and screening for sexually transmitted infections is the best next step. This patient does not require treatment due to a lack of diagnostic criteria. A serum Beta-hCG is not indicated in the setting of normal menstrual cycles with last menstrual period a week ago and a negative urine pregnancy test. Guidelines for initiation of cervical cancer screening is recommended at age 21 regardless of coitarche. A pelvic ultrasound would not be indicated at this time especially since the pregnancy test is negative and given her lack of menstrual or pelvic symptoms.

A 33-year-old G2P1 presents at 36 weeks gestation for consultation because ultrasound revealed a fetus with biometry consistent with 30 5/7 weeks gestation. The EFW is less than the 5th percentile. Umbilical artery Doppler studies are abnormal. There is reverse end diastolic flow and the amniotic fluid volume is decreased. The AFI is 1.1 cm. Which of the following is the most appropriate next step in the management of this patient? A. Close observation with twice weekly NSTs and amniotic fluid assessments B. Close observation with twice weekly NSTs, amniotic fluid assessments and weekly umbilical artery Doppler studies C. Induction of labor D. Induction of labor at term (37 weeks gestation) E. Delivery by Cesarean section

C. Delivery is indicated in a fetus with IUGR at 36 weeks gestation with oligohydramnios and abnormal umbilical artery Doppler studies. Although there is an increased incidence of fetal intolerance of labor, induction of labor is generally preferred over elective Cesarean delivery. Delivery at term is indicated in fetuses with IUGR with reassuring fetal testing including a normal amniotic fluid volume.

A 38-year-old G0 woman presents for a preconception evaluation. She has a history of long-time anxiety and depression, and is interested in continuing her medications in pregnancy, which includes sertraline (Zoloft). She expresses concern because sertraline is a Category C drug. Which of the following descriptions is associated with FDA Category C drug classification? A. Studies in humans have demonstrated fetal abnormalities and/or there is positive evidence of human fetal risk based on adverse reaction data B. Positive evidence of human fetal risk based on adverse reaction data from studies in humans C. Animal reproduction studies have shown an adverse effect on the fetus and there are no adequate and well controlled studies in humans D. There are no adequate and well controlled studies in pregnant women; animal reproduction studies have failed to demonstrate a risk in the fetus E. Adequate, well-controlled studies in pregnant women that have not shown an increased risk of fetal abnormalities to the fetus in any trimester of pregnancy

C. Depression is more common in women than men. Appropriate treatment, including during the antepartum period, is a component of good medical care. As in all cases, when considering treatments, the benefits should outweigh the risks. With Category A drugs, there are adequate, well-controlled studies in pregnant women that have not shown an increased risk of fetal abnormalities to the fetus in any trimester of pregnancy. With Category B, animal studies have revealed no evidence of harm to the fetus; however, there are no adequate and well-controlled studies in pregnant women or animal studies that have shown an adverse effect, but adequate and well-controlled studies in pregnant women have failed to demonstrate a risk to the fetus in any trimester. Category C drugs have animal studies that show an adverse effect and there are no adequate and well-controlled studies in humans, but potential benefits may warrant use of the drug in pregnant women despite potential risks. Category D drugs have adequate well-controlled or observational studies in pregnant women and are known risks to the fetus. Category X drugs should not be used in pregnancy, because adequate well-controlled or observational studies in animals or pregnant women have demonstrated positive evidence of fetal abnormalities or risks.

A 14-year-old G0 female reports menarche six months ago, with increasingly heavy menstrual flow causing her to miss several days of school. Three months ago, her pediatrician started her on oral contraceptives to control her menstrual periods, but she continues to bleed heavily. Her previous medical history is unremarkable. The patient has a normal body habitus for her age. Appropriate breast and pubic hair development is present. Her hemoglobin is 9.1 mg/dl, hematocrit 27.8%, a urine pregnancy test is negative. Which of the following etiologies for menorrhagia is most likely the cause of her symptoms? A. Uterine leiomyoma B. Thyroid disorder C. Coagulation disorder D. Endometrial hyperplasia E. Chronic endometritis

C. Disorders of clotting may present with menstrual symptoms in young women, with Von Willeberand disease being most common. Leiomyomas typically present in women in their 30's and 40's. Endometrial hyperplasia can occur in younger anovulatory patients, but the short duration of this patient's symptoms makes this less likely. She does not have any signs of infection or thyroid disease.

A 32-year-old G3P2 woman presents at 40 1/7 weeks gestation because of regular uterine contractions every five minutes for the last two hours. Her prenatal course was unremarkable. She states the baby is moving, but she has had a bright red, bloody discharge for the last 30 minutes. She does not think she has ruptured her membranes. Her blood pressure is 120/70; pulse 80; and she is afebrile. Her abdomen is soft and she has regular contractions of moderate intensity. Fetal heart tones have a baseline of 130 with a Category I fetal heart rate tracing. Pelvic ultrasound reveals a fundal placenta and cephalic presentation of the fetus. Cervical examination reveals a friable cervix that bleeds easily and is 5 centimeters dilated and completely effaced. Membranes are confirmed to be intact. Which of the following is the most likely source of bleeding? A. Placental abruption B. Placenta previa C. Bloody show D. Cervical cancer E. Cervicitis

C. During pregnancy the cervix is extremely vascular, and with dilation a small amount of bleeding may occur. This bloody show is not of clinical significance and often occurs with normal labor. Serious causes of bleeding, such as placental abruption and placenta previa, need to be ruled out in order to make the proper delivery plans. Cervical cancer and cervicitis are very unlikely causes for the bleeding in this situation.

A 19-year-old G0 woman presents with severe menstrual pain which causes her to miss school. She takes 600 mg of ibuprofen every four to six hours to control the pain, but this does not relieve the discomfort. She is sexually active, with one partner (she has had two lifetime partners) and uses condoms for contraception. Examination is normal. What is the most appropriate next step in the management of this patient? A. Copper IUD B. GnRH agonist C. Oral contraceptives D. Continuous medroxyprogesterone E. Laparoscopy

C. Dysmenorrhea or painful menstrual cramps is often incapacitating. Oral contraceptives will not only relieve primary dysmenorrhea, but also provide more reliable contraception. Copper IUD's have the potential to cause heavier and more painful periods. Continuous oral Medroxyprogesterone may be effective, but will not provide contraception. Depo-Provera would be a better choice. GnRH agonists are too expensive and have too high a side effect profile to be used for this purpose.

A 36-year-old G2P0 woman at 11 weeks gestational age requests a surgical termination of pregnancy. She had a manual vacuum aspiration last year and would like to undergo the same procedure again. She has chronic hypertension and diabetes well controlled on medications. Vital signs reveal a blood pressure of 120/80 and fasting blood glucose of 100. Which of the following is a contraindication for manual vacuum aspiration of this patient? A. Age B. Parity C. Gestational age D. Chronic hypertension E. Diabetes

C. Manual vacuum aspiration is more than 99% effective in early pregnancy (less than eight weeks). Age, parity and medical illnesses are not contraindications for manual vacuum aspiration. Although the risk of Asherman's syndrome increases with each subsequent pregnancy termination, this patient may still undergo surgical termination as long as she understands risks and benefits. Complications of pregnancy termination increase with increasing gestational age.

A 19-year-old G1P0 woman at 41 weeks with spontaneous rupture of membranes for 13 hours presented to labor and delivery. She had an uncomplicated prenatal course. Her vital signs are: blood pressure 120/70; pulse 72; afebrile; fundal height 36 cm; and estimated fetal weight of 2700 gm. Cervix is dilated to 4 cm, 100% effaced, + 1 station. What does the fetal heart rate tracing seen below show? (fetal heart rate trough min prior to max of CTX) A. Late deceleration B. Variable decelerations C. Early decelerations D. Sinusoidal rhythm E. Normal fetal heart rate pattern

C. Early decelerations are physiologic caused by fetal head compression during uterine contractions, resulting in vagal stimulation and slowing of the heart rate. This type of deceleration has a uniform shape, with a slow onset that coincides with the start of the contraction and a slow return to the baseline that coincides with the end of the contraction. Thus, it has the characteristic mirror image of the contraction. A late deceleration is a symmetric fall in the fetal heart rate, beginning at or after the peak of the uterine contraction and returning to baseline only after the contraction has ended. Late decelerations are associated with uteroplacental insufficiency. Variable decelerations show an acute fall in the FHR with a rapid down slope and a variable recovery phase. They are characteristically variable in duration, intensity, and timing, and may not bear a constant relationship to uterine contractions. The true sinusoidal pattern is a regular, smooth, undulating form typical of a sine wave that occurs with a frequency of two to five cycles/minute and an amplitude range of five to 15 beats per minute. It is also characterized by a stable baseline heart rate of 120 to 160 beats per minute and absent beat-to-beat variability.

A 22-year-old G1P0 woman at 38 weeks gestation presents in labor. Her prenatal course and past history are uncomplicated. She is having regular contractions and, on examination, the cervix is 5 cm, 100% effaced and fetal head at +1 station. The fetal heart rate tracing is shown below. What is the most likely interpretation? Trough of fetal heart rate prior to peak of CTX A. Normal reassuring B. Bradycardia C. Early deceleration D. Variable decelerations E. Late decelerations

C. Early decelerations are thought to represent the fetal response to head compression during the contraction and the fetal heart rate inversely mirrors the changes noted during the contraction. Variable contractions are thought to be due to cord compression and can occur at any time in relation to a contraction. Generally, they have an abrupt onset and return of the fetal heart rate deceleration to the baseline heart rate. Late decelerations are thought to represent uteroplacental insufficiency. The deceleration of the fetal heart rate occurs at or after the peak of the uterine contraction and returns to baseline after complication of the contraction. Bradycardia is defined as fetal heart rate less than 110 beats per minute.

A 25-year-old G2P1 woman at eight weeks gestation is diagnosed with a spontaneous abortion. Her husband is 40 years old. The patient's past medical history is noncontributory. She gets some exercise regularly and smokes two packs of cigarettes a day. Three years ago, she had a full-term delivery that was complicated by mild preeclampsia. Which of the following factors is most likely the cause of this spontaneous abortion? A. Infection B. Advanced paternal age C. Environmental factors D. Uterine anomaly (i.e. unicornuate uterus) E. History of preeclampsia

C. Environmental factors, such as smoking, alcohol and radiation are causes of spontaneous abortion. Although the risk increases with infections, such as listeria, mycoplasma, ureaplasma, toxoplasmosis and syphilis, advancing maternal or paternal age, advancing parity and some mullerian anomalies, the clinical scenario does not support these as possible causes. An isolated history of preeclampsia confers no increase in risk of spontaneous abortion.

A 28-year-old G0 woman presents with her husband for preconception counseling. Her family is Ashkenazi Jewish from Poland. Her husband is 30 years old and is also Jewish. They seek information about preconception and prenatal screening. The patient is at increased risk for having a fetus affected with all of the following conditions except: A. Fanconi anemia B. Tay-Sachs disease C. Beta thalassemia anemia D. Cystic fibrosis E. Niemann-Pick disease

C. Fanconi anemia, Tay-Sachs disease, Cystic Fibrosis, and Niemann-Pick disease are all autosomal recessive conditions that occur at an increased incidence in Jews of Ashkenazi descent. The Beta thalassemia is seen mainly in Mediterranean populations.

A 28-year-old G1P0 woman is at 15 weeks gestation. Her husband's cousin has moderate mental retardation. The most common cause of inherited mental retardation in this patient's child would be? A. Undiagnosed phenylketonuria (PKU) B. Neonatal hypothyroidism C. Fragile X syndrome D. Down syndrome E. Autism

C. Fragile X syndrome is the most common form of inherited mental retardation. The syndrome occurs in approximately 1 in 3,600 males and 1 in 4,000 to 6,000 females. Down syndrome is genetic but the majority of cases are not inherited.

A 23-year-old G2P0 woman at 33 weeks gestation presents to labor and delivery with acute nausea, vomiting and epigastric pain. Her blood pressure is 145/90; she has 1+ protein on a urinalysis. Her labs are shown below: Hematocrit: 42% White blood cell count: 11,000/mcL Bilirubin: 1.4 mg/dL Platelets: 42,000/mcL Lipase: 11 u/L Aspartate aminotransferase (AST): 391 u/L Creatinine: 0.8 mg/dL Alanine aminotransferase (ALT): 444 u/L Uric acid: 7.7 mg/dL Glucose: 100 mg/dL Fibrinogen: 405 mg/dL Which of the following is the most likely diagnosis in this patient? A. Mild preeclampsia B. Hepatitis C. HELLP syndrome D. Cholecystitis E. Acute fatty liver

C. HELLP syndrome is a disease process in the spectrum of severe preeclampsia. The acronym stands for "hemolysis, elevated liver enzymes, low platelets" and can lead to swelling of the liver capsule and possibly liver rupture. It may or may not be accompanied by right upper quadrant pain. It is possible to only have thrombocytopenia and elevated transaminases without clear hemolysis (elevated bilirubin and anemia), especially if a diagnosis is made early. This patient does not have seizures and, therefore, does not have eclampsia. The clinical scenario is not consistent with hepatitis or cholecystitis. Acute fatty liver almost always manifests late in pregnancy. Symptoms develop over several days to weeks and include malaise, anorexia, nausea and vomiting, epigastric pain, and progressive jaundice. In many women, persistent vomiting in late pregnancy is the major symptom. About half of all women have hypertension, proteinuria, and edema signs suggestive of preeclampsia. There is usually severe liver dysfunction with hypofibrinogenemia, hypoalbuminemia, hypocholesterolemia, and prolonged clotting times. As acute fatty liver worsens there is marked hypoglycemia.

A 26-year-old G2P1 woman at 41 weeks gestation is brought in by ambulance. The emergency medical technician reports that a pelvic examination performed 20 minutes ago when the patient had a severe urge to push revealed that she was fully dilated and the fetal station was +2. Fetal heart tones were confirmed to be in the 150s, with no audible decelerations. When the patient is placed on the fetal monitor, the heart rate is noted to be in the 60s. The maternal heart rate is recorded as 100. Without pushing, the fetal scalp is visible at the introitus. A repeat pelvic exam shows that the infant is in the occiput anterior position. What is the most appropriate next step in the management of this patient? A. Emergent Cesarean delivery B. Amnioinfusion C. Assisted operative vaginal delivery D. Confirm the fetal heart rate with an internal fetal scalp electrode E. Use ultrasound to assess the fetal heart rate

C. If the patient cannot deliver the infant with one or two pushes, the next best choice given the fetal station and presentation is to perform an emergent outlet forceps or vacuum-assisted delivery. None of the other options offer an expedient mode of delivery. Since the patient's heart rate is distinct from the fetal heart rate, it is not necessary to check the fetal heart rate with an ultrasound. This will potentially delay the time until delivery of the fetus. Amnioinfusion is not indicated given the imminent delivery.

A 35-year-old G4P3 woman comes in for a postpartum visit. She had a normal uncomplicated vaginal delivery two weeks ago. She has a history of postpartum depression, which required treatment with antidepressants with her last pregnancy. Which of the following signs or symptoms of postpartum depression are most useful to distinguish it from postpartum blues and normal changes that occur after delivery? A. Anhedonia B. Crying spells C. Ambivalence toward the newborn D. Sleeplessness E. Weight loss

C. In addition to the more common symptoms of depression, the postpartum patient may manifest a sense of incapability of loving her family and manifest ambivalence toward her infant. Anhedonia is an inability to experience pleasure from normally pleasurable life events such as eating, exercise, and social or sexual interaction.

A 19-year-old G1P0 woman at 28 weeks gestation comes to labor and delivery because of the onset of contractions. The patient describes the contractions as progressively becoming more painful, each lasting 40 seconds and now occurring every five minutes. She reports good fetal movement and does not have any bleeding or leakage of fluid. On evaluation in triage, it is noted that she is having regular contractions, approximately every five minutes, has intact membranes and her cervical exam is 3 cm dilated and 50% effaced. What is the most frequent cause of this condition? A. Dehydration B. Fetal anomalies C. Idiopathic D. Uterine fibroids E. Cervical incompetence

C. In most cases, preterm labor is idiopathic (i.e. no cause can be identified). Dehydration and uterine distortion (from uterine fibroids or structural malformations) can be associated with preterm labor. In some cases, preterm labor is due to iatrogenic causes; for example, when a physician induces a preterm patient who has severe preeclampsia. Fetal anomalies typically do not cause preterm labor. Cervical incompetence is usually diagnosed earlier in the second trimester and is associated with painless cervical dilation.

A 28-year-old G1 woman at 31 weeks gestation presents with complaints of fluid leaking from the vagina. Preterm premature rupture of membranes is diagnosed. The patient has mild uterine tenderness concerning for early chorioamnionitis. An amniocentesis is performed. Which of the following amniotic fluid results is indicative of an intra-amniotic infection? A. Presence of leukocytes B. Low Interleukin-6 C. Amniotic glucose less than 20 mg/dl D. Elevated level of bilirubin E. Lecithin/sphingomyelin (L/S) ratio <2

C. In some cases of preterm rupture of the membranes, amniocentesis may be performed to detect intra-amniotic infection. The presence of amniotic leukocytes has the lowest predictive value for the diagnosis of chorioamnionitis. Interleukin-6 would be increased in the setting of chorioamnionitis. A low amniotic fluid glucose is an indication of intra-amniotic infection. L/S ratio is a marker for fetal lung maturity.

A 17-year-old G0 female presents with vaginal spotting for the last three days. Her last menstrual period was six weeks ago. Vitals signs are normal. Abdominal and pelvic examination reveals a 10-week sized uterus. Beta-HCG is 80,000 mIU. What is the best next step in the management of this patient? A. Repeat Beta-HCG in 24 hours B. Repeat Beta-HCG in 48 hours C. Pelvic ultrasound D. Dilation and curettage E. Routine prenatal care

C. In the face of discrepancy between dates and uterine size, a pelvic ultrasound is indicated to confirm dates, exclude multiple gestation, uterine abnormalities, and molar pregnancy. There is no single Beta-hCG value that is diagnostic for a molar pregnancy. A quantitative Beta-hCG, though, is crucial at determining whether or not a pelvic (transvaginal) ultrasound will confirm a very early gestation. With a Beta-hCG above the discriminatory zone (>1500 mIU), an IUP should be easily identified on transvaginal ultrasound. If an IUP is not seen, the ultrasound findings (in conjunction with the Beta-hCG level) should identify a mole (multiple internal echoes) or an ectopic (absence of intra-uterine gestation). Additional Beta-HCG levels are not indicated at this time. Suction curettage will provide a pathologic specimen that can distinguish between a normal and molar pregnancy, but it is used only as a therapeutic intervention. Routine prenatal care would be appropriate only after establishing a normal pregnancy.

A 29-year-old G2P1 woman at 36 weeks gestation is seen for management of her gestational diabetes. Despite diet modification, the patient has required insulin to control her serum glucose levels. She has gained 25 pounds with the pregnancy. Which of the following complications is least likely to occur? A. Polyhydramnios B. Neonatal hypoglycemia C. Intrauterine growth restriction D. Preeclampsia E. Fetal macrosomia

C. Intrauterine growth restriction is typically seen in women with pre-existing diabetes and not with gestational diabetes. Shoulder dystocia, metabolic disturbances, preeclampsia, polyhydramnios and fetal macrosomia are all associated risks of gestational diabetes.

A 48-year-old G4P4 woman with last menstrual period four weeks ago presents with a one-year history of non-cyclical pelvic pain, dysmenorrhea and dyspareunia. She has a past history of endometriosis, diagnosed 10 years ago by laparoscopy. She had previously been on oral contraceptives for birth control and menstrual cycle regulation, but elected for permanent laparoscopic sterilization 14 months ago. Minimal endometriosis was noted at the time of laparoscopy. She now has recurrent symptoms and desires definitive treatment. Which of the following is the most appropriate surgical option for this patient?A 48-year-old G4P4 woman with last menstrual period four weeks ago presents with a one-year history of non-cyclical pelvic pain, dysmenorrhea and dyspareunia. She has a past history of endometriosis, diagnosed 10 years ago by laparoscopy. She had previously been on oral contraceptives for birth control and menstrual cycle regulation, but elected for permanent laparoscopic sterilization 14 months ago. Minimal endometriosis was noted at the time of laparoscopy. She now has recurrent symptoms and desires definitive treatment. Which of the following is the most appropriate surgical option for this patient? A. Hysteroscopy and dilation and curettage B. Diagnostic laparoscopy C. Hysterectomy with bilateral salpingo-oophorectomy D. Endometrial ablation E. Wedge resection of the ovaries

C. It is estimated that chronic pelvic pain is the principal preoperative indication for 10-12% of hysterectomies. Since the patient had a tubal ligation and does not desire any more children, the best option is removal of ovaries with or without a hysterectomy. Repeat laparoscopy with treatment of endometriosis and adhesions can be helpful; however, the patient will continue to be at increased risk of recurrent disease. An endometrial ablation or wedge resection of ovaries alone would not be very helpful in the setting of non-cyclical pain.

A 32-year-old G3P0 woman presents to the clinic for preconception counseling. Her prior three pregnancies resulted in first trimester losses. Which of the following tests should be ordered for this patient? A. Adrenal stimulation test B. Clomiphene citrate-FSH challenge test C. Lupus anticoagulant test D. Pelvic MRI E. CT scan of the pelvis

C. It is important to rule out systemic disease in a patient with recurrent abortion (three successive first trimester losses). Testing for lupus anticoagulant, diabetes mellitus and thyroid disease are commonly performed. Maternal and paternal karyotypes should also be obtained. Infectious causes should also be considered. Uterine imaging to exclude a septum or other anomaly is routinely done using hysteroscopy or hysterography and not CT or MRI scanning. There is no role for clomiphene citrate-FSH challenge in the evaluation of this patient.

A 26-year-old G0 woman presents to the reproductive endocrinology clinic seeking an infertility evaluation for failing to conceive after 14 months of unprotected intercourse with her boyfriend, who has fathered two other children. She works as a janitor in a nearby elementary school and currently has Medicaid for her health insurance. The physician discourages her from pursuing treatment because she will likely have to pay for her visit with cash, check or charge, and is told that treatment for infertility often involves procedures and technology that are very expensive. She is also informed that, in most states, many of these therapies are not covered by insurance or Medicaid. This situation violates which of the following ethical principles? A. Patient autonomy B. Beneficence C. Justice D. Physician autonomy E. Non-malfeasance

C. Justice requires that we treat like cases alike. It is the physician's duty to educate the patient about all her treatment options in a non-judgmental way regardless of the nature of the treatment and her socioeconomic status.

29-year-old G1P0 woman at 42 weeks gestation presents in labor. She denies ruptured membranes. Her prenatal course was complicated by chronic hypertension. Her vital signs are: blood pressure 130/80; pulse 72; afebrile; fundal height 36 cm; and estimated fetal weight of 2100 gm. Cervix is dilated to 4 cm, 100% effaced, +1 station. The fetal heart rate tracing is shown below. What is the most likely diagnosis? (fetal heart rate min after peak of CTX) A. Normal fetal heart rate pattern B. Sinusoidal rhythm C. Late deceleration D. Variable decelerations E. Early decelerations

C. Late decelerations are a symmetric fall in the fetal heart rate, beginning at or after the peak of the uterine contraction and returning to baseline only after the contraction has ended. Late decelerations are associated with uteroplacental insufficiency. Variable decelerations show an acute fall in the FHR with a rapid down slope and a variable recovery phase. They are characteristically variable in duration, intensity, and timing, and may not bear a constant relationship to uterine contractions. Early decelerations are physiologic caused by fetal head compression during uterine contraction, resulting in vagal stimulation and slowing of the heart rate. This type of deceleration has a uniform shape, with a slow onset that coincides with the start of the contraction and a slow return to the baseline that coincides with the end of the contraction. Thus, it has the characteristic mirror image of the contraction. The true sinusoidal pattern is a regular, smooth, undulating form typical of a sine wave that occurs with a frequency of two to five cycles/minute and an amplitude range of five to 15 beats per minute. It is also characterized by a stable baseline heart rate of 120 to 160 beats per minute and absent beat-to-beat variability.

A 52-year-old G0 woman presents with long-standing vulvar and vaginal pain and burning. She has been unable to tolerate intercourse with her husband because of pain at the introitus. She has difficulty sitting for prolonged periods of time or wearing restrictive clothing because of worsening vulvar pain. She recently noticed that her gums bleed more frequently. She avoids any topical over-the-counter therapies because they intensify her pain. Her physical examination is remarkable for inflamed gingiva and a whitish reticular skin change on her buccal mucosa. A fine papular rash is present around her wrists bilaterally. Pelvic examination reveals white plaques with intervening red erosions on the labia minora as shown in below picture. A speculum cannot be inserted into her vagina because of extensive adhesions. The cervix cannot be visualized. Which of the following is the most likely diagnosis in this patient? A. Squamous cell hyperplasia B. Lichen sclerosus C. Lichen planus D. Genital psoriasis E. Vulvar cancer

C. Lichen planus is a chronic dermatologic disorder involving the hair-bearing skin and scalp, nails, oral mucous membranes and vulva. This disease manifests as inflammatory mucocutaneous eruptions characterized by remissions and flares. The exact etiology is unknown, but is thought to be multifactorial. Vulvar symptoms include irritation, burning, pruritus, contact bleeding, pain and dyspareunia. Clinical findings vary with a lacy, reticulated pattern of the labia and perineum, with or without scarring and erosions as well. With progressive adhesion formation and loss of normal architecture, the vagina can become obliterated. Patients may also experience oral lesions, alopecia and extragenital rashes. Treatment is challenging, since no single agent is universally effective and consists of multiple supportive therapies and topical high potency corticosteroids.

A 19-year-old G1P0 woman at 40 weeks gestation has an uncomplicated vaginal delivery followed by a brisk hemorrhage. Her past medical history is significant for steroid-dependent asthma. Her blood pressure is 110/70; pulse 84; and she is afebrile. Which of the following uterotonic agents should not be used in this patient? A. Intramuscular oxytocin B. Intravenous oxytocin C. Prostaglandin F2-alpha D. Prostaglandin E1 (Misoprostol) E. Methylergonovine

C. Methergine, prostaglandins and oxytocin are all uterotonics and used to increase uterine contractions and decrease uterine bleeding. Prostaglandin F2-alpha (Hemabate) is a potent smooth muscle constrictor, which also has a bronchio-constrictive effect. As such, it should be used with caution in any patient with a reported history of asthma. It is absolutely contraindicated in patients with poorly controlled or severe asthma. Prostaglandin E1 (Misoprostol), often used for cervical ripening and labor induction, is frequently used in oral or rectal suppository form for treatment of uterine atony, although not FDA approved for this use.

A 30-year-old G0 woman presents with her husband for preconception counseling. The patient is of Ashkenazi Jewish descent. Her husband is Irish. The patient has a brother who has a child diagnosed with attention deficit hyperactivity disorder. Which of the following genetic diseases is the most likely to affect their future children? A. Canavan disease B. Bloom syndrome C. Cystic fibrosis D. Tay-Sachs disease E. Gaucher's disease

C. Non-Hispanic white individuals, including Ashkenazi Jews, are at increased risk for being carriers for cystic fibrosis. The carrier frequency is approximately 1/25 in the non-Hispanic white population. Since the patient's husband is not of Ashkenazi Jewish or French Canadian descent, he is not at increased risk for being a carrier for Tay-Sachs disease. The carrier frequency for Tay-Sachs disease is estimated at 1/30 for Ashkenazi Jews. The gene occurs at a much lower frequency (1 in 300) in most other populations. Canavan disease, Bloom syndrome and Gaucher's disease occur at an increased incidence in the Ashkenazi Jewish population. The carrier frequency for Canavan is 1:55, for Bloom 1:134, and for Gaucher is approximately 1/15 in Ashkenazi Jews.

A 15-year-old female mentions to her doctor that she has never had a menstrual cycle. She is healthy, active in school activities and eats a normal diet. She denies ever being sexually active. On physical examination, she has Tanner stage II breast and pubic hair growth, and average weight and height. Vaginal opening is present and appears normal. What is the most appropriate next step in her management? A. Pelvic ultrasound B. Oral contraceptive pills C. Reassurance D. MRI of sella turcica E. Cortisol challenge test

C. Normal age for menarche is between nine and 17. Since this patient has secondary sexual characteristics and normal anatomy, she should be offered reassurance that she is normal and her menses will probably start soon.

A 22-year-old G2P1 woman is at 42 weeks gestation dated by an ultrasound performed five weeks ago. Her cervix is long and closed. She does not report contractions and states there is good fetal movement. She would like to wait until she goes into labor spontaneously. Which of the following treatment options is optimal at this time? A. Allow the patient to go into labor spontaneously B. Perform an ultrasound to determine gestational age C. Perform a non-stress test (NST) and amniotic fluid index (AFI) twice a week, with induction of labor for a nonreactive NST or oligohydramnios. D. Patient should perform daily fetal movement counts and proceed with induction for decreased fetal movement. E. Perform daily biophysical profiles and deliver if 4 or less

C. Optimal management for the patient with an unfavorable cervix at an uncertain 42 weeks gestation is arguable. Given the uncertainty of her dates, it is reasonable to follow this patient with antepartum fetal testing, such as twice weekly non-stress tests with amniotic fluid index. The risk of fetal death is 1-2/1000 high-risk pregnancies with a reassuring NST, contraction stress test or biophysical profile. The addition of amniotic fluid assessment may improve the predictive value of a reactive NST and reduce the risk of antepartum fetal demise to even lower levels. Ultrasound for gestational age determination in the third trimester is not useful since the measurement error is +/- 3 weeks. Allowing spontaneous onset of labor is okay, but not without some type of antepartum surveillance testing.

A 23-year-old G0 woman presents to the office because she has not had any menses for four months. She has a long history of irregular menstrual cycles since menarche at age 14. She is in good health and is not taking any medications. She is sexually active with her partner of six months, and uses condoms for contraception. She is 5 feet 4 inches tall and weighs 170 pounds. On exam, she has noticeable hair growth on her upper lip and chin. The rest of her examination including a pelvic examination is normal. Her Beta-hCG is < 5 mIU/mL, and her prolactin and TSH levels are normal. In addition to recommending weight loss, what is the most appropriate next step in the management of this patient? A. Treatment with gonadotropin releasing hormone level (GnRH) agonist B. Treatment with clomiphene citrate C. Treatment with oral contraceptives D. Check progesterone levels E. Check cortisol levels

C. Oral contraceptives (OCPs) are the most appropriate treatment for this patient who most likely has the diagnosis of polycystic ovarian syndrome (PCOS). The constellation of findings support this clinical diagnosis (irregular cycles, obesity, and hirsutism). Because she is using condoms for contraception and is sexually active, OCPs would help regulate her cycles and further provide effective contraception. When she desires pregnancy, however, she will most likely need treatment for ovulation induction due to the anovulatory cycles as the leading cause of her oligomenorrhea. Clomiphene citrate is not indicated at this time.

A 27-year-old G2P0 woman is diagnosed with an early first trimester spontaneous abortion. She has a history of type 1 diabetes mellitus, mild chronic hypertension and one prior termination of pregnancy. Which of the following is the most likely cause of this spontaneous abortion? A. Prior termination of pregnancy B. Chronic hypertension C. Diabetes mellitus D. Intrauterine adhesions E. Infection

C. Systemic diseases such as diabetes mellitus, chronic renal disease and lupus are associated with early pregnancy loss. In women with insulin-dependent diabetes, the rates of spontaneous abortion and major congenital malformations are both increased. The risk appears related to the degree of metabolic control in the first trimester. There are many other causes of spontaneous abortion, including genetic factors, endocrine abnormalities, reproductive tract abnormalities, immunologic factors and environmental factors. The patient's history of mild chronic hypertension and one prior termination of pregnancy do not increase her risk of a first trimester loss. Additionally, an uncomplicated termination of pregnancy, intrauterine adhesions and infection are not likely causes in this scenario.

A 26-year-old G0 woman returns for a follow-up visit regarding endometriosis. She has been using NSAIDs to manage her pelvic pain, but had to miss four days of work in the last two months. She is sexually active with her husband of two years, although it has been more painful recently. She has regular menstrual cycles and is using condoms for contraception. On pelvic examination, she has localized tenderness in the cul de sac and there are no palpable masses. What is the most appropriate next step in the management of this patient? A. GnRH agonist B. Danazol C. Oral contraceptives D. Laparoscopy and ablation of endometriosis E. Progesterone intrauterine device

C. Oral contraceptives will be the next best choice for this patient. They provide negative feedback to the pituitary-hypothalamic axis which stops stimulation of the ovary resulting in ovarian suppression of sex hormone production, such as estrogen. Since estrogen stimulates endometrial tissue located outside of the endometrium and uterus, endometriosis can be suppressed by OCPs especially when prescribed in a continuous fashion (omit the week of placebo pills resulting in no withdrawal bleed). GnRH agonists also exert negative feedback, but can be used short term only and have more side effects. Danazol is a synthetic androgen used to treat endometriosis, but due to its androgenic side effects (weight gain, increased body hair and acne, and adverse affect on blood lipid levels) it is not usually the first choice of treatment. Laparoscopy is indicated in the patient who fails medical treatment and/or is planning pregnancy in the near future. A progesterone intrauterine device might potentially help alleviate some of her symptoms, but is not the best management for endometriosis.

A 76-year-old G3P3 woman presents to your office with worsening urinary incontinence for the past three months. She reports increased urinary frequency, urgency and nocturia. On examination, she has a mild cystocele and rectocele. A urine culture is negative. A post-void residual is 400 cc. Which of the following is the most likely diagnosis in this patient? A. Genuine stress incontinence B. Detrusor instability C. Overflow incontinence D. Functional incontinence E. Mixed incontinence

C. Overflow incontinence is characterized by failure to empty the bladder adequately. This is due to an underactive detrusor muscle (neurologic disorders, diabetes or multiple sclerosis) or obstruction (postoperative or severe prolapse). A normal post-void residual (PVR) is 50-60 cc. An elevated PVR, usually >300 cc, is found in overflow incontinence. Stress incontinence occurs when the bladder pressure is greater than the intraurethral pressure. Overactive detrusor contractions can override the urethral pressure resulting in urine leakage. The mixed variety includes symptoms related to stress incontinence and urge incontinence.

A 68-year-old G2P2 woman who has recently moved in with her daughter (a long-standing patient of yours) comes in for a health maintenance examination. A vaginal hysterectomy was done in her fifties for uterine prolapse. She is not sure if her ovaries were removed. She has never had an abnormal mammogram or Pap test and has had yearly exams. She stopped hormone replacement therapy 10 years ago. She was recently widowed after being married for 50 years. She does not smoke or drink. Her diabetes is well-controlled with Metformin; she takes a daily baby aspirin and is on a lipid-lowering agent. On examination, she is a thin elderly woman with a dowager's hump. Her breast exam is unremarkable. Her lower genital tract is notable for atrophy. No masses are noted on bimanual and recto-vaginal exam. A fecal occult blood test is negative. Which of the following tests is not necessary? A. Bone density B. Colonoscopy C. Pap test D. Mammogram E. Annual bimanual and recto-vaginal exam

C. Pap test screening is not indicated in patients who have had a hysterectomy, unless it was done for cervical cancer or a high-grade cervical dyspalsia. Patients with a uterus can discontinue cervical cancer screening between the ages of 65-70 if they have had three consecutive negative smears or two negative consecutive cotesting in the last 10 years and no history of high-grade cervical intraepithelial neoplasia or cancer. Patients still need yearly bimanual and rectovaginal exam. Mammograms are done annually, as breast cancer increases with age. Colon cancer screening is recommended at age fifty. The patient has an exaggerated thoracic spine curvature, termed a dowager's hump, likely secondary to thoracic compression fractures secondary to osteoporosis. If this is confirmed on a bone density test, she may benefit from the addition of bisphosphonates

A 25-year-old G3P2 woman, who had recently undergone a primary Cesarean section, had her HIV status revealed to her mother when a nurse left her chart open in the recovery room. She speaks to patient relations and is thinking about seeking damages through legal avenues. When trying to explain the concept of patient privacy, which of the following statements is correct? A. Patient privacy is based on the ethical principle of justice B. Patient privacy is protected by federal law, primarily with the Federal Emergency Medical Treatment and Labor Act (EMTALA) statute C. Patient privacy is the responsibility of physicians; physicians may be fined and/or assessed criminal penalties for violating the privacy of a patient's protected health information D. The patient cannot win a lawsuit in this case because the mother should not have looked at the open record E. Patient privacy is based on the ethical principal of beneficence

C. Patient privacy is the responsibility of physicians. Physicians may be fined and/or assessed criminal penalties for violating the privacy of a patient's protected health information. It was the responsibility of the physicians and the other health care providers in this case to make sure the chart is not left open so someone walking by sees the information.

A 65-year-old G3P3 woman presents with symptoms of vaginal pressure and heaviness, which seem to worsen towards the end of the day. She has a history of three vaginal deliveries. Her surgical history is significant for hysterectomy for abnormal vaginal bleeding at age 45. On exam, she is found to have a large pelvic prolapse. Which of the following is the most appropriate initial treatment of this patient's prolapse? A. Sacrospinous ligament suspension B. Transvaginal tape C. Pessary fitting D. Anterior repair E. Topical vaginal estrogen

C. Pessary fitting is the least invasive intervention for this patient's symptomatic prolapse. Although a sacrospinous ligament suspension would be an appropriate procedure for this patient, it is invasive and not an appropriate first step. Transvaginal tape is used for urinary incontinence and has no role in the management of this patient. An anterior repair can potentially help with her symptoms, depending on what is contributing most to her prolapse but, again, it is invasive. Topical estrogen is unlikely to properly treat her prolapse and related symptoms.

A 34-year-old G3P1 woman at 26 weeks gestation reports "difficulty catching her breath," especially after exertion for the last two months. She is a non-smoker. She does not have any history of pulmonary or cardiac disease. She denies fever, sputum, cough or any recent illnesses. On physical examination, her vital signs are: blood pressure 108/64; pulse 88; respiratory rate 15; and she is afebrile. Pulse oximeter is 98% on room air. Lungs are clear to auscultation. Heart is regular rate and rhythm with II/VI systolic murmur heard at the upper left sternal border. She has no lower extremity edema. A complete blood count reveals a hemoglobin of 10.0 g/dL. What is the most likely explanation for this woman's symptoms? A. Pulmonary embolism B. Mitral valve stenosis C. Physiologic dyspnea of pregnancy D. Peripartum cardiomyopathy E. Anemia

C. Physical examination findings are not consistent with pulmonary embolus (e.g tachycardia, tachypnea, hypoxia, chest pain, signs of a DVT) or mitral stenosis (diastolic murmur, signs of heart failure). Physiologic dyspnea of pregnancy is present in up to 75% of women by the third trimester. Peripartum cardiomyopathy is an idiopathic cardiomyopathy that presents with heart failure secondary to left ventricular systolic function towards the end of pregnancy or in the several months following delivery. Symptoms include fatigue, shortness of breath, palpitations, and edema. The history and physical do not suggest a pathologic process, nor does her hemoglobin level.

A 28-year-old G3P2 woman presents in labor at 39 weeks gestation and delivers a 3500 gram infant spontaneously after oxytocin augmentation of labor. Thirty minutes later, the placenta has not delivered. Her past medical history is significant for leiomyoma uteri. Her prenatal course was uncomplicated. What is the most likely risk factor for retained placenta in this case? A. Placental abruption B. Labor augmentation C. Leiomyomas D. Multiparity E. Circumvallate placenta

C. Placental abruptions, labor augmentation, degree of parity and circumvallate placenta have no impact on the risk of retained placenta. The following are associated with retained placenta: prior Cesarean delivery, uterine leiomyomas, prior uterine curettage and succenturiate lobe of placenta.

A 45-year-old G2P2 woman presents for a six-week postpartum check. She reports crying spells, loss of appetite, difficulty sleeping and a feeling of low self-worth that began one week after her delivery. She denies any suicidal or homicidal ideations. She is frustrated because she has not been able to breastfeed and feels that she is a bad mother. She has a previous history of anxiety. Which of the following is the most likely diagnosis in this patient? A. Normal puerperium B. Postpartum blues C. Postpartum depression D. Anxiety disorder E. Bipolar disorder

C. Postpartum depression is a common condition estimated to affect approximately 10-15% of women and often begins within two weeks to six months after delivery. Signs and symptoms of depression which last for less than two weeks are called postpartum blues; it occurs in 40-85% of women in the immediate postpartum period. It is a mild disorder that is usually self-limited. This patient does not have signs/symptoms of anxiety disorder or bipolar disorder.

A 28-year-old G3P3 woman status post an uncomplicated spontaneous vaginal delivery of 4150 gram infant experiences profuse vaginal bleeding of 700 cc. Prior obstetric history was notable for a previous low uterine segment transverse Cesarean section, secondary to transverse fetal lie. The patient had no antenatal problems. The placenta delivered spontaneously without difficulty. Which of the following is the most likely cause of this patient's hemorrhage? A. Vaginal lacerations B. Cervical lacerations C. Uterine atony D. Uterine dehiscence E. Uterine rupture

C. Postpartum hemorrhage (PPH) is an obstetrical emergency that can follow vaginal or Cesarean delivery. Uterine atony is the most common cause of PPH and occurs in one in every twenty deliveries. It is important to detect excessive bleeding quickly and determine an etiology and initiate the appropriate treatment as excessive bleeding may result in hypovolemia, with associated hypotension, tachycardia or oliguria. The most common definition of PPH is an estimated blood loss of greater than or equal to 500 ml after vaginal birth, or greater than or equal to 1000 ml after Cesarean delivery.

A 36-year-old G1P1 woman comes to the office due to hair loss. She delivered a healthy infant girl three months ago. She is currently on a progestin-only oral contraceptive pill since she is breastfeeding. In the last month, she has noticed a large amount of hair on her brush each morning. Her father has male pattern baldness and her mother, who is postmenopausal, has had some thinning of her hair, as well. Testosterone and TSH levels are within the normal range. Which of the following is the most likely underlying cause for alopecia in this patient? A. Genetic predisposition B. Progesterone only pills C. High estrogen levels during pregnancy D. Stress during pregnancy and delivery E. Breastfeeding

C. Postpartum telogen effluvium (hair loss) affects 40-50% of women postpartum. High estrogen levels in pregnancy increase the synchrony of hair growth. Therefore, hair grows in the same phase and is shed at the same time. Occasionally, this can result in significant postpartum hair loss at 1 to 5 months postpartum with 3 months after delivery being most common time. In the non-pregnant state, asynchronous hair growth occurs such that a portion of hair is in one of the three hair growth cycles at all times. While genetic predisposition and stress can be causes of hair loss, they are unlikely explanation in this post partum woman.

A 38-year-old G4P2 woman was diagnosed with triplets when an ultrasound was performed at 12 weeks gestational age. Which of the following is the most concerning complication for this multiple gestation? A. Preeclampsia B. Intrauterine growth restriction C. Preterm birth D. Gestational diabetes E. Abnormal placentation

C. Preterm delivery increases the risk of morbidity and mortality, increasing with higher orders of multiples. Preterm birth occurs in over 50% of twin pregnancies, 90% of triplet pregnancies, and almost all quadruplet pregnancies. While all the choices may occur with a multiple gestation, prematurity has the most significant consequences as it is associated with an increased risk of respiratory distress syndrome (RDS), intracranial hemorrhage, cerebral palsy, blindness, and low birth weight. Intrauterine growth restriction, intrauterine death of one or more fetuses, miscarriage and congenital anomalies are all more common with multiple gestations, as are the complications of preeclampsia, diabetes and placental abnormalities.

A 19-year-old G1P0 woman with a desired pregnancy notes vaginal spotting early this morning and it has slightly increased. Her last normal menstrual period occurred six weeks ago. She has no pain or other symptoms. Her medical history is noncontributory. Vital signs are: blood pressure 120/68; pulse 68; respirations 20; and temperature 98.6°F (37.0°C). On pelvic exam, her cervix is normal; her uterus is small and nontender; there are no masses palpable. Labs show: quantitative Beta-hCG 750 mIU/ml; progesterone 3.8 ng/ml; hematocrit 38%. Which of the following is the most appropriate next step in the management of this patient? A. Order a transvaginal ultrasound B. Repeat Beta-hCG level in 24 hours C. Repeat Beta-hCG level in 48 hours D. Dilation and curettage E. Bed rest

C. Repeating the Beta-hCG level will show whether the pregnancy is viable or failing. The appropriate time interval for repeating the initial level is 48 hours, since during the first 42 days of gestation levels increase by approximately 50% every 48 hours in most viable pregnancies. Ordering an ultrasound would not be helpful, since the patient's Beta-hCG level is lower than the discriminatory zone (the level at which an intrauterine pregnancy should be seen on ultrasound, usually 2000 mIU/ml). There is no need to repeat the progesterone level. Dilation and curettage or treatment with methotrexate are both inappropriate without a diagnosis, since both could interrupt a viable pregnancy. Bedrest is not indicated in this patient.

A 22-year-old G1P1 woman delivered her first baby five days ago after a prolonged labor and subsequent Cesarean section for arrest of cervical dilation at 7 cm. Fever was noted on postoperative day two and, despite broad spectrum antibiotics, she continues to have temperature spikes above 101.3° F (38.5° C). She is eating a normal diet and ambulating normally. On physical examination, her breasts have no erythema and nipples are intact. Her abdomen is soft, uterine fundus is firm and nontender, and her incision is healing without induration or erythema. She has normal lochia and her urinalysis is normal. Pelvic examination reveals a firm nontender uterus and no adnexal masses or tenderness. Which of the following is the most likely cause of her fevers? A. Mastitis B. Endometritis C. Ovarian abscess D. Cystitis E. Septic pelvic thrombophlebitis

C. Septic thrombophlebitis involves thrombosis of the venous system of the pelvis. Diagnosis is often one of exclusion of other causes, but sometimes a CT scan will reveal thrombosed veins. Treatment requires addition of anticoagulation to antibiotics and resolution of fevers is rapid. Anticoagulation treatment is short-term. Classic clinical findings for endometritis include fever and maternal tachycardia, uterine tenderness and no other localizing signs of infection. The clinical manifestations of cystitis include lower abdominal pain, frequency, urgency and dysuria. The clinical findings in patients with mastitis include fever, tenderness, induration and erythema of the affected breast.

A 21-year-old G1P1 woman presents to the office with amenorrhea since the birth of her one-year-old daughter. She reports extreme fatigue, forgetfulness, and depression. She was unable to breastfeed because her milk never came in. She notes hair loss including under her arms and in her pubic area. Her delivery was complicated by a postpartum hemorrhage, hypovolemic shock, requiring aggressive resuscitation. She is afebrile. Vital signs are: blood pressure 90/50; pulse 84. The patient appears tired. Her exam is normal but she is noted to have dry skin. A urine pregnancy test is negative. Which of the following is the most likely diagnosis in this patient? A. Hyperprolactinemia B. Hyperthyroidism C. Sheehan Syndrome D. Asherman Syndrome E. Major depressive disorder

C. Sheehan Syndrome is a rare occurrence. When a patient experiences a significant blood loss, this can result in anterior pituitary necrosis, which may lead to loss of gonadotropin, thyroid-stimulating hormone (TSH) and adrenocorticotropic hormone (ACTH) production, as they are all produced by the anterior pituitary. Signs and symptoms of Sheehan syndrome may include slow mental function, weight gain, fatigue, difficulty staying warm, no milk production, hypotension and amenorrhea. Sheehan's syndrome frequently goes unnoticed for many years after the inciting delivery. Treatment includes estrogen and progesterone replacement and supplementation with thyroid and adrenal hormones.

A 38-year-old G1P1 woman who delivered by a Cesarean delivery three weeks ago presents to the clinic with concerns that the baby is not getting enough milk and is fussy. She reports that she is feeding on demand and not supplementing. Which of the following is indicative that the baby is getting adequate milk? A. Sleeps through the night B. Spits up a small amount of milk after feeding C. 3-4 stools in 24 hours D. 3-4 diapers wet with urine in 24 hours E. Coverage of the entire areola with his mouth when he breastfeeds

C. Signs that a baby is getting sufficient milk include 3-4 stools in 24 hours, six wet diapers in 24 hours, weight gain and sounds of swallowing.

A 25-year-old G0 woman presents to her doctor for preconception counseling. She is healthy without significant medical problems. She takes no medications. She smokes one pack of cigarettes per day since age 16 and drinks occasionally. She weighs 140 pounds and her vital signs and examination are normal. The patient is at increased risk of which of the following during her pregnancy? A. Fetal chromosomal abnormality B. Breech presentation C. Placental abruption D. Cerebral palsy E. Neural tubal defect

C. Smoking increases the risk of several serious complications of pregnancy, including placental abruption, placenta previa, fetal growth restriction, preeclampsia and infection. Women who smoke should be counseled vigorously to quit smoking prior to conception and to resist restarting after the baby is born.

A 17-year-old G1P0 woman at 32 weeks gestation complains of right flank pain that is "colicky" in nature and has been present for two weeks. She denies fever, dysuria and hematuria. Physical examination is notable for moderate right costovertebral angle tenderness. White blood cell count is 8,800/mL and urine analysis is negative. A renal ultrasound reveals no signs of urinary calculi, but there is moderate (15 mm) right hydronephrosis. Which of the following is the most likely cause of these findings? A. Smooth muscle relaxation due to declining levels of progesterone B. Smooth muscle relaxation due to increasing levels of estrogen C. Compression by the uterus and right ovarian vein D. Elevation of the bladder in the second trimester E. Iliac artery compression of the ureter

C. Some degree of dilation in the ureters and renal pelvis occurs in the majority of pregnant women. The dilation is unequal (R > L) due to cushioning provided by the sigmoid colon to the left ureter and from greater compression of the right ureter due to dextrorotation of the uterus. The right ovarian vein complex, which is remarkably dilated during pregnancy, lies obliquely over the right ureter and may contribute significantly to right ureteral dilatation. High levels of progesterone likely have some effect but estrogen has no effect on the smooth muscle of the ureter

A 68-year-old woman with a history of breast cancer presents for evaluation of endometrial cancer risk. She was treated with lumpectomy and axillary node dissection and radiation therapy. She has been on tamoxifen therapy for the past year. She denies any vaginal bleeding or discharge. She is 5 feet 3 inches tall and weighs 140 pounds. Her pelvic examination is notable only for severe vulvovaginal atrophy. What is the next best step in the management of this patient? A. Endometrial biopsy now to obtain a baseline B. Annual endometrial biopsy C. Annual exams D. Annual pelvic ultrasound E. Endometrial biopsy upon completion of five years of tamoxifen therapy

C. Tamoxifen is known to increase the risk of endometrial cancer. However, diagnostic studies, such as endometrial biopsy, are reserved for when the patient develops symptoms of bleeding or abnormal vaginal discharge. Ultrasound is not helpful because Tamoxifen is known to cause changes to the endometrium, including thickening. Endometrial biopsy is not indicated as a screening tool for endometrial cancer.

A 35-year-old G1P0 woman at 30-weeks gestation is transferred from an outside hospital in preterm labor. Her cervix is 3 cm dilated, 50% effaced and the vertex is at 0 station. She is having contractions every five minutes and has no signs consistent with an intra-amniotic infection (chorioamnionitis). She was initially treated with terbutaline prior to her transfer. Which of the following side effects would you expect? A. Premature constriction of the ductus arteriosus B. Respiratory depression C. Tachycardia D. Tachypnea E. Headache

C. Terbutaline is a beta-adrenergic agent. Side effects include tachycardia, hypotension, anxiety and chest tightening or pain. Tachypnea and headaches are not usual side effects. The FDA made a formal announcement in 2011 warning against using terbutaline to stop preterm labor stating that terbutaline is both ineffective and dangerous if used for longer than 48 hours. The drug may still be used on a short-term basis in patients with active contractions, such as those being transferred to another hospital for tertiary care. Alternative tocolytic agents should be used for longer term treatment of preterm labor. Non-steroidal anti-inflammatory agents, such as indomethacin can be used as a tocolytic agent, but would have the side effect of premature closure of the ductus arteriosus if used beyond 32 weeks gestation. Magnesium sulfate can also be used as a tocolytic and has the potential side effect of respiratory depression.

A 21-year-old G0 woman presents for her first pelvic examination. She is completely asymptomatic, healthy, and reports having only one sexual partner. She uses condoms for contraception. On examination, the patient has a normal appearing cervix except for minimal, non-malodorous vaginal discharge. Chlamydia and gonorrhea screening is performed, as well as a Pap test. The Pap test is read as ASCUS (atypical squamous cells of undetermined significance), HPV negative, and her cultures are negative. Which of the following is the most appropriate management strategy for this patient? A. Repeat the Pap test in 4-6 weeks after antibiotic treatment for bacterial vaginosis B. Pap test in one year C. Pap test in three years D. Colposcopy with endocervical curettage and directed biopsies E. Cervical conization

C. The ASCCP (American Society of Colposcopy and Cervical Pathology) recommends that management options for ASCUS include performing HPV DNA testing or repeat cytology at 12 months following the abnormal Pap test result. If the HPV testing is negative (as was reported in this case), then routine screening can be resumed at three years. If HPV is positive, or if repeat cytology at 12 months reveals ASCUS or higher, then colposcopy should be performed. For women ages 21-24, if HPV is positive, then repeat cytology at 12 months is recommended with colposcopy performed only if the repeat cytology reveals ASC-H (atypical squamous cell - cannot rule out high grade squamous intraepithelial lesion), AGC (atypical glandular cells) or HSIL (high-grade squamous intraepithelial lesion). The presence of an underlying infection does not affect the triage of an abnormal Pap smear but may explain the presence of ASCUS. See ASCCP guidelines: http://www.asccp.org/Portals/9/docs/Algorithms%207.30.13.pdf

A 25-year-old G1P0 woman is seen for an initial obstetrical appointment at eight weeks gestation. She has had a small ventricular septal defect (VSD) since birth. She has no surgical history and no limitations on her activity. Vital signs are: respiratory rate 12; heart rate 88; blood pressure 112/68. On physical examination: her skin appears normal; lungs are clear to auscultation; heart is a regular rate and rhythm. There is a grade IV/VI coarse pansystolic murmur at the left sternal border, with a thrill. Chest x-ray and ECG are normal. Which of the following is the correct statement regarding cardiovascular adaptation in this patient? A. Approximately 2% of women will normally have a diastolic murmur B. Maternal pulmonary vascular resistance is normally less than systemic vascular resistance C. The maternal cardiac output will increase up to 33% during pregnancy D. Maternal systemic vascular resistance increases throughout pregnancy E. The increase in cardiac output is only due to the increase in the maternal stroke volume

C. The cardiac output increases up to 33% due to increases in both the heart rate and stroke volume. The SVR falls during pregnancy. Up to 95% of women will have a systolic murmur due to the increased volume. Diastolic murmurs are always abnormal. The systemic vascular resistance (SVR) is normally greater than the pulmonary vascular resistance. If the pulmonary vascular resistance exceeds the SVR, right to left shunt will develop in the setting of a VSD, and cyanosis will develop.

A 12-year-old girl is brought to the office by her mother who complains that her daughter has never been interested in dolls and pretty dresses, but prefers to play with tools and mechanical things. The mother also divulges that her brother is gay and is worried that her daughter will grow up as a lesbian and be stigmatized. A private conversation with the girl reveals that she is starting to show an interest in boys, and even has a "boyfriend." Examination reveals a normal pre-pubertal phenotype. Which of the following is the most appropriate course of action at this time? A. Encourage mother to support more gender-appropriate activities including dressing like and playing with other girls B. Refer the mother to a family counselor C. Reassure the mother that her daughter's behavior is normal D. Inform mother that her daughter may ultimately develop an alternative sexual lifestyle E. Have the patient return for a repeat assessment once she has begun menarche

C. The division of interests based on gender is inappropriate. Children should be encouraged to follow their own interests and desires. The girl has a normal physical exam; therefore, diagnostic studies are not indicated.

An obese 30-year-old G3P1 Asian woman undergoes an uncomplicated dilation and curettage for a first-trimester miscarriage. Pathology reveals a molar pregnancy. The patient's medical history is significant for chronic hypertension. She has a history of a previous uncomplicated term pregnancy, and termination of a pregnancy at 16 weeks gestation for trisomy 18. What aspect of the patient's history places her at increased risk for a molar pregnancy? A. Obesity B. Previous history of fetal aneuploidy C. Asian race D. Chronic hypertension E. Prior term pregnancy

C. The incidence of molar pregnancy is approximately 1 per 1,500-2,000 pregnancies among Caucasians in the United States. There is a much higher incidence among Asian women in the United States (1/800.) Molar pregnancy occurs more frequently in women less than 20 or older than 40 years of age. The incidence is higher in areas where people consume less beta-carotene and folic acid. There is no known association between molar pregnancy and obesity, a previous history of fetal aneuploidy, chronic hypertension and parity. The risk of having a molar pregnancy is increased in women with two or more miscarriages.

A 23-year-old G0 woman with severe dysmenorrhea that is unresponsive to non-steroidal anti-inflammatory agents and oral contraceptives is taken to the operating room for a laparoscopy. Blue-black powder burn lesions are seen in the pelvis. A biopsy is performed and sent to pathology. Which of the following pathologic findings would you expect to see in this patient? A. Hyperplastic overgrowth of endometrial glands/stroma B. Decidual effect in the endometrium C. Endometrial glands/stroma and hemosiderin-laden macrophages D. Invasion of endometrial glands into the myometrium E. Well-circumscribed, non-encapsulated myometrium

C. The lesions described are classic for endometriosis. One would therefore expect to see endometrial glands/stroma with hemosiderin-laden macrophages. Hyperplastic overgrowth of endometrial glands/stroma is consistent with endometrial polyps. Decidual effect on the endometrium are seen during pregnancy. Invasion of endometrial glands into the myometrium is seen with adenomyosis. Well-circumscribed, non-encapsulated myometrium is consistent with myomas.

A 49-year-old G5P5 woman presents for her first health maintenance examination since she had her last child 10 years ago. She has no health complaints. She has had two sexual partners. She smokes three to five cigarettes per day, and has been smoking for the past 15 years. Last month, her mother underwent a radical hysterectomy for Stage 1B cervical carcinoma. Her pelvic examination is normal, except for mucopurulent discharge and vaginal condyloma. Which of the following is the patient's greatest risk for developing cervical cancer? A. Family history of cervical cancer B. Smoking history C. Vaginal condyloma D. Multiparity E. Age

C. The majority of risk factors for cervical cancer are related to HPV exposure and include early-onset sexual activity, multiple sexual partners, a sexual partner with multiple partners, history of HPV or other sexually transmitted diseases, immunosuppression, smoking, low socioeconomic status and a lack of regular Pap tests. In this patient with multiple risk factors, the presence of an HPV-related condition (vaginal condyloma) already indicates infection with HPV. Although the HPV type associated with condyloma is typically a low risk strain (e.g. types 6 and 11), she is also at risk of having been exposed to high-risk types that are typically associated with high-grade dysplasia and cervical cancer (e.g. types 16 and 18).

An 88-year-old G2P2 woman is brought in for evaluation of blood found in her diapers. She is a nursing home resident and has a long-standing history of incontinence. This is the first time that her caregivers have noted blood. They describe it as "quarter size." Her nurses think that she may have been itching, as they frequently find her scratching through the diaper. On review of her medical record, biopsy-documented lichen sclerosus of the vulva was diagnosed fifteen years ago. She has not been on any therapy for this condition for years. Examination of the external genitalia reveals an elevated, white, firm irregular lesion arising from the upper middle left labia. The lesion measures 2.5 cm in greatest dimension. The remainder of the external genitalia shows evidence of excoriation of thin, white skin with a wrinkled parchment appearance. The vagina and cervix are atrophic. No masses are noted on bimanual or rectovaginal exam and a sample of her stool is negative for blood. No adenopathy is noted in her groin. Which of the following is the most appropriate next step in the management of this patient? A. Begin topical steroids B. Begin topical benadryl C. Biopsy the lesion D. Obtain cultures of the lesion E. Complete vulvectomy and lymph node dissection

C. The most important step is to first biopsy the lesion. It would be inappropriate to treat the lichen sclerosus first with steroids, as the lesion is suspicious for malignancy. Treatment with benadryl would also be inappropriate given the suspicious nature of the lesion. Diagnostic studies such as cultures and cytology of such a lesion are not appropriate given the exophytic, nodular lesion seen on examination. A biopsy should be performed to make a definitive diagnosis and rule out malignancy. It would also be inappropriate to treat the patient with a vulvectomy and lymph node dissection before obtaining a clear diagnosis.

A 30-year-old G1P1 woman who underwent an urgent vacuum extraction of a baby girl two months ago is experiencing persistent depressive symptoms suggestive of postpartum depression. She is recently divorced and has no immediate family or close friends. She works as a mechanic in a local garage and is planning on going back to school. She contemplated terminating the pregnancy but ultimately decided to have the baby despite no support from her ex-husband. She has a history of depression in the past but has not required any medications for the last three years. Which of the following is her most significant risk factor for postpartum depression? A. Single parent B. Consideration to terminate the pregnancy C. Personal history of depression D. Urgent delivery E. Social isolation

C. The most significant risk factor for developing postpartum depression is the patient's prior history of depression. Other risk factors for postpartum depression include marital conflict, lack of perceived social support from family and friends, having contemplated terminating the current pregnancy, stressful life events in the previous twelve months, and a sick leave in the past twelve months related to hyperemesis, uterine irritability or psychiatric disorder.

A 24-year-old G1P0 at 32 weeks gestation presents with vaginal bleeding most likely caused by placental abruption. She receives a standard dose of 300 micrograms of RhoGAM. What amount of fetal blood is neutralized by this dose? A. 10 cc B. 20 cc C. 30 cc D. 40 cc E. 50 cc

C. Thirty (30) cc of fetal blood is neutralized by the 300 micrograms dose of RhoGAM. This is equivalent to 15 cc of fetal red blood cells. At 28-weeks gestation, 300 micrograms of Rh-immune globulin is routinely administered after testing for sensitization with an indirect Coombs' test. Administration is given following amniocentesis at any gestational age.

A 72-year-old G3P2 postmenopausal woman is referred by her internist after work-up for abdominal bloating revealed a large pelvic mass on transvaginal ultrasound and an elevated CA-125. The patient had a normal colonoscopy and mammography two months ago. The patient's greatest complaint is early satiety and upper abdominal discomfort. Her physical exam is notable for moderate abdominal distension and a significant fluid wave. Pelvic examination confirms a smooth, but fixed pelvic mass filling the cul de sac and extending to the umbilicus. Which of the following tests would be most helpful in assessing the extent of disease? A. Barium enema B. PET scan C. CT scan of abdomen and pelvis D. Chest X-ray E. Intravenous pyelogram

C. The most useful radiologic tool for evaluating the entire peritoneal cavity and the retroperitoneum is computerized tomography. Specifically in this patient, it would be important to look for significant involvement of the omentum. A chest x-ray provides adequate evaluation of the chest, unless it is abnormal. If there is a suspicion for chest involvement on the chest film, then a chest CT is necessary. With a normal colonoscopy and no symptoms suggestive of colonic obstruction, a barium enema would not be useful. PET scan, to date, has not been shown to play a role in the initial evaluation of women with a suspected ovarian malignancy. However, PET scan may play a role in evaluating women with a known diagnosis of ovarian cancer who have a suspected recurrence. An IVP would be useful if there was suspected ureteral obstruction, but otherwise is quite limited in assessing the entire abdominal/pelvic cavity.

A 47-year-old G2P2 woman comes to see you because she is concerned that she has uterine fibroids, as she recently gained about 20 pounds. Her mother had a hysterectomy for large fibroids that "made her look like she was 40 weeks pregnant." She has smoked one pack of cigarettes a day for the last 35 years and reports no other medical problems. She has normal menstrual cycles. Her weight is 216 pounds and she is 5 feet 4 inches tall (BMI 37). Her exam is extremely limited by her body habitus. A Beta-hCG is negative. A pelvic ultrasound shows a 4 cm intramural fibroid. What is the next best step in the management of this patient? A. Obtain a pelvic MRI B. Perform laparoscopic myomectomy C. Counsel her on diet and exercise D. Perform a hysterectomy E. Recommend bariatric surgery

C. The mostly likely cause of this patient's weight gain is excessive dietary intake and lack of exercise. She should be counseled on healthy habits and quitting smoking. The treatment of asymptomatic relatively small fibroids is not indicated. She does not qualify for bariatric surgery based on her BMI. Bariatric surgery may be considered when BMI is greater than 40, or is between 35 to 39.9 accompanied by a serious weight-related health problem, such as type 2 diabetes, high blood pressure or severe sleep apnea.

A 23-year-old G0 woman comes to the office to discuss contraception. Her past medical history is remarkable for hypothyroidism and mild hypertension. She has a history of slightly irregular menses. Her best friend recently got a "patch," so she is interested in using a transdermal system (patch). Her vital signs are: blood pressure 130/84; weight 210 pounds; height 5 feet 4 inches. What is the most compelling reason for her to use a different method of contraception? A. Age B. Hypothyroidism C. Weight D. Unpredictable periods E. Her blood pressure

C. The patch has comparable efficiency to the pill in comparative clinical trials, although it has more consistent use. It has a significantly higher failure rate when used in women who weigh more than 198 pounds. The patch is a transdermal system that is placed on a woman's upper arm or torso (except breasts). The patch (Ortho Evra®) slowly releases ethinyl estradiol and norelgestromin, which establishes steady serum levels for seven days. A woman should apply one patch in a different area each week for three weeks, then have a patch-free week, during which time she will have a withdrawal bleed.

A 42-year-old G5P4 woman is exclusively breastfeeding her two-month-old baby when she develops a fever and a red tender wedge-shaped area on the outer quadrant of her left breast. Which of the following is the most appropriate treatment for this condition? A. Cessation of breastfeeding for 48 hours B. Cessation of breastfeeding until afebrile C. Antibiotics D. Warm compresses E. Incision and drainage

C. The patient has a classic picture of mastitis that is usually caused by streptococcus bacteria from the baby's mouth. Mastitis is easily treated with antibiotics. The initial choice of antimicrobial is influenced by the current experience with staphylococcal infections at the institution. Most are community-acquired organisms, and even staphylococcal infections are usually sensitive to penicillin or a cephalosporin. If the infection persists, an abscess may ensue which would require incision and drainage. However, this patient's presentation is that of simple mastitis. There is no need for the mother to stop breastfeeding because of the mastitis.

A 22-year-old G4P1 woman at 26 weeks gestation presents complaining of a postcoital musty odor and increased milky, gray-white discharge for the last week. This was an unplanned pregnancy. She had her first pregnancy at age 15. She reports that she has no new sex partners, but the father of the baby may not be monogamous. On examination, there is a profuse discharge in the vaginal vault, which covers the cervix. Pertinent labs: wet mount pH >4.5 and whiff test positive. Microscopic exam reveals clue cells, but no trichomonads or hyphae. Which of the following is the most appropriate next step in the management of this patient? A. Delay treatment until postpartum B. Treat her now and again during labor C. Treat her now D. Treat her and her partner E. No treatment necessary

C. The patient has bacterial vaginosis. All symptomatic pregnant women should be tested and treatment should be not be delayed because treatment has reduced the incidence of preterm delivery. The optimal regimen for women during pregnancy is not known, but the oral metronidazole regimens are probably equally effective. Once treated antepartum, there is no need to treat during labor unless she is reinfected.

A 30-year-old G3P2 woman, whose last normal menstrual period was eight weeks ago, began spotting three days ago and developed cramping this morning. She has a history of a chlamydia infection with a previous pregnancy. She smokes one pack of cigarettes per day and denies alcohol or drug use. On physical exam: blood pressure 120/70; pulse 82; respirations 20; and temperature 98.6°F (37.0°C). Abdominal examination is normal. Pelvic examination reveals old blood in the vaginal vault, closed cervix without lesions, slightly enlarged uterus and no adnexal tenderness. Pertinent labs: quantitative Beta-hCG is 1000 mIU/ml; urinalysis normal; hematocrit = 32%. Transvaginal ultrasound shows no intrauterine pregnancy, no adnexal masses, and no free fluid in pelvis. Which of the following is the most appropriate next step in the management of this patient? A. Treat with methotrexate B. Exploratory surgery C. Repeat Beta-hCG in 48 hours D. Repeat Beta-hCG in one week E. Admit the patient to the hospital for observation

C. The patient has risk factors for ectopic pregnancy, but needs an accurate diagnosis before a treatment plan is entertained. Repeating the Beta-hCG is the next step in this patient's management. Inappropriately rising Beta-hCG levels (less than 50% increase in 48 hours) or levels that either do not fall following diagnostic dilation and curettage would be consistent with the diagnosis of ectopic pregnancy. Alternatively, a fetal pole must be visualized outside the uterus on ultrasound. The patient would need a Beta-hCG level over the discriminatory zone (the level where an intrauterine pregnancy can be seen on ultrasound) with an empty uterus. The level commonly used is 2000 mIU/ml. Treatment with methotrexate may be appropriate, but only after a definitive diagnosis is made. The patient does not yet have this level and is stable. She is, therefore, not a candidate for exploratory surgery. If she had unstable vital signs or an acute abdomen, a diagnostic laparoscopy/laparotomy would be indicated. Repeating the ultrasound in one week is not recommended because a delay in diagnosis could result in a ruptured ectopic pregnancy and increased risk to the patient. The patient is hemodynamically stable; therefore, she does not need to be admitted to the hospital.

A 29-year-old G0 woman presents due to the inability to conceive for the last year. Her cycles are regular every 28 days, but she has very painful periods, occasionally requiring that she miss work despite the use of non-steroidal anti-inflammatory drugs (NSAIDs). She also reports painful intercourse, which is becoming a problem as she now tries to avoid intercourse, even though she would like to conceive. She is otherwise in good health and has been married for five years. She is 5 feet 4 inches tall and weighs 130 pounds. She has a history of pelvic inflammatory disease at age 19, for which she was hospitalized. Her mother had a history of ovarian cancer at age 49. On physical examination, she has abdominal and pelvic lower quadrant tenderness. Uterus is normal in size, but there is a slightly tender palpable left adnexal mass. A pelvic ultrasound shows a 5 cm left complex ovarian cyst and two simple cysts measuring 2 cm in the right ovary. What best explains the underlying pathophysiology of the disease process in this patient? A. Chronic pelvic inflammatory disease B. Family history of ovarian cancer C. Endometrial glands outside the uterine cavity D. Polycystic ovarian syndrome E. Functional hemorrhagic cysts

C. The patient has typical signs of endometriosis which is characterized by the presence of endometrial glands and stroma outside of the uterus. Endometriosis is present in about 30% of infertile woman. She does not have the signs and symptoms of chronic pelvic inflammatory disease. She also does not have the signs and symptoms of polycystic ovarian syndrome, which typically presents with oligomenorrhea in overweight patients. The complex ovarian cyst is most likely an endometrioma (chocolate cyst). The duration of her symptoms makes functional hemorrhagic cyst a less likely option.

A 28-year-old G1P1 woman presents to your office. She delivered four weeks ago and tearfully reports that she is not sleeping, feels anxious and has thoughts of jumping out her 15th floor window. What is the most likely diagnosis? A. Postpartum anxiety B. Postpartum blues C. Postpartum depression D. Postpartum psychosis E. Bipolar disorder

C. The patient is describing symptoms of depression. Symptoms such as mood changes, insomnia, phobias and irritability are more pronounced than with the "blues." She has not described any of the even more advanced psychotic symptoms of visual or auditory hallucinations.

A 20-year-old G1 woman at 40 weeks gestation presents to labor and delivery complaining of painful contractions every 3-4 minutes for the last eight hours. Cervical examination on admission was 2 centimeters dilated, 90% effaced and 0 station. Three hours later, her exam is unchanged. The patient is still having contractions every 3-4 minutes. She is discouraged about her lack of progress. Which of the following is the most appropriate next step in the management of this patient? A. Laminaria placement B. Artificial rupture of membranes C. Counseling about latent phase of labor and rest D. Manual cervical dilation E. Cesarean section for arrest of labor

C. The patient is in the latent phase of labor and has not yet reached the active phase (more than 4 cm). A prolonged latent phase is defined as >20 hours for nulliparas and >14 hours for multiparas, and may be treated with rest or augmentation of labor. Artificial rupture of membranes is not recommended in the latent phase as it places the patient at increased risk of infection. Cervical dilation or laminaria placement are not indicated.

A 33-year-old G3P3 woman presents to the office complaining of a new onset vaginal discharge of four days duration. The discharge is thick and white. She has noted painful intercourse and itching since the discharge began. Her vital signs are: blood pressure 120/76 and pulse 78. The pelvic examination reveals excoriations on the perineum, thick white discharge, and is otherwise non-contributory. What is the most likely diagnosis in this patient? A. Herpes simplex virus B. Primary syphilis C. Candida vaginalis D. Bacterial vaginosis E. Trichomonas

C. The patient is most likely has candida vaginalis. Clinically women have itching and thick white cottage cheese like discharge. They may also have burning with urination and pain during intercourse. Herpes simplex viral infections are characterized by viral like symptoms preceding the appearance of vesicular genital lesions. A prodrome of burning or irritation may occur before the lesions appear. With primary infection, dysuria due to vulvar lesions can cause significant urinary retention requiring catheter drainage. Pain can be a very significant finding as well. Syphilis is a chronic infection caused by the Treponema pallidum bacterium. Transmission is usually by direct contact with an infectious lesion. Early syphilis includes the primary, secondary, and early latent stages during the first year after infection, while latent syphilis occurs after that and the patient usually has a normal physical exam with positive serology. In primary syphilis, a painless papule usually appears at the site of inoculation. This then ulcerates and forms the chancre, which is a classic sign of the disease. Left untreated, 25% of patients will develop the systemic symptoms of secondary syphilis, which include low-grade fever, malaise, headache, generalized lymphadenopathy, rash, anorexia, weight loss, and myalgias. Bacterial vaginosis is due to an overgrowth of anaerobic bacteria and characterized by a grayish / opaque foul-smelling discharge. Trichomonas is a protozoan and is transmitted via sexual contact. It typically presents with a non-specific yellow or greenish vaginal discharge. It does not have a systemic manifestation.

A 42-year-old G2P2 woman presents with chronic pelvic pain of two years duration. She describes the pain as constant ever since she underwent a laparoscopic-assisted vaginal hysterectomy for menorrhagia and dysmenorrhea. She did not have any evidence of endometriosis or obvious ovarian pathology at the time of surgery. During the postoperative period, she developed pelvic pain and fever, and was diagnosed with a pelvic/vaginal cuff abscess that was treated with antibiotics and percutaneous drainage. Her pain persisted in the subsequent months. Follow-up imaging over the next two years indicated transient ovarian cysts. Her abdominal examination is notable for mild-moderate tenderness across the lower quadrants, and her pelvic examination is notable for severe tenderness at the vaginal cuff with fullness noted in the midline. Which of the following is the most likely diagnosis in this patient? A. Endometriosis B. Pelvic inflammatory disease C. Pelvic adhesive disease D. Ovarian remnant syndrome E. Ovarian cancer

C. The patient most likely has pelvic adhesive disease as a result of her prior hysterectomy. The development of a postoperative pelvic infection likely has contributed to the further development of pelvic adhesions involving the tubes and ovaries that were retained. Although she did have a pelvic infection, it is unlikely that her pain resulted from classic PID since her adnexa appeared normal at the time of the hysterectomy. It is likely that her tubes and ovaries were affected by the postoperative infection, and as a result she may develop chronic pain from the adhesive disease and tubal damage from that acute infection. The cyclical nature of the ovarian cyst essentially rules out ovarian cancer. Ovarian remnant syndrome occurs following surgical removal of the ovaries, with subsequent development of cyclical pain due to ovarian tissue that was left behind inadvertently.

A 48-year-old G2P2 woman complains of progressively heavier and longer menstrual periods over the last year. Prior to this year the patient had normal periods. She denies any symptoms other than fatigue over the last few months. Physical examination is unremarkable except for the pelvic examination. The patient is noted to have an irregularly shaped 16-week size uterus. The patient's hematocrit is 28%. What is this patient's most likely diagnosis? A. Endometrial hyperplasia B. Endometrial carcinoma C. Uterine fibroids D. Uterine leiomyosarcoma E. Adenomyosis

C. The patient's history and physical examination is typical for a perimenopausal woman with probable uterine fibroids. Although it is possible that she could have underlying endometrial hyperplasia, the most likely diagnosis is uterine fibroids. Uterine leiomyosarcoma should be considered in a postmenopausal woman with bleeding, pelvic pain coupled with uterine enlargement, and vaginal discharge, but it is exceedingly rare. Endometrial hyperplasia is more common in perimenopausal women who do not ovulate regularly and postmenopausal women. Endometrial carcinoma is typically a disease of postmenopausal women, although 5-10% of cases occur in women who are menstruating and 10-15% of cases occur in perimenopausal women. For this reason, she should still undergo an endometrial biopsy. Adenomyosis may result in a symmetrically enlarged "boggy" uterus, but usually presents with dysmenorrhea in addition to menorrhagia.

A 29-year-old G3P0 woman presents for evaluation and treatment of pregnancy loss. Her past medical history is remarkable for three early (< 10 weeks gestation) pregnancy losses and a deep vein thrombosis two years ago. Her work up includes: prolonged dilute Russell viper venom test, elevated anticardiolipin antibodies, normal thyroid function, normal prolactin, and normal MRI of the pelvis. She wishes to get pregnant soon. In addition to aspirin, which of the following treatments is appropriate for this patient? A. No additional treatment B. Corticosteroid C. Heparin D. 17-OH progesterone E. Bromocriptine

C. The prolonged dilute Russell viper venom time, history of three early pregnancy losses, and a history of venous thrombosis leads one to suspect that the etiology of recurrent pregnancy loss is due to antiphospholipid antibody syndrome. The treatment is aspirin plus heparin. There is roughly a 75% success rate with combination therapy versus aspirin alone. There is conflicting evidence regarding steroid use for treatment. 17-OH progesterone is used for the prevention of preterm delivery and not recurrent pregnancy loss. Bromocriptine would be indicated for hyperprolactinemia.

A 45-year-old G5P5 premenopausal woman was initially seen in your office for work-up and evaluation of a FIGO grade 3 endometrial cancer that was diagnosed by her gynecologist. Which of the following is the most appropriate treatment for this patient? A. Chemotherapy B. Radiation treatment C. Total abdominal hysterectomy, bilateral salpingo-oophorectomy, bilateral pelvic and para-aortic lymphadenectomy D. Supracervical abdominal hysterectomy with ovarian preservation E. Medroxprogesterone (Megace)

C. The recommended components of the surgical approach to an early endometrial cancer are the extrafascial total abdominal hysterectomy, bilateral salpingo-oophorectomy, and pelvic and para-aortic lymphadenectomy. Alternative surgical approaches to early endometrial cancer include a total vaginal hysterectomy with or without a bilateral salpingo-oophorectomy in women who are medically unstable or have contraindications to major abdominal surgery. Ideally, this approach would only be utilized in patients with well-differentiated endometrioid adenocarcinomas and avoided in patients with high-grade lesions or aggressive cell types, such as clear cell or papillary serous carcinomas. A total laparoscopic hysterectomy, BSO, with or without staging is being utilized more and more in lieu of the traditional open approach for select patients in many centers, and is a reasonable alternative. Although chemotherapy, radiation, and hormonal therapy may be utilized, it is usually in an adjuvant setting.

A 16-year-old G1P0 African-American female presents at eight weeks gestation for prenatal care. She reports occasional spotting but denies pain or fever. The laboratory reports hemoglobin of 8 g/dL and a peripheral smear reveals hypochromia and microcytosis. Which of the following is the most likely diagnosis for this patient? A. Sickle cell anemia B. Folate deficiency C. Iron deficiency D. β-thalassemia E. Acute blood loss

C. The two most common causes of anemia during pregnancy and the puerperium are iron deficiency and acute blood loss. Classical morphological evidence of iron-deficiency anemia includes erythrocyte hypochromia and microcytosis. Serum ferritin levels are lower than normal and there is no stainable bone marrow iron on examination of a bone marrow aspirate. The spotting she reports would not lead to anemia due to blood loss

A 33-year-old G3P1 woman presents with left lower quadrant pain of two days duration and seven weeks of amenorrhea. She describes her pain as mild and intermittent. Past medical history is significant for smoking during her teens, ectopic pregnancy with salpingostomy four years ago, multiple Chlamydia infections in her teens, and an uncomplicated Cesarean delivery for breech presentation. Which of the following risk factors is most likely associated with recurrent ectopic pregnancy in this patient? A. Age B. History of chlamydia infections C. History of ectopic pregnancy D. Prior Cesarean delivery E. Interval between pregnancies

C. There is a tenfold increase risk for ectopic pregnancy in women with a prior history of ectopic pregnancy. Age between 35 and 44 years old is associated with a threefold increase in ectopic pregnancy. Prior abdominal surgery and history of sexually transmitted infections as well as sterilization failures, endometriosis and congenital uterine malformations are all associated with an increased risk of ectopic pregnancy. The interval between pregnancies and past smoking history is not associated with an increased risk of ectopic pregnancy.

An 18-year-old G1P0 woman presents for prenatal care at 6 weeks gestation. Her medical, surgical, gynecologic, social and family history are unremarkable. Her dietary history includes high carbohydrate intake with no fresh vegetables. Her physical examination is within normal limits except that she is pale and has a BMI of 42. Nutritional counseling should include the following: A. 25-30 grams of protein in her diet every day B. A strict diet to maintain her current weight C. Folic acid supplementation D. Intake of 1200 calories a day E. Initiation of a vigorous weight loss exercise program

C. There should be folic acid supplementation, as well as evaluation for deficiencies in her iron, protein and other nutrient stores. In general, a patient needs approximately 70 grams of protein a day, along with her other nutrients. It would be prudent to caution her that, though aerobic exercise is recommended and would be a benefit to her, it is not advisable to initiate a vigorous program in a woman who has not been routinely working out. Women should gain weight during their pregnancy, and 1200 calories a day is not sufficient for a pregnant woman.

A 29-year-old G1P0 woman is at 11 weeks gestation. She has a history of depression which has been well controlled with fluoxetine (Prozac). Although the medication is very helpful in controlling her depression, she is concerned about potential side effects on her neonate. Which of the following conditions in the neonate is associated with maternal use of Fluoxetine during pregnancy? A. Necrotizing enterocolitis B. Intracranial hemorrhage C. Agitation and poor feeding D. Temperature instability E. Persistent pulmonary hypertension

C. Third trimester maternal use of SSRIs including Fluoxetine has been associated with abnormal muscle movements (extrapyramidal signs or EPS) and withdrawal symptoms which may include agitation, abnormally increased or decreased muscle tone, tremor, sleepiness, severe difficulty breathing, and difficulty in feeding. In some newborns, the symptoms subside within hours or days and do not require specific treatment; other newborns may require longer hospital stays. SSRI use during pregnancy is not associated with newborn seizures, intracranial hemorrhage or temperature instability. The FDA has concluded that, given the conflicting results from different studies, it is premature to reach any conclusion about a possible link between SSRI use in pregnancy and persistent pulmonary hypertension.

A 32-year-old G1 is at 36 weeks gestation. Ultrasound reveals limited fetal growth over the past three weeks. Biometry is consistent with 30-5/7, EFW 1900 g, less than 10th percentile. Which of the following is LEAST likely to be associated with this pregnancy? A. Fetal demise B. Perinatal demise C. Polyhydramnios D. Meconium aspiration E. Polycythemia

C. This fetus had intrauterine growth restriction and, with the exception of polyhydramnios, all of the morbidities listed above may complicate intrauterine growth restriction. In general, the causes of polyhydramnios relate to amniotic fluid production (abnormalities of the fetal urinary tract) and removal (abnormalities of fetal swallowing and intestinal reabsorption of fluid). Some investigators report an increase in fetal urinary output when there is hyperglycemia and increased renal osmotic load, thus resulting in polyhydramnios. Abnormal fetal swallowing may be a result of a CNS or gastrointestinal tract abnormalities, such as anencephaly, esophageal or duodenal atresia, diaphragmatic hernia or primary muscular disease. Typically, polyhydramnios is not associated with asymmetric growth restriction (the most common form of IUGR), since an asymmetric growth pattern reflects poor uterine blood flow and limited substrate availability. In fact, oligohydramnios is frequently identified in pregnancies complicated by fetal growth restriction.

A 42-year-old G2P2 woman presents to the office because she recently noticed a pigmented lesion on her vulva. She does not know how long it has been there and it doesn't bother her except that she is worried that she may have warts. Her screening Paps have been negative. Her prior exams have been reported as normal. She is a nonsmoker. Examination of her vulva reveals a pigmented, flat lesion, approximately 1.5 cm in largest diameter at the base of the right labia. It is non-tender. No induration is present. Her groin examination reveals no adenopathy. Her vagina and cervix are well estrogenized and without obvious lesions. Which of the following is the most likely diagnosis? A. Vulvar condyloma B. Squamous cell carcinoma C. Melanoma in situ D. Paget's disease E. Lichen sclerosus

C. This lesion may be melanoma and a biopsy must be done to exclude this diagnosis. The concerning features are the size and irregularity of the lesion. Melanoma represents 5% of vulvar cancer, which is not insignificant given the lack of sun exposure and the relatively small surface area. There is no variability in the coloration, ulceration or thickening of the lesion to suggest malignancy at this time. Squamous cell carcinoma is typically not pigmented. Paget's disease is usually erythematous with a lacy white mottling of the surface. Condyloma lesions have a characteristic verrucous appearance. With lichen sclerosus, the skin appears thin, inelastic and white, with a "crinkled tissue paper" appearance. Although not listed as an option, the most likely diagnosis is high-grade vulvar intraepithelial neoplasia, which can have the same clinical appearance.

A 74-year-old G0 woman complains of vulvar pain. She reports that the pain is present every day and she has had it for the past year. It now limits her ability to exercise, and she is no longer able to have sexual relations with her partner. On exam, her BMI is 32; blood pressure is 100/60; and heart rate is 77. Her vulva has an ulcerated lesion near the left labial edge. Which of the following is the next best step in the management of this patient? A. Estrogen cream B. Clobetasol cream C. Vulva biopsy D. Laser vaporization of the lesion E. Vulvectomy

C. This patient has a vulvar lesion causing her pain. The next step is to perform a biopsy to evaluate for vulvar cancer. Estrogen cream and clobetasol (a high potency steroid) are treatments for vulvadynia. To diagnosis vulvadynia, all other causes of pain must first be excluded, including infectious etiologies as well as other vulvar conditions. Laser vaporization and vulvectomy are contraindicated until a definitive diagnosis is made.

A 41-year-old G2P2 woman presents with menstrual pain, menorrhagia, irregular periods and intermenstrual bleeding. She describes the pain as pressure and cramps. Ibuprofen improves the pain, but does not entirely eliminate the discomfort. Pelvic examination reveals a 14-week size uterus with irregular masses within the uterus. Pelvic ultrasound confirms the diagnosis of fibroids. What is the most appropriate next step in the management of this patient? A. CA125 assay B. CT scan of the pelvis C. Endometrial biopsy D. GnRH agonist E. Hysterectomy

C. This patient has classic symptoms of leiomyomata, including menorrhagia. An endometrial biopsy should be performed on all women over age 40 with irregular bleeding to rule out endometrial carcinoma. The CA125 assay measures the level of CA125 in the blood and is increased in some types of cancer, including ovarian cancer or other conditions. This non-specific marker is not indicated in this patient. A CT scan of the pelvis is also not indicated. A simple pelvic ultrasound could be used to help confirm the clinical diagnosis. GnRH agonist and hysterectomy are not used until the diagnosis of leiomyomata uteri is confirmed. Her desire for future fertility should be discussed.

A 37-year-old G2P1011 woman comes to the clinic with her husband due to the inability to conceive for the last year. She reports being in good health and not having problems with her prior pregnancy two years ago, except for some postpartum depression for which she was placed on imipramine and which she continues to take. She took birth control pills after her pregnancy and stopped one year ago, when she began trying to conceive. Her periods were regular on the pills, but have been irregular since she stopped taking them. She has no history of sexually transmitted infections or abnormal Pap smears. Her husband is also healthy and he fathered their first child. Her physical examination is completely normal. Laboratory tests show: Results Normal Values TSH 2.1 mIU/ml 0.5-4.0 mIU/ml Free T4 1.1 ng/dl 0.8-1.8 ng/dl Prolactin 60 ng/ml <20 ng/ml FSH 6 mIU/ml 5-25 mIU/ml LH 4 mIU/ml 5-25 mIU/ml What is the most appropriate next step in the management of this patient's subfertility? A. Begin bromocriptine B. Ovulation induction with clomiphene citrate (Clomid) C. Wean off imipramine D. Perform a visual field examination E. Obtain a brain MRI

C. This patient has hyperprolactinemia due to imipramine. The patient has to be weaned off imipramine (instead of abrupt discontinuation to minimize withdrawal symptoms) and placed on a more appropriate medication. Once she is off imipramine and the cause of her elevated prolactin levels is confirmed, her normal menses should resume. Although MRI of the brain would be a reasonable step, it would be premature, and visual field examination does not aid in the diagnostic work-up. It would be premature to obtain an MRI or begin bromocriptine without this intermediate step. An endometrial biopsy is not indicated at this point, especially since the patient had normal cycles on OCPs. Although Clomid is used to help with ovulatory dysfunction, the hyperprolactinemia must be addressed first.

A 27-year-old G1 woman at 36 weeks gestation is undergoing an induction of labor for preeclampsia with severe features. She complains of a headache, right upper quadrant pain and seeing spots. Admission vital signs are: blood pressure 180/120, respiratory rate 20, pulse 92. In addition to 10 hours of oxytocin, she is receiving intravenous magnesium sulfate 2 g /hour. During the past two hours her urine output has decreased to 15 mL per hour (down from 40 mL/h) and her respiratory rate is now 10. What is the next best step in the management of this patient? A. Continue magnesium sulfate B. Decrease magnesium sulfate to 1 gram/hour C. Administer calcium gluconate D. Increase oxytocin drip E. Cesarean delivery now

C. This patient has respiratory depression likely secondary to magnesium toxicity. In addition to discontinuing the magnesium sulfate, she needs a dose of calcium gluconate to restore her respiratory function. The classic signs of magnesium toxicity include muscle weakness and loss of deep tendon reflexes, nausea, and respiratory depression. If magnesium is given in high doses, cardiac arrest is possible. Altering the oxytocin drip and a Cesarean delivery are not indicated at this time.

A 30-year-old G2P1 woman at 38 weeks gestation presents to labor and delivery with contractions every 2-3 minutes. Her membranes are intact. Her cervical examination is 5 centimeters dilated, 100% effaced, and -1 station. The fetal heart rate tracing is Category I. Two hours later, she progresses to 7 cm and 0 station and receives an epidural for pain. Four hours after that, her exam is unchanged (7/100/0). Fetal heart rate tracing remains Category I. Which of the following is the most appropriate next step in the management of this patient? A. Allow her to ambulate and return when she is ready to push B. Perform a contraction stress test C. Perform an amniotomy D. Perform a Cesarean delivery E. Place an internal fetal scalp electrode

C. This patient has secondary arrest of dilation, as she has not had any further cervical change in the active phase for over four hours. Amniotomy is often recommended in this situation. After it is performed, if the patient is still not in an adequate contraction pattern, augmentation with oxytocin can be attempted after careful evaluation. Although the patient requires close monitoring, it is too early to proceed with a Cesarean delivery. An internal scalp electrode is not necessary, since the fetal heart monitoring is reassuring.

A 32-year-old G0 woman presents with a one-month history of profuse vaginal discharge with mild odor. She has a new sexual partner with whom she has had unprotected intercourse. She reports mild to moderate irritation, pruritus and pain. She thought she had a yeast infection, but had no improvement after using an over-the-counter antifungal cream. She is concerned about sexually transmitted infections. Her medical history is significant for lupus and chronic steroid use. Pelvic examination shows normal external genitalia, an erythematous vagina with a copious, frothy yellow discharge and multiple petechiae on the cervix. Vaginal pH is 7. Which of the following findings on a wet prep explains the etiology of this condition? A. Hyphae B. Clue cells C. Trichomonads D. Lactobacilli E. Normal epithelial cells

C. This patient has signs and symptoms of trichomoniasis, which is caused by the protozoan, T. vaginalis. Many infected women have symptoms characterized by a diffuse, malodorous, yellow-green discharge with vulvar irritation. However, some women have minimal or no symptoms. Diagnosis of vaginal trichomoniasis is performed by saline microscopy of vaginal secretions, but this method has a sensitivity of only 60% to 70%. The CDC recommended treatment is metronidazole 2 grams orally in a single dose. An alternate regimen is metronidazole 500mg orally twice daily for seven days. The patient's sexual partner also should undergo treatment prior to resuming sexual relations.

An 18-year-old G1 woman presents for prenatal care at 16 weeks gestation without complaints. The patient denies any history of sexually transmitted disease, although admits to a history of multiple sex partners, with irregular use of condoms. She is allergic to penicillin, which causes anaphylaxis. Physical exam is unremarkable. Pertinent labs: rapid plasma reagin test (RPR) positive (titer = 32); fluorescent treponemal antibody absorption test (FTA-ABS) is positive. Which of the following is the best treatment for this patient? A. Oral erythromycin B. Oral doxycycline C. Desensitization and penicillin D. Intravenous erythromycin E. Intravenous cefazolin

C. This patient has syphilis, and the fluorescent treponemal antibody absorption test (FTA-ABS) confirms the diagnosis. The transmission rates for primary and secondary disease are approximately 50-80%. There are no proven alternatives to penicillin therapy during pregnancy and penicillin G is the therapy of choice to treat syphilis in pregnancy. Women with a history of penicillin allergy can be skin tested to confirm the risk of immunoglobulin E (IgE)-mediated anaphylaxis. If skin tests are reactive, penicillin desensitization is recommended and is followed by intramuscular benzathine penicillin G treatment. Erythromycin has an 11% failure rate. Doxycycline is contraindicated in pregnancy. Cefazolin is commonly used to treat urinary tract infections and is not effective in the treatment of syphillis.

A 27-year-old G0 woman presents with a one-year history of dysmenorrhea and dyspareunia. Pain, when present, is 7/10 in strength and requires that she miss work. She now avoids intercourse and no longer finds it pleasurable. She is otherwise in good health. Her last menstrual period was 17 days ago and her menses are typically 28 days apart. She had chlamydia once, at age 19. Physical examination is notable for mild tenderness on abdominal examination in the lower quadrants, and bilateral adnexal tenderness on pelvic examination. Uterus is normal in size and there is uterosacral ligament nodularity. What is the most likely diagnosis in this patient? A. Adenomyosis B. Chronic pelvic inflammatory disease C. Endometriosis D. Endometritis E. Leiomyoma

C. This patient has typical symptoms of endometriosis, including dysmenorrhea and dyspareunia. In addition, the nodularity along the back of the uterus along the uterosacral ligaments is suggestive of endometriosis. Chronic pelvic inflammatory disease would not present this far out from a known infection. Adenomyosis is endometrial glands embedded in the wall of the uterus. Endometritis is an infection of the endometrium. Premenstrual dysphoric disorder (PMDD) is a condition in which a woman has severe depressive symptoms, irritability, and tension before menstruation.

A 57-year-old G2P2 woman presents with a six-month history of urinary incontinence, urgency, and nocturia. She describes the amount of urine loss as large and lasting for several seconds. The urine loss occurs when she is standing or sitting and is not associated with any specific activity. A post-void residual is 50cc. What is the most likely cause of this patient's symptoms? A. Stress incontinence B. Overflow incontinence C. Urge incontinence D. Mixed incontinence E. Vesicovaginal fistula

C. This patient has urge incontinence, which is caused by overactivity of the detrusor muscle resulting in uninhibited contractions, which cause an increase in the bladder pressure over urethral pressure resulting in urine leakage. Stress incontinence is caused by an increase in intra-abdominal pressure (coughing, sneezing) when the patient is in the upright position. This increase in pressure is transmitted to the bladder that then rises above the intra-urethral pressure causing urine loss. Associated structural defects are cystocele or urethrocele. Overflow incontinence is associated with symptoms of pressure, fullness, and frequency, and is usually a small amount of continuous leaking. It is not associated with any positional changes or associated events. Mixed incontinence occurs when increased intra-abdominal pressure causes the urethral-vesical junction to descend causing the detrusor muscle to contract. A vesicovaginal fistula typically results in continuous loss of urine.

A 40-year-old G6P5 woman delivered a 5020 grams live born male infant vaginally 20 minutes ago. The nurse notes that the patient is having ongoing bleeding that she estimates to be 1000 cc. Her past medical history is significant for chronic hypertension and asthma. She has received oxytocin and misoprostol. Her examination reveals a boggy uterus. Which of the following is the next step in management of her bleeding? A. Uterine artery embolization B. B-Lynch compression stitch C. Bakri balloon placement D. Uterine artery ligation E. Hysterectomy

C. This patient has uterine atony, which accounts for about 80-90% of all postpartum hemorrhages. Risk factors for PPH include uterine over distension (polyhydramnios, macrosomia, and multiple gestation), prolonged labor, chorioamnionitis, and grandmultiparity. This patient has already received uterotonic agents and while additional agents are available, because of her hypertension and asthma both methergine and prostaglandin F2 alpha are contraindicated. All of the choices listed are treatments, the least invasive treatment is placement of the Bakri balloon, which is a device placed into the uterus with a balloon that is filled with up to 500 cc of sterile fluid. This places pressure on the inside of the uterus. Uterine artery ligation, B-lynch compression stitch and hysterectomy all require a laparotomy and should be reserved for recalcitrant cases. Uterine artery embolization requires placement of embolization catheters as well as interventional radiology.

A 22-year-old G1P0 woman presents to the emergency department at eight weeks gestation experiencing heavy vaginal bleeding. On physical exam: blood pressure 94/60; pulse 108; respirations 20; and temperature 98.6°F (37.0°C). Pelvic examination demonstrates brisk bleeding through a dilated cervical os. The patient's hemoglobin is 7 g/dL (hematocrit 21%). Which of the following is the most appropriate next step in the management of this patient? A. Administration of intravaginal misoprostol B. Administration of oral misoprostol C. Dilation and suction curettage D. Endometrial ablation E. Expectant care to permit spontaneous abortion

C. This patient is actively bleeding and is anemic. She, therefore, requires immediate surgical treatment consisting of dilation and suction curettage. Although clinicians increasingly utilize both expectant management and various drug regimens to treat spontaneous abortion, a prerequisite for either is that the patient is hemodynamically stable and reliable for follow-up care. She is not hemodynamically stable. Endometrial ablation will not work in this case, as the products of conception need to be evacuated to control the bleeding.

A 23-year-old G2P1 woman at 36 weeks gestation presents with her third episode of heavy vaginal bleeding. She has normal prenatal labs and a known placenta previa. She denies uterine contractions or abdominal pain and reports good fetal movement. Her vital signs are: blood pressure 100/60; pulse 110; and she is afebrile. Her abdomen and uterus are non-tender. Fundal height measures 35 centimeters and fetal heart tones reveal a baseline of 140 and are reassuring. Pelvic ultrasound confirms a placenta previa and the fetus is in the cephalic presentation. Hematocrit is 29%. Which of the following is the most appropriate next step in the management of this patient? A. Tocolysis B. Induction of labor C. Cesarean delivery D. Amniocentesis E. Administer steroids

C. This patient is near term with a third episode of active bleeding from a placenta previa. The appropriate next step would be to move towards delivery via Cesarean section. The patient is not experiencing contractions, so tocolysis is not necessary and would not be used with heavy vaginal bleeding. Catastrophic bleeding could occur due to disruption of blood vessels as the cervix dilates if a vaginal delivery is pursued, and induction of labor would therefore be contraindicated. An amniocentesis is not indicated in this situation. Although the patient is not yet at term, delivery is appropriate due to the third episode of heaving bleeding at near term. Administering steroids is not appropriate at this gestational age.

A 39-year-old G1P0 woman at 37 weeks gestation is being induced secondary to polyhydramnios. She has received cervical ripening with prostaglandin E2, and is now on oxytocin. Her cervix on last check was 3cm, 75% effaced, -4 station. Her water just broke, and the nurse reports that the patient is now contracting every minute and notes that she is having a large amount of bleeding. On exam, her blood pressure is 100/80; pulse 100; temperature of 100.0°F (37.8°C); and she has a Category II tracing. Her cervical exam is 10cm, 100% effaced, +2 station. Which of the following is the most likely explanation for her bleeding and labor progression? A. Oxytocin B. Rupture of membranes C. Abruptio placentae D. Chorioamnionitis E. Normal labor progression

C. This patient likely is experiencing a placenta abruption. Her biggest risk factor is polyhydramnios with rapid decompression of the intrauterine cavity. While oxytocin is used to augment labor, in a nulliparous patient, the difference between her two exams is extreme. Typically, normal labor progresses about 1 cm per hour in the active phase of labor (multiparous woman about 1-2 cm/hour). While there may be some vaginal bleeding (bloody show, or light bleeding with cervical dilation) it is not normal to have a large amount of bleeding. This patient has no signs or symptoms of chorioamnionitis.

A 38-year-old G0 woman comes to the office because she noted a persistent yellow, frothy discharge associated with mild external vulvar irritation. She denies any odor. She tried over the counter anti-fungal medication without success. The discharge has been present for over three months, gradually increasing in amount. Douching has resulted in temporary relief, but the symptoms always recur. Pelvic examination reveals mild erythema at the introitus and a copious yellow frothy discharge fills the vagina. The cervix has erythematous patches on the ectocervix. A sample of the discharge is examined under the microscope. What is the most likely finding? A. Strong amine fishy odor when KOH applied to sample B. Marked polymorphonuclear cells with multi-nucleate giant cells C. Motile ovoid protozoa with flagella D. Budding yeast and pseudo-hyphae E. Clue cells

C. This patient most likely has trichomoniasis. The erythematous patches on the cervix are characteristic of "strawberry cervicitis." Trichomonads are unicellular protozoans, which are easily seen moving across the slide with flagella. The slide must be examined immediately. The discharge is mixed with saline and placed on the slide with a cover slip. Women with trichomonas vaginal infections may have a frothy, yellow-green vaginal discharge. Clue cells are seen on a saline wet mount in women who have bacterial vaginosis. Clue cells are characterized by adherent coccobacillary bacteria that obscure the edges of the cells. A drop of KOH releases amines from the cells and a fishy odor is noted if bacterial vaginosis is present. Yeast vaginitis is characterized by a thick white clumpy discharge which results in erythema, swelling and intense pruritus. Multinucleate giant cells and inflammation may be herpes.

A 45-year-old G3P3 woman comes to the office because she has been unable to conceive for the last two years. She is healthy and has three children, ages 10, 12 and 14, whom she conceived with her husband. She used a copper IUD after the birth of her last child and had it removed two years ago, hoping to have another child. She has no history of sexually transmitted infections or abnormal Pap smears. Her cycles are regular every 28 to 32 days. She is not taking any medications. She has been married for the last 16 years, and her husband is 52-years-old and in good health. Her physical examination, including a pelvic exam, is completely normal. Which of the following is the most appropriate next step in the management of this patient? A. Hysteroscopy B. Hysterosalpingogram C. Clomiphene challenge test D. Sperm penetration assay E. Basal body temperatures for six months

C. This patient, most likely, has decreased ovarian reserve due to her age. A clomiphene challenge test, which consists of giving clomiphene citrate days five to nine of the menstrual cycle and checking FSH levels on day three and day 10, will help determine ovarian reserve. This will help counsel the patient on appropriate options to have a child, as most women will not be able to conceive at this age and would not be good candidates for ovarian stimulation or IVF. Even though this patient had an IUD in the past, there is no reason to believe that this contributed to her inability to conceive, as IUDs do not cause infertility and she has no risk factors for tubal disease. Having her keep basal body temperatures for six months would be a waste of time for this patient, who is already 45. A semen analysis or sperm penetration assay is not necessary as a first step as she was able to conceive from her husband previously without problems. Most likely this patient will have to use a donor egg if she wants to carry the pregnancy herself.

A 38-year-old G3P3 woman presents to the office because she has noted dark spots on her vulva. She states that the lesions have been present for at least two years and are occasionally itchy. She has a history of laser therapy for cervical intraepithelial neoplasia ten years ago, and has not had a pelvic exam since then. She has had multiple partners and uses condoms. Her menses are regular and she had a tubal ligation. She has a history of genital herpes, but has only one or two recurrences a year. She has smoked since age 14. On examination, multicentric brown-pigmented papules are noted on the perineum, perianal region and labia minora. No induration or groin adenopathy is noted. The vagina and cervix are normal in appearance. Which of the following is the most likely diagnosis? A. Hidradenitis suppurativa B. Molluscum contagiosum C. Vulvar intraepithelial neoplasia D. Melanoma E. Paget's disease

C. This presentation is classic for human papilloma virus (HPV) related vulvar intraepithelial neoplasia. Melanoma would be unlikely to be multifocal and warts have a characteristic verrucous appearance, although pigmentation can occur. Molluscum, a poxvirus, is characterized by multiple shiny non-pigmented papules with a central umbilication. Paget's disease, although multicentric, does not have brown pigmentation. Hidradenitis is a chronic, unrelenting skin infection causing deep, painful scars and foul discharge.

A 7-year-old female is undergoing evaluation for vaginal bleeding. On physical examination, she has Tanner stage III breasts, tall stature and an otherwise normal examination. An MRI of the brain and a pelvic ultrasound are normal. LH and FSH levels are in the pubertal levels and she has normal DHEAS and androgen levels. What is the most likely diagnosis in this patient? A. Pituitary adenoma B. Congenital adrenal hyperplasia C. True precocious puberty D. McCune Albright Syndrome E. Ovarian neoplasm

C. True precocious puberty is a diagnosis of exclusion where the sex steroids are increased by the hypothalamic-pituitary-gonadal axis, with increased pulsatile GnRH secretion. CNS abnormalities associated with precocious puberty include the following: tumors (e.g., astrocytomas, gliomas, germ cell tumors secreting human chorionic gonadotropin [hCG]); hypothalamic hamartomas; acquired CNS injury caused by inflammation, surgery, trauma, radiation therapy, or abscess; or congenital anomalies (e.g. hydrocephalus, arachnoid cysts, suprasellar cysts). These conditions are not likely in the presence of a normal work-up in this patient. Congenital adrenal hyperplasia usually presents in the neonatal period and is associated with ambiguous genitalia. McCune Albright Syndrome is characterized by premature menses before breast and pubic hair development. An ovarian neoplasm is unlikely with a normal pelvic ultrasound.

A 29-year-old G4P2 woman with no previous prenatal care presents at 24 weeks gestation with signs and symptoms of preterm labor. Her cervix is 3 cm dilated and 80% effaced. Fundal height is 30 cm and an ultrasound examination reveals a twin gestation. Estimated fetal weights on the twins are 850 gm and 430 gm. The maximum vertical amniotic fluid pocket around the smaller twin is 1 cm; the maximum vertical amniotic fluid pocket around the larger twin is 8 cm. Which of the following is the most likely associated with these ultrasound findings? A. Dichorionic diamniotic twins B. Monochorionic monoamniotic twins C. Monochorionic diamniotic twins D. Superfecundation E. Rh-isoimmunization

C. Twin-twin transfusion syndrome is the result of an intrauterine blood transfusion from one twin to the other. It most commonly occurs in monochorionic, diamniotic twins. The donor twin is often smaller and anemic at birth. The recipient twin is usually larger and plethoric at birth. Clues to the presence of the twin-twin transfusion syndrome include the large weight discordance (although this is not necessary for diagnosis), polyhydramnios around the larger (recipient) twin, and oligohydramnios around the smaller (pump) twin. The two different placental types in twin gestation are monochorionic and dichorionic. Monozygotic conceptions may have either monochorionic or dichorionic placentation, depending upon the time of division of the zygote. Dizygotic conceptions always have dichorionic placentas. Diamniotic dichorionic placentation occurs with division prior to the morula state (within three days post fertilization). Diamniotic monochorionic placentation occurs with division between days four and eight post fertilization. Monoamniotic, monochorionic placentation occurs with division between days eight and 12 post fertilization. Superfecundation is the fertilization of two different ova at two separate acts of intercourse in the same cycle. Isoimmunization is associated with polyhydramnios and fetal hydrops and does not cause twin-twin transfusion.

A 25-year-old G1P0 woman at 29 weeks gestation presents to labor and delivery complaining of contractions every 3 minutes. On exam, her blood pressure is 120/70; heart rate is 100; temperature is 100.7°F (38.2°C); and fetal heart tones are in the 160's. The fetal heart rate tracing is showing deep variable decelerations, but is otherwise is reassuring. Her pelvic examination reveals gross rupture of membranes and her cervix is dilated to 6 cm and ultrasound reveals the baby is in the breech presentation. What of the following explains the variable decelerations? A. Chorioamnionitis B. Labor C. Premature Rupture of Membranes D. Breech presentation E. Gestational age

C. Variable decelerations are a result of cord compression. With cord compression, initially there is a short increase in fetal heart rate followed by an abrupt drop in heart rate. As the compression is released, the fetal heart rate returns to baseline, frequently with a brief, slight increase. Variable decelerations can occur at any point. This patient has PPROM and a frequent cause of cord compression can be lack of amniotic fluid. Chorioamnionitis is an infection of the amniotic fluid and chorion that explains this patient's fever, but is not a cause of variable decelerations. Breech presentation, and gestational age do not cause variable decelerations.

A 37-year-old G0 woman presents with a one-week history of a mildly painful vulvar ulcer. She reports no fevers, malaise or other systemic symptoms. She recently started use of a topical steroid ointment for a vulvar contact dermatitis. She is married and has no prior history of sexually transmitted infections. She reports no travel outside the United States by her husband or herself. Her last Pap smear, six months ago, was normal. A vulvar herpes culture later returns positive for herpes simplex virus type 2. A Rapid Plasma Reagin (RPR) is nonreactive, and HIV testing is negative. Which of the following is the most likely diagnosis in this patient? A. Primary HSV episode B. Recurrent HSV-1 episode C. Recurrent HSV-2 episode D. Atypical HSV episode E. Contact dermatitis

C. Two serotypes of HSV have been identified: HSV-1 and HSV-2. Most cases of recurrent genital herpes are caused by HSV-2. Up to 30% of first-episode cases of genital herpes are caused by HSV-1, but recurrences are much less frequent for genital HSV-1 infection than genital HSV-2 infection. Genital HSV infections are classified as initial primary, initial nonprimary, recurrent and asymptomatic. Initial, or first-episode primary genital herpes is a true primary infection (i.e. no history of previous genital herpetic lesions, and seronegative for HSV antibodies). Systemic symptoms of a primary infection include fever, headache, malaise and myalgias, and usually precede the onset of genital lesions. Vulvar lesions begin as tender grouped vesicles that progress into exquisitely tender, superficial, small ulcerations on an erythematous base. Initial, nonprimary genital herpes is the first recognized episode of genital herpes in individuals who are seropositive for HSV antibodies. Prior HSV-1 infection confers partial immunity to HSV-2 infection and thereby lessens the severity of type 2 infection. The severity and duration of symptoms are intermediate between primary and recurrent disease, with individuals experiencing less pain, fewer lesions, more rapid resolution of clinical lesions and shorter duration of viral shedding. Systemic symptoms are rare. Recurrent episodes involve reactivation of latent genital infection, most commonly with HSV-2, and are marked by episodic prodromal symptoms and outbreaks of lesions at varying intervals and of variable severity. Clinical diagnosis of genital herpes should be confirmed by viral culture, antigen detection or serologic tests. Treatment consists of antiviral therapy with acyclovir, famciclovir or valacyclovir.

A 28-year-old G3P3 woman experiences profuse vaginal bleeding of 700 cc in one hour following an uncomplicated spontaneous vaginal delivery of a 4150 gram infant. The placenta delivered spontaneously without difficulty. Prior obstetric history is notable for a previous low transverse Cesarean delivery, secondary to transverse fetal lie. The patient had no antenatal complications. Which of the following is the most likely cause of this patient's hemorrhage? A. Vaginal or cervical lacerations B. Uterine inversion C. Uterine atony D. Uterine dehiscence E. Uterine rupture

C. Uterine atony is the most common cause of postpartum hemorrhage. Risk factors for uterine atony include precipitous labor, multiparity, general anesthesia, oxytocin use in labor, prolonged labor, macrosomia, hydramnios, twins and chorioamnionitis. Patients at risk for genital tract lacerations are those who have a precipitous labor, macrosomia or who have an instrument-assisted delivery or manipulative delivery (i.e. breech extraction). Factors that lead to an over-distended uterus are risk factors for uterine inversion. Grand multiparity, multiple gestation, polyhydramnios and macrosomia are all risk factors. The most common etiology of uterine inversion, however, is excessive (iatrogenic) traction on the umbilical cord during the third stage of delivery. Although the patient is at risk for uterine dehiscence/uterine rupture because of her history of a prior Cesarean delivery, these are infrequent occurrences so the most likely cause of postpartum hemorrhage in this patient is uterine atony.

A 45-year-old G3P3 woman presents to the office because of a large dark spot on her vulva. She states that the lesion has been present for at least two years and is occasionally itchy. She has smoked since age 20. She has a history of genital herpes, but only has one or two recurrences a year. On examination, a 2.5 cm lesion is noted. No induration or groin lymphadenopathy is noted. The vagina and cervix appear normal. There are no additional lesions noted on colposcopic examination of the vulva. A biopsy of the lesion returns as vulvar intraepithelial neoplasia grade 3 (VIN 3). What is the most appropriate next step in the management of this patient? A. Imiquimod (Aldara) treatment B. Trichloroacetic acid (TCA) treatment C. Wide local excision D. Cryotherapy E. Radical vulvectomy

C. VIN III should be treated with local superficial excision. Even with complete removal of all gross disease, recurrence is still possible and the patient will need close surveillance. It is inappropriate to do radical surgery in this setting as cancer has not been diagnosed. Treatment with TCA and Aldara are reserved for condyloma, although some studies have shown utility in the use of Aldara in treating low grade VIN. Cryotherapy is primarily used to treat cervical dysplasia.

A 20-year-old G1P0 woman at 18 weeks gestation with a history of epilepsy has conceived while taking valproic acid. She is scheduled for an ultrasound. What is the most common anomaly associated with prenatal exposure to valproic acid? A. Cardiac defects B. Caudal regression syndrome C. Neural tube defects D. Cleft lip and palate E. Holoprosencephaly

C. Valproic acid use during pregnancy is associated with a 1 to 2% incidence of neural tube defects, specifically lumbar meningomyelocele. Fetal ultrasound examination at approximately 16 to 18 weeks gestation is recommended to detect neural tube defects. Other malformations have been reported in the offspring of women being treated with valproic acid and a fetal valproate syndrome has been described which includes spina bifida, cardiac defects, facial clefts, hypospadius, craniosynostosis, and limb defects, particularly radial aplasia. Case reports have associated prenatal exposure to valproic acid with omphalocele and lung hypoplasia. Caudal regression syndrome is a rare syndrome observed in offspring of poorly controlled diabetics.

A 24-year-old G2P1 woman has a fetus that is affected by Rh disease. At 30 weeks gestation, the delta OD450 (optical density deviation at 450 nm) results plot on the Liley curve in Zone 3, indicating severe hemolytic disease. Which of the following is the most appropriate next step in the management of this patient? A. Immediate Cesarean delivery B. Induction of labor C. Intrauterine intravascular fetal transfusion D. Umbilical blood sampling E. Maternal plasmapheresis

C. Values in Zone 3 of the Liley curve indicate the presence of severe hemolytic disease, with hydrops and fetal death likely within 7-10 days, thus demanding immediate delivery or fetal transfusion. At 30 weeks gestation, the fetus would benefit from more time in utero. An attempt should be made to correct the underlying anemia. Intravascular transfusion into the umbilical vein is the preferred method. Intraperitoneal transfusion is used when intravascular transfusion is technically impossible. If fetal hydrops is present, the reversal of the fetal anemia occurs much more slowly via intraperitoneal transfusion. Percutaneous umbilical blood sampling should not be used as a first-line method to evaluate fetal status. Maternal plasmapheresis is used in severe disease when intrauterine transfusions are not possible.

A 27-year-old G0 woman presents with a three-year history of dyspareunia. She reports a history of always having painful intercourse, but she is now unable to tolerate intercourse at all. She has avoided sex for the last six months. She describes severe pain with penile insertion. On further questioning, she reports an inability to use tampons because of painful insertion. She also notes a remote history of frequent yeast infections while she was on antibiotics for recurrent sinusitis that occurred years ago. Her medical history is unremarkable, and she is not on medications. Pelvic examination is remarkable for normal appearing external genitalia. Palpation of the vestibule with a Q-tip elicits marked tenderness and slight erythema. A normal-appearing discharge is noted. Saline wet prep shows only a few white blood cells, and potassium hydroxide testing is negative. Vaginal pH is 4.0. The cervix and uterus are unremarkable. Which of the following is the most likely diagnosis in this patient? A. Vaginal cancer B. Genital herpes infection C. Vestibulodynia D. Contact dermatitis E. Chlamydia infection

C. Vestibulodynia (formally vulvar vestibulitis) syndrome consists of a constellation of symptoms and findings limited to the vulvar vestibule, which include severe pain on vestibular touch or attempted vaginal entry, tenderness to pressure and erythema of various degrees. Symptoms often have an abrupt onset and are described as a sharp, burning and rawness sensation. Women may experience pain with tampon insertion, biking or wearing tight pants, and avoid intercourse because of marked introital dyspareunia. Vestibular findings include exquisite tenderness to light touch of variable intensity with or without focal or diffuse erythematous macules. Often, a primary or inciting event cannot be determined. Treatment includes use of tricyclic antidepressants to block sympathetic afferent pain loops, pelvic floor rehabilitation, biofeedback, and topical anesthetics. Surgery with vestibulectomy is reserved for patients who do not respond to standard therapies and are unable to tolerate intercourse.

A 28-year-old G0 woman comes to the office for preconception counseling and the inability to conceive for one year. She and her husband of three years are both in good health. She has normal cycles every 28-33 days. She has intercourse about once a month, depending on her schedule. She is an airline pilot and travels a lot. Her examination is normal. She asks about when to best have intercourse during her cycle to maximize her chances of pregnancy. What is the most appropriate advice to give her? A. Keep basal body temperatures and try to attempt intercourse immediately after the rise in body temperature B. Best to attempt intercourse after she is done with her menses C. Use ovulation predictor kits and attempt intercourse after it turns positive D. Take a leave from her work so she can have intercourse three times a week until she gets pregnant E. Attempt intercourse on day 18 of her cycle

C. Women are most fertile during the middle of their cycle when they are ovulating. Assuming normal cycles every 28 days, a woman is most likely to ovulate on day 14. Since sperm can live for up to three days, intercourse up to three days before ovulation can still result in pregnancy. Since this patient has cycles that vary in length, she can best tell when she is ovulating by using an ovulation predictor kit. The basal body temperature charts tell when a patient ovulated retrospectively, so it cannot be used to time intercourse to conceive, as the egg is only viable for about 24 hours. Although having intercourse more frequently will increase her likelihood of conceiving, it is not a practical solution for a working person to stop their work in order to conceive.

A 51-year-old G4P4 woman presents for her health maintenance examination. She has not seen a physician for the past two years as she was caring for her sick husband who passed away two months ago. Her last menstrual period was four years ago and she denies any bleeding since that time. Her past medical and surgical histories are negative. Her mother was diagnosed with ovarian cancer and died at age 54. Her Pap smears have always been normal. Her last one was two years ago and it was negative for high-risk HPV types. Her exam is normal. Which of the following is the most appropriate screening test for this patient? A. Pelvic ultrasound B. Endometrial biopsy C. Colonoscopy D. DEXA scan e. Pap smear

C. Women should be offered colorectal cancer screening starting at age 50. Options include yearly hemoccult testing, flexible sigmoidoscopy every five years, or colonoscopy every 10 years. Ultrasound is not a good screening modality for pelvic pathology. An endometrial biopsy is indicated if a patient is experiencing irregular bleeding. A DEXA scan is only recommended in patients with risk factors for osteoporosis prior to age 65. This patient's history does not indicate that she is high risk. A Pap smear is not indicated as she has no recent history of abnormal Pap smears, and her last one with HPV testing was two years ago.

A 36-year-old G3P2 woman presents in active labor at full term with a known placenta previa. She reports brisk vaginal bleeding. Evaluation shows that fetus and patient are currently hemodynamically stable. She has had two normal vaginal deliveries in the past. She declines your recommendation to undergo Cesarean section. Which of the following is not advisable during your initial management of this patient? A. Soliciting her reasons for not undergoing a Cesarean section B. Obtaining hospital Ethics Committee recommendation C. Proceeding with an emergency Cesarean section D. Explaining your reasons for recommending a Cesarean section E. Informing risk management of the situation that has developed

C. You should not perform any procedure on the patient without her consent. It is best in these situations to explain your reasons for the recommended Cesarean section and elicit the patient's reasons for not wanting to undergo the procedure. A court order should only be obtained as a last resort.

A 33-year-old G5P4 woman just delivered her fourth baby without complications. She had gained 50 pounds during this pregnancy and would like to begin a weight loss program as soon as possible. She desires long-term effective contraception, because she doubts she wants more children. She also desires to breastfeed exclusively for six months and has had trouble with this in the past. Which of the following is the most appropriate contraceptive choice for this patient? A. Depot medroxyprogesterone B. Combined estrogen-progestin contraceptives C. Tubal ligation D. Intrauterine device (IUD) E. Essure (Bilateral occluding tubal coils)

D. A paucity of data exists regarding the effect of hormonal contraception on breastfeeding. There are concerns that hormones, especially estrogen, may have a negative impact on the quantity or quality of breast milk. Although Depot medroxyprogesterone is a progesterone only contraceptive, it is known to cause weight gain and would not be a good choice in this patient. The IUD is the best choice because it is long term but reversible, and does not affect milk production. Tubal ligation and Essure are permanent sterilization and would not be best for a patient who may desire more children.

A 35-year old G2P1 woman is at 11 weeks gestation. She had a triple screen with her last pregnancy and would like to have aneuploidy screening with the current pregnancy. Which of the following screening tests will provide the highest detection rate for trisomy 21 for this patient? A. First trimester combined test B. Sequential screen C. Quad screen D. Cell-free DNA screen E. Serum integrated screen

D. All of the tests screen for trisomy 21 and trisomy 18. Cell-free DNA screening has a trisomy 21 detection rate of over 99% at a 0.2% false-positive rate. The other options may also be used to screen for trisomy 21. Detection rates provided at a 5% false positive screen rate. • First trimester combined test: first trimester nuchal translucency, PAPP-A (pregnancy associated plasma protein A) and Beta-hCG - 85% Detection Rate • Triple screen: second trimester AFP (alpha fetoprotein), Beta-hCG, uE3 (unconjugated estriol) - 69% Detection Rate • Quad screen: (second trimester Triple screen + inhibin A) - 81% Detection Rate • Sequential screen: (first trimester NT and PAPP-A + second trimester quad screen) - 93% Detection Rate • Serum integrated screen, when unable to obtain nuchal translucency: (first trimester PAPP-A + second trimester quad screen) - 85-88% Detection Rate

A 42-year-old G2P2 woman complains of bloating, mood swings and irritability the week prior to her menses. She is convinced that something is wrong with her hormone levels. In addition to a complete physical examination, which of the following diagnostic tools would provide information to accurately determine the diagnosis? A. Pelvic ultrasound B. Estradiol level C. CAGE questionnaire D. Prospective symptom calendar E. Mini mental status examination

D. A calendar of symptoms can clarify if there is a cyclic or constant nature of the symptoms. Often women will mistakenly attribute their symptoms to their menstrual cycle. Different self-reporting scales have been written to assist patients track their symptoms. Because she is menstruating regularly, there is no role for obtaining serum hormone levels.

A 20-year-old G0 previously healthy woman presents to the emergency department with painless vaginal bleeding. Her last menstrual period was 16 weeks ago. On physical exam, her vital signs are: temperature 98.6°F (37.0°C); heart rate 120 beats/minute; and blood pressure 140/90. Abdominal and pelvic examination confirms a 20-week sized uterus with a small amount of blood in the vagina. Beta-hCG is 68,000 mIU/mL. Fetal doppler tones are not auscultated. Which of the following findings would you expect to see on a pelvic ultrasound of this patient? A. Fetus with no cardiac activity C. Multifetal gestation D. Uterus with a snowstorm appearance E. Empty uterus with an enlarged, complex adnexal mass

D. A complete mole has a characteristic "snowstorm" appearance on ultrasound. This is due to the presence of multiple hydropic villi. This patient has a classic presentation for a molar pregnancy. Vaginal bleeding is universal in molar pregnancies. Uterine size greater than dates (weeks from LMP) can be seen in 25-50% of moles, although size less than dates can be seen in 14-33% of moles. There is no fetus seen in cases of a complete mole. There can be a fetus, which is usually grossly abnormal, in cases of a partial mole. There is detectable Beta-hCG in molar pregnancies. The Beta-hCG values are generally higher than the values observed in normal pregnancy. Caution should be taken against the use of a single-value of Beta-hCG to rule in or out a molar pregnancy. However, when combined with the findings of an enlarged uterus and vaginal bleeding, a Beta-hCG value >1,000,000 mIU/mL may be diagnostic. Tachycardia from hyperthyroidism (10% serum diagnosis; 1% clinical diagnosis) and hypertension from preeclampsia (12-25%) can occur in molar pregnancy.

A 31-year-old G3P0 woman presents with amenorrhea for six months. She is otherwise in good health and is not taking any medications. She had a miscarriage seven months ago, which was complicated by an infection and required antibiotics and a dilation and curettage procedure. Her examination is normal. Her laboratory results show a Beta-hCG <5 mIU/mL, and normal TSH and prolactin levels. What is the most likely underlying cause of this patient's amenorrhea? A. Chronic endometritis B. Recurrent miscarriages C. Hypothalamic-pituitary amenorrhea D. Asherman's syndrome E. Sheehan's syndrome

D. Asherman's syndrome can be caused by curettage or endometritis. The intrauterine synechiae or adhesions result from trauma to the basal layer of the endometrium, which causes amenorrhea. Chronic endometritis may be associated with abnormal uterine bleeding and not amenorrhea. Hypothalamic amenorrhea is unlikely because of the temporal relationship of her amenorrhea to the procedure. Sheehan's syndrome is typically due to severe postpartum hemorrhage leading to pituitary apoplexy.

A 41-year-old G3P3 woman reports heavy menstrual periods occurring every 26 days lasting eight days. The periods have been increasingly heavy over the last three months. She reports soaking through pads and tampons every two hours. She has a history of three uncomplicated spontaneous vaginal deliveries and a tubal ligation following the birth of her last child. On pelvic examination, the cervix appears normal and the uterus is normal in size. Which of the following tests or procedures would be most useful in further evaluation of this patient's complaint? A. Follicle stimulating hormone level B. Prolactin level C. Coagulation studies D. Pelvic ultrasound E. Hysteroscopy

D. A pelvic ultrasound would image the endometrium and assess for endometrial pathology such as polyps or submucosal fibroids. In the absence of menopausal symptoms, FSH is unlikely to be helpful. The patient is unlikely to have a coagulation disorder, as she has had three spontaneous vaginal deliveries without postpartum hemorrhage. Hysteroscopy is more invasive than an ultrasound as a first step and would not be helpful if the cause of abnormal bleeding is myometrial pathology such as intramural and subserosal fibroids or adenomyosis. Hyperprolactinemia is found with prolactin-secreting adenomas associated with amenorrhea.

A 42-year-old G3P3 woman comes to the office after noticing a breast mass while performing a breast self-exam. She is in good health and has normal menstrual cycles. Physical exam is significant for a 2 cm dominant breast mass. The remainder of the exam is normal. A mammogram obtained today shows no abnormalities. A fine needle aspiration was negative, and the mass persisted. What is the most appropriate next step in the management of this patient? A. Reassurance and observation B. Obtain an MRI of the chest C. Breast ultrasound D. Perform an excisional biopsy E. Repeat mammogram in two months

D. A specimen obtained on fine-needle aspiration (FNA) is examined both histologically and cytologically. An excisional biopsy should be performed when the results are negative, due to the possibility of a false-negative result. FNA can, however, prevent the need for other diagnostic testing and is the appropriate first step in the evaluation of a palpable breast mass. Breast ultrasound can be used to distinguish between a cyst and a solid mass. Fine needle aspiration under ultrasound guidance can help distinguish a fibroadenoma from a cyst and exclude cancer in certain situations. A normal mammogram does not rule out breast cancer and there is no need to repeat it in two months. There are no indications for obtaining an MRI of the chest in the initial diagnosis of this patient.

A 23-year-old G1P0 woman presents in labor at term. Her prenatal course was uncomplicated. She delivers a 3500 gram infant spontaneously after oxytocin augmentation of labor. Immediately postpartum, there is excessive bleeding greater than 2000 cc. There are no lacerations and the uterus is found to be boggy. Her blood pressure is 90/40; pulse is 120. Conservative and medical management have failed and you proceed with an exploratory laparotomy. Which of the following is the most appropriate next step in the management of this patient? A. Cervical artery ligation B. Ovarian artery ligation C. External iliac artery ligation D. B-Lynch suture E. Hysterectomy

D. A uterine compression suture such as a B-Lynch has been shown to be effective in the management of unresponsive uterine atony. Ligation of a number of pelvic vessels can lead to reduction in the vascular pressure in the pelvis thus controlling hemorrhage. This is especially true with internal iliac artery (hypogastric artery) ligation. However, ligation of the ovarian arteries should not be undertaken as a primary approach. Ligation of the external iliac artery results in devascularization of the leg and, therefore, should not be performed. If these more conservative maneuvers fail, hysterectomy may be necessary but should be a last resort considering the age and parity of the patient.

A 23-year-old G1P1 woman delivered vaginally at 42-weeks gestation after a prolonged induction of labor. She had an epidural with an indwelling catheter for 36 hours and three IV sites for her intravenous medications. She now complains of lower abdominal pain, frequency and dysuria. Her vital signs are: temperature 98.6° F (37° C); pulse 70; blood pressure 100/60; and respirations 12. On examination, her lungs are clear, cardiac exam is normal, abdomen is soft, uterine fundus is firm and nontender, and she has mild suprapubic tenderness. Which of the following organisms is most likely causing her discomfort? A. Group A streptococcus B. Proteus mirabilis C. Klebsiella pneumoniae D. Escherichia coli E. Group B Streptococcus

D. Acute cystitis is a common complication after vaginal delivery and the risk increases with the use of an indwelling catheter. The most common cause of acute cystitis infection is gram-negative bacteria. The major pathogens are E. coli (75%), P. mirabilis (8%), K. pneumoniae (20%), S. faecalis (<5%), and S. agalactiae.

A 68-year-old G3P3 woman comes to the office due to breast tenderness. She is in good health and not taking any medications. Family history is significant for her 70-year-old sister recently diagnosed with breast cancer. On breast examination, her breasts have no lesions; there are no palpable masses, nodules or lymphadenopathy. Her last mammogram was four months ago and was normal. What is the most appropriate next step in the management of this patient? A. Order a mammogram B. Order a breast ultrasound C. Obtain genetic testing (BRCA-1 and BRCA-2 mutations) D. Reassurance E. Order a breast MRI

D. Age and gender are the greatest risk factors for developing breast cancer. Having one first-degree relative with breast cancer does increase the risk. A women's risk of developing breast cancer before menopause is increased if she is BRCA-1 or BRCA-2 positive; however, these genetic mutations occur in a low percentage of the general population. There is no indication for a mammogram since the patient's last mammogram was normal four months ago. Ultrasound and MRI would not add valuable information especially in the setting of a normal mammogram and no masses on physical examination. Genetic testing is not indicated in this case as there is no strong family history and the sister with breast cancer was postmenopausal at time of diagnosis.

A 24-year-old G2P1 woman is undergoing a Cesarean delivery for placental abruption. She presented to labor and delivery with severe abdominal pain and heavy vaginal bleeding. The fetus was delivered uneventfully. The placenta delivered with a significant clot attached to the maternal surface. The patient continues to bleed from the placental bed. Estimated blood loss is 1500 ml. The operative team decides to give her fresh frozen plasma (FFP) to replace which of the following components? A. Platelets B. Von Willebrand's factor C. Red blood cells D. Fibrinogen E. Factor X

D. Correcting coagulation deficiencies requires replacing all necessary blood components. Fresh frozen plasma contains fibrinogen, as well as clotting factors V and VIII. Cryoprecipitate contains fibrinogen, factor VIII and von Willebrand's factor. Neither of these preparations contains red blood cells or platelets, which must be given separately.

A 28-year-old G1P0 woman presents for prenatal care. Her periods have been irregular and she does not recall when the last one occurred. She is healthy and denies any medical problems. The uterus is 10 weeks in size and there are no adnexal masses. At this point in time, what is the best way to date the pregnancy? A. Serum Progesterone B. Quantitative serum Beta-hCG C. Ultrasound measurement, gestational sac D. Ultrasound measurement, crown-rump length E. Uterine size on pelvic exam

D. All the above can potentially be used to help date a pregnancy; however, ultrasound measurement of crown-rump length is considered the most reliable (+/- 4 to 5 days) in the first trimester. Other means to date the pregnancy include: fetal heart tones that have been documented for 20 weeks by a non-electronic fetoscope or for 30 weeks by Doppler; it has been 36 weeks since a positive serum or urine Beta-hCG pregnancy test was performed by a reliable laboratory; an ultrasound measurement of the crown-rump length obtained at six to twelve weeks supports a gestational age of at least 39 weeks; and an ultrasound obtained at 13-20 weeks confirms the gestational age of at least 39 weeks determined by clinical history and physical examination. Clearly, these means are not as useful in early pregnancy, but in confirming the length of pregnancy. Serum progesterone levels are used to help establish if a pregnancy is progressing normally and not an ectopic, miscarriage, or fetal demise.

A 17-year-old G0 sexually active female presents to the emergency room with acute right lower quadrant pain and nausea for 12 hours. Her periods have always been irregular, with her last one six weeks ago. She is otherwise completely healthy. She appears in mild distress. Physical examination: temperature 100.2°F (37.9°C); blood pressure 110/60; heart rate 108 beats/min. She has moderate abdominal tenderness with right greater than left pelvic tenderness. Pelvic examination reveals normal external genitalia and pink-tinged discharge is noted on speculum examination. Bimanual/rectovaginal examination confirms mild cervical motion tenderness and fullness in the right adnexa with moderate tenderness and some voluntary guarding. What is the single most important test to obtain? A. Pelvic ultrasound B. CT scan of the abdomen and pelvis C. GC and chlamydia DNA probe D. Beta-hCG E. CBC with differential

D. Although all of the tests listed above may be considered, it is imperative to obtain a Beta-HCG to rule out an ectopic pregnancy.

A 36-year-old G5P4 woman with no prenatal care presented in active labor with a blood pressure of 170/105 and 3+ proteinuria. Fundal height is 28 cm. Fetal heart tones were found to be in the 170s with decreased variability and a sinusoidal pattern. Resting uterine tone was noted to be increased and she was having frequent contractions (every 1-2 minutes). The patient complained of bright red vaginal bleeding for the past hour. Based on this history, what is the most likely etiology of her vaginal bleeding? A. Uterine rupture B. Placenta previa C. Bloody show D. Abruptio placentae E. Cervical trauma

D. Although all the options above can result in third trimester vaginal bleeding, the most likely cause in this patient is placental abruption. This diagnosis goes along with the tachysystole on tocometer and evidence of fetal anemia (tachycardia and sinusoidal heart rate pattern) on the heart rate tracing. Hypertension and preeclampsia are risk factors for abruption. She has no history of cervical trauma.

A 65-year-old G2P2 postmenopausal woman with a remote history of stage I, grade 1 endometrial cancer treated with surgery 15 years ago returns to your office for a health maintenance examination. During a review of systems, the patient reports several months of a dry cough, progressive dyspnea on exertion, and swelling in her legs. She is a non-smoker, but her now deceased husband smoked heavily. She saw her family physician, who initially treated her with a short course of antibiotics; however, because of persistent symptoms a chest x-ray was obtained and revealed a bilateral pleural effusion and a suspicious pulmonary nodule. Her examination is notable for decreased breath sounds at the lung bases, a normal abdominal exam, and a pelvic exam without any suspicious masses or nodularity. She has pitting edema in both of her lower extremities. What is the most appropriate next step in the management of this patient? A. Obtain a Doppler of her lower extremities B. Refer to oncologist C. Refer to palliative care D. Refer to pulmonologist E. Repeat chest x-ray in three months

D. Although recurrent endometrial cancer can present as multiple pulmonary nodules, this patient is unlikely to have a recurrence of her endometrial cancer given the initial early stage and remote timing of her cancer diagnosis. The most appropriate next step is to refer her to a pulmonologist (or cardiologist) for a thorough work-up. The finding of pleural effusions and lower extremity edema point towards a cardiopulmonary etiology; however, the finding of a solitary lung nodule in a patient exposed to second hand smoke certainly suggests the possibility of a primary lung cancer. Referral to palliative care would be premature at this point. A Doppler ultrasound to rule out a deep venous thrombosis is reasonable, but typically of more utility in the setting of unilateral edema, and still would not address the need to evaluate her lung findings

A 36-year-old G0 woman presents to the emergency department accompanied by her female partner. The patient notes severe abdominal pain. She states that this pain began 2-3 days ago and was associated with diarrhea as well as some nausea. It has gotten progressively worse and she has now developed a fever. Neither her partner, nor other close contacts, report any type of viral illness. She had her appendix removed as a teenager. On examination, her temperature is 102.0°F (38.9°C), her abdomen is tender with mild guarding and rebound, and she has an elevated white count. On pelvic examination, she is exquisitely tender, such that you cannot complete the examination. Pelvic ultrasound demonstrates bilateral 3-4 cm complex masses. What is the most likely underlying pathogenesis of her illness? A. Diverticulitis B. Gastroenteritis C. Reactivation of an old infection D. Ascending infection E. Pyelonephritis

D. Although salpingitis is most often caused by sexually transmitted agents such as gonorrhea and chlamydia, any ascending infection from the genitourinary tract or gastrointestinal tract can be causative. The infection is polymicrobial consisting of aerobic and anaerobic organisms such as E. coli, Klebsiella, G. vaginalis, Prevotella, Group B streptococcus and/or enterococcus. Although diverticulitis and gastroenteritis should be part of the differential diagnosis initially, the specific findings on examination and ultrasound are more suggestive of bilateral tubo-ovarian abscesses. Even though this patient does not have the typical risk factors for salpingitis, the diagnosis should be considered and explained to the patient in a sensitive and respectful manner. The patient should also be questioned separate from her partner regarding the possibility of other sexual contacts.

A 16-year-old G0 female presents to the emergency department with a two-day history of abdominal pain, nausea and vomiting. She is sexually active with a new partner and is not using any form of contraception. On examination, her temperature is 100.2°F (37.9°C), and she has bilateral lower quadrant pain, with slight rebound and guarding. On pelvic examination, she has purulent cervical discharge and cervical motion tenderness. Her white count is 14,000/mcL. What is the most appropriate next step in the management of this patient? A. Oral amoxicillin clavunate and doxycycline B. Oral metronidazole and doxycycline C. IV metronidazole and doxycycline D. IV cefotetan and doxycycline E. No treatment until culture results are back

D. Although some patients can be treated with an outpatient regimen, this patient should be hospitalized for IV treatment, as she has nausea and vomiting so she might not be able to tolerate oral medications. While adolescents have no better outcomes from inpatient vs outpatient therapy, each patient should be assessed for compliance. It is important to treat aggressively in order to prevent the long-term sequelae of acute salpingitis. You would not wait for culture results before initiating treatment. Her recent sexual contacts should also be informed (by her and/or with her consent) and treated. According to the 2010 CDC treatment guidelines, there are two options for parenteral antibiotics covering both gonorrhea and chlamydia. Cefotetan or cefoxitin PLUS doxycycline or clindamycin PLUS gentamicin. For outpatient treatment, the 2010 CDC guidelines recommend ceftriaxone, cefoxitin, or other third-generation cephalosporin (such as ceftizoxime or cefotaxime) PLUS doxycycline WITH or WITHOUT metronidazole. There are alternative oral regimens as well. http://www.cdc.gov/std/treatment/2010/pid.htm

A 36-year-old G2P1 woman presents for her first prenatal visit at 11 weeks gestation. She has a two-year history of chronic hypertension treated with lisinopril and labetalol. In addition, she has hypothyroidism treated with levothyroxine, and recurrent herpes, for which she is on chronic acyclovir suppressive therapy. She takes amitriptyline for migraine headaches. Which of her medications is contraindicated in pregnancy? A. Levothyroxine B. Labetalol C. Acyclovir D. Lisinopril E. Amitriptyline

D. Amitriptyline, levothyroxine, labetalol and acyclovir are medications that are frequently used in pregnancy and are felt to have acceptable safety profiles. The use of angiotensin converting enzyme inhibitors, such as Lisinopril, beyond the first trimester of pregnancy has been associated with oligohydramnios, fetal growth retardation and neonatal renal failure, hypotension, pulmonary hypoplasia, joint contractures and death. Amitriptyline is used in pregnancy to treat migraine headaches.

A 33-year-old G2P1 woman at eight weeks presents to the clinic. This is an unplanned pregnancy. She had planned a tubal ligation six years ago when she was diagnosed with pulmonary hypertension, but was unable to have the procedure. She states her pulmonary hypertension has been stable, but she gets short of breath when climbing stairs. She sleeps on one pillow at night. What is the concern for her during this pregnancy? A. There are no additional concerns compared to a normal pregnancy B. She will need a Cesarean section at delivery C. Her baby is at increased risk for pulmonary hypoplasia D. The mother's mortality rate is above 25% E. Epidural analgesia is contraindicated

D. Among women with cardiac disease, patients with pulmonary hypertension are among the highest risk for mortality during pregnancy, a 25-50% risk for death. Management of labor and delivery is particularly problematic. These women are at greatest risk when there is diminished venous return and right ventricular filling which is associated with most maternal deaths. Similar mortality rates are seen in aortic coarctation with valve involvement and Marfan syndrome with aortic involvement. The baby is not at risk for pulmonary hypoplasia unless there is very preterm rupture of the fetal membranes.

A 24-year-old G0 woman comes into the office because she has not had her menses for six months. She is in good health and not taking any medications. She is not sexually active. She does well in graduate school, despite her demanding new program. Her height is 5 feet 6 inches and her weight is 104 pounds. Her vital signs are normal. Her physical examination, including a pelvic examination, is completely normal. What is the most likely reason for her amenorrhea? A. Ovarian dysfunction B. Thyroid disease C. Premature ovarian failure D. Hypothalamic-pituitary dysfunction E. Pregnancy

D. Anorexia nervosa or significant weight loss may cause hypothalamic-pituitary dysfunction that can result in amenorrhea. A lack of the normal pulsatile secretion of gonadotropin releasing hormone (GnRH) leads to a decreased stimulation of the pituitary gland to produce follicle stimulating hormone (FSH) and luteinizing hormone (LH). This leads to anovulation and amenorrhea. Although testing for thyroid dysfunction may be indicated, she has no other symptoms to suggest thyroid disease. While ovarian dysfunction/failure, premature ovarian failure and pregnancy cause amenorrhea, they are unlikely in this case.

A 24-year-old G2P2 woman with a history of two prior Cesarean deliveries desires a tubal ligation for permanent sterilization. She has two daughters, who are 3 and 1 years old. She is very sure she does not desire any more children. She is happily married and is a stay-at-home-mom. What is the strongest predictor of post-sterilization regret for this patient? A. Not working outside the home B. Parity C. Marital status D. Age E. Children's gender

D. Approximately 10% of women who have been sterilized regret having had the procedure with the strongest predictor of regret being undergoing the procedure at a young age. The percentage expressing regret was 20% for women less than 30 years old at the time of sterilization. For those under age 25, the rate was as high as 40%. The regret rate was also high for women who were not married at the time of their tubal ligation, when tubal ligation was performed less than a year after delivery, and if there was conflict between the woman and her partner.

A 32-year-old G0 woman presents for preconception visit. She has regular periods reports tension, depressed mood and decreased productivity towards the end of each cycle. She is otherwise healthy and maintains a high-profile job. Her past medical history is benign and she denies prior psychiatric problems. She denies smoking and drinks alcohol socially. What is the next best step in the management of this patient? A. Reassure her that her monthly symptoms are normal B. Initiate anti-depressant therapy C. Psychiatry consult D. Ascertain the timing of her symptoms each month E. Initiate psychotherapy

D. Ascertaining the timing of her symptoms each month is an important first step in establishing the proper diagnosis. Symptoms of Premenstrual Dysphoric Disorder occur in the luteal phase and are absent in the beginning of the follicular phase. It is therefore important to document the timing of symptoms each month when considering a diagnosis of Premenstrual Dysphoric Disorder. Additionally, it is important to ascertain that these symptoms are not an exacerbation of an underlying psychiatric disorder before initiating therapy as this potentially can have more consequences during her pregnancy and postpartum period.

A 23-year-old G1P1 woman delivered her first baby two days ago after an uncomplicated labor and vaginal delivery. She wants to breastfeed and has been working with the lactation team. Prior to discharge, her temperature was 100.4° F (38° C) and other vitals were normal. She denies urinary frequency or dysuria and her lochia is mild without odor. On examination, her lungs are clear, cardiac exam normal, and abdomen and uterine fundus are nontender. Her breasts are firm and tender throughout, without erythema, and nipples are intact. Which of the following is the most likely cause of her fever? A. Endomyometritis B. Septic pelvic thrombophlebitis C. Mastitis D. Breast engorgement E. Vaginitis

D. Breast engorgement is an exaggerated response to the lymphatic and venous congestion associated with lactation. Milk "let-down" generally occurs on postpartum day two or three. If the baby is not feeding well, the breast can become engorged, which can cause a low-grade fever. Lactating women are encouraged to feed their baby frequently, and use a breast pump to prevent painful engorgement and mastitis. Postpartum fever differential includes endometritis, cystitis and mastitis. These are easy distinguished, based on clinical findings. Vaginitis is not accompanied by fever. Septic pelvic thrombophlebitis is a rare condition and characterized by high fever not responsive to antibiotics and is a diagnosis of exclusion.

A 58-year-old G3P3 woman has been postmenopausal for five years and is concerned about osteoporosis. She has declined hormone therapy in the past. Her mother has a history of a hip fracture at age 82. A physical exam is unremarkable. In addition to weight bearing exercise and vitamin D supplementation, what optimal daily calcium intake should she take? A. None B. 400 mg C. 800 mg D. 1200 mg E. 1600 mg

D. Calcium absorption decreases with age because of a decrease in biologically active vitamin D. A positive calcium balance is necessary to prevent osteoporosis. Calcium supplementation reduces bone loss and decreases fractures in individuals with low dietary intakes. In order to remain in zero calcium balance, postmenopausal women require a total of 1200 mg of elemental calcium per day.

A 45-year-old G4P3 woman presents with vaginal bleeding. Last week, she performed a home pregnancy test that was positive. She thinks her last menstrual period was four months ago. The last time she saw her doctor was eight years ago, with the birth of her last child. She has no serious medical problems, has smoked a pack of cigarettes a day since the age of 20, occasionally has a beer and does not exercise. Abdominal examination reveals a soft abdomen and the fundus palpable just below the umbilicus. Pelvic ultrasound reveals a fundal placenta and a fetus measuring 18 weeks with normal cardiac activity. Vaginal examination reveals a 3-centimeter lesion on the posterior lip of the cervix. It easily bleeds with palpation and is hard in consistency. Which of the following is the most likely cause of the bleeding? A. Cervicitis B. Cervical polyp C. Endometrial polyp D. Cervical cancer E. Nabothian cyst

D. Cervical cancer can unfortunately complicate pregnancies and presents with bleeding. She is at risk due to lack of screening as well as her history of smoking. Other causes of bleeding need to be ruled out such as cervical incompetence, infection or trauma. Treatment for cervical cancer during pregnancy requires difficult decisions that consider the stage of cancer, appropriate therapy, maternal welfare and fetal welfare. Cervical polyps occur during pregnancy and can be a cause of bleeding, but are typically soft and not hard or nodular on examination. Nabothian cysts are very common, but do not typically cause bleeding.

A 28-year-old G0 woman with a low-grade squamous intraepithelial lesion (LSIL) on a Pap test presents for evaluation. A colposcopy is performed and is satisfactory. A lesion is seen at 3:00 that turns white with acetic acid, has punctations and mosaicism, and is friable. This lesion is biopsied with a pathology report of CIN 1. The patient's endocervical curettage (ECC) is positive for a high-grade lesion. Which of the following is the most appropriate next step in the management of this patient? A. Follow up Pap test in six months B. Repeat colposcopy in six months C. Cryotherapy D. Cervical conization E. Hysterectomy

D. Cervical conization is indicated in this patient who has a positive ECC. Hysterectomy is the treatment for invasive cancer. Waiting six months can potentially be harmful, as the lesion can progress or a higher-grade lesion might already be present. Cryotherapy will not provide a pathologic specimen to rule out invasive cancer, but can be used to treat cervical dysplasia once cancer has been completely ruled out and the entire lesion can be visualized.

A 19-year-old G1P0 woman presents at 41 weeks gestation with a fever, spontaneous ruptured membranes and no contractions. Her temperature is 102.6° F (39.2° C); pulse 126. Ultrasound reveals a singleton with decreased amniotic fluid and placenta partially covering the os. The cervix appears long and closed. Which of the following is an indication for Cesarean delivery in this patient? A. Chorioamnionitis B. Unfavorable cervix C. Oligohydramnios D. Placenta covering the cervical os E. Spontaneous ruptured membranes not in labor

D. Cesarean delivery is indicated in this patient because of a placenta previa (placenta covering the internal os). A vaginal delivery is contraindicated in patients with a placenta previa. Post-term pregnancies, chorioamnionitis, oligohydramnios, and term premature rupture of membranes are all acceptable indications for induction of labor and delivery if the patient is a good candidate for initiation of labor. An unfavorable cervix is not a contraindication for a vaginal delivery.

A 16-year-old female has a new boyfriend and comes in to discuss contraception. She is well aware of the importance of preventing sexually transmitted infections and specifically wants to know about prevention of pregnancy. Other than abstinence, the most effective method of birth control in this patient is: A. The male condom B. The diaphragm with spermicide C. Oral contraceptives D. Depo-Provera E. The contraceptive ring

D. Contraceptive methods with <1% pregnancy rates (typical use) are Depo-Provera, IUD, sterilization (male or female), and Implanon. Oral contraceptives have a 3-5% pregnancy rate with typical use, and the male condom has a 12% pregnancy rate. Eight percent of women will experience an unintended pregnancy after one year of typical use with a contraceptive ring. Of the methods listed, the diaphragm with spermicide has the highest failure rate (18%) with typical use.

A 23-year-old G0 woman reports having a solitary, painful vulvar lesion that has been present for three days. This lesion has occurred twice in the past. She states that herpes culture was done by her doctor during her last outbreak and was negative. She is getting frustrated in that she does not know her diagnosis. She has no significant previous medical history. She uses oral contraceptives and condoms. She has had four sexual partners in her lifetime. On physical examination, a cluster of three irregular erosions with a superficial crust is noted on the posterior fourchette. Urine pregnancy test is negative. You suspect recurrent genital herpes. How do you explain the negative culture? A. Cultures were taken too early B. Oral contraceptives affect the growth of the virus C. The cultures were refrigerated prior to transport to the lab D. Herpes cultures have a 10-20% false negative rate E. The herpes virus cannot be recovered with recurrent infections

D. Culture is the gold standard in the diagnosis of herpes. They are highly specific, yet sensitivity is limited. It is best to culture the lesion very early in the course. The blister is unroofed and the base is vigorously scraped. The herpes virus can theoretically be isolated from both primary and recurrent infections. This patient very likely presented too late in the course for a useful culture. Oral contraceptives do not affect the growth of viruses. While serum antibody screening can be performed, it indicates lifetime exposure and would not answer the question as to the etiology of the specific lesion. Alternatively, DNA studies such as the polymerase chain reaction can be done, if available.

A 28-year-old G1P1 woman delivered three days ago and desires to breastfeed her infant, but is having problems since her milk came in with full tender breasts. She is uncomfortable and has engorged breasts. Which of the following strategies may help relieve her discomfort? A. Discontinue breastfeeding for 24 hours to decrease the milk supply B. Cover the breast with cool lettuce leaves C. Increase the interval between breastfeeding sessions to decrease the milk supply D. Nurse every 1.5-3 hours around the clock E. Don't wear a bra until the engorgement subsides

D. Engorgement commonly occurs when milk comes in. Strategies that may help include frequent nursing, taking a warm shower or warm compresses to enhance milk flow, massaging the breast and hand expressing some milk to soften the breast, wearing a good support bra and using an analgesic 20 minutes before breastfeeding.

A 54-year-old G4P4 woman who has been menopausal for four years recently underwent a total vaginal hysterectomy and bilateral salpingo-oophorectomy for vaginal prolapse. She comes in for a postoperative check up and complains of hot flashes and wonders why she is experiencing menopause again. Which of the following most likely explains why she is experiencing these symptoms? A. Increased postoperative liver metabolism B. Decreased adrenal estrogen production C. Removal of an occult estrogen-producing tumor D. Decreased circulating androgens E. Cessation of ovarian estrogen production

D. Estrogen production by the ovaries does not continue beyond menopause. However, estrogen levels in postmenopausal women can be significant due to the extraglandular conversion of androstenedione and testosterone to estrogen. This conversion occurs in peripheral fat cells and, thus, body weight has been directly correlated with circulating levels of estrone and estradiol. Since menopausal ovaries are known to continue production of androgens, surgical removal of postmenopausal ovaries may result in the resurgence of menopausal symptoms from the abrupt drop in circulating androgens.

A 54-year-old G2P2 presents for her health maintenance examination. She has a history of breast cancer treated with mastectomy with reconstruction, chemotherapy, and is currently on tamoxifen. She has been in remission for two years and has been menopausal since the initiation of her chemotherapy. She experiences very mild hot flashes, and is not sleeping well. She appears apprehensive during the examination, although her examination is completely normal except for severe vaginal atrophy. At the conclusion of the office visit, she finally opens up and admits that she has a new boyfriend. She has not had a sexual relationship since her divorce five years earlier, but has been enjoying masturbation. Although excited about initiating sexual activity again, she is obviously concerned. Which of the following is most likely to contribute to sexual dysfunction? A. Sexual desire B. Arousal C. Orgasm D. Dyspareunia E. Body image

D. Female sexual dysfunction can be classified as disorders in sexual desire, arousal, orgasm, or sexual pain, and can include any combination of these. In this case, because she states a desire to initiate in sexual activity and has been enjoying masturbation, it is unlikely that she will experience any problems related to desire, arousal, or orgasm. However, in the presence of severe atrophy and lack of estrogen, she may in fact experience pain related to dyspareunia. She should be encouraged to use some form of water-based lubricant to diminish the effects of the vaginal dryness since estrogen is likely contraindicated with her breast cancer diagnosis. Although body image may play a role, it would be classified under the category of sexual desire.

A 24-year-old Rh-negative G2P1 woman is found at 10 weeks gestation to have anti-D antibodies. You follow her closely during this pregnancy and order serial ultrasound examinations. Which of the following fetal ultrasound findings would be most explained by the presence of Rh disease? A. Meconium B. Fetal bladder obstruction C. Oligohydramnios D. Pericardial effusion E. Placenta previa

D. Fetal hydrops is easily diagnosed on ultrasound. It develops in the presence of decreased hepatic protein production. It is defined as a collection of fluid in two or more body cavities, such as ascites, pericardial and/or pleural fluid and scalp edema. On occasion, when extramedullary hematopoiesis is extensive, there will be evidence of hepatosplenomegaly. Placentomegaly (placental edema) and polyhydramnios are also seen on ultrasound. Meconium, fetal bladder obstruction, oligohydramnios and placenta previa do not fit the clinical scenario.

A 29-year-old G1P0 woman at 28 weeks gestation who is the wife of basketball player is diagnosed with gestational diabetes. Her BMI is 23.8. Her mother had a delivery complicated by shoulder dystocia and she is concerned about her own risk. Which of the following is her biggest risk factor for shoulder dystocia? A. Family history B. Tall husband C. Maternal weight D. Gestational diabetes E. Parity

D. Fetal macrosomia, maternal obesity, diabetes mellitus, postterm pregnancy, a prior delivery complicated by a shoulder dystocia, and a prolonged second stage of labor are all associated with an increased incidence of shoulder dystocia. Although a family history can be indicative of large babies which might place her at additional risk, her gestational diabetes represents her largest risk factor.

A 24-year-old woman complains of cyclic mastalgia since the onset of her period at age 12. The symptoms have increased over the years but were less troublesome when she took oral contraceptives a few years ago. Currently, she takes no medications and is not sexually active. She is a strict vegetarian and eats soy products. She does not smoke and she drinks a glass of wine three times a week, and several diet colas every day. Her mother was diagnosed with breast cancer at age 55. Her breast exam is normal, except for some mild fibrocystic changes. Which of the following elements in her history contributes to her increasing pain? A. Alcohol intake B. Vegetarian diet C. Family history of breast cancer D. Caffeine intake E. Age at menarche

D. Fibrocystic breast changes are the most common type of benign breast conditions and occur most often during the reproductive years. Fibrocystic disease is often associated with cyclic mastalgia, possibly related to a pronounced hormonal response. Caffeine intake can increase the pain associated with fibrocystic breast changes, so recommending that she decrease her caffeine intake may be helpful. Pain is not related to alcohol intake, her vegetarian diet or age of menarche. There is an increased risk of breast cancer when atypia is present.q

A 42-year-old G3P3 woman comes to the office after noticing a breast mass while performing a breast self-exam. She is in good health and has normal menstrual cycles. Family history is significant for multiple first and second-degree relatives having breast cancer. Physical exam reveals a 2 cm dominant breast mass. The remainder of the exam is normal. A mammogram obtained today shows no abnormalities. What is the most appropriate next step in the management of this patient? A. Reassurance and observation B. Obtain genetic testing for BRCA1 and BRCA2 C. MRI of the breast D. Fine needle aspiration E. Repeat mammogram in two months

D. Given her positive family hx, can assume that the mammogram might be a false negative. she is >25yo, so go for the FNA.

A 34-year-old G2P2 woman presents with biopsy-proven vulvar intraepithelial neoplasia, grade 2 (VIN 2). She had undergone routine examination by her primary physician, who performed a Pap smear (normal) and noted multiple warty-type lesions on the labia. The patient describes some mild itching that she self-treated for a yeast infection, with minimal relief. Otherwise, she is completely healthy, except for smoking a half-pack of cigarettes per day. She is sexually active, and is concerned about the impact this will have on her sex life. Examination confirms multiple, whitish raised 0.5 - 1.5 cm papules throughout her labial minora, majora, clitoral hood and perineum. Which of the following is the most appropriate treatment option for this patient? A. Trichloroacetic acid (TCA) B. Skinning vulvectomy C. Observation and expectant management D. CO2 laser ablation of the lesions E. Smoking cessation

D. Given the multifocality of the vulvar dysplasia (VIN 2), laser treatment is the best choice. In order to adequately treat these lesions, a complete (skinning) vulvectomy would be the other choice, but would be disfiguring and require removal of the clitoris which would have detrimental effects on her sexual function. Treatment with TCA is recommended for treatment of warts and not VIN. Smoking cessation is strongly recommended regardless, but would not be the sole means of addressing these lesions. Observation is not ideal, given her mild symptoms, moderate grade, and diffuse nature of the lesions.

A 49-year-old G0 woman reports that her periods have become heavier over the last year. The patient's physical exam is notable for a slightly enlarged, irregularly shaped uterus, measuring approximately eight weeks in size. A pelvic ultrasound confirms the presence of two 2 x 2 cm intramural uterine fibroids. Her endometrial biopsy reveals proliferative endometrium. The patient's friend recently had a hysterectomy due to uterine fibroids and menorrhagia, but she would like to avoid having surgery. She is interested in the medical options for treating symptomatic uterine fibroids, but has tried NSAIDs which did not seem to help much. What is the next best step in the management of this patient? A. Aspirin B. Methotrexate C. Estrogen D. Gonadotropin-releasing hormone agonists E. Indomethacin

D. Growth of uterine fibroids is stimulated by estrogen. Gonadotropin-releasing hormone agonists inhibit endogenous estrogen production by suppressing the hypothalamic-pituitary-ovarian axis. They can result in a 40-60% reduction in uterine size. This treatment is commonly used for three to six months before a planned hysterectomy in an attempt to decrease the size of the uterus, which may lead to a technically easier surgery and decreased intraoperative blood loss. In patients who are not yet menopausal, once the gonadotropin-releasing hormone agonist therapy is discontinued, the fibroids may grow again with re-exposure to endogenous estrogen. Thus, this therapy may be most useful for women who are close to menopause, as this patient is at age 49. Aspirin and methotrexate are not effective treatments for fibroids. Methotrexate is used in ectopic pregnancies. Aspirin and indomethacin will likely not help, as she did not respond to NSAIDs.

A 42-year-old G2P2 woman presents for a health maintenance examination. Her past medical history is negative. Her family history is significant for hypertension and hypercholesterolemia in her father and diabetes mellitus in her mother. Her body mass index (BMI) is 23. What lifestyle modification is most important for this patient? A. Starting a weight loss diet B. Starting a sugar-free and cholesterol-free diet C. Recording a daily blood pressure D. Starting an aerobic exercise program E. Recording a weekly blood pressure

D. Heart disease is the number one killer of women. Lifestyle modifications to reduce her risk, especially considering her family history, are important proactive changes that she can make. Studies show an inverse relationship between the level of physical activity and incidence of death from coronary disease. Exercise would be an appropriate first step with this patient. She does not need to lose weight (normal BMI) and does not need to be on a special diet (normal labs). Recording daily or weekly blood pressures is not necessary, but her blood pressure should be checked once a year.

A 23-year-old G3P2 woman wants to exclusively breastfeed her baby. She is deciding at which hospital she will deliver. Hospital policies that promote breastfeeding include which of the following? A. Uninterrupted sleep for the mother on her first night in the hospital B. Use of a breast pump to help increase the milk supply C. Use of pacifiers to prevent sore nipples D. Unlimited access of mother to baby E. Use of metoclopramide to increase the milk supply

D. Hospital policies that promote breastfeeding include getting the baby on the breast within a half hour of delivery and rooming-in for the baby to ensure frequent breastfeeding on demand (i.e. unlimited access).

A 23-year-old G1P1 woman delivered a healthy infant two days ago. She has had difficulty breastfeeding despite multiple attempts. Her nipples are sore and cracked and she is thinking about exclusively bottlefeeding. The patient's pregnancy was complicated by gestational diabetes and the patient has chronic hypertension and a history of an abnormal Pap. She had a cone biopsy two years ago and had a normal Pap with the current pregnancy. The patient's mother has a history of endometrial and colon cancer and her maternal grandmother and grandfather both had fatal heart attacks in their early sixties. Breastfeeding decreases the risk of which of the following for this patient? A. Type 2 diabetes B. Coronary artery disease C. Cervical cancer D. Ovarian cancer E. Colon cancer

D. Human milk is recognized by the American Academy of Pediatrics as an optimal feeding for all infants. The American Academy of Pediatrics recommends exclusive breastfeeding for the first six months after birth. Physicians can influence a patient's feeding choice, and prenatal education is important in the initiation and maintenance of breastfeeding. Nationally representative surveys have noted that women were more likely to initiate breastfeeding if their physicians or nurses encouraged it. Benefits to the mother include increased uterine contraction due to oxytocin release during milk let down and decreased blood loss. Breastfeeding is associated with a decreased incidence of ovarian cancer. Some studies have reported a decreased incidence of breast cancer. Breastfeeding has not been shown to decrease the risk for developing coronary artery disease, cervical dysplasia and cervical cancer or colon cancer in the mother. Breast milk is a major source of Immunoglobulin A which is associated with a decrease of newborn's gastrointestinal infections.

A 26-year-old G0 woman comes to the office due to irregular menses since menarche, worsening for the last six months. The patient has noted increasing hair growth on her chin and most recently hair growth on her chest, requiring that she shave periodically. No one in her family has hirsutism. On exam, you also notice acne on her chin, acanthosis nigricans and temporal balding. Her serum testosterone is elevated. You suspect hyperthecosis. Which of the following might also be associated with this condition? A. Hyperthyroidism B. Hyperprolactinemia C. Atrophic changes of external genitalia D. Deepening of the voice E. Hyperparathyroidism

D. Hyperthecosis is a more severe form of polycystic ovarian syndrome (PCOS). It is associated with virilization due to the high androstenedione production and testosterone levels. In addition to temporal balding, other signs of virilization include clitoral enlargement and deepening of the voice. Hyperthecosis is more difficult to treat with oral contraceptive therapy. It is also more challenging to achieve successful ovulation induction. Hyperthyroidism and hyperparathyroidism are not typically associated with hyperthecosis. Hyperprolactinemia is typically associated with amenorrhea and does not cause hirsutism.

A 42-year-old G4P4 woman presents for management of suspected adenomyosis. She had a tubal ligation four years ago. A pelvic examination shows an enlarged, soft, boggy uterus. A pregnancy test is negative and she is mildly anemic. An ultrasound shows an enlarged uterus with no fibroids. The patient desires definitive treatment for this condition. What is the most appropriate next step in her management? A. Continuous estrogen/progestin therapy B. Endometrial ablation C. GnRH agonist D. Hysterectomy E. Insertion of a levonorgestrel containing intrauterine system

D. Hysterectomy is nearly 80% effective in eliminating pain and abnormal bleeding, if she is willing to undergo surgery. Gonadotropin releasing agents are the first choice for medical therapy for the pain, but the problem is that the adenomyosis seems to recur after discontinuing the therapy. Endometrial ablation and insertion of a levonorgestrel-containing intrauterine system are options in women who decline hysterectomy or desire to maintain fertility. For abnormal bleeding problems and desire for uterine conservation, a progesterone intrauterine contraceptive device can also be used to improve irregular bleeding. Hysteroscopic endometrial ablation can be a treatment for adenomyosis.

A 37-year-old G3P3 woman presents for contraceptive counseling. She and her husband have decided that they no longer plan to have children and desire permanent sterilization. Her husband refuses to have a vasectomy. On exam, her BMI is 52; blood pressure is 140/80; and heart rate is 86. She has had three previous Cesarean deliveries. Which of the following options would be the be the best method of permanent sterilization? A. Laparoscopic tubal ligation B. Mini-laparotomy with tubal ligation C. Hysterectomy D. Hysteroscopic tubal occlusion (Essure) E. Endometrial ablation

D. Hysteroscopic tubal occlusion is the best option for this patient. Hysteroscopic tubal occlusion (Essure®) can be performed in the office and places coils into the fallopian tubes that cause scarring that blocks the tubes. Patients are required to use a back up method of contraception for three months following the procedure until a hysterosalpingogram is performed confirming complete occlusion of the tubes. While tubal ligation, either by laparoscopy or mini-laparotomy, are common and effective forms of permanent sterilization, for this patient with her BMI and previous surgeries, this would carry more surgical risks. Hysterectomy is not an indicated procedure for sterilization. Endometrial ablation, or thermal destruction of the endometrial tissue, is an effective treatment for menorrhagia but is not reliable for permanent sterilization.

A 32-year-old G3P2 woman has delivered a previous child with anencephaly. What is the appropriate recommended dose of folic acid for this woman? A. 0.4 mg B. 0.8 mg C. 1.0 mg D. 4 mg E. 8 mg

D. In 1991, the Centers for Disease Control and Prevention recommended that all women with a previous pregnancy complicated by a fetal neural tube defect ingest 4 mg of folic acid daily before conception and through the first trimester. In one analysis, this dose of folic acid in women at high risk reduced the incidence of neural tube defects by 85%. According to ACOG, tThe recommended dose for non-high risk patients is at least 0.6 mg/day.

A 23-year-old G1P0 at 39 weeks gestation presents in spontaneous labor. Pregnancy was complicated by gestational diabetes. She delivers a 4200 gram infant with ruddy color and jitteriness. The infant is at immediate risk for which of the following conditions? A. Hyperglycemia B. Anemia C. Thrombocytopenia D. Polycythemia E. Hypercalcemia

D. Infants born to diabetic mothers are at increased risk for developing hypoglycemia, polycythemia, hyperbilirubinemia, hypocalcemia and respiratory distress. Thrombocytopenia is not a risk.

A 34-year-old G2P2 woman presents with inter-menstrual bleeding for one year. The bleeding typically occurs two weeks after her menses and last two to three days. The symptoms began one year ago and the bleeding has not changed recently. She is currently taking oral contraceptives. On pelvic examination, the cervix appears normal and the uterus is normal in size and shape. Her urine pregnancy test is negative; an endometrial biopsy is negative for hyperplasia. Which of the following tests or procedures would be indicated for further work-up? A. Prolactin level B. Progesterone level C. Hysterosalpingogram (HSG) D. Pelvic ultrasound E. Colposcopy

D. Intermenstrual bleeding is frequently caused by structural abnormalities of the endometrial cavity, such as myomas, polyps or malignancy. An ultrasound would be helpful as the next step in diagnosis. Although an HSG might reveal structural abnormalities, it is too invasive as the next step. A colposcopy would not be helpful in the diagnosis, nor would obtaining a Prolactin level, as it would be indicated for the evaluation of anovulatory bleeding. Progesterone levels are not helpful in a patient on oral contraceptives.

A 32-year-old G3P2 woman with a last menstrual period two weeks ago presents with a six-month history of abdominal pain. She has noncyclic intermittent pain, which she describes as crampy and diffuse across the lower abdomen. Her pain is typically relieved with defecation and is associated with loose, watery stools. Onset of the symptoms is associated with a change in stool frequency from once daily to multiple times daily. She also experiences bloating and abdominal distention several times a week. Her medical history is significant for chronic migraines and she denies previous surgery. Her gynecological history is unremarkable. Her abdominal examination is notable for mild tenderness to palpation in the left lower quadrant, and her pelvic examination is normal. What is the most likely diagnosis in this patient? A. Pelvic adhesions B. Diverticulosis C. Endometriosis D. Irritable bowel syndrome E. C. difficile colitis

D. Irritable bowel syndrome (IBS) is a common functional bowel disorder of uncertain etiology. It is characterized by a chronic, relapsing pattern of abdominal and pelvic pain, and bowel dysfunction with constipation or diarrhea. IBS is one of the most common disorders associated with chronic pelvic pain. IBS appears to occur more commonly in women with chronic pelvic pain than in the general population. Diagnosis is based on the Rome II Criteria for IBS, which includes at least 12 weeks (need not be consecutive) in the preceding 12 months of abdominal discomfort or pain that has two of three features: 1) relief with defecation; 2) onset associated with a change in frequency of stool; or 3) onset associated with a change in stool form or appearance. The patient's history does not support pelvic adhesions, and diverticulosis (although very common) typically may be asymptomatic unless inflammation/infection develops. In this case, the symptoms for IBS may be indistinguishable from diverticulitis or severe diverticular disease. Although severe endometriosis may affect the lower bowel with constricting and invasive implants, the lack of any gynecologic/menstrual symptoms and the normal pelvic examination essentially excludes this diagnosis. The lack of recent antibiotic exposure essentially rules out the diagnosis of C. difficile.

A 32-year-old G3P1 woman presents to your office today because of exposure to hepatitis B. She had vaginal and anal intercourse with a new partner three days ago and did not use condoms. The partner informed her today he was recently diagnosed with acute hepatitis B acquired from intravenous drug use and needle sharing. She has no prior history of hepatitis B infection and has not been vaccinated. She is currently asymptomatic and her examination is normal. Her urine pregnancy test is negative. What is the next best step in the management of this patient? A. Check AST, ALT, and HBsAg B. Administer HBIG one dose C. Administer HBIG two doses D. Administer HBIG and start hepatitis B vaccine series E. Administer hepatitis B vaccine series only

D. It is estimated that 38% of hepatitis B cases worldwide are acquired from sexual transmission. Post-exposure prophlaxis should be inititated as soon as possible but not later than 7 days after blood contact and within 14 days after sexual exposure. In individuals who are unvaccinated but exposed to persons who are HBsAG positive, recommendations are to receive one dose of HBIG (Hepatitis B Immune Globulin) and the HBV (Hepatitis B Vaccine Series). If the source is HBsAG negative or unknown status, then only the HBV series is used. If the exposed individual has been vaccinated and is a responder then no further treatment is necessary. If the exposed individual is vaccinated and a non-responder, then HBIG plus HBV or HBIG times two doses is used. Because the incubation period for the virus is six weeks to six months, checking liver function and immunologic status at this time is not indicated.`

A 69-year-old G3P3 comes in for a health maintenance examination. Her younger sister was recently diagnosed with endometrial cancer and she is concerned about her risk. Your patient experienced her last menstrual period at age 49, and she has not had any bleeding since. Her medications include only a multivitamin and supplemental calcium. She has no other significant family history. Her physical examination including a pelvic examination is normal. She is 5 feet 5 inches tall and weighs 120 pounds. What is the most appropriate management for this patient? A. Endometrial biopsy B. CA125 level C. Ultrasound with measurement of the endometrial lining D. Annual exams E. Refer to genetic counselor for risk assessment

D. Less than 5% of women diagnosed with endometrial cancer are asymptomatic. Approximately 80-90% of women with endometrial carcinoma present with vaginal bleeding or discharge as their only presenting symptom. Since this patient does not have any symptoms or risk factors for endometrial cancer, she does not need to have any diagnostic testing. Risk factors for endometrial cancer include late menopause, unopposed estrogen therapy, nulliparity, obesity, Tamoxifen therapy and diabetes mellitus. Although sometimes associated with Hereditary Non-polyposis Colorectal Cancer Syndrome (HNPCC, or Lynch II), endometrial cancer is typically not a genetically-inherited malignancy, and so genetic counseling for risk assessment would not be recommended unless a more significant family history existed. Endometrial cancer ranks as the fourth most common cancer detected in women in the US. In 2010, according to the American Cancer Society, there will be an estimated 43,470 new endometrial cancer cases. It is the most common gynecologic malignancy. Top Five Cancers Detected in Women: • Breast 28% • Lung 14% • Colon 10% • Uterine 6% • Ovary 3% Gynecologic Cancers: • Uterine 52% • Ovary 26% • Cervix 14% • Vulva 5% • Vagina 3%

A 13-year-old female is brought to the physician for increasingly severe abdominal pain. The pain is now constant and mildly uncomfortable, but every month she has a week when it is more severe. She has Tanner stage II breasts and pubic hair development. On genital examination, there is a bluish mass pushing the labia open. What is the most likely cause of this patient's abdominal pain? A. Turner's syndrome B. Transverse vaginal septum C. Isolated atresia of the cervix D. Imperforate hymen E. Synechiae of the uterine cavity

D. Lower genital tract malformations occur in 1 in 10,000 females and are most commonly an imperforate hymen where the genital plate canalization is incomplete. Amenorrhea and abdominal pain are also associated with isolated atresia of the vagina or cervix. The menstrual blood will collect in the vagina and uterus causing pain. Treatment involves surgical correction. When a transverse vaginal septum is present, a normal vaginal opening with a short blind vagina and pelvic mass may be located above the level of the obstruction found on exam. Asherman's syndrome is associated with secondary amenorrhea resulting from intrauterine scarring/synechiae.

A 25-year-old G1P0 woman at six weeks gestation comes to the office because of undesired pregnancy. You discuss with her the risks and benefits of surgical versus medical abortion using misoprostol and mifepristone. Compared to surgical abortion, which of the following is increased in a woman undergoing a medical abortion? A. Post abortion pain B. Lower failure rate C. Long-term psychological sequelae D. Blood loss E. Future infertility

D. Medical abortion is associated with higher blood loss than surgical abortion. Early in pregnancy (less than 49 days) both medical and surgical procedures can be offered. Mifepristone (an antiprogestin) can be administered, followed by misoprostol (a prostaglandin) to induce uterine contractions to expel the products of conception. This approach has proven to be effective (96%) and safe. A surgical termination is required in the event of failure or excessive blood loss. Medical termination may be more desirable by some patients since they do not have to undergo a surgical procedure. It does not affect future fertility. Any termination of pregnancy, whether medical or surgical, can have psychological sequelae.

A 19-year-old G0 woman presents to the office with a two-week history of low pelvic pain and cramping. She has a new sexual partner and is on oral contraception and uses condoms. She is one week into her cycle. She has noted no vaginal discharge, itch or odor. She denies fevers or chills. She does note that she is on a new diet and has started drinking lots of water. As such, she notes that she is urinating much more frequently. Her examination is entirely unremarkable. Which of the following is the most appropriate next step in the management of this patient? A. Pelvic ultrasound B. Pap test C. Wet prep D. Urinalysis . Testing for chlamydia

D. Mildly symptomatic or asymptomatic urinary tract infections are common in female patients. Urinary tract infection must be considered in patients who present with low pelvic pain, urinary frequency, urinary urgency, hematuria or new issues with incontinence. While yearly screening for chlamydia is recommended for patients less than 25 years old, this patient's symptoms are most consistent with a UTI. A pelvic ultrasound is not indicated at this point.

A 23-year-old G1P1 woman is five days post-operative from a Cesarean section for arrest of labor at 7 cm. She now complains of minimal abdominal pain and drainage from the right side of the incision. Lochia is normal and she has no urinary complaints. Her vital signs are normal and she is afebrile. On physical exam, her lung and cardiac examinations are normal. Her abdomen and uterine fundus are nontender. Her Pfannenstiel incision has tenderness extending 3 cm from the incision and there is serous, bloody drainage coming from the right side. What is the next best step in the management of this patient? A. Initiate intravenous antibiotics B. Initiate oral antibiotics C. Occlusive dressing to the wound D. Open drainage of wound E. Tropical antibiotics to the wound

D. Mixed bacteria originating from the skin, uterus and vagina cause wound infections after a Cesarean section. Prior to establishing a diagnosis of surgical site infection, evaluation requires opening the wound, checking for fascial dehiscence, drainage and assessment of the fluid. Packing the wound until it has healed from the base of the wound facilitates the healing process. Broad spectrum antibiotics are indicated if you suspect cellulitis or abscess.

A 35-year-old G1P0 woman with last menstrual period one week ago presents with an eight-month history of pelvic pain. She reports regular menstrual cycles with moderate flow and dysmenorrhea, relieved with ibuprofen. She describes her pain as a deep, achy sensation with frequent sharp exacerbations. She has not been sexually active for the last several months because of dyspareunia and some arguments with her new partner of one year. She has no history of sexually transmitted infections. Her medical history is significant for irritable bowel syndrome, managed with a fiber supplement. She is a business executive. She has smoked one pack of cigarettes a day since age 25, and drinks a glass of wine three times a week. She tries to exercise regularly by running three to four times a week. This new pain is distinctly different from her IBS symptoms. Which of the following risk factors can contribute to increased incidence of pelvic pain in this patient? A. Alcohol use B. Smoking habit C. Occupation D. New partner E. Age

D. Most published evidence suggests a significant association of physical and sexual abuse with various chronic pain disorders. The arguments with the new partner allude to possible abuse. Studies have found that 40-50% of women with chronic pelvic pain have a history of abuse. Whether abuse (physical or sexual) specifically causes chronic pelvic pain is not clear, nor is a mechanism established by which abuse might lead to the development of chronic pelvic pain. Women with a history of sexual abuse and high somatization scores have been found to be more likely to have non-somatic pelvic pain, suggesting the link between abuse and chronic pelvic pain may be psychologic or neurologic. However, studies also suggest that trauma or abuse may also result in biophysical changes, by literally heightening a person's physical sensitivity to pain. While smoking can be associated with dysmenorrhea, she has been smoking for 10 years and her symptoms are recent onset.

A 32-year-old G2P2 woman delivered five days ago by uncomplicated vaginal delivery. Her postpartum course thus far has been unremarkable and she is breastfeeding without difficulty. She woke up in the middle of the night with intense upper abdominal pain and chills. She admits that she has had pain like this before, but never this severe. Her vital signs reveal blood pressure 120/70; pulse 110; and temperature 101.8° F (38.8° C). On physical examination, she has abdominal pain located in the right upper quadrant with rebound tenderness. Her uterine fundus is well below the umbilicus and nontender. Her lochia is normal. Laboratory tests reveal mild anemia, a slightly elevated white count and slightly elevated liver function tests. What is the most likely etiology of her pain? A. Endomyometritis B. Ruptured ovarian abscess C. Hepatitis D. Cholecystitis E. Appendicitis

D. Non-pregnancy related conditions must be considered when evaluating women in the postpartum period. Pregnancy puts women at risk for cholelithiasis and, therefore, cholecystitis. Classic symptoms include nausea, vomiting, dyspepsia and upper abdominal pain after eating fatty foods. Treatment would be dependent on the severity of symptoms, but often involves cholecystectomy that is usually performed laparoscopically. Classic clinical findings for endomyometritis include fever and maternal tachycardia, uterine tenderness and no other localizing signs of infection. This patient is unlikely to have an ovarian cyst. Appendicitis presents with nausea, vomiting, anorexia and abdominal pain. Although hepatitis may be associated with elevated liver function tests, patients don't typically present with acute severe abdominal pain.

A 24-year-old G2P1 woman at 30 weeks gestation is sensitized to the D antigen. She is Rh-negative and received RhoGAM after her first delivery one year ago. Which of the following statements best explains these findings? A. The patient initiated her prenatal care late during the present pregnancy B. The patient was sensitized during the previous pregnancy by receiving the RhoGAM C. Current pregnancy is too close to the first pregnancy D. The amount of fetal maternal hemorrhage was more than previously estimated E. The cause is most likely idiopathic in this case

D. On rare occasion, an Rh-negative woman will subsequently be sensitized, despite prophylaxis. The protection afforded by a standard RhoGAM administration is dose-dependent. One dose will prevent Rh sensitization to an exposure of as much as 30 cc of Rh-positive red blood cells. With greater exposure, there is only partial protection and Rh sensitization may occur as a result of failure to diagnose massive transplacental hemorrhage. Alternatively, an Rh-negative woman may be sensitized in the latter part of pregnancy or soon after delivery before the post-delivery prophylaxis dose is given. Inadvertent maternal transfusion of Rh-positive blood may result in Rh sensitization to the D or another red blood cell antigen. Patients may become sensitized if they do not receive RhoGAM following an episode of antenatal bleeding or after an invasive procedure, such as amniocentesis or chorionic villus sampling. In addition, RhoGAM only confers protection against the D antigen. Therefore, despite administration of RhoGAM to Rh-negative patients, they may still become sensitized to other red blood cell antigens. Pregnancy spacing does not affect the presence of the antibody.

A 38-year-old G1P0 woman undergoes dilation and curettage for a partial molar pregnancy. The patient and her husband are very devastated by the loss of this much-desired pregnancy. Because she feels that her "reproductive clock" is ticking away, the patient would like to get pregnant as soon as possible. How long should she wait before attempting pregnancy? A. After recovery from the dilation and curettage B. After the Beta-hCG normalizes C. After she has one normal menstrual cycle D. Six months after negative Beta-hCG levels E. Two years

D. Once evacuation has been accomplished, patients must be followed regularly with serial Beta-hCG levels to insure spontaneous regression. Pregnancy should be avoided during this follow-up period, and for the following six months. Effective contraception (OCP or other hormonal contraception) is strongly recommended to prevent confusion in interpreting a rising Beta-hCG as a post-molar recurrence/progression versus a new, spontaneous pregnancy. Given this patient's age and desire for a pregnancy, waiting two years decreases her fertility and increases her risks of pregnancy complications.

A 36-year-old G1 with type 1 diabetes is diagnosed with intrauterine growth restriction at 33 weeks gestation. What is the most appropriate next step in management? A. Amniocentesis B. Immediate delivery C. Weekly ultrasounds to assess fetal growth D. Antenatal testing of fetal well-being E. Observation

D. Once intrauterine growth restriction is detected, the fetus needs to be evaluated periodically for evidence of well-being until delivery is deemed necessary. This will result in once or twice weekly testing, depending on the modality of assessment that is being used. Testing includes: non-stress test (NST), where the fetal heart beat is recorded over a period of at least 30 minutes while looking for accelerations with fetal movement, and the biophysical profile, which includes an ultrasound evaluation of fetal movement, fetal tone, amniotic fluid and breathing. NSTs should be performed twice weekly with at least a weekly AFI. The BPP may be performed weekly. Ultrasound for fetal growth is not useful if more frequent than every two weeks. An amniocentesis for fetal lung maturity can be considered at more advanced gestational age.

A 32-year-old G2P1 woman is at 42 weeks gestation. Her prenatal course was uncomplicated and she had a first trimester ultrasound confirming dates. Her cervix is 4 cm dilated and 100% effaced. She does not report contractions and states there is good fetal movement. What is the next best step in the management of this patient? A. Ultrasound to assess amniotic fluid volume B. Twice weekly non-stress test (NST) and amniotic fluid index (AFI) C. Daily biophysical profiles D. Admit for induction E. Ultrasound to assess fetal growth

D. Optimal management for the patient with a favorable cervix at greater than or equal to 41 weeks gestation is delivery. Her dilation and effacement make it likely her induction will be successful. Induction of labor in a patient with an unfavorable cervix increases the risk of Cesarean section significantly, compared to a patient who goes into spontaneous labor. It is not advisable to follow a patient who is >42 weeks with antepartum fetal testing, such as twice weekly non-stress tests with amniotic fluid index, if the gestational age is certain. Performing an ultrasound to assess fetal growth and/or amniotic fluid volume should not change the management plan which should be induction of labor at this gestational age.

A 23-year-old G1P0 comes into the office after having some light inter-menstrual spotting and cramping. She is currently sexually active and has had unprotected intercourse with two different partners over the past three months. A urine pregnancy test is positive. She does not desire to keep the pregnancy and, after an ultrasound scan in the office reveals a six-week viable intrauterine pregnancy, the patient asks about an abortion, but has no health insurance. What is the most appropriate next step in the management of this patient? A. You inform her that state Medicaid programs are not allowed to cover this service; therefore, you cannot perform the procedure B. You recommend against the procedure due to potential complications with future infertility C. You request she seeks the opinion of both of her partners before undergoing the procedure D. You support her decision for abortion after appropriate counseling E. e. You inform her that abortion should only be performed after six weeks gestation

D. Patients requesting abortion should be counseled appropriately regardless of their insurance status and do not have to obtain the consent of their partner to undergo the procedure. Although there are complications associated with pregnancy termination, they are significantly fewer than complications with carrying a pregnancy. Fewest complications occur when termination is done in the first trimester. From 1990 (the year in which the number of abortions was highest) to 1995, the annual number of legally induced abortions in the United States declined by 15%. Since 1990, factors contributing to the continued decrease in the proportion of pregnancies that ended in abortion might include a decrease in the number of unintended pregnancies, changes in contraceptive practices (including an increased use of condoms among young women), reduced access to abortion services and possible changes in attitudes concerning abortion. Clinical guidelines from the Society of Family Planning state that surgical abortion can be performed successfully and safely as early as three weeks from the onset of last menses if a protocol exists that includes sensitive pregnancy testing, immediate and meticulous examination of the aspirate, and assiduous follow-up of questionable specimens to rule out ectopic pregnancy or continuing gestation.

A 33-year-old G2P2 woman reports a two-year history of severe dysmenorrhea, menorrhagia and pelvic pain following the delivery of her last child. She describes her pelvic pain as primarily in the right lower quadrant, radiating into the vagina. Her pain worsens throughout the day with standing and is associated with pelvic pressure and fullness. Her pelvic examination reveals a mildly enlarged uterus with marked tenderness to palpation of the right adnexa, and no other significant findings. A vaginal ultrasound with color-flow Doppler reveals multiple dilated vessels traversing the right broad ligament to the lower uterus and cervix. The uterus shows no fibroids or other significant changes. Endometrial thickness appears normal. Which of the following is the most likely diagnosis in this patient? A. Endometriosis B. Endometritis C. Adenomyosis D. Pelvic congestion E. Pelvic floor relaxation

D. Pelvic congestion syndrome is a cause of chronic pelvic pain occurring in the setting of pelvic varicosities. The unique characteristics of the pelvic veins make them vulnerable to chronic dilatation with stasis leading to vascular congestion. These veins are thin walled and unsupported, with relatively weak attachments between the supporting connective tissue. The cause of pelvic vein congestion is unknown. Hormonal factors contribute to vasodilatation when pelvic veins are exposed to high concentration of estradiol, which inhibits reflex vasoconstriction of vessels, induces uterine enlargement with selective dilatation of ovarian and uterine veins. This pain may be of variable intensity and duration, is worse premenstrually and during pregnancy, and is aggravated by standing, fatigue and coitus. The pain is often described as a pelvic "fullness" or "heaviness," which may extend to the vulvar area and legs. Associated symptoms include vaginal discharge, backache and urinary frequency. Menstrual cycle defects and dysmenorrhea are common. No signs of pelvic floor relaxation were noted on exam.

A 30-year-old G4P3 woman at 24 weeks gestation is found to have an anterior placenta previa. She has a history of three prior Cesarean deliveries. What is the most likely serious complication that can lead to obstetric hemorrhage in this woman? A. Placental abruption B. Uterine dehiscence prior to labor C. Uterine inversion D. Placenta accreta E. Uterine atony

D. Placental abruption and uterine atony are both common, but, in the presence of a low-lying anterior placenta in a patient with a history of multiple Cesarean births, the diagnosis of the placenta accreta must be entertained. Placenta accreta is an abnormally firm attachment of the placenta to the uterine wall. The incidence of placenta accreta may be increasing because of the rise in the number of women with previous Cesarean deliveries. This is a serious obstetric complication leading to retained placenta and severe postpartum hemorrhage. Hysterectomy is frequently required due to intractable hemorrhage at delivery.

A 20-year-old G2P1 woman is at 41 weeks gestation. Her prenatal course and past medical history are unremarkable. She has not had any complications with her pregnancy and fetal surveillance is reassuring. Which of the following complications is most likely to occur in this pregnancy? A. Preeclampsia B. Retained placenta C. Postpartum hemorrhage D. Macrosomia E. Placenta abruption

D. Postterm pregnancies are associated with macrosomia, oligohydramnios, meconium aspiration, uteroplacental insufficiency and dysmaturity. Although postterm infants are larger than term infants and have an increased incidence of fetal macrosomia, there is no evidence to support induction of labor as a preventive measure for macrosomia in these cases. There is no associated risk for preeclampsia in postterm gestations.

A 32-year-old G0 woman presents with amenorrhea for the last three months. She has a long history of irregular cycles, 26 to 45 days apart, for the last two years. She is otherwise in good health and is not taking any medications. She is sexually active with her husband and uses condoms for contraception. She is 5 feet 4 inches tall and weighs 140 pounds. On exam, she has a slightly enlarged, non-tender uterus. There are no adnexal masses. Which of the following is the most appropriate test to obtain in this patient? A. Thyroid stimulating hormone (TSH) B. Progesterone and estrogen C. Follicle stimulating hormone (FSH) D. Urine pregnancy test E. Pelvic ultrasound

D. Pregnancy is the most common cause of amenorrhea. It is important to consider it early in the workup to avoid unnecessary tests, procedures and treatments that may be contraindicated during pregnancy. Although the patient has a history of irregular cycles and is using condoms for contraception, it is important to first rule out pregnancy before initiating further work-up.

A 22-year-old G2P1 woman presents for prenatal care at approximately 10 weeks gestation. Her first pregnancy was complicated by preterm premature rupture of the membranes at 28 weeks gestation. Which of the following interventions could reduce the risk of preterm premature rupture of the membranes during this pregnancy? A. Bedrest B. Placement of a cerclage C. Placement of a Tertbutaline pump D. 17 alpha-hydroxyprogesterone E. Nifedipine

D. Premature rupture of the membranes occurs in approximately 10-15% of all pregnancies. Preterm premature rupture of the membranes between 16 and 26 weeks gestation is identified in 1% of pregnancies. Preterm premature rupture of the membranes occurs in 1/3 of all preterm deliveries. The reported recurrence rate for preterm premature rupture of the membranes is approximately 32% when it occurred in the index pregnancy. Bedrest and tocolytics have not been shown to reduce the risk for PPROM, and may have detrimental effects to the mother. A cerclage may be indicated for patients with a history of an incompetent cervix. 17 alpha-hydroxyprogesterone has been shown to reduce the risk of premature labor. 17 alpha-hydroxyprogesterone is administered weekly (starting between 16-20 weeks) until 36 weeks gestation.

A 24-year-old G1P0 woman at 34 weeks gestation is planning to breastfeed her baby. Several hormones of pregnancy are responsible in order for the breasts to produce milk. Which of the following hormones is responsible for synthesis of milk? A. Estrogen B. Oxytocin C. Cortisol D. Prolactin E. Human placental lactogen

D. Progesterone, estrogen, and placental lactogen, as well as prolactin, cortisol, and insulin, appear to act in concert to stimulate the growth and development of the milk-secreting apparatus of the mammary gland. Prolactin is responsible for the synthesis of milk, but although present in large quantities during gestation, its action is inhibited by the hormones of pregnancy, particularly estrogen and progesterone. After delivery, large amounts of prolactin continue to be secreted, milk is produced after the inhibitory action of estrogen and progesterone is lifted.

A 22-year-old G1P0 woman presents at 42 weeks gestation. Her cervix is long and closed. She does not report contractions and states there is good fetal movement. You discuss the benefits of induction at this time versus waiting until she goes into labor spontaneously. She agrees to proceed with an induction. Which of the following is the best next step in the management of this patient? A. Artificial rupture of membranes B. Membrane stripping C. Oxytocin infusion D. Prostaglandin E1 tablet E. RU486 (progesterone antagonist)

D. Prostaglandins applied locally are the most commonly-used cervical ripening agents. RU486 is not used for cervical ripening. Membrane stripping (digital separation of chorioamnion from lower uterine segment) and artificial rupture of membranes cannot be performed in a patient with a closed cervix. The American College of Obstetricians and Gynecologists (ACOG) recommendations for the management of postterm pregnancy includes patient records fetal kick counts and fetal surveillance using one of the following: NST, CST, biophysical profile and delivery for nonreassuring testing. If the patient has a favorable cervix, induce at 42 weeks and, if the cervix is unfavorable, use cervical ripening agents.

An African-American couple comes to you for preconception counseling. Neither one has any significant family or genetic history. Based on their African-American descent, which of the following blood tests would you recommend? A. MCV and CBC B. Sickle cell preparation and CBC C. Peripheral blood smear and CBC D. Hemoglobin electrophoresis and CBC E. Sickle cell preparation with a hemoglobin electrophoresis, if the sickle preparation is abnormal

D. Screening for carriers of both alpha and beta thalassemia is possible by evaluation of red cell indices. Although solubility tests for hemoglobin S or sickle cell preparations can be used for screening, hemoglobin electrophoresis is definitive and preferable because other hemoglobinopathies can also be detected including hemoglobin C trait and thalassemia minor. Although sickle cells can be identified on a blood smear in individuals with sickle cell disease, the cells may be absent in individuals with milder types of sickle cell disease and even in some individuals with severe sickle cell disease. Evaluation of a peripheral smear is not useful in detecting carriers for sickle cell disease.

A 34-year-old G1 woman at eight weeks gestation presents for prenatal care. She is healthy and takes no medications. Family history reveals type 2 diabetes in her parents and brothers. She is 5 feet 2 inches tall and weighs 220 pounds (BMI 40.2 kg/m2). Which of the following is the best recommendation to screen her for gestational diabetes? A. Screen at 24-28 weeks with a 50-g oral glucose challenge test B. Screen at 16-20 weeks with a 50-g oral glucose challenge test C. Screen at 12 weeks with a 50-g oral glucose challenge test D. Screen now with a 50-g oral glucose challenge test E. Begin an oral hypoglycemic agent now

D. Screening should be performed between 24 and 28 weeks in those women not known to have glucose intolerance earlier in pregnancy. This evaluation can be done in two steps: a 50-g oral glucose challenge test is followed by a diagnostic 100-g oral glucose tolerance test (OGTT) if initial results exceed a predetermined plasma glucose concentration. Patients at low risk are not routinely screened. For those patients of average risk screening is performed at 24-28 weeks while those at high risk (severe obesity and strong family history) screening should be done as soon as feasible.

A 37-year-old G1P1 woman has experienced symptoms of depression and difficulty concentrating the week prior to her menstrual period for the last three years, since her tubal ligation. She kept a symptom diary for three months revealing symptoms clustered around her menstrual cycle. She was diagnosed with premenstrual syndrome and began a regular exercise routine with dietary modifications, but only noticed mild relief in her symptoms. Work-up is otherwise unremarkable. Which of the following will most likely alleviate her symptoms? A. Evening primrose oil B. Ginkgo C. Progesterone cream D. Fluoxetine hydrochloride E. Levothyroxine sodium

D. Selective serotonin reuptake inhibitors increase the amount of active serotonin in the brain and have been found to be effective in alleviating PMS and PMDD symptoms. Patients can take the medication either every day or for 10 days during the luteal phase. Progesterone cream will not help her symptoms. Herbal therapies such as evening primrose oil and ginkgo have not been shown to improve PMDD symptoms. Symptoms due to hypothyroidism would be present throughout her cycle.

A 23-year-old woman presents with complaints of a bilateral nipple itchy sensation for six months. There is no nipple discharge or dry skin. She reports her nipple appears to be swollen at times and there is an erythematous fine rash. She had breast implants placed five years ago, but otherwise has no significant medical problems or surgical history. What is the most likely cause of her symptoms? A. Fibroadenoma B. Breast cancer C. Rupture of breast implants D. Chemical irritants E. Mastitis

D. She's young, likely not to be breast cancer.

Mary is a 65-year-old G2P2 woman with lung metastases from cervical cancer. She was recently weaned from mechanical ventilation after being on the ventilator for four weeks. She has a tracheostomy. Mary currently has worsening pulmonary function and needs to go back on the ventilator or she will die within a few days. Mary's husband, Jim, has power of attorney for Mary's health care decisions. The attending offers Mary a choice of either no ventilation with morphine for comfort or resumption of mechanical ventilation. Mary decides she prefers to go back on the ventilator. Jim prefers that she does not go back on the ventilator because the doctor has said that Mary may never wean off of the ventilator again. Who should make the decision about whether to put Mary back on the ventilator? A. Jim B. All of them together C. Mary's doctor in consultation with Jim D. Mary E. If Jim and Mary cannot agree, consult the hospital ethics committee

D. Since Mary is still competent, she can make her own decisions despite the fact that her husband has power of attorney.

A 25-year-old G1P1 woman presents with complaints of a white, watery nipple discharge for four months. She discontinued breastfeeding six months ago. She has been told in the past she had fibrocystic breast changes, but otherwise has no significant medical problems or surgical history. A white nipple discharge is noted on manual expression, but the exam is otherwise normal. She was then sent for a serum prolactin level which was 45 ng/ml (normal below 40 ng/ml). What is the most appropriate next step in the management of this patient? A. Obtain a brain MRI B. Obtain a beta-hCG C. Begin Bromocriptine D. Obtain a fasting prolactin level E. Order a ductogram

D. Stimulation of the breast during the physical examination may give rise to an elevated prolactin level. Accurate prolactin levels are best obtained in the fasting state. If still elevated, then a TSH level and brain MRI would be indicated to rule out a pituitary tumor. Post partum women may continue to produce milk for up to two years after cessation of breastfeeding. Although pathologic factors such as hypothyroidism, hypothalamic disorders, pituitary disorders (adenomas, empty sella syndrome), chest lesions (breast implants, thoracotomy scars, and herpes zoster) and renal failure can elevate prolactin levels, a non-significant benign elevation needs to be ruled out first. A ductogram is usually indicated in patients who have bloody discharge from a single breast duct.

A 39-year-old G4P3 woman with an ultrasound report suggestive of a molar pregnancy is referred for evaluation. She is asymptomatic. Her uterus is 16-weeks size and her Beta-hCG is >200,000 mIU/mL. What is the recommended treatment for this patient? A. Expectant management B. Induction with oxytocin C. Methotrexate D. Suction curettage E. Hysterectomy

D. Suction curettage is the standard management for molar pregnancies. Hysterectomy can be considered in women who have completed childbearing, however, the morbidity of a hysterectomy is still considered greater than suction curettage. Induction with oxytocin would result in severe bleeding once cervical dilation and contractions developed, and expectant management would risk increased growth and progression of the mole (as well as the similar unnecessary risk of bleeding.) Methotrexate may become necessary if she develops post-molar GTD, but not as a sole means of primary treatment.

A 32-year-old G2P2 woman complains of depression, weight gain and premenstrual bloating. She has suffered from these symptoms for 18 months and they have not responded to dietary changes and avoidance of alcohol and caffeine. Her only medications are multivitamins and herbs to increase her energy. She is very concerned about fatigue that often interferes with caring for her two children. A prospective symptom diary completed by the patient indicates mood symptoms, fatigue and bloating almost every day of the past two months, and regular menstrual cycles accompanied by breast tenderness. She denies feelings of wanting to hurt herself or others. Physical examination is unremarkable. Which of the following conditions is the most likely explanation for this patient's symptoms? A. Panic disorder B. Anxiety disorder C. Anemia D. Hypothyroidism E. Premenstrual dysphoric disorder

D. Symptoms of hypothyroidism can mimic typical symptoms of PMS, but symptoms occur more constantly throughout the cycle. Diagnosis involves complete work-up to rule out medical illnesses, including hypothyroidism. Although fatigue can be associated with anemia, her presentation is not consistent with this diagnosis.

A 19-year-old G3P0 woman at 39 weeks gestation with spontaneous rupture of membranes for 13 hours presented to labor and delivery. Her vital signs are: blood pressure 120/70; pulse 72; afebrile; fundal height 36 cm; and estimated fetal weight of 2700 gm. Cervix is dilated to 1 cm, 50% effaced, -2 station. Which statement best describes the tracing seen below? (fetal baseline in 170s, flattish line til end then irregular decel; CTX regular) A. Normal fetal heart rate with good variability and regular contractions B. Fetal tachycardia with good variability and regular contractions C. Normal fetal heart rate with poor variability and irregular contractions D. Fetal tachycardia with poor variability and regular contractions E. Normal fetal heart rate with good variability and irregular contractions

D. The baseline fetal heart rate is >160 with no accelerations or variability. There are regular contractions. Prolonged periods of fetal tachycardia are frequently found with maternal fever or chorioamnionitis.

A 23-year-old G0 woman presents to the office because she has not had any menses for four months. She has a long history of irregular menstrual cycles since menarche at age 14. She is otherwise in good health and is not taking any medications. She is thin and has chronic anxiety (BMI 16). Her Beta-hCG is < 5 mIU/mL, and her prolactin and TSH levels are normal. What would be the next best diagnostic test to order? A. Estrogen level B. Progesterone level C. Gonadotropin releasing hormone level (GnRH) D. Follicle stimulating hormone (FSH) E. Dehydroepiandrosterone sulfate (DHEAS)

D. The causes of hypothalamic-pituitary amenorrhea are functional (weight loss, obesity, excessive exercise), drugs (marijuana and tranquilizers), neoplasia (pituitary adenomas), psychogenic (chronic anxiety and anorexia nervosa), and certain other chronic medical conditions. In this case, the next step to make a diagnosis is to obtain FSH which would be expected to be in the low range. You already know that her prolactin level is normal, which is consistent with the diagnosis. Prolactin would be elevated with a prolactin-secreting pituitary adenoma.

A 22-year-old G2P1 woman has a history of a previous postterm pregnancy. She delivered a 3500 g healthy male infant at 42-½ weeks gestation via a Cesarean section secondary to fetal distress. She is currently 15 weeks pregnant, based on an irregular last menstrual period. What is the most appropriate management at this time? A. Plan for a repeat Cesarean section at 38 completed weeks B. Schedule for a repeat Cesarean section if she does not go into spontaneous labor by 40 weeks gestation C. Plan to admit the patient for an induction of labor (a VBAC) if she does not go into spontaneous labor by 41 weeks gestation D. Obtain a fetal ultrasound to date the pregnancy E. Start weekly non-stress tests and amniotic fluid indexes at 40 weeks gestation and proceed with either induction of labor or Cesarean section for a nonreactive non-stress test or oligohydramnios or if patient has not gone into spontaneous labor by 41 weeks gestation

D. The diagnosis of postterm pregnancy is based on the establishment of an accurate gestational age. In a patient with irregular menses, it is important to obtain an ultrasound prior to 20 weeks to accurately date the pregnancy. It is reasonable to allow a patient with reassuring fetal surveillance to go past 41 weeks gestation if her gestational age is accurately known. However, because of a prior Cesarean birth, consideration should be given to delivery before 41 weeks. Approximately 50% of patients with a history of a postterm pregnancy will experience prolonged pregnancy with the next gestation. There is currently no indication to begin antenatal testing at 40 weeks in this pregnancy.

A 47-year-old G3P3 presents with a several month history of progressive abdominal bloating. She has had regular menses her entire life, but recently notes her bleeding to be heavier and occurring "twice a month." She is otherwise healthy and does not smoke or drink. On examination, she is 5 feet 5 inches tall and weighs 130 pounds. Her abdominal exam is notable for some mild distension, but no palpable masses. Her pelvic examination is notable for a normal appearing cervix, a deviated, but non-enlarged uterus, and a 10 cm mobile, non-tender right adnexal mass. An office endometrial biopsy reveals complex endometrial hyperplasia without atypia. What is the most likely explanation for the adnexal mass and the findings seen on the endometrial biopsy? A. Fibroid uterus B. Endometrioma C. Metastatic endometrial cancer D. Granulosa cell tumor E. Theca lutein cyst

D. The finding of an adnexal mass in a perimenopausal woman raises the suspicion of a neoplastic process. Because of the new onset of irregular bleeding and the finding of hyperplasia, the most likely explanation would be that of a granulosa cell tumor, an estrogen-secreting tumor. A theca lutein cyst is typically seen in the setting of pregnancy (molar pregnancy) and is often bilateral. A fibroid uterus may present with heavy irregular bleeding, but a pedunculated fibroid mimicking an adnexal mass is unlikely to present with such a bleeding pattern and has no correlation with hyperplasia. Severe endometriosis often presents with dysmenorrhea and is unlikely to develop in the perimenopause.

A mother brings her 16-year-old daughter to the doctor because she has not begun menses. She performs poorly in school because of dyslexia. On physical examination, she is 4 feet 11 inches tall, 100 pounds and has Tanner stage I breast and pubic hair growth. Her chest is broad, breast nipples are widely spaced and her neck is thickened. No genital tract abnormalities are noted on exam. Which of the following is the most likely cause of her delayed sexual maturation? A. Partial deletion of the long arm of the X chromosome B. Down Syndrome C. Noonan Syndrome D. Turner Syndrome E. Rokitansky-Kuster-Hauser Syndrome

D. The genetic defect of Turner syndrome is the absence of one of the X chromosomes. These females have failure to establish secondary sexual characteristics, short stature and characteristic physical features: pterygium colli, shield chest and cubitus valgus. Partial deletions of the long arm of the X chromosome also cause premature ovarian failure. The average age of puberty in females with Down syndrome is not significantly different than normal females. Clinically, patients with Noonan syndrome typically have normal puberty and fertility. They may have short stature, webbed neck, heart defects, and abnormal faces. Individuals with Noonan syndrome have a normal karyotype. Rokitansky-Kuster-Hauser Syndrome causes vaginal and uterine agenesis and is not suspected in this case due to the normal pelvic exam findings.

A 23-year-old G1 woman with six weeks amenorrhea presents with lower abdominal pain and vaginal bleeding. Her temperature is 102.0°F (38.9°C) and the cervix is 1 cm dilated. Uterus is eight-week size, tender and there are no adnexal masses. Urine pregnancy test is positive. Which of the following is the most appropriate next step in the management of this patient? A. Observation B. Single-agent antibiotics C. Medical termination of pregnancy plus antibiotics D. Uterine evacuation plus antibiotics E. Laparoscopy plus antibiotics

D. The management of septic abortion includes broad-spectrum antibiotics and uterine evacuation. Single agent antimicrobials do not provide adequate coverage for the array of organisms that may be involved and therefore are not indicated. A laparoscopy can be indicated if ectopic pregnancy is suspected, but it is unlikely in this case. Medical termination is not the best option since prompt evacuation of the uterus is indicated for septic abortion.

A 30-year-old G1P1 woman presents to the emergency department with left-sided abdominal pain. Physical examination is notable for a 5 x 6 cm mobile left adnexal mass. An ultrasound is performed, which shows a left ovarian mass with cystic and solid components. Which of the following is the most likely diagnosis in this patient? A. Serous cystadenoma B. Mucinous cystadenoma C. Endometrioid tumor D. Dermoid tumor E. Brenner tumor

D. The most common tumor found in women of all ages is the dermoid. The median age of occurrence is 30 years, and 80% occur during the reproductive years. Dermoids may contain differentiated tissue from all three embryonic germ layers. Dermoid tumors can contain teeth, hair, sweat and sebaceous glands, cartilage, bone, and fat.

A 38-year-old G1P0 woman is admitted at 42 weeks gestation with an anencephalic infant for induction of labor. The attending physician decides not to monitor the baby's heart rate during labor because he would not intervene with a Cesarean section in the event of fetal distress or demise. The physician's action is justified by which one of the following concepts? A. Beneficence to the fetus B. Respect for autonomy to the patient C. Maleficence to the fetus D. Non-maleficence to the patient E. Justice for the patient

D. The non-malfeasance principle expresses the concept that professionals have a duty to protect the patient from harm. Since an anencephalic infant will not survive, performing a Cesarean section on this patient will cause her harm. Beneficence principle expresses the concept that professionals have a duty to act for the benefit of others, and, in this case, performing a Cesarean section will not benefit the fetus.

A 34-year-old G4P3 woman at 36 weeks with a twin gestation presents in labor. She has three prior normal spontaneous vaginal deliveries at term, with the largest infant weighing 3400 grams. Twin A is breech with an estimated fetal weight of 2800 gm and twin B is vertex, with an estimated fetal weight of 3200 gm. Which of the following is an appropriate delivery option for this patient? A. Total breech extraction of twin A, vaginal delivery of twin B B. External cephalic version for twin A, vaginal delivery twin of B C. Operative vaginal delivery for twin A and vaginal delivery for twin B D. Cesarean delivery E. Vaginal delivery for twin A and Cesarean delivery for twin B

D. The optimal mode of delivery for twins in which the first twin is in the breech presentation is by Cesarean section. Similar to singletons, if the first twin is breech problems can occur including head entrapment and umbilical cord prolapse. When the presenting twin is vertex and twin B is not vertex, controversy exists as to the optimal mode of delivery. A small randomized study comparing Cesarean delivery with vaginal delivery for vertex-non-vertex twins failed to show an advantage for Cesarean delivery, but did not have statistical power to address rare neonatal morbidities. Some authors have advocated external cephalic version for management of the second twin; however, observational studies have not shown any advantage of this approach compared to total breech extraction.

A 23-year-old G2P1 woman with six weeks amenorrhea presents with lower abdominal pain and vaginal bleeding. Her temperature is 102.0°F (38.9°C) and the cervix is 1 cm dilated. Uterus is eight-week size and tender. There are no adnexal masses. Urine pregnancy test is positive. What is the most likely diagnosis? A. Threatened abortion B. Missed abortion C. Normal pregnancy D. Septic abortion E. Ectopic Pregnancy

D. The patient has a septic abortion. She has fever and bleeding with a dilated cervix which are findings seen with septic abortion. Threatened abortions clinically have vaginal bleeding, a positive pregnancy test and a cervical os closed or uneffaced. Missed abortions have retention of a nonviable intrauterine pregnancy for an extended period of time (i.e. dead embryo or blighted ovum). A normal pregnancy would have a closed cervix. Ectopic pregnancy would likely present with bleeding, abdominal pain, possibly have an adnexal mass, and the cervix would typically be closed.

A 35-year-old G3P2 woman presents for her initial prenatal care visit at 15 weeks gestation, according to her last menstrual period. She reports that a home pregnancy test was positive about five weeks ago. Review of her history is unremarkable and her entire family is in good health. Physical examination reveals a ten-week size uterus. Which of the following is the most appropriate next step in establishing this pregnancy's gestational age? A. Checking fetal heart tones B. Hysterosonogram C. Quantitative Beta-hCG D. Obstetrical ultrasound E. Quadruple screen

D. The patient's gestational age based on her LMP and the findings on physical exam are discordant. In this case, the most reliable method of confirming gestational age is a dating ultrasound. A quantitative Beta-hCG will not reliably predict the gestational age. The uterine size on physical exam is not the most accurate way to date a pregnancy. An ultrasound performed between 14 and 15 6/7 weeks gestation should be used to date the pregnancy if there is greater than a 7 day discrepancy from the menstrual dates or more than a 10 day discrepancy if the ultrasound is performed between 16 and 21 6/7 weeks. . First trimester ultrasound provides the most accurate assessment of gestational age and can give an accurate estimated date of confinement (EDC) to within 3-5 days.

A 33-year-old G0 woman presents with amenorrhea for the past 12 months. She also reports a recent onset of dyspareunia, causing her to feel anxious about having intercourse. She had menarche at age 15. Her cycles were normal until two years ago when she began skipping menses. She is otherwise in good health. She is 5 feet 4 inches tall and weighs 130 pounds. Her physical examination is completely normal. TSH and prolactin levels are normal. Urine pregnancy test is negative. What is the most likely cause of this patient's amenorrhea? A. Psychogenic B. Genital tract outflow obstruction C. Asherman's syndrome D. Premature ovarian failure E. Pituitary adenoma

D. The patient's symptom of dyspareunia is likely caused by vaginal dryness, which is associated with estrogen deficiency. Hypergonadotropic amenorrhea is the result of ovarian failure or follicular resistance to gonadotropin stimulation. The history, physical exam and labs make the other possibilities less likely: psychogenic disorder (no chronic anxiety or anorexia nervosa), outflow obstruction (previously had periods), Asherman's syndrome (no history of pregnancy or intrauterine procedures), or a pituitary tumor (normal labs).

A 22-year-old G0 woman presents with painful menstruation that limits her activities each month. She describes the pain as spasmodic occurring on days one to three of bleeding since her cycles began. Other symptoms include nausea, nervousness, diarrhea, and headache. Her physical exam is normal with a soft, non-tender abdomen. Bimanual exam reveals a fixed uterus with uterosacral ligament nodularity. There are no adnexal masses noted. Which of the following is the most likely diagnosis in this patient? A. Premenstrual syndrome B. Premenstrual dysphoric disorder C. Primary dysmenorrhea D. Secondary dysmenorrhea E. Adenomyosis

D. The physical examination in patients with primary dysmenorrhea is normal. There should not be any palpable abnormalities on abdominal, speculum, pelvic, bimanual, and rectal examinations. The restricted uterine motion found on exam suggests the possibility of endometriosis or pelvic scarring from inflammation or adhesions. These conditions must be considered in establishing the etiology of her diagnosis. Childbearing does not affect the occurrence of either diagnosis. Although the patient's symptoms, including the associated symptoms, timing of initial onset, and cyclic nature of her pain are consistent with primary dysmenorrhea, the finding on physical examination makes secondary dysmenorrhea the likely diagnosis.

A 20-year-old G1P0 woman has vaginal spotting and mild cramping for the last three days. Her last normal menstrual period was approximately seven weeks ago. She had a positive home pregnancy test. Vital signs are: blood pressure 120/72; pulse 64; respirations 18; temperature 98.6°F (37°C). On pelvic exam, she has scant old blood in the vagina, with a normal appearing cervix and no discharge. On bimanual exam, her uterus is nontender and small, and there are no adnexal masses palpable. Quantitative Beta-hCG 48 hours ago was 750 mIU/ml. Today, current Beta-hCG 760 mIU/ml; progesterone 3.2 ng/ml; hematocrit 37%. Transvaginal ultrasound shows a fluid collection in the uterus with a yolk sac but no fetal pole. A 3x3 cm cyst is seen on the left ovary. There is no free fluid in the pelvis. Which of the following is the most appropriate next step in the management of this patient? A. Exploratory laparoscopy B. Treat with methotrexate C. Treat with mifepristone D. Expectant management E. Repeat ultrasound in one week

D. The pregnancy is abnormal based on the abnormal Beta-hCG levels and the progesterone level. In a normal pregnancy, the level should rise by at least 50% every 48 hours until the pregnancy is 42 days old (after that time, the rise in level may not follow the projection curve). A progesterone level of <5 ng/ml suggests an abnormal or extrauterine pregnancy. In this instance, the pregnancy is intrauterine because of the presence of a yolk sac. Expectant management is appropriate and may avoid the risks of surgery. Other options include misoprostol, manual vacuum aspiration, or dilation and curettage. Laparoscopy and methotrexate are not indicated as this is a confirmed intrauterine pregnancy. Mifepristone is a progestin receptor antagonist and can be used as emergency contraception to prevent ovulation and blocks the action of progesterone which is needed to maintain pregnancy. In the US, Mifepristone is also used with misoprostol for pregnancy termination.

A 32-year-old G0 woman comes to your office because she has been unable to conceive for one year. She is currently in a mutually monogamous relationship with her husband, has intercourse three times per week, and has no dyspareunia. Her menstrual cycles occur every 26-34 days. She has had seven sexual partners in the past. She was treated for multiple sexually transmitted infections including gonorrhea, chlamydia and pelvic inflammatory disease in her early twenties. She had an abnormal Pap test about four years ago and was treated with a LEEP. What is the most likely underlying cause of infertility in this patient? A. Luteal phase defect B. Cervical stenosis C. Ovulatory dysfunction D. Tubal disease E. Endometriosis

D. The rate of tubal infertility has been reported as 12% after one episode of PID, 25% after 2 episodes and 50% after three episodes. Salpingitis can develop in 15-30% of women with inadequately treated gonococcal or chlamydial infections and can result in significant long-term sequelae, such as chronic pelvic pain, hydrosalpinx, tubal scarring and ectopic pregnancy. Given this patient's history, her inability to conceive is most likely due to the long-term sequelae of a sexually transmitted infection. Although the patient had a LEEP, risk for cervical stenosis is low. She is having regular cycles; therefore, anovulation and luteal phase defect is less likely. This case emphasizes the importance of aggressive screening and treatment protocols for sexually transmitted infections, as well as counseling regarding abstinence and safer sex practices. While endometriosis can cause tubal occlusion, her clinical presentation is not consistent with endometriosis.

A 35-year-old G0 Asian woman presents with irregular menses and hirsutism of three months duration. The patient has no family history of hirsutism. On exam, the patient was noted to have terminal hair growth on her chest and recently had laser treatment to remove similar hair on her chin. Her total testosterone is 76 ng/dl (normal) and her DHEAS is 1500µg/dl (elevated). Which of the following is the most likely diagnosis in this patient? A. Pituitary adenoma B. Ovarian tumor C. Cushing's syndrome D. Adrenal tumor E. Idiopathic

D. The short duration of symptoms and the significantly elevated DHEAS support the diagnosis of an adrenal tumor as the etiology of the patient's symptoms. In addition, the patient is Asian and is less likely to have a predisposition to idiopathic hirsutism. Asians with polycystic ovarian syndrome are less likely to present with overt hirsutism than other ethnic groups.

A 26-year-old G0 woman presents with severe right lower quadrant pain associated with nausea for the last six hours, which began shortly after she finished her aerobic exercises. She has a history of suspected endometriosis, which was diagnosed two years ago, based on her severe dysmenorrhea. She has been using NSAIDs during her menses to control the pain. She is not sexually active, and is otherwise in good health. Her menstrual cycles are regular and her last menstrual period was three weeks ago. She has no history of sexually transmitted infections. Her vital signs are: blood pressure 145/70; pulse 100; temperature 99.2°F (37.3°C). She appears uncomfortable. On abdominal examination, she has moderate tenderness to palpation in the right lower quadrant. On pelvic exam, she has no lesions or discharge. A thorough bimanual examination is difficult to perform due to her discomfort. Beta-hCG <5 mIU/ml and WBC 8,500 /microliter. A pelvic ultrasound shows a 6 cm right ovarian mass. The uterus and left ovary appear normal. There is a moderate amount of free fluid in the pelvis. What is the most likely diagnosis in this patient? A. Appendicitis B. Exacerbation of the endometriosis C. Ovarian carcinoma D. Ovarian torsion E. Ectopic pregnancy

D. The sudden onset of pain and nausea, as well as the presence of a cyst on ultrasound, suggest ovarian torsion. Although appendicitis is in the differential diagnosis, it is unlikely to have such a sudden onset of pain and a normal white count. Her endometriosis can get worse but it would be unlikely to be of such sudden onset. Although she has an adnexal mass, a negative Beta-hCG rules out pregnancy.

A 22-year-old G1P0 woman currently at eight weeks gestation is noted to have a missed abortion on ultrasound, along with a retroverted uterus. She elects to undergo suction dilation and curettage. During the procedure, "fatty appearing tissue" is noted to be coming through the curette. What is the next best step in the management of this patient? A. Continuing with the suction curettage B. Remove the tissue from the curette and replace it into the uterus C. Cut the tissue off at the cervical os D. Proceed with laparoscopy E. Stop the procedure and observe her the hospital for 48-hours

D. The tissue is consistent with omental tissue and may include segments of bowel. The suction should be turned off and the tissue gently removed from the curette. Laparoscopy will allow closer examination and should bowel appear to be involved, the surgeon should consider laparotomy for closer evaluation of the bowel for damage. The other options would place the patient at increased risk of complications and delay diagnosis.

A 36-year-old G1 began prenatal care at eight weeks gestation. At that time, the gestational age was confirmed by a transvaginal ultrasound. She is now at 36 weeks gestation. Her previous medical history reveals hypertension for eight years and class F diabetes for five years (baseline proteinuria = 1 g). She smokes one half-pack of cigarettes per day. On examination at 32 weeks gestation, her fundal height was 29 cm. At 33 weeks, biometry was consistent with 31-3/7, EFW 1827g, 25th percentile. Today, ultrasound reveals limited fetal growth over the past three weeks. Biometry is consistent with 31-5/7, EFW 1900 g, <10th percentile. What is the most likely etiology of the intrauterine growth restriction in this case? A. Genetic factors B. Congenital anomaly C. Tobacco use D. Uteroplacental insufficiency E. Perinatal infection

D. There is substantial evidence from experimental animal studies that suggests that alterations in uteroplacental perfusion affect the growth and status of the fetus, as well as the placenta. This patient has significant medical diseases that are affecting her vasculature and, ultimately, limiting the substrate availability to the fetus with resultant uteroplacental insufficiency. The vascular disease is evidenced by retinopathy and proteinuria. The other choices above may all result in fetal growth restriction; however, they are not the most likely etiology in this clinical scenario.

A 33-year-old G1P0 woman at 38 weeks gestation with pregnancy complicated by type 1 diabetes was admitted for induction due to oligohydramnios. She received Cervidil (prostaglandin E2) overnight and her cervix was noted to be 3 cm dilated in the morning so oxytocin was started. After three hours on oxytocin induction, fetal heart rate was noted to be in the 160s with minimal variability and late decelerations despite resuscitation with oxygen, fluids and left lateral position. Thirty minutes after discontinuing the oxytocin, she continued to have contractions every three to four minutes with late decelerations. Her blood pressure was noted to be 138/88 and her pulse was 110. Her cervical exam was noted to be 4 cm dilated. What is the most appropriate next step in the management of this patient? A. Perform a biophysical profile B. Begin amnioinfusion C. Administer terbutaline D. Proceed with a Cesarean section E. Restart the oxytocin

D. This fetus is clearly not tolerating labor. Unfortunately, there is no good way to assess fetal status at this point. A biophysical profile is not of any value in labor. Amnioinfusion may be used for repetitive variable decelerations and not for recurrent lates. The presence of late decelerations in a patient with diabetes and oligohydramnios is not reassuring and unlikely to recover. Although terbutaline may slow down the contractions, it is not recommended in a patient whose heart rate is 110.

A 70-year-old G3P3 woman presents with a four-year history of constant leakage of urine. Her history is significant for abdominal hysterectomy and bilateral salpingo-oophorectomy for endometriosis. She had two anterior repairs in the past for recurrent cystocele. The leakage started six months after her last anterior repair. Pelvic examination reveals no evidence of pelvic relaxation. The vagina is well-estrogenized. Q-tip test reveals a fixed, immobile urethra. Cystometrogram shows no evidence of detrusor instability. Cystourethroscopy showed no evidence of any fistula and reveals a "drain pipe" urethra. Which of the following is the best first treatment for this patient? A. Retropubic urethropexy B. Needle suspension C. Artificial urethral sphincter D. Urethral bulking procedure E. Sling procedure

D. This is a classic example of intrinsic sphincteric deficiency. Urethral bulking procedures are minimally invasive and have a success rate of 80% in these specific patients. The success rates for retropubic urethropexies, needle suspension and slings are less than 50%. An "obstructive or tight" sling can be performed to increase the success rate, but the voiding difficulties are significant, even requiring prolonged or lifelong self-catheterization. Artificial sphincters should be used in patients as a last resort.

A 20-year-old G2P1 woman at 28 weeks gestation presents to labor and delivery with contractions every four minutes. On physical examination, her vital signs are: temperature 101°F (38.3°C); heart rate 120; respiratory rate 18; and blood pressure 110/65. Her uterine fundus is tender and the rest of the physical exam is normal. Her cervix is dilated 1 cm and is 50% effaced. The fetus is in vertex presentation. Fetal heart tones are in the 150s with a Category I tracing. Her white blood cell count (WBC) is 18,000/mcL. Which of the following is the most appropriate next step in the management of this patient? A. Observation B. Tocolysis C. Contraction stress test D. Labor induction E. Cesarean section

D. This patient has a fever, a tender fundus, and elevated white blood cell count, which are concerning for an intra-amniotic infection. Delivery is warranted and in the case of reassuring heart tones, there are no contraindications for labor induction and a Cesarean section is not indicated at this time. Tocolytics should not be used in the case of an intra-amniotic infection. Conservative management with observation would delay diagnosis and would not be appropriate. A contraction stress test is not indicated since the patient is already contracting with reassuring fetal heart tones.

A 33-year-old G2P1 woman presents at 12 weeks gestation for routine prenatal visit. She has had an uncomplicated prenatal course. Doppler fetal heart tones are not heard and the ultrasound today shows a crown rump length of 8 mm with no cardiac activity and a retroverted uterus. What is the next step in the management of this patient? A. Check a serum progesterone level B. Obtain serial Beta-HCG levels every two days C. Repeat the ultrasound in seven days D. Medical induction with misoprostol E. Medical induction with methotrexate

D. This patient has a missed abortion and should be offered uterine evacuation. Ultrasound criteria for a missed abortion are a CRL of > 7 mm with no cardiac activity. Medical induction using misoprostol has been shown to be efficacious and associated with less complications when compared to surgical evacuation. Checking a serum progesterone and following serial Beta-hCG may be indicated in confirming a viable pregnancy. Methotrexate is used in the treatment of selected ectopic pregnancies and can be used to induce medical terminations of pregnancies if the LMP was < six weeks ago.

A 23-year-old G1P0 woman at 10 weeks gestation presents with an intrauterine embryonic demise. On exam, her blood pressure is 120/80; heart rate is 67; and she is afebrile. Her cervix is closed and there is no evidence of bleeding. She desires to have the most minimally invasive treatment as possible. Which of the following options is best for this patient? A. Dilation and curettage B. Manual vacuum aspiration C. Mifepristone D. Misoprostol E. Oxytocin

D. This patient has a missed abortion. Expectant management is the least invasive treatment. Of the options listed, Misoprostol (prostaglandin E1) is the least invasive best option for this patient. Misoprostol can be administered orally or vaginally and will induce uterine cramping with expulsion of products of conception. Potential risk factors of use include hemorrhage as well as failure. Dilation and curettage and manual vacuum aspiration are effective methods for treatment of a missed abortion, but are invasive procedures. Mifepristone, a progesterone receptor blocker, is used for pregnancy termination. It is recommended for use within 49 days of the last menstrual period, but there is data to show that it can be effective up to nine weeks. Oxytocin would not be effective as there has not been up-regulation of receptors at this gestational age.

An 18-year-old G1 woman at 32 weeks gestation presents with severe abdominal pain and a small amount of bleeding. She has received routine prenatal care, smokes one pack of cigarettes per day and admits to using crack cocaine. On exam, her blood pressure is 140/80; pulse 100; and she is afebrile. Her uterus is tense and very tender. Pelvic ultrasound reveals a fundal placenta, cephalic presentation of the fetus and no other abnormalities. Cervical examination reveals blood coming through the os and is one centimeter dilated. Fetal heart tones have a baseline of 160s, with a category III tracing. Which of the following is the most likely diagnosis? A. Placenta previa B. Premature rupture of the membranes C. Preterm labor D. Placental abruption E. Chorioamnionitis

D. This patient has a placental abruption. Common presenting signs of an abruption include abdominal pain, bleeding, uterine hypertonus and fetal distress. Risk factors include smoking, cocaine use, chronic hypertension, trauma, prolonged premature rupture of membranes, and history of prior abruption. Treatment would involve an emergent Cesarean delivery with appropriate resuscitation, including intravenous fluids and blood products as needed. A placenta previa is an abnormal location of the placenta. Chorioamnionitis is an infection that typically occurs following prolonged rupture of membranes, and is frequently accompanied by a maternal fever. Premature rupture of membranes presents with loss of fluid not active bleeding.

A 58-year-old G3P1 woman presents to your office for her a health maintenance examination. She became menopausal at age 54. Her past medical history is significant for angina. She experienced a hip fracture 14 months ago when she tripped and fell while running after her grandson. She has not had any surgeries. She takes no medications and has no known drug allergies. She smokes 10 cigarettes a day and drinks a glass of red wine at dinner. Her father was diagnosed with colon cancer at the age of 72. Physical exam revealed a blood pressure of 120/68, pulse of 64, and BMI of 22. Her heart, lung, breast and abdominal exams were normal. Pelvic exam was consistent with vaginal atrophy and a small uterus. There was no adnexal tenderness and no masses were palpated. In addition to obtaining a bone mineral density scan, what is the next step in the management plan for this patient? A. Repeat bone mineral density in one year B. Repeat bone mineral density at age 65 C. Begin hormone replacement therapy D. Begin treatment with bisphosphonates E. Test for the presence of biochemical bone markers in the blood

D. This patient has many of the major risk factors for osteoporosis including history of fracture as an adult, low body weight and being a current smoker. Patients who already have had an osteoporotic fracture may be treated on this basis alone. Prior to beginning treatment with bisphosphonates, a bone mineral density (BMD) should be documented and repeated at two-year intervals to monitor treatment. DEXA is the test of choice for measuring BMD. A nuclear medicine bone scan may be useful to rule out a pathologic fracture from metastatic disease. General recommendations for the prevention of osteoporosis include eating a balanced diet that includes adequate intake of calcium and vitamin D, regular physical activity, avoidance of heavy alcohol consumption, and smoking cessation. Bone markers are used in research but are not yet a reliable predictor of BMD. Hormone replacement therapy is not recommended long term for disease prevention especially in patients with cardiovascular disease.

A 27-year-old G0 woman presents to the clinic because of concerns that she has not been able to get pregnant for the last year. She has been married for two years ago and was using birth control pills for contraception. She stopped using birth control pills when she decided to start a family one year ago. She is in good health and her only medication is a prenatal vitamin. Her periods are regular, every 28 days, with normal flow; her last period was two weeks ago. She has no history of sexually transmitted infections and no abnormal Pap smears. Her husband is also healthy with no medical problems. She is 5 feet 4 inches tall and weighs 130 pounds. Her examination, including a pelvic exam, is completely normal. Laboratory results show normal thyroid function tests and normal prolactin level. What is the most appropriate next step in the management of this patient? A. Reassurance and observation B. Perform a pelvic ultrasound C. Order a hysterosalpingogram D. Order a semen analysis E. Recommend a diagnostic laparoscopy

D. This patient has primary infertility, since she has not been able to conceive for one year. She does not appear to have underlying pathology to explain why she has not conceived, and her husband's semen has not yet been examined. The male factor plays a role in about 35% of infertility cases. A pelvic ultrasound is unlikely to add any information, as the patient has normal cycles and normal exam. Although a hysterosalpingogram might be ordered in the future, the male factor needs to be ruled out first, as it is less invasive to perform. Even though this patient had been on birth control pills previously, this should not be affecting her fertility a year later. In patients who use OCPs for prolonged periods, there might be a few months delay in returning to normal fertility.

An 18-year-old G1P0 woman presents at 32 weeks for a routine visit. She complains of intense itching for the past two weeks and cannot stop scratching her arms, legs, and soles of her feet. She has tried over the counter lotions and antihistamines with no relief. She also states that her family noticed she is slightly yellow. Her vital signs are normal and there are scattered excoriations over her arms and legs. Which of the following is the best treatment in the management of this patient? A. Aggressive hydration B. Antivirals such as Acyclovir C. Antihistamines D. Ursodeoxycholic acid E. Steroids

D. This patient has pruritus gravidarum, a common pregnancy-related skin condition that is a mild variant of intrahepatic cholestasis of pregnancy. There is retention of bile salt, and as serum levels increase they are deposited in the dermis. This, in turn, causes pruritus. The skin lesions are secondary to scratching and excoriation. Antihistamines and topical emollients may provide some relief and should be used initially. Ursodeoxycholic acid relieves pruritus and lowers serum enzyme levels. Another agent reported to relieve the itching is the opioid antagonist naltrexon. Hydroxychloroquine is used to treat lupus and is not indicated in this patient.

A 27-year-old G2P1 woman presents to the emergency department with increasing lower abdominal pain, nausea, scant bleeding, and fever. She is two days postop from a suction dilatation and curettage for an incomplete abortion. Vital signs: blood pressure 120/80, pulse 104, respiratory rate 20, and temperature 100.4°F (38.0°C). Physical examination reveals rebound tenderness and abdominal guarding, uterus soft and slightly tender. Which of the following is most likely in this patient? A. Normal recovery symptoms B. Retained products of conception C. Hematometra D. Perforated uterus E. Cervical laceration

D. This patient has the classic signs and symptoms of a uterine perforation. Hematometra can develop after an abortion, but the patient would complain of cyclic midline abdominal cramping pain. Retained products of conception would cause profuse vaginal bleeding and if not removed may lead to a septic abortion. Complications that may occur secondary to suction dilatation and curettage include anesthesia risk, bowel and bladder injury, cervical lacerations, and uterine perforations. If the patient had a cervical laceration she would have more vaginal bleeding than scant.

A 56-year-old G3P3 woman presents to the office for her annual health maintenance examination. She is in good health and is not taking any medications. She has been postmenopausal for three years. She had an abnormal Pap test 10 years ago, but results have been normal every year since. She is sexually active with her husband. On examination, her cervix is 1 cm above the vaginal introitus and there is moderate bladder prolapse. Her uterus is normal in size and she has no adnexal masses or tenderness. In addition to recommending a mammogram, what is the most appropriate next step in the management of this patient? A. Cystocele repair B. Pelvic ultrasound C. Total hysterectomy D. Observation E. Topical estrogen

D. This patient is asymptomatic from her prolapse; therefore, no intervention is necessary at this point. Cystocele repairs and hysterectomies are invasive procedures which are not indicated in this asymptomatic patient. It is not necessary to obtain a pelvic ultrasound, as her uterus is normal in size and she has no adnexal masses. Topical estrogen would not help improve the prolapse, although it might help with her vaginal dryness. She seems to be doing well with the lubricants and it is not necessary to expose her to estrogen.

A 52-year-old G3P2 woman reports vaginal spotting and bleeding after intercourse for the past 18 months. She stopped having menses at the age of 48 and has not been on hormone replacement therapy. She also notes new onset low back pain. She has smoked two packs a day for the past thirty years. Her last gynecologic exam was 10 years ago. On physical examination, she is a thin female who appears older than her stated age. She weighs 120 pounds and is 5 feet 6 inches tall. Her pelvic examination reveals atrophy of the external genitalia and vagina, a minimal amount of dark brown blood in the vault, and a large parous cervix with a friable lesion on the anterior lip of the cervix. The uterus is normal size, non-mobile and fixed in a retroverted position. There are no palpable adnexal masses, but there is firm nodularity in the posterior cul-de-sac on rectal examination. Which of the following is the most appropriate next step in the management of this patient? A. Computerized tomography of the lower spine and pelvis B. Pap test C. Colposcopy D. Cervical biopsy E. Pelvic ultrasound

D. This patient is at high-risk for cervical cancer. Her risk factors include tobacco use and a poor screening history. The symptoms of postmenopausal and postcoital bleeding should be taken seriously, and a cervical biopsy of the suspicious cervical lesion performed. Her physical examination with fixation of the uterus and thickening of the rectovaginal septum and back pain suggests involvement of the parametria (Stage II) and possible extension to the sidewall (Stage III). A Pap test should not be used to exclude cervical cancer, as it is a screening test and not a diagnostic test, and colposcopy would not be useful since a clinically visible lesion is already present. Although a CT scan may ultimately be needed as part of the evaluation of cervical cancer, a diagnosis must first be made by biopsy. Ultrasonography may be helpful in the diagnostic evaluation of post-menopausal bleeding, but not in the setting of an obvious cervical lesion.

A 34-year-old G1P0 woman at 39 weeks gestation presents in active labor. Her cervical examination an hour ago was 5 cm dilated, 90 percent effaced and 0 station. She just had spontaneous rupture of membranes and is found to be completely dilated with the fetal head is at +3 station. The fetal heart rate tracing is shown below. What is the most likely etiology for these decelerations? (late decels) A. Oligohydramnios B. Rapid change in descent C. Umbilical cord compression D. Uteroplacental insufficiency E. Head compression

D. This patient is having late decelerations. Late decelerations are associated with uterine contractions. The onset, nadir, and recovery of the decelerations occur, respectively, after the beginning, peak and end of the contraction. Late decelerations are associated with uteroplacental insufficiency. A rapid change in cervical dilation and descent are not associated with late decelerations. Umbilical cord compression is associated with variable decelerations. Oligohydramnios can increase a patient's risk of having umbilical cord compression; however, it does not cause late decelerations. Head compression is associated with early decelerations.

A 29-year-old G1P0 woman at 28 weeks gestation presents with preterm labor. She reports having contractions every 2 minutes and describes them as painful. On exam, her blood pressure is 130/70; pulse 92; and is afebrile. Her cervix is dilated to 2 cm/50%effaced/-4 station. The fetus has a Category I tracing with contractions every 2 minutes. She is admitted and started on nifedipine and betamethasone. Which of the following medications is also indicated for this patient? A. Terbutaline B. Prostaglandin E1 C. Prostaglandin E2 D. Ampicillin E. No additional medications required

D. This patient is in preterm labor. Ampicillin is indicated for this patient as her Group B Strep status is unknown and should be continued until a culture result is negative or her labor stops. Nifedipine is a tocolytic used to delay the progression of labor to allow for the benefit of betamethasone to hasten pulmonary maturation. Both prostaglandin E1 and E2 are uterotonic agents and would likely increase the rate of this patient's contractions. Terbutaline is a tocolytic. The FDA has indicated that terbutaline should not be used secondary to its side effects and lack of efficacy.

A 22-year-old G1P0 woman, who is at 38 weeks gestation with an estimated fetal weight of 2500 g, presents in active labor. She is completely dilated and effaced. The fetus is at +4 station and left occiput anterior with no molding. She has an epidural and has been pushing effectively for three hours. She is exhausted. What is the next step in management? A. Allow to continue pushing until the baby delivers B. Start Oxytocin to strengthen contractions C. Discontinue the epidural D. Forceps-assisted vaginal delivery E. Cesarean section

D. This patient meets all the requirements for an operative vaginal delivery. Forceps application requires complete cervical dilation, head engagement, vertex presentation, clinical assessment of fetal size and maternal pelvis, known position of the fetal head, adequate maternal pain control and rupture of membranes. Strict adherence to the guidelines suggested by the American College of Obstetricians and Gynecologists (ACOG) for low forceps delivery does not increase the fetal or maternal risks when performed by an experienced operator.

A 26-year-old G2P1 woman presents with early pregnancy and vaginal spotting. Her last normal menstrual period was three months ago, and she had a positive home pregnancy test two weeks ago. She has been experiencing severe morning sickness. She denies any pelvic cramping or abdominal pain. She is otherwise healthy. On physical examination, she has a palpable uterus just above the symphysis. Pelvic examination is notable for a 2 cm fleshy friable nodular lesion along the left lateral vaginal sidewall, the cervix is multiparous, with a small amount of blood in the vault. Bimanual examination confirms a 10-week sized uterus. A pelvic ultrasound confirms a snowstorm pattern, and Beta-hCG is 52,000 mIU. What is the next step in the management of this patient? A. Chest X-Ray B. Biopsy the vaginal lesion C. PET scan D. CT scan of the chest/abdomen/pelvis E. Evacuation of the uterus

D. This patient most likely has metastatic GTD given the constellation of findings, and elevated Beta-hCG with no evidence of an intrauterine pregnancy. Although evacuation is likely necessary, the finding of a vaginal nodule raises the suspicion of metastasis and further warrants a full staging evaluation with a CT scan of the chest, abdomen and pelvis. A brain MRI is also likely. A simple CXR would not be sufficient since she already has evidence of metastasis to the vagina. Since metastatic GTD is known to be quite vascular, suspicious lesions should not be biopsied. A PET scan has not been shown to be a preferred method of evaluation at this time.

An 18-year-old G1P0 woman at 12 weeks gestation reports nausea, vomiting, scant vaginal bleeding and a "racing heart." These symptoms have been present on and off for the past four weeks. The patient has no significant past medical, surgical or family history. Vital signs are: temperature 98.6°F (37°C); heart rate 120 beats/minute; blood pressure 128/78. On physical examination: uterine fundus is 4 cm below the umbilicus; no fetal heart tones obtained by fetal Doppler device; cervix is 1 cm dilated with pinkish/purple "fleshy" tissue protruding through the os. Labs show: hemoglobin 8.2 gm/dL, quantitative Beta-hCG 1.0 Million IU/mL; thyroid-stimulating hormone (TSH) undetectable; free T4 3.2 (normal 0.7 - 2.5). An ultrasound reveals heterogeneous cystic tissue in the uterus (snowstorm pattern). Which of the following is the most appropriate next step in the management of this patient? A. Repeat quantitative Beta-hCG B. Repeat transvaginal ultrasound C. PET scan D. Chest x-ray E. CBC

D. This patient's presentation is classic for a molar pregnancy. Beta-hCG levels in normal pregnancy do not reach one million. A chest x-ray would be the most appropriate step, as the lungs are the most common site of metastatic disease in patients with gestational trophoblastic disease. Though a repeat quantitative Beta-hCG will be required on a weekly basis, an immediate post-operative value will be of little clinical utility. A PET scan is not indicated and the patient already had a CBC done.

A 32-year-old G5P3 woman presents with left-sided abdominal pain. Her last normal menstrual period was eight weeks ago. She began having pain early this morning and it has increased to a severity of 8/10. She denies nausea or vomiting or vaginal bleeding. Her gynecological history is notable for a right-sided ectopic pregnancy four years ago. At that time, she had a right salpingectomy and a left tubal ligation. On physical examination: blood pressure is 90/54; pulse 108; respirations 22; and temperature 98.6°F (37.0°C). On abdominal examination, she has rebound and guarding in all quadrants, and on pelvic exam, her uterus is very tender and there is left adnexal fullness. Urine pregnancy test is positive. A transvaginal ultrasound shows a thickened endometrium, left pelvic mass with a gestational sac and fetal pole, and a large amount of free fluid in the pelvis. Her hematocrit is 26%. What would be the next best step in the management? A. Admit for observation B. Repeat Beta-hCG level in 48 hours C. Treat with methotrexate D. Perform a laparoscopy E. Perform a dilation and curettage

D. This scenario is consistent with the patient having a ruptured ectopic pregnancy. Signs of hypovolemia (tachycardia, hypotension) with peritoneal signs (rebound, guarding and severe abdominal tenderness) and a positive pregnancy test lead to the diagnosis of ruptured ectopic pregnancy. Conservative management, with observation and repeating the Beta-hCG level in 48 hours is not indicated since a diagnosis is clear and delaying surgery can potentially be dangerous to the patient. Dilation and curettage would only be considered after laparoscopy, if needed.

A 27-year-old G1P0 woman at 32 weeks gestation presents complaining of cough, fever, chest pain, and dyspnea. Vital signs are pulse 108; temperature 100.5° F (38.0° C); respiratory rate 22 per minute. Physical examination reveals right lower lobe bronchial breath sounds. Which of the following tests would be most appropriate for making a diagnosis for this patient? A. Blood cultures B. Sputum culture C. Lower extremity dopplers D. Chest x-ray E. Pulmonary function tests

D. This woman presents with classic symptoms and findings for pneumonia. The typical symptoms include cough, dyspnea, sputum production, and pleuritic chest pain. Mild upper respiratory symptoms and malaise usually precede these symptoms, and mild leukocytosis is usually present. Chest radiography is essential for diagnosis, although radiographic appearance does not accurately predict the etiology of the pneumonia. Pulmonary function tests, blood and sputum cultures, serological testing, cold agglutinin identification, and tests for bacterial antigens are not recommended in uncomplicated pneumonia.

A 39-year-old G4P1 woman at 36 weeks gestation presents to labor and delivery. Upon initial evaluation, no fetal heart tones were noted on Doptone. Ultrasound confirms fetal demise. Problems during the pregnancy include diagnosis of an open neural tube defect, estimated fetal weight >90th percentile, polyhydramnios and a nonreactive NST (non-stress test) the week prior to admission. What is the most likely etiology of this fetal demise? A. Uncontrolled hypertension B. In-utero viral infection C. Antiphospholipid antibody syndrome (APAS) D. Uncontrolled diabetes E. Untreated maternal hypothyroidism

D. Uncontrolled diabetes during organogenesis is associated with a high rate of birth defects. The most common sites affected are the spine and the heart of the fetus, although all birth defects are increased. Fetuses in utero exposed to high levels of glucose transplacentally have increased growth and polyuria resulting in an increase in the amniotic fluid. While some viral infections are also associated with placentomegaly and polyhydramnios, the fetus will have normal or decreased growth depending on the timing of the infection. Severe hypertension and active APAS is often associated with oligohydramnios and intrauterine growth restriction. The risk of miscarriage is increased if hypothyroidism goes untreated.

A 29-year-old G2P1 woman at 39 weeks gestation presents in early labor after spontaneous rupture of the fetal membranes. Thirty minutes after arrival, she delivers a 2650 gram male infant. A globular pale mass appears at the introitus when attempting to deliver the placenta. Her blood pressure is 90/60; pulse 104; and temperature is 98.6°F (37.0°C). What is the most likely etiology for this event in this patient? A. Multiparity B. Twin gestation C. Leiomyoma D. Uterine inversion E. Rapid labor

D. Uterine inversion is an uncommon etiology of postpartum hemorrhage. Factors that lead to an over-distended uterus are risk factors for uterine inversion. Grand multiparity, multiple gestation, polyhydramnios and macrosomia are all risk factors. The most common risk factor, however, is excessive (iatrogenic) traction on the umbilical cord during the third stage of delivery. Although leiomyomas may spontaneously prolapse, it is unlikely during the peripartum period.

A 34-year-old chronic hypertensive G1 comes to see you for a consultation at 34 weeks for size less than dates. Her prenatal course has been uncomplicated and the genetic amniocentesis obtained at 15 weeks revealed a normal male. Biometrics today reveal a biparietal diameter consistent with 33 weeks, abdominal circumference of 28 weeks, EFW 1600 g, less than 10th percentile, and an amniotic fluid index of 6. What is the most likely cause of fetal growth restriction in this patient? A. Chromosomal abnormality B. Fetal infection with Rubella C. Fetal infection with cytomegalovirus (CMV) D. Uteroplacental insufficiency E. Maternal infection with Varicella

D. Uteroplacental insufficiency can lead to asymmetric growth restriction. Asymmetric growth restricted infants typically have a normal length, but their weight is below normal. On ultrasound, there is a head-sparing effect, meaning that the head/brain is spared of the reduced blood flow that is a result of uteroplacental insufficiency. Thus, the fetal abdomen measures below normal and the head remains very close to normal. There is an asymmetrical growth pattern that is usually detected during the third trimester and reflects uteroplacental insufficiency. Symmetric fetal growth restriction indicates that all fetal measurements are below normal. As a general rule, such a finding indicates an intrinsic growth failure or an "early event" secondary to one or more organ system anomalies, fetal aneuploidy or chronic intrauterine infection. Infectious diseases are known to cause IUGR, but the number of organisms is poorly defined. There is sufficient evidence to show a causal relationship between rubella and CMV infections and fetal growth restriction. Other viruses to consider are syphilis and varicella. The protozoan toxoplasmosis results in IUGR as well. There are no bacteria known to cause IUGR. Symmetrical growth restriction is usually detected in the mid-trimester of pregnancy.

A 24-year-old G0 woman presents to you for preconception counseling. Her medical history is notable for type 1 diabetes mellitus, hypertension, epilepsy, and hypothyroidism. Her medications include insulin, methyldopa, valproic acid and levothyroxine. Based on her medication exposure, her infant is at greatest risk of which of the following anatomical defects? A. Duodenal atresia B. Skeletal anomalies C. Renal tubular dysgenesis D. Neural tube defects E. Omphalocele

D. Valproic acid is associated with an increased risk for neural tube defects, hydrocephalus and craniofacial malformations. Insulin and methyldopa are not associated with fetal defects. Omphalocele and duodenal atresia are not increased in type 1 diabetic patients.

A 19-year-old G1P0 woman at 42 weeks gestation presents to labor and delivery with spontaneous rupture of membranes for 13 hours and spontaneous onset of labor. Her vital signs are: blood pressure 120/70; pulse 72; afebrile; fundal height 36 cm; and estimated fetal weight of 2700 gm. Cervix is dilated to 4 cm, 100% effaced, +1 station. Which statement best describes the tracing seen below? (fetal heart rate baseline 130s with high moderate variability and decels. no relationship to CTX) A. Normal fetal heart rate pattern B. Sinusoidal rhythm C. Late deceleration D. Variable deceleration E. Early deceleration

D. Variable decelerations show an acute fall in the FHR, with a rapid down slope and a variable recovery phase. They are characteristically variable in duration, intensity, and timing, and may not bear a constant relationship to uterine contractions. They are typically associated with cord compression, especially in the setting of low amniotic fluid volume. Early decelerations are physiologic caused by fetal head compression during uterine contraction, resulting in vagal stimulation and slowing of the heart rate. This type of deceleration has a uniform shape, with a slow onset that coincides with the start of the contraction and a slow return to the baseline that coincides with the end of the contraction. Thus, it has the characteristic mirror image of the contraction. A late deceleration is a symmetric fall in the fetal heart rate, beginning at or after the peak of the uterine contraction and returning to baseline only after the contraction has ended. Late decelerations are associated with uteroplacental insufficiency. The true sinusoidal pattern is a regular, smooth, undulating form typical of a sine wave that occurs with a frequency of two to five cycles/minute and an amplitude range of five to 15 beats per minute. It is also characterized by a stable baseline heart rate of 120 to 160 beats per minute and absent beat-to-beat variability.

A 37-year-old G1P1 woman suffers from severe mood swings the week before her menstrual cycle. The mood swings resolve after she stops bleeding. You diagnose her with premenstrual syndrome (PMS) after obtaining further history and a normal examination. In addition to exercise, which of the following might be suggested to help decrease this patient's symptoms? A. Folic acid B. Ginkgo C. Fish oil D. Vitamin B6 E. Potassium

D. Vitamin deficiency of A, E and B6 have been associated with an increase in PMS. Replacement of these vitamins might improve PMS symptoms and avoid further medical therapy.

A 29-year-old G1P0 woman presents at 31 weeks gestation with preterm rupture of membranes six hours ago. She notes that for the last hour she has had some occasional contractions. Her prenatal course has been uncomplicated and she takes prenatal vitamins and iron. She denies substance abuse, smoking or alcohol use. Her blood pressure is 110/70; pulse 84; temperature 98.6°F (37.0°C). What is the role of tocolysis in this patient? A. Prevent delivery B. Delay delivery until fetal lung maturity is reached C. Delay delivery for one week D. Delay delivery in order to administer steroids E. Contraindicated

D. While the role of tocolysis in the setting of preterm rupture of membranes is controversial, it may be appropriate in limited settings. Tocolysis may be administered in an attempt to prolong the interval to delivery to gain time for steroids to obtain maximum benefit for the fetus. The risks of chorioamnionitis with continuing tocolytics beyond 48 hours outweighs the benefit of awaiting lung maturity. This may be reasonable in women without evidence of infection or advanced preterm labor. Admittedly, the likelihood of success in this setting is relatively poor, but the potential benefit to the fetus probably outweighs any maternal complication from tocolysis.

A 22-year-old G0 woman presents with five months of amenorrhea since discontinuing her oral contraceptive pills. She had been on the pill for the last six years and had normal menses every 28 days while taking them. She is in good health and not taking any medications. She is 5 feet 4 inches tall and weighs 140 pounds. Her examination, including a pelvic examination, is normal. Which of the following historical elements would be most useful in determining the cause of amenorrhea in this patient? A. Age at first intercourse B. History of sexually transmitted infections C. Parity D. History of oligo-ovulatory cycles E. Recent history of weight loss

D.Since most women resume normal menstrual cycles after discontinuing oral contraceptive pills (OCPs), they are not usually considered the cause of the amenorrhea. A history of irregular cycles prior to pill use may increase the risk of amenorrhea upon discontinuation. This is sometimes referred to as "post pill amenorrhea." A complete work-up should be performed to properly find the cause. Although the other historical elements are all important components of a complete gynecological history, they are not helpful to find the etiology of amenorrhea in this patient. Significant weight loss might cause amenorrhea; however, this patient still has normal body mass index, which makes it unlikely cause of amenorrhea.

A 21-year-old G1P0 woman delivered a 4000 gram infant by a low-forceps delivery after a protracted labor course that included a three-hour second stage. Her prenatal course was notable for development of anemia, poor weight gain and maternal obesity. Following the delivery, the patient was noted to have a vaginal sulcus laceration and a third-degree perineal laceration, which required extensive repair. Her hematocrit was noted to be 30% on postpartum day one. Which of the following factors places this patient at greatest risk for developing a puerperal infection? A. Third-degree perineal laceration B. Poor nutrition C. Obesity D. Anemia E. Protracted labor

E. Endometritis in the postpartum period is most closely related to the mode of delivery. Endometritis can be found in less than 3% of vaginal births and this is contrasted by a 5-10 times higher incidence after Cesarean deliveries. Factors related to increased rates of infection with a vaginal birth include prolonged labor, prolonged rupture of membranes, multiple vaginal examinations, internal fetal monitoring, removal of the placenta manually and low socioeconomic status.

A 19-year-old G1P0 woman at 41-weeks gestation with two prior prenatal visits at 35-weeks and 40-weeks, presents in active labor. Review of available maternal labs shows: blood type O+; RPR non-reactive; HBsAg negative; and HIV negative. She delivers a small female infant who cries spontaneously. On examination, you find the infant has a slightly flattened nasal bridge. Her ears are small and slightly rotated. What is the most appropriate next step in the management of this patient? A. Tell the mother the infant will be fine B. Tell the mother that her newborn has Down syndrome C. Question the patient why an amniocentesis was not performed D. Further examine the infant for wide-spaced nipples and lymphedema E. Further examine the infant for sandal gap toes and hypotonia

E. A flattened nasal bridge, small size and small rotated, cup-shaped ears may be associated with Down syndrome and should prompt a survey looking specifically for other features seen with Down syndrome that include sandal gap toes, hypotonia, a protruding tongue, short broad hands, Simian creases, epicanthic folds, and oblique palpebral fissures. The initial physical findings may be a variant of normal, therefore, you should not share any concerns with the mother until you perform a detailed physical examination. Wide-spaced nipples and lymphedema are associated with Turner syndrome. It is not standard of care to offer amniocentesis to a 19-year-old, unless she has specific risk factors.

A 26-year-old G2P2 woman presents with urinary urgency and dysuria for the past three days. She has a history of a urinary tract infection once. She is sexually active and uses condoms for contraception. She is otherwise healthy and does not take any medications or supplements. She does not have fever, chills, flank pain or vaginal discharge. Which of the following organisms is the most likely cause of this patient's symptoms? A. Enterococcus faecalis B. Klebsiella pneumoniae C. Proteus mirabilis D. Staphylococcus saprophyticus E. Escherichia coli

E. Acute cystitis in a healthy, non-pregnant woman is considered uncomplicated and is very common. Escherichia coli causes 80 to 85 percent of cases. The other major pathogens are Staphylococcus saprophyticus, Klebsiella pneumoniae, Enterococcus faecalis and Proteus mirabilis. The physician must consider antibiotic resistance when determining treatment.

A 32-year-old G1P0 woman at 10 weeks gestation presents to your office after an ultrasound evaluation has revealed a diamniotic, dichorionic twin gestation. She is very concerned about the risk for preterm delivery. Which intervention would you recommend as a possible means to reduce the risk of a preterm, low-birthweight infant? A. Bed rest B. Cervical cerclage C. Tocolytics starting at 24 weeks D. Home uterine monitoring E. Early, good weight gain

E. Although prematurity has been recognized as a major cause of morbidity and mortality among twin gestations, interventions for prevention of prematurity have, in general, been unsuccessful. Studies show that an adequate weight gain in the first 20 to 24 weeks of pregnancy is especially important for women carrying multiples and may help to reduce the risk of having preterm and low-birth weight babies. These pregnancies tend to be shorter than singleton pregnancies, and studies suggest that a good early weight gain aids in development of the placenta, possibly improving its ability to pass along nutrients to the babies. Bed rest, long prescribed by obstetricians for the prevention of preterm birth, has never been shown to be efficacious, and may be associated with thromboembolic complications. An observational study of prophylactic cerclage for twin gestations failed to show any benefit. Tocolytic drugs for prevention of preterm labor in asymptomatic women with twin gestations have not been shown to be effective. Home uterine activity monitoring is another intervention that has been shown to be ineffective.

A 29-year-old G3P0 woman presents for evaluation and treatment of pregnancy loss. Her past medical history is remarkable for three early (<14 weeks gestation) pregnancy losses. Parental karyotype was normal. Which of the following is the most appropriate next step in the management of this patient? A. Prophylactic cerclage with her next pregnancy B. Serial cervical length with her next pregnancy C. 17-hydroxyprogesterone with her next pregnancy D. Check for Factor V Leiden mutation E. Check antiphospholipid antibodies

E. Antiphosphospholipid antibodies are associated with recurrent pregnancy loss. The workup for antiphospholipid syndrome includes assessment of anticardiolipin and beta-2 glycoprotein antibody status, PTT, and Russell viper venom time. Recurrent pregnancy loss is defined as > two consecutive or > three spontaneous losses before 20 weeks gestation. Etiologies include anatomic causes, endocrine abnormalities such as hyper- or hypothyroidism and luteal phase deficiency, parental chromosomal anomalies, immune factors such as lupus anticoagulant and idiopathic factors. Her history is not consistent with cervical insufficiency which is diagnosed in the second trimester by history, physical exam and other diagnostic tests, such as ultrasound. Serial cervical lengths or placement of a cerclage are not indicated in this patient. Treatment with 17-hydroxyprogesterone is indicated in patients with a history of prior preterm birth. Factor V Leiden mutation has not been associated with recurrent pregnancy loss. It can be associated with thrombotic events.

A 34-year-old G4P4 woman is diagnosed with endometritis following a Cesarean delivery three days ago. Which of the following is the most likely causative agent(s) of endometritis in this patient? A. Aerobic streptococcus B. Anaerobic streptococcus C. Aerobic staphylococcus D. Anaerobic staphylococcus E. Aerobic and anaerobic bacteria

E. Bacterial isolates related to postpartum endometritis are usually polymicrobial resulting in a mix of aerobes and anaerobes in the genital tract. The most causative agents are Staphylococcus aureus and Streptococcus.

A 32-year-old G3P3 woman comes to the office to discuss permanent sterilization. She has a history of hypertension and asthma (on corticosteroids). She has been married for 10 years. Vital signs show: blood pressure 140/90; weight 280 pounds; height 5 feet 9 inches; and BMI 41.4kg/m2. You discuss with her risks and benefits of contraception. Which of the following would be the best form of permanent sterilization to recommend for this patient? A. Laparoscopic bilateral tubal ligation B. Mini laparotomy tubal ligation C. Exploratory laparotomy with bilateral salpingectomy D. Total abdominal hysterectomy E. Vasectomy for her husband

E. Both vasectomy and tubal ligation are 99.8% effective. Vasectomies are performed as an outpatient procedure under local anesthesia, while tubal ligations are typically performed in the operating room under regional or general anesthesia; therefore carrying slightly more risk to the woman, assuming both are healthy. She is morbidly obese, so the risk of anesthesia and surgery are increased. In addition, she has chronic medical problems that put her at increased risk of having complications from surgery.

32-year-old G1P1 is status post uncomplicated normal spontaneous vaginal delivery. She is taking sertraline (Zoloft), a selective serotonin uptake inhibitor (SSRI) as an antidepressant and wants to breastfeed. What is the next best step in management of this patient? A. Decrease her SSRI dose by 50%, since these drugs are concentrated in the breast milk B. Consult psychiatry about changing medications and discard the expressed milk in the meantime C. Discontinue the medications so she can breastfeed D. Increase her SSRI dose, since these drugs are not concentrated in the breast milk and she is at great risk for postpartum depression E. Continue the medications, since there is negligible risk for the newborn

E. Breastfeeding is beneficial to both mother and infant. Current recommendations state that SSRI medications can be safely used during lactation. Several studies show that SSRIs are secreted in breast milk, however no detectable levels of the drug were found in the infants' serum. In addition, no adverse effects were noted in the infants by either their parents or pediatricians following the infants.

A 48-year-old G3P3 woman recently had an abnormal Pap test with high grade squamous intraepithelial lesion (HSIL). Colposcopically-directed biopsy revealed cervical intraepithelial neoplasia 3 (CIN 3). A loop electrosurgical excision procedure (LEEP) is subsequently performed. In reviewing the pathologic specimen with the pathologist, abnormal squamous cells are seen extending 2 mm beyond the basement membrane. What is the patient's diagnosis? A. CIN 1 B. CIN 2 C. CIN 3 D. Carcinoma in situ E. Microinvasive cervical cancer

E. Cervical dysplasia is graded based on extent of involvement of the epithelial layer but does not extend below the basement membrane. Carcinoma in situ (CIS) represents abnormal cells involving the entire epithelium to the basement membrane. In cancer, the cells invade beyond the basement membrane. In microinvasive cancer, they invade less than 3 mm.

An 18-year-old G0 woman presents with a one-year history of hirsutism and acne. She had menarche at age 14 and her menses have been irregular every 26-60 days. Her sister has a similar pattern of hair growth. The patient is 5 feet 4 inches tall and weighs 180 pounds. On exam, a few terminal hairs were identified on her chin and upper lip. TSH, prolactin, total testosterone, and DHEAS levels are normal. Which of the following is the most appropriate next test to evaluate this patient's condition? A. Estradiol B. Serum cortisol C. Urinary cortisol D. Random blood glucose E. 17-hydroxyprogesterone

E. Checking 17-hydroxyprogesterone would rule out late onset 21-hydroxylase deficiency. Normal TSH, Prolactin, total testosterone and DHEAS levels rule out pituitary or adrenal tumors. The patient could have polycystic ovarian syndrome; however, normal serum testosterone levels make it less likely. Blood glucose would not help determine the etiology of hisrsutism.

A 28-year-old G2P2 woman returns today for follow up on her abnormal Pap test which reveals atypical squamous cells of undetermined significance (ASCUS). Reflex HPV testing is positive for high risk type. She has never had a prior abnormal Pap test, and has been following the recommended screening guidelines. She is asymptomatic. Her pelvic exam reveals a normal cervix with a small amount of cervical mucous. What is the next best step in the management of this patient? A. Routine screening B. Repeat Pap test in one year C. Repeat HPV testing in one year D. Repeat co-testing with Pap and HPV in one year E. Colposcopy

E. Colposcopy is indicated for all abnormal Pap test results including ASCUS Pap test when HPV is positive. Reflex HPV testing for high-risk DNA types should be performed in patients with ASCUS. If negative, then co-testing with cytology and HPV can be repeated in three years. Repeat cytology in one year is also an acceptable option for ASCUS if HPV testing cannot be done. http://www.asccp.org/Portals/9/docs/ASCCP%20Updated%20Guidelines%20Algorithms%206.3.13.pdf

A 32-year-old G2P1 woman is 20 weeks gestation. Her prior pregnancy was complicated by postpartum endometritis and her son was diagnosed with early-onset neonatal sepsis due to group B streptococcus. Which of the following management options regarding Group B streptococcus is most appropriate for this patient? A. Recto-vaginal culture at 35-37 weeks and antibiotic treatment during labor if positive B. Recto-vaginal culture at 35-37 weeks and antibiotic treatment at the time the culture result returns if positive C. Recto-vaginal culture at 24-28 weeks and antibiotic treatment during labor if positive D. Recto-vaginal culture at 24-28 weeks and antibiotic treatment at the time the culture result returns if positive E. Do not perform recto-vaginal cultures and treat with antibiotics during labor

E. Cultures for group B streptococcus are not required in women who have group B streptococcal bacteriuria during the current pregnancy or who have previously given birth to a neonate with early-onset group B streptococcal disease because these women should receive intrapartum antibiotic prophylaxis. Universal screening with a recto-vaginal culture at 35-37 weeks of gestation is recommended for all women who do not have an indication for intrapartum antibiotic prophylaxis. All women with positive cultures for group B streptococci should receive intrapartum antibiotic in labor unless a cesarean delivery is performed before onset of labor in a woman with intact amniotic membranes.

A 26-year-old G0 women returns for a follow-up visit regarding suspected endometriosis. She has been using NSAIDs and oral contraceptive pills to help manage her pelvic pain which has been getting worse. While discussing further treatment options, she asks if there is any test or procedure you can perform to confirm her diagnosis. Which of the following would you recommend? A. CA-125 B. Pelvic ultrasound C. CT scan of the abdomen and pelvis D. MRI of the pelvis E. Diagnostic laparoscopy

E. Definitive diagnosis is based on exploratory surgery and biopsies, although endometriosis is usually initially treated based on the clinical presentation. In addition, this patient can benefit from laparoscopy, since she has failed the two most common treatments for endometriosis, NSAIDs and OCPs. There is no imaging study or blood test that can confirm the diagnosis of endometriosis. CA-125 is non-specific and can be elevated in patients with endometriosis, and therefore not helpful.

A 23-year-old G0 woman with last menstrual period 14 days ago presents to the office because she had unprotected intercourse the night before. She does not desire pregnancy at this time and is requesting contraception. She has no medical problems and is not taking any medications. In addition to offering her counseling and testing for sexually transmitted infections, which of the following is the most appropriate next step in the management of this patient? A. Observation for two weeks to establish if pregnancy occurred before initiating treatment B. Oral contraceptives now C. Oral contraceptives after her next normal menstrual period D. Emergency contraception and follow-up after next menstrual period E. Provide emergency contraception, then begin oral contraceptives immediately

E. Emergency contraceptive pills are not an abortifacient, and they have not been shown to cause any teratogenic effect if inadvertently administered during pregnancy. They are more effective the sooner they are taken after unprotected intercourse, and it is recommended that they be started within 72 hours, and no later than 120 hours. Plan B®, the levonorgestrel pills, can be taken in one or two doses and cause few side effects. Emergency contraceptive pills may be used anytime during a woman's cycle, but may impact the next cycle, which can be earlier or later with bleeding ranging from light, to normal, to heavy.

A 35-year-old G0 woman presents with irregular menstrual periods occurring every six to twelve weeks with occasional inter-menstrual bleeding. Currently, she has been bleeding daily for the last four weeks. She reports that her periods have always been irregular, but have become more so with heavier flow and cramping in the last year. She is sexually active with one partner. On physical exam, she is morbidly obese with no abnormalities detected on pelvic exam. Which of the following is the most appropriate next step in the management of this patient? A. Luteinizing hormone level (LH) B. Follicle stimulating hormone level (FSH) C. Testosterone level D. Pelvic CT E. Endometrial biopsy

E. Endometrial biopsy should be performed to rule out endometrial hyperplasia or carcinoma given the history of irregular bleeding, coupled with the increased risk of these diagnoses in morbidly obese patients. While an ultrasound may be helpful, a pelvic CT is not useful in the workup for potential endometrial neoplasia. LH and FSH levels would not aid in the diagnostic workup and testosterone levels would not be useful, unless signs of hirsutism or virilization are present.

A 57-year-old G0 postmenopausal woman presents to her gynecologist with a complaint of vaginal bleeding for one week. The patient reports the cessation of normal menses approximately four years ago. She has had no previous episodes of irregular bleeding except when she took hormonal replacement therapy for six months. She saw her nurse practitioner five months ago and reports having a normal gynecologic evaluation and a normal Pap smear. Her past medical history is significant for hypercholesterolemia and diet-controlled diabetes mellitus. Physical exam reveals a 5 feet 3 inches tall, 275-pound woman in no acute distress. Pelvic exam demonstrated a normal vulva, urethra, vagina and cervix. Bimanual exam was normal. An endometrial biopsy was obtained and demonstrated complex atypical hyperplasia. Which of the following is this patient's greatest risk factor for developing endometrial cancer? A. Nulliparity B. Obesity C. Postmenopausal status D. Use of hormone replacement therapy E. Complex atypical hyperplasia

E. Endometrial cancer is a gynecologic malignancy that has easily identifiable risk factors and typically presents with symptoms that lead to an early diagnosis. Risk factors include nulliparity, obesity, late menopause, hypertension and exposure to unopposed estrogens. Of these risk factors, obesity confers the greatest risk of developing endometrial carcinoma, especially when the patient is more than 50 pounds over ideal body weight (10-fold increase). However, in this case, the patient's greatest risk for developing an endometrial cancer is the presence of complex atypical hyperplasia (CAH) on endometrial biopsy. If left untreated, this process has approximately a 28% chance of progressing to an invasive cancer. More importantly, approximately 30% of women with a diagnosis of CAH will be found to have an invasive endometrial cancer on final pathology. Most women who develop endometrial cancer are postmenopausal, but this is less of an issue because of the finding of CAH.

A 65-year-old G2P1 woman has been referred to you for further evaluation of postmenopausal bleeding. She initially was seen by her internist after two weeks of intermittent vaginal spotting. She reports a similar episode approximately two months ago. A recent exam and Pap smear were normal. A transvaginal ultrasound showed a homogeneous endometrial lining measuring 5.0 mm. A subsequent office endometrial sample was obtained and returned with rare atypical cells. What is the most appropriate next step in the management of this patient? A. Repeat office endometrial sample B. Follow-up ultrasound in eight to twelve weeks C. Return visit in three to six months D. Abdominopelvic CT scan E. Dilation and curettage

E. Enough to think it was endometrial cancer

A 32-year-old G1 is seeing you in consultation at 35 weeks gestation. Ultrasound reveals limited fetal growth over the past three weeks. Biometry is consistent with 30-5/7, EFW 1900 g, less than 10th percentile. You counsel her about short and long-term complications for her baby. This fetus is at increased risk for all of the following adult disorders EXCEPT: A. Cardiovascular disease B. Chronic hypertension C. Chronic obstructive lung disease D. Diabetes E. Osteoporosis

E. Epidemiologic studies indicate that fetal growth restriction is a significant risk factor for the subsequent development of cardiovascular disease, chronic hypertension, chronic obstructive lung disease and diabetes. Researchers suggest that the phenomenon of programming may be operable and that an adverse fetal environment during a critical period of fetal growth helps to promote these adult diseases. Osteoporosis risk factors include family history, slender body composition, prior history of osteoporosis, Asian and Caucasian ethnicity, alcohol consumption, smoking, sedentary lifestyle, excess thyroid or corticosteroids and use of anticonvulsant medications.

At one minute of life, an infant has a heart rate greater than 120 beats/minute, is crying, has acrocyanosis, gags when suctioned and is vigorously moving all four extremities. What is the APGAR score for this infant? A. 5 B. 6 C. 7 D. 8 E. 9

E. Heart rate= 2, Respiratory rate= 2, Reflex = 2, Activity =2, Color =1. Therefore, the one-minute APGAR score is 9.

A 35-year-old G3P3 woman requests contraception. Her youngest child is seven years old. Her periods have been regular since she discontinued breastfeeding five years ago. Her past medical history includes depression that is controlled with antidepressants, and a history of deep venous thrombosis. She denies smoking or alcohol use. In the past, oral contraceptive pills have caused her to have severe gastrointestinal upset. What in her history makes her an ideal candidate for progestin-only pills? A. Depression B. Smoking history C. Severe nausea on combined oral contraceptives D. Lactation history E. Deep venous thrombosis

E. Ideal candidates for progestin-only pills include women who have contraindications to using combined oral contraceptives (estrogen and progestin containing). Contraindications to estrogen include a history of thromboembolic disease, women who are lactating, women over age 35 who smoke or women who develop severe nausea with combined oral contraceptive pills. Progestins should be used with caution in women with a history of depression.

A 24-year-old G2P1 woman is diagnosed with Rh hemolytic disease at 24 weeks gestation. Measurement of which of the following in the amniotic fluid is best indicative of the severity of the disease? A. Hemoglobin B. Iron C. Anti-D antibody titer D. Ferritin E. Bilirubin

E. In the presence of a severely erythroblastotic fetus, the amniotic fluid is stained yellow. The yellow pigment is bilirubin, which can be quantified most accurately by spectrophotometric measurements of the optical density between 420 and 460nm, the wavelength absorbed by bilirubin. The deviation from linearity of the optical density reading at 450nm is due to the presence of heme pigment, an indicator of severe hemolysis. Amniotic fluid ferritin, an acute-phase reactant, is associated with spontaneous preterm delivery and not with Rh isoimmunization.

An 18-year-old G0 woman comes to the office due to vaginal spotting for the last two weeks. Her menstrual periods were regular until last month, occurring every 28-32 days. Menarche was at age 13. She started oral contraceptives three months ago. On pelvic examination, the uterus is normal in size, slightly tender with a mass palpable in the right adnexal region. No adnexal tenderness is noted. Which of the following tests is the most appropriate next step in the management of this patient? A. Endometrial biopsy B. Pelvic MRI C. Pelvic ultrasound D. Abdominal CT Scan E. Urine pregnancy test

E. It is vitally important to rule out pregnancy in the evaluation of abnormal uterine bleeding. Pelvic ultrasound could be considered as a next step if the pregnancy test is negative in order to evaluate the adnexal finding. Abdominal CT or MRI would not be performed in this patient unless advanced adnexal pathology was found on pelvic sonography. Endometrial biopsy would rarely be indicated in a teen with abnormal bleeding, unless morbidly obese and anovulatory.

A 24-year-old G1P0 woman at 41 weeks gestation was noted to have meconium-stained amniotic fluid after an amniotomy was performed. What is the most appropriate management of this patient to attempt to prevent her newborn from experiencing meconium aspiration syndrome? A. Initiate amnioinfusion with sterile saline to dilute the meconium-stained amniotic fluid B. Suction the oropharynx and nasopharynx on the perineum after the delivery of the head but before the delivery of the shoulders C. Suction the oropharynx and nasopharynx on the perineum immediately after delivery before the baby takes his first breath D. Intubate the trachea and suction meconium and other aspirated material from beneath the glottis immediately after delivery E. Intubate the trachea and suction meconium and other aspirated material from beneath the glottis immediately after delivery only if the infant is depressed

E. Meconium-stained amniotic fluid is present in 12-22% of women in labor. Meconium aspiration syndrome occurs in up to 10% of infants who have been exposed to meconium-stained amniotic fluid. It is associated with significant morbidity and mortality. The American College of Obstetrics and Gynecology, the American Academy of Pediatrics and the American Heart Association recommend that all infants with meconium-stained amniotic fluid should not routinely receive suctioning at the perineum. If meconium is present and the newborn is depressed, the clinician should intubate the trachea and suction meconium or other aspirated material from beneath the glottis. If the newborn is vigorous, defined as having strong respiratory efforts, good muscle tone, and a heart rate greater than 100 beats per minute, there is no evidence that tracheal suctioning is necessary. Injury to the vocal cords is more likely to occur when attempting to intubate a vigorous newborn. Routine prophylactic amnioinfusion for meconium-stained amniotic fluid is not recommended as there is no definitive benefit.

A 39-year-old G1P0 woman presents in labor at term. The estimated fetal weight is 3200 g. She is 10 cm dilated with left sacrum anterior at +2 station. Which of the following is the most appropriate next step in the management of this patient? A. Attempt external version B. Attempt internal version C. Apply forceps D. Apply a vacuum E. Recommend a Cesarean section

E. Most recent data suggests that breech infants delivered vaginally are at higher risk for neonatal complications. Therefore, it would be recommended that this patient undergo a Cesarean section, especially since this is her first pregnancy. External cephalic version and internal versions are contraindicated in active labor. Forceps are used in breech deliveries to assist in flexion of the head and vacuum applications on breech presentations are contraindicated.

A 31-year-old G0 woman has been diagnosed with uterine fibroids. An ultrasound confirmed the presence of two intramural fibroids measuring 5 x 6 cm and 2 x 3 cm that appear to be distorting the patient's uterine cavity. The patient has a two-year history of infertility. She has had a thorough infertility work up. No etiology for her infertility has been identified. Which of the following treatments is most appropriate for this patient? A. Hysteroscopy B. Uterine curettage C. Gonadotropin-releasing hormone agonist D. Uterine artery embolization E. Myomectomy

E. Myomectomy is warranted in younger patients whose fertility is compromised by the presence of fibroids that cause significant distortion of the uterine cavity. A myomectomy may be indicated in infertility patients when the fibroids are of sufficient size or location to be a probable cause of infertility and when no more likely explanation exists for the failure to conceive. Hysteroscopy is a procedure that involves placing a scope through the cervical os to assess the endometrial cavity. The patient has already been diagnosed with uterine fibroids that are distorting her cavity and she has already had a fluid contrast ultrasound, so it is unnecessary to perform hysteroscopy on this patient. Treatment with GnRH agonists can be useful to shrink fibroids in anticipation of surgery, or if menopause is expected soon. This patient desires future childbearing, therefore, its use would not be an appropriate option. Uterine artery embolization can be recommended for women who have completed child-bearing because of the unclear long-term effects on fertility.

A 24-year-old Rh-negative G2P1 woman at 18 weeks gestation is positive for anti-D antibodies. In discussing the risks of Rh sensitization with her, you tell her that her fetus may be at increased risk of significant perinatal disease including fetal anemia. Which of the following non-invasive tests can detect severe fetal anemia? A. Umbilical artery systolic-diastolic ratio B. Biophysical profile C. Amniotic fluid index D. Umbilical artery blood flow E. Middle cerebral artery peak systolic velocity

E. Noninvasive diagnosis of fetal anemia can be performed with Doppler ultrasonography. The use of middle cerebral artery peak systolic velocity in the management of fetuses at risk for anemia because of red cell alloimmunization has emerged as the best test for the noninvasive diagnosis of fetal anemia. All the other listed tests are for assessment of fetal well-being and non-specific to detect fetal anemia. Amniocentesis and cordocentesis have been used for many years to diagnose fetal anemia due to red cell alloimmunization. These techniques, however, are invasive and many complications are associated with their use.

A 27-year old G3P1 woman is admitted to the orthopedic service after open reduction and internal fixation of her femur status post a motor vehicle accident. Her past medical history is significant for diabetes (controlled with metformin) and a history of a deep venous thrombosis three years ago while taking an oral contraceptive. She has been receiving ibuprofen for pain control and oxycodone for breakthrough pain as well as docusate sodium (Colace). Additionally, anticoagulation therapy was began with IV heparin, and is now therapeutic on warfarin. At a follow up visit, she has a positive pregnancy test and an ultrasound confirms a six-week intrauterine pregnancy. Which of the following medications should be discontinued now? A. Metformin B. Docusate sodium C. Ibuprofen D. Oxycodone E. Warfarin

E. Of the medications she is currently taking, none are contraindicated at this gestational age. Ibuprofen is safe to take until around 32 weeks gestation, when premature closure of the ductus arteriosis is a risk. While heparin is safe during pregnancy, warfarin has known teratogenic affects and should not be given. If continued anticoagulation is necessary, low molecular weight heparin is the drug of choice.

A 24-year-old G1P1 woman comes to the office requesting contraception. Her past medical history is unremarkable, except for a family history of ovarian cancer. She denies alcohol, smoking and recreational drug use. She is in a monogamous relationship. She wants to decrease her risk of gynecological cancer. Of the following, what is the best method of contraception for this patient? A. Diaphragms B. Condoms C. Copper containing intrauterine device D. Progesterone containing intrauterine device E. Combined oral contraceptives

E. Oral contraceptives will decrease a woman's risk of developing ovarian and endometrial cancer. The first developed higher dose oral contraceptive pills have been linked to a slight increase in breast cancer, but not the most recent (current) lower dose pills. Women who use oral contraceptive pills have a slightly higher risk of developing cervical intraepithelial neoplasia, but their risk of developing PID, endometriosis, benign breast changes and ectopic pregnancy are reduced. Both hypertension and thromboembolic disorders can be a potential side effect from using oral contraceptive pills. Diaphragms, condoms and intrauterine devices will not lower her risk of ovarian cancer. The progesterone IUD may decrease a woman's risk for endometrial cancer but would not effect her risk for ovarian cancer, and have been associated with increased ovarian cysts.

A 34-year-old G1P0 woman presents with vaginal spotting. On physical exam: blood pressure 120/70; pulse 82; respirations 20; and temperature 98.6°F (37.0°C). An ultrasound confirms a non-viable intrauterine pregnancy. She is otherwise healthy. Her partner accompanies her and is supportive. The patient wishes to avoid any unnecessary medical interventions and asks whether she can safely let nature take its course. What is the best next step in the management of this patient? A. Immediate dilation and suction curettage B. Dilation and suction curettage in one week C. Immediate treatment with misoprostol D. Treatment with misoprostol in one week E. Expectant management

E. Patients experiencing early pregnancy loss can safely consider several different treatments, including expectant management, medical treatment to assist with expulsion of the pregnancy or surgical evacuation. Provided the patient is hemodynamically stable and reliable for follow-up, expectant management is appropriate therapy. At the gestational age described, expectant management portends no increase in risk of either hemorrhage or infection compared with surgical or medical evacuation. Regardless of method chosen, the patient's blood type should be checked and rhogam administered as indicated.

A 28-year-old G0 woman has a high-grade squamous intraepithelial lesion (HSIL) on a Pap test. She has no complaints. She smokes one pack of cigarettes per day. Her pelvic exam is normal. Colposcopy is performed. The cervix is noted to have an ectropion and there is abundant acetowhite epithelium. Mosaicism, punctations and several disorderly atypical vessels are noted. Three biopsies are obtained and sent to pathology. Which of the findings on this patient's colposcopy is most concerning? A. Ectropion B. Acetowhite epithelium C. Mosaicism D. Punctations E. Disorderly atypical vessels

E. Punctations and mosaicism represent new blood vessels on end and on their sides, respectively. Atypical vessels usually represent a greater degree of angiogenesis and, thus, usually a more concerning lesion. An ectropion is an area of columnar epithelium that has not yet undergone squamous metaplasia. It appears as a reddish ring of tissue surrounding the external os. Acetowhite epithelium can represent dysplasia but, in most cases, is less concerning than the above vascular changes.

A 49-year-old G1P1 woman comes to your office for menopause counseling. She has been experiencing severe sleep disturbances and night sweats for the past four months. She would like to begin hormone therapy, but is concerned because she has elevated cholesterol levels for which she takes medication. You explain to her that hormone therapy has the following effect on a lipid/cholesterol profile: A. Both HDL and LDL levels increase B. Both HDL and LDL levels decrease C. HDL and LDL levels are unaffected D. HDL levels increase and LDL levels are unaffected E. HDL levels increase and LDL levels decrease

E. Recent data have confirmed the overall positive effects of hormone therapy on serum lipid profiles. The most important lipid effects of postmenopausal hormone treatment are the reduction in LDL cholesterol and the increase in HDL cholesterol. Estrogen increases triglycerides and increases LDL catabolism, as well as lipoprotein receptor numbers and activity, therefore causing decreased LDL levels. Hormones inhibit hepatic lipase activity, which prevents conversion of HDL2 to HDL3, thus increasing HDL levels. Hormone therapy is not currently recommended for the primary prevention of heart disease.

A 17-year-old G1P0 female at 39 weeks gestation presents with increased swelling in her face and hands over the last two days. Her blood pressure is 155/99. She has 2 plus pitting edema of the lower extremities. A 24-hour urine collection shows 440 mg of protein. What is the next best step in the management of this patient? A. Fluid restriction B. Magnesium sulfate C. Furosemide D. Hydralazine E. Delivery

E. Regardless of disease severity, the only definitive therapy for preeclampsia is delivery of the fetus and placenta. This solution can occasionally be delayed in the setting of stable disease (mild or severe) when it occurs at an extremely early gestational age. Fluid management must be monitored closely in this person. Magnesium sulfate is the mainstay of therapy during labor and for 24 hours postpartum to lower the seizure threshold in women with severe disease. Low-dose aspirin may have some benefit in decreasing the risk of preeclampsia in a subset of high-risk patients. Hydralazine is often the antihypertensive agent of choice for controlling elevated blood pressures in the acute setting.

A 22-year-old G1P1 woman delivered her first baby five days ago after a prolonged labor and subsequent Cesarean section for arrest of cervical dilation at 7 cm. Fever was noted on postoperative day two and, despite intravenous broad spectrum antibiotics, she continues to have temperature spikes above 101.3° F (38.5° C). She is eating a normal diet and ambulating normally. On physical examination, her breasts have no erythema and nipples are intact. Her abdomen is soft, uterine fundus is firm and nontender, and her incision is healing without induration or erythema. She has normal lochia and her urinalysis is normal. Pelvic examination reveals a firm nontender uterus and no adnexal masses or tenderness. Which of the following treatments is indicated for this patient? A. Addition of antifungal therapy B. Addition of oral antibiotic therapy C. Addition of antiviral therapy D. Surgical exploration E. Heparin anticoagulation

E. Septic thrombophlebitis involves thrombosis of the venous system of the pelvis. Diagnosis is often one of exclusion of other causes, but sometimes a CT scan will reveal thrombosed veins. Treatment requires addition of anticoagulation to antibiotics and resolution of fevers is generally rapid. Anticoagulation treatment is short-term. The addition of oral antibiotics has no extra benefit on a patient who is already on broad spectrum IV antibiotics. She has no evidence of fungal or viral infections, so therapy for these is not indicated. There is also no indication she needs surgery.

A 27-year-old G2P1 woman at 18 weeks gestation presents to the emergency department complaining of fever, nausea, vomiting, and mid-abdominal pain for the last 24 hours. For the last 12 hours, she has had no appetite. She has been healthy, but reports that her three-year-old son has had diarrhea for two days. Physical examination reveals a blood pressure of 100/60; pulse 88; respiratory rate 18; and temperature 102.0°F (38.9°C). Abdominal examination reveals decreased bowel sounds and tenderness more pronounced on the right than the left. Which of the following is the next best step in the management of this patient? A. Complete blood count B. X-Ray of the abdomen C. Helical CT scan D. MRI E. Graded compression ultrasound

E. Suspected appendicitis is one of the most common indications for surgical abdominal exploration during pregnancy. The diagnosis is made based on clinical findings and graded compression ultrasonography that is sensitive and specific especially before 35 weeks gestation. This noninvasive procedure should be considered first in working up suspected acute appendicitis. Selective imaging of the appendix using helical computed tomography may be a safe and potentially reliable tool to accurately identify appendiceal changes in appendicitis, except that radiation exposure using this test is higher than graded compression ultrasonography. An MRI would not expose the patient to radiation but is not the best diagnostic study for appendicitis. A plain abdominal radiograph can be used to identify air fluid levels or free air but offers little diagnostic value for appendicitis. The diagnosis of appendicitis is more difficult to make in pregnancy because anorexia, nausea, and vomiting that accompany normal pregnancy are also common symptoms of appendicitis. In addition, the enlarged uterus shifts the appendix upward and outward toward the flank, so that pain and tenderness may not be located in the right lower quadrant. Appendicitis is easily confused with preterm labor, pyelonephritis, renal colic, placental abruption, or degeneration of a uterine myoma. Peritonitis and appendiceal rupture are more common during pregnancy.

A 54-year-old G2P2 presents for a health maintenance examination. She has a history of breast cancer treated with mastectomy with reconstruction, chemotherapy, and is currently on tamoxifen. She has been in remission for two years and has been menopausal since the initiation of her chemotherapy. She experiences very mild hot flashes and is not sleeping well. She appears apprehensive during the examination, although her examination is completely normal except for some mild vaginal atrophy. At the conclusion of the office visit, she finally opens up and admits that she has a new boyfriend after having gone through a divorce five years earlier. She is anxious about initiating sexual activity again and wants your advice on what she should do. Which of the following is likely contributing to her anxiety? A. Breast cancer diagnosis B. Menopausal symptoms C. Body image D. Vaginal atrophy E. All of the above

E. Taking a good sexual history is critical to understanding the root cause of a woman's concerns about possible sexual dysfunction. Etiologies may be physiologic, hormonal, psychologic, and often multifactorial. To distinguish among these causes, a thorough inquiry into a woman's hormonal, sexual, medical, and social history is necessary. A sexual history must include a non-judgemental and candid discussion related to her interests, desires, and practices to better understand potential causes for concern and dysfunction.

A 74-year-old G2P2 post-menopausal woman presents for a health maintenance examination. She notes the new onset of a lump in her vagina, but denies any pain, abnormal bleeding or vaginal discharge. She has well-controlled diabetes mellitus and hypertension. She is recently sexually active with a new partner since the death of her husband three years ago. She smokes a half-pack per day, and has done so since age 18. On examination, she is noted to have a somewhat firm and fixed non-tender 4 cm mass in her labia majora at the level of the Bartholin gland on the right. There is no associated erythema or discharge, and the remaining vulvar exam and pelvic exam are unremarkable. Her groin examination reveals no adenopathy. What is the most likely diagnosis? A. Lipoma B. Fibroma C. Bartholin gland cyst D. Bartholin gland abscess E. Bartholin gland malignancy

E. The finding of a mass in the Bartholin gland is highly suspicious for malignancy and requires excision/biopsy, especially in a post-menopausal women. Primary vulvar adenocarcinomas most likely arise from the Bartholin gland, but other histologies such as squamous cell, transitional, adenosquamous, and adenoid cystic carcinomas can also arise from this location. This is unlikely to be a fibroma or lipoma given the recent onset and fixed nature of the mass. A benign Bartholin gland cyst is also unlikely given the patient's age and rather abrupt onset, and any finding of a new Bartholin gland cyst in a post-menopausal woman should be further investigated. For any woman over the age of 40 with a mass in this area, a biopsy should be obtained. This is not an abscess given the absence of signs and symptoms of cellulitis or infection.

A 35-year-old G3P3 woman comes to the office because she desires contraception. Her past medical history is significant for Wilson's disease, chronic hypertension and anemia secondary to menorrhagia. She is currently on no medications. Her vital signs reveal a blood pressure of 144/96. Which of the following contraceptives is the best option for this patient? A. Progestin-only pill B. Low dose combination contraceptive C. Continuous oral contraceptive D. Copper containing intrauterine device E. Levonorgestrel intrauterine device

E. The levonorgestrel intrauterine device has lower failure rates within the first year of use than does the copper containing intrauterine device. It causes more disruption in menstrual bleeding, especially during the first few months of use, although the overall volume of bleeding is decreased long-term and many women become amenorrheic. The levonorgestrel intrauterine device is protective against endometrial cancer due to release of progestin in the endometrial cavity. She is not a candidate for oral contraceptive pills because of her poorly controlled chronic hypertension. The progestin-only pills have a much higher failure rate than the progesterone intrauterine device. She is not a candidate for the copper-containing intrauterine device because of her history of Wilson's disease.

A 17-year-old G0 sexually active female presents to the emergency department with pelvic pain that began 24 hours ago. She reports menarche at the age of 15 and coitarche soon thereafter. She has had four male partners, including her new boyfriend of a few weeks. Her blood pressure is 100/60; pulse 100; and temperature 102.0°F (38.9°C). On speculum examination, you note a foul-smelling mucopurulent discharge from her cervical os and she has significant tenderness with manipulation of her uterus. What is the next best step in the management of this patient? A. Outpatient treatment with oral broad spectrum antibiotics B. Outpatient treatment with intramuscular and oral broad spectrum antibiotics C. Intravenous antibiotics and dilation and curettage D. Inpatient treatment, laparoscopy with pelvic lavage E. Inpatient treatment and intravenous antibiotics

E. The most likely cause of the symptoms and signs in this patient is infection with a sexually transmitted organism. The most likely organisms are both N. gonorrhoeae and chlamydia, and the patient should be treated empirically for both after appropriate blood and cervical cultures are obtained. There is no evidence that adolescents have better outcomes from inpatient therapy. However, since the patient also has a high fever, inpatient admission is recommended for aggressive intravenous antibiotic therapy in an effort to prevent scarring of her fallopian tubes and possible future infertility.

A 35-year-old G5P4 woman status post vaginal delivery desires postpartum tubal ligation. In obtaining informed consent, which of the following is the most likely risk associated with this procedure? A. Chronic cyclic pain with menstrual cycles B. Increased risk for ovarian cancer C. Decreased enjoyment with sexual intercourse D. Aspiration with general anesthesia E. Unplanned pregnancy

E. The most likely complication she will experience is future pregnancy. The failure rate associated with surgical sterilization is approximately one percent. Approximately one-third of pregnancies after tubal ligation are ectopic. The existence of a "post-tubal ligation syndrome" in which disruption of blood flow in the area of the fallopian tubes leads to menstrual dysfunction and dysmenorrhea has not been substantiated. Tubal ligation appears to have a protective effect on ovarian cancer incidence. There is no proven association between decreased sexual enjoyment and tubal ligation. Tubal ligations may be performed under regional or general anesthesia. Postpartum tubal ligations are generally performed using a spinal or epidural anesthesia. The risk of aspiration are low and range from 1/1000 to 1/14,000.

A 23-year-old G1P0 woman at 40 weeks gestation presents to labor and delivery with contractions. At 10:00 am, her cervical exam is 2 centimeters dilated, 70% effaced and the vertex at 0 station. Clinical pelvimetry reveals an adequate pelvis and membranes are intact. The fetus is in a cephalic presentation and EFW is 3500 gms. Contractions are occurring every 3-4 minutes, based on the external monitor. Her labor slowly progresses and, at 1:00 pm, the patient has spontaneous rupture of membranes. Fetal surveillance remains Category I. Her cervical exam is 5 centimeters dilated, 100% effaced, and 0 station. At 4:00 pm, the patient's cervical exam is unchanged. Contractions are occurring every 5-6 minutes. Which of the following is the most appropriate next step in the management of this patient? A. Perform a biophysical profile B. Have the patient ambulate C. Perform a Cesarean delivery D. Continue fetal surveillance and reexamine the patient in two hours E. Begin oxytocin augmentation

E. The patient has an arrest of dilatation in the active phase of labor. She is only having contractions every 5-6 minutes, so it is reasonable to start oxytocin to increase the frequency and strength of this patient's contractions. If the patient does not have cervical change once she is having more frequent contractions on oxytocin, it would be reasonable to place an IUPC (intrauterine pressure catheter) to assess the strength of the contractions. It is not yet necessary to perform a Cesarean delivery. Further observation and having the patient ambulate do not facilitate delivery. A biophysical profile is not indicated in this situation.

19-year-old G1 woman presents at 28 weeks gestation for prenatal care. Her past medical history is unremarkable except for a splenectomy following a motor vehicle accident four years ago. Prenatal labs today show a hemoglobin of 12 g/dL; blood type O positive; Rh negative with antibody screen positive for Lewis (titer 1:16). What is the next best step in the management of this pregnancy? A. Check father of the baby's antibody status B. Biophysical profile C. Serial amniocentesis D. Percutaneous umbilical blood sampling E. Reassurance

E. The patient should be reassured that the fetus is not at risk even though the antibody titer is 1:16. Lewis antibodies are IgM antibodies and do not cross the placenta, therefore are not associated with isosensitization or hemolytic disease of the fetus. The father of the baby does not need to be tested nor does this unaffected fetus need a biophysical profile. The other tests listed above are invasive and used to monitor fetuses at risk for anemia, hydrops and fetal death.

A 37-year-old G3P3 woman complains of severe premenstrual symptoms for the past two years. She finds her mood swings and irritability troubling and requests a hysterectomy, as she thinks that this procedure will alleviate her symptoms. Past medical history is only remarkable for high cholesterol and her physical examination, including pelvic examination, is normal. The patient's physician does not recommend a hysterectomy. Which of the following is the most likely explanation for the physician's recommendation not to perform a hysterectomy in this patient? A. An endometrial ablation would be preferable B. Past medical history C. Influence of thyroid hormone on symptoms D. Influence of adrenal gland on symptoms E. Influence of ovaries on symptoms

E. The patient's mood swings are influenced by the hormonal shifts controlled by the hypothalamic-pituitary-ovarian axis. A hysterectomy or endometrial ablation would only resolve the menstrual bleeding component of this patient's symptoms, and have no effect on the hormonal production of the ovaries. Removal of her ovaries may improve her symptoms but would increase her risk for future problems, including osteoporosis and menopausal symptoms. A bilateral oophorectomy would be the last option for this patient as many medical options are available to treat her symptoms.

A 42-year-old G5P2 woman at 36 weeks gestation is diagnosed with preeclampsia. Her previous pregnancy was complicated by twins and preeclampsia at 36 weeks gestation. She also has had two spontaneous abortions at seven weeks gestation. Which of the following conditions is not associated with her increased risk for preeclampsia in this pregnancy? A. Previous history of preeclampsia B. Chronic hypertension C. Multifetal pregnancy D. Age E. Previous spontaneous abortion

E. The previous history of spontaneous abortion does not put the patient at increased risk. The incidence of preeclampsia is commonly cited to be about 5 percent and is markedly influenced by parity. It is related to race, ethnicity and genetic predisposition. Environmental factors are also likely to play a role. Other risk factors for preeclampsia include a previous history of the disease, chronic hypertension, multifetal pregnancy and molar pregnancy. In addition, patients at extremes of maternal age or with diabetes, chronic renal disease, antiphospholipid antibody syndrome, vascular or connective tissue disease or triploidy are at increased risk for developing preeclampsia

A 19-year-old G2P1 African American woman at 30 weeks gestation presents with preterm rupture of membranes six hours ago. Her prenatal course has been complicated by two episodes of bacterial vaginosis for which she was treated. She takes prenatal vitamins and iron. She denies substance abuse or alcohol use, but admits to smoking five cigarettes each day. Her prior pregnancy was delivered vaginally at 41 weeks after spontaneous rupture of membranes. Her blood pressure is 110/70; pulse 84; temperature 98.6°F (37.0°C). Pertinent sonographic findings reveal oligohydramnios and a cervical length of 30 mm. Which of the following is the most likely cause of preterm premature rupture of membranes in this patient? A. Ethnicity B. Smoking C. Previous premature rupture of membranes D. Cervical length E. Genital tract infections

E. The primary risk factor for preterm rupture of membranes is genital tract infection, especially associated with bacterial vaginosis. All of the other listed options are risk factors. Smoking and prior preterm premature rupture of membranes (which she did not have previously because she delivered at 41 weeks) increases the likelihood of preterm rupture of membranes two-fold. A shortened cervical length is also a risk factor, but her cervical length is normal.

A 16-year-old G0 female presents to the emergency department with a two-day history of abdominal pain. She is sexually active with a new partner and is not using any form of contraception. Temperature is 101.8°F (38.8°C). On examination, she has lower abdominal tenderness and guarding. On pelvic exam, she has diffuse tenderness over the uterus and bilateral adnexal tenderness. Beta-hCG is <5. What is the most likely diagnosis for this patient? A. Ectopic pregnancy B. Appendicitis C. Acute cystitis D. Endometriosis E. Acute salpingitis

E. The signs and symptoms of acute salpingitis can vary and be very subtle with mild pain and tenderness, or the patient can present in much more dramatic fashion with high fever, mucopurulent cervical discharge and severe pain. Important diagnostic criteria include lower abdominal tenderness, uterine/adnexal tenderness and mucopurulent cervicitis.

A 34-year-old G2P1 woman at 18 weeks gestation presents with a newly discovered lump in her left breast. Fine needle aspiration reveals adenocarcinoma. Which of the following is the least likely recommended therapy for breast cancer during pregnancy? A. Wide local excision biopsy B. Modified radical mastectomy C. Total mastectomy and node dissection D. Chemotherapy E. Radiotherapy

E. There is no doubt that breast cancer is more aggressive in younger women. Whether it is more aggressive during pregnancy in young women is debatable. Slight delays (1 to 2 months) in clinical assessment, diagnostic procedures, and treatment of pregnant women with breast tumors are common. Approximately 30 percent of pregnant women with breast cancer have stage I disease, 30 percent have stage II, and 40 percent stages III or IV. Many clinical reports maintain that when breast cancer is diagnosed during pregnancy, the regional lymph nodes are more likely to contain microscopic metastases. Surgical treatment may be definitive for breast carcinoma during pregnancy and in the absence of metastatic disease a wide excisional biopsy, modified radical mastectomy, or total mastectomy with axillary node staging can be performed. Non-pregnant women receive adjunctive radiotherapy with breast-conserving surgery. However, this is not recommended during pregnancy due to sizeable abdominal scatter placing the fetus at significant risk for excessive radiation.

A 23-year-old G1P0 woman at six weeks gestation undergoes a medical termination of pregnancy. One day later, she presents to the emergency department with bleeding and soaking more than a pad per hour for the last five hours. Her blood pressure on arrival is 110/60; heart rate 86. On exam, her cervix is 1 cm dilated with active bleeding. Hematocrit on arrival is 29%. Which of the following is the most appropriate next step in the management of this patient? A. Admit for observation B. Repeat hematocrit in six hours C. Begin transfusion with O-negative blood D. Give an additional dose of prostaglandins E. Perform a dilation and curettage

E. This patient is having heavy bleeding as a complication of medical termination of pregnancy. The most likely etiology for her bleeding is retained products of conception. This is managed best by performing a dilation and curettage. It is not appropriate to wait six hours before making a decision regarding next step in management, or to just admit her for observation. Since the patient is not symptomatic from her anemia, it is not necessary to transfuse her at this time.

A 58-year-old G2P2 woman presents to your office complaining of two years of a vulvar rash. She has seen multiple physicians without a clear definitive diagnosis. The patient has experienced intermittent pruritus for one year. She has been prescribed "every yeast medication known" and has also used multiple over-the-counter products. She was recently given topical steroid cream, which did not alleviate her symptoms. She is a breast cancer survivor and was diagnosed and treated one year ago. She is presently on tamoxifen. No vaginal bleeding has occurred since her menopause. On examination, her vulva is fiery red mottled background with whitish hyperkeratotic areas. A distinct lesion is not seen. No nodularity or tenderness is noted. With the exception of vaginal atrophy, the rest of her pelvic exam is normal. What is the most likely diagnosis in this patient? A. Lichen sclerosus of the vulva B. Contact dermatitis C. Yeast vulvitis D. Psoriasis of the vulva E. Paget's disease of the vulva

E. This is a typical description of Paget's disease of the vulva. Paget's is an in situ carcinoma of the vulva. The association with breast cancer is significant, but not as high as Paget's disease of the nipple. It would be unlikely for psoriasis to present this late in life. Contact dermatitis is unlikely to last for years and this woman has had therapy for yeast. Lichen sclerosus is possible and more common, but does not have the hyperkeratotic overlay and would have more likely responded to steroid use.

A 24-year-old G1P0 woman presents in active labor at 39 weeks gestation. She reports leaking fluid for the last two days. She develops a temperature of 102.0°F (38.9°C) and fetal heart rate is 180 beats/min with minimal variability. Maternal labs show: blood type O+; RPR non-reactive; HBsAg, negative; HIV negative; and GBS unknown. What will be the expected appearance of the baby at delivery? A. Vigorous, pink with normal temperature B. Vigorous, pale with low temperature C. Lethargic, pink with high temperature D. Lethargic, pale with low temperature E. Lethargic, pale with high temperature

E. This patient clearly has chorioamnionitis. The fetal tachycardia may be in response to the maternal fever. Fetal tachycardia coupled with minimal variability is a warning sign that the infant may be septic. A septic infant will typically appear pale, lethargic and have a high temperature.

A 20-year-old G0 woman presents for prenatal care at 10 weeks gestation. She states that she is healthy and denies vaginally bleeding. Prenatal labs reveal a hemoglobin of 9.1 g/dL with the following indices: MCV (mean cellular volume): 72 femtoliter MCH (mean cellular hemoglobin): 22 picograms/cell Serum ferritin: 108 mcq/L Hemoglobin A2: normal Hemoglobin F: normal Hemoglobin electrophoresis: normal What is the most likely cause of anemia in this patient? A. Acute blood loss B. Iron deficiency anemia C. Hemoglobin H disease D. Beta thalassemia E. Alpha thalassemia trait

E. This patient has alpha thalassemia trait characterized by mild anemia, macrocytic and hypochromic anemia and a normal hemoglobin electrophoresis. She denies blood loss therefore acute blood loss is unlikely and her serum ferritin is normal ruling out iron deficiency anemia. Hemoglobin H disease and beta thalassemia are characterized by moderate to severe anemia. Beta-thalassemia would have hemoglobin F as well as hemoglobin A2 on hemoglobin electrophoresis.

A 34-year-old G2P1 woman is 40 weeks gestation. She was admitted to labor and delivery in active labor 2 hours ago. Her cervix was 6 cm dilated and 100% effaced on admission. Her fetus was vertex and - 3 station. You are called to examine the patient after she experiences spontaneous rupture of membranes. The cervix is completely dilated and the fetal head is occiput anterior (OA) at +1 station. You palpate a 5 cm long section of umbilical cord in the patient's vagina. The fetal heart tracing is reassuring. The baseline is 130 beats per minute. There are multiple accelerations and no decelerations. The patient is having regular uterine contractions every 2-3 minutes. She has an epidural and is not feeling the contractions. What is the most appropriate next step in the management of this patient? A. Allow for passive descent of the fetal head with continuous fetal monitoring B. Have the patient start pushing with the contractions C. Gently attempt to replace the umbilical cord segment back up into the uterus D. Perform a forceps assisted vaginal delivery E. Elevate the fetal head with a vaginal hand and perform a Cesarean delivery

E. This patient has an umbilical cord prolapse. Although fetal surveillance is reassuring, the most appropriate management is to continue to elevate the fetal head with a hand in the patient's vagina and call for assistance to perform a Cesarean delivery. It is important to elevate the fetal head in an attempt to avoid compression of the umbilical cord. Once an umbilical cord prolapse is diagnosed, expeditious arrangements should be made to perform a cesarean section. It is not appropriate to replace the umbilical cord into the uterus or allow the patient to continue to labor or perform a forceps-assisted vaginal delivery.

A 24-year-old G0 woman presents with multiple painful ulcers involving the vulva. The sores were initially fluid filled, but are now open, weeping and crusted. She reports a fever and is having difficulty voiding due to pain. She uses a vaginal ring for contraception. She has multiple sexual partners and uses condoms for vaginal intercourse. She is distraught that she may have a sexually transmitted infection. She is healthy and does not smoke or use drugs. On physical exam, she is in obvious distress. Temperature is 100.2°F (37.9°C); pulse 100. Examination of the genital tract is limited due to her discomfort. Multiple ulcers and erosions of variable size are localized to the perineum, labia minora and vestibule. Swelling is diffuse. The lesions are eroded, some with a purulent eschar. There is exquisite tenderness to touch. What further testing should be offered to this patient? A. RPR (rapid plasma regain) B. HIV C. Herpes culture D. DNA probe for gonorrhea and chlamydia E. All of the above

E. This patient has classic primary herpes with painful genital ulcerations, fever and dysuria. Given the presence of one sexually transmitted infection, screening should be offered for other STIs. Resolution of the acute episode is required before a speculum can be inserted to allow endocervical sampling for gonorrhea and chlamydia. If it was a high-risk exposure, prophylactic empiric treatment could be offered to cover gonorrhea and chlamydia. The patient should be counseled that primary herpes can be acquired despite condoms and even by oral-genital inoculation. Hepatitis B vaccination should be offered to protect her against any future exposures. She should be encouraged to discuss her diagnosis with all sexual partners and to continue to reliably use latex condoms.

A 23-year-old G0 woman comes to the clinic because she is interested in becoming pregnant. She is in good health; however, she has not had any menses for the last two years. She had menarche at age 15, had normal periods until three years ago, when she started having periods irregularly every three months until it stopped two years ago. She has no history of pelvic infections or abnormal Pap smears. She exercises every day by running and has run four marathons in the last three years. She is 5 feet 10 inches tall and weighs 115 pounds. Her examination including a pelvic exam is normal. Laboratory results show: Results Normal Values TSH 3.5 mIU/ml 0.5-4.0 mIU/ml Free T4 0.9 ng/dl 0.8-1.8 ng/dl Prolactin 10 ng/ml <20 ng/ml FSH 6 mIU/ml 5-25 mIU/ml LH 4 mIU/ml 5-25 mIU/ml BHCG 2 mIU/ml <5 mIU/ml What is the most appropriate next step in the evaluation of this patient? A. Check cortisol levels B. Order a brain MRI C. Obtain a pelvic ultrasound D. Check testosterone levels E. Check estrogen levels

E. This patient most likely has exercise-induced hypothalamic amenorrhea, which is characterized by normal FSH and low estrogen levels. The other studies will not help determine the diagnosis. The best treatment is to encourage the patient to gain weight by decreasing exercise and increasing caloric intake. If her menses fail to resume, she may be treated with exogenous gonadotropins (LH and FSH) to help her conceive. Clomiphene citrate tends not to work as well, due to the baseline hypoestrogenic state.

A 22-year-old G0 woman presents with hirsutism which has been present since menarche. She states that she has laser treatments done to remove the hair on her chin every couple of months, and was wondering if there are additional treatments which might help her. She is otherwise in good health. She has normal menstrual cycles every 28 days. She is sexually active and uses birth control pills for contraception. The patient is adopted and has no information about family history. She is 5 feet 4 inches tall and weighs 125 pounds. On examination, the patient was noted to have terminal hair growth on her chest. Her TSH, Prolactin, total testosterone, DHEAS, 17-hydroxyprogesterone levels are normal. Which of the following is the most likely underlying etiology for the hirsutism in this patient? A. Polycystic ovarian syndrome B. Side effects of the oral contraceptives C. Cushing's syndrome D. Adrenal tumor E. Idiopathic hirsutism

E. This patient most likely has idiopathic hirsutism. She has no other clinical signs of polycystic ovaries, such as irregular cycles or obesity. Normal laboratory values rule out other pathogenic causes of hirsutism, such as Cushing's syndrome or adrenal tumor. Oral contraceptives are actually used for the treatment of hirsutism because they establish regular menses and lower ovarian androgen production. Additionally, they cause an increase in SHBG (sex hormone binding globulin) which allows more testosterone to be bound and unavailable at the hair follicle.

A 26-year-old G0 presents to the emergency department with eight hours of severe right lower quadrant pain associated with nausea. She has a history of suspected endometriosis, which was diagnosed two years ago based on severe dysmenorrhea. She has been using NSAIDs during her menses to control the pain. She is not sexually active. She is otherwise in good health. Her menstrual cycles are regular and her last menstrual period was three weeks ago. She has no history of sexually transmitted infections. Her vital signs are: blood pressure 145/70; pulse 100; temperature 98.6°F (37.0°C). She appears uncomfortable. On abdominal examination, she has moderate tenderness to palpation in the right lower quadrant. On pelvic examination, she has no lesions or discharge. A thorough bimanual examination was difficult to perform due to her discomfort. Beta-hCG <5 mIU/ml and hematocrit 29%. A pelvic ultrasound shows a 6 cm right ovarian mass. The uterus and left ovary appear normal. There is a moderate amount of free fluid in the pelvis. What is the most appropriate next step in the management of this patient? A. Begin oral contraceptives B. MRI of the pelvis C. Doppler pelvic ultrasound D. CT scan of the pelvis E. Surgical exploration

E. This patient most likely has ovarian torsion and needs to be surgically explored. Further imaging studies will not help beyond the information obtained on the ultrasound. A Doppler ultrasound to check the blood flow to the ovaries is controversial, as normal flow does not rule out ovarian torsion. Although oral contraceptives can help decrease the development of further cyst formation and control the pain associated with endometriosis, this patient needs immediate surgical attention due to suspected ovarian torsion.

A 31-year-old G1 woman presents at term in active labor. After four hours her cervix is still 5 cm dilated and the fetal station is zero. The fetal heart rate tracing is Category III. The patient's contractions have increased in intensity and are occurring every 2 to 3 minutes. Which of the following is the most appropriate next step in the management of this patient? A. Ambulation B. Ultrasound for estimated fetal weight C. Continued monitoring of labor D. Augmentation with oxytocin E. Cesarean delivery

E. This patient needs a Cesarean delivery since she is remote from delivery with a Category III fetal heart rate tracing. Continued monitoring of labor would be appropriate if the patient was making adequate progress and there was a Category I fetal heart rate tracing. Augmentation of labor would be indicated if there was a reassuring fetal heart rate tracing and the patient was not making adequate progress of labor. An ultrasound at this stage of labor is inaccurate and ambulation in the presence of a non-reassuring fetal heart tracing is contraindicated.

A 23-year-old G1P1 woman diagnosed with postpartum depression at three months after a spontaneous vaginal delivery has suicidal ideation and is desperate for help. Which of the following is the most appropriate next step in the treatment of this patient? A. Behavioral psychotherapy B. Anti-depressant medication C. Anti-psychotic medication D. Electroconvulsive therapy (ECT) E. Inpatient psychiatric admission

E. This patient offered thoughts of suicidal ideation, thus inpatient management is the most appropriate choice. While behavioral psychotherapy is necessary to establish long-term strategies for coping skills, newer regimens for postpartum depression include the use of SSRI medication. SSRI medications have been shown to hasten recovery to a fully functioning state. Antipsychotic medication is not indicated without an established diagnosis. Electroconvulsive therapy may be indicated for patients who don't respond to standard depression treatments.

A 22-year-old G2P1 woman comes to your clinic today with her three-month-old daughter. She was breastfeeding without problems until about two weeks ago, when she began to experience sore nipples. The nipples are very sensitive and there is a burning pain in the breasts, which is worse when feeding. The tips of the nipples are pink and shiny with peeling at the periphery. Which of the following organism is the most likely cause of these findings? A. Group A streptrococcocus B. Group B streptrococcocus C. Staphylococcus aureus D. Staphylococcus epidermidis E. Candida

E. This presentation is classic for candidiasis and should prompt an inspection of the baby's oral cavity. Candida of the nipple is associated with severe discomfort and pain. All the other above organisms are associated with classic mastitis and do not usually cause intense nipple pain. Localized candida of the nipple may be treated with an antifungal, topical medication such as clotrimazole or miconazole cream. The treatment plan may include a topical antibiotic ointment because nipple fissures can concurrently present with candida of the nipples, and S. aureus is significantly associated with nipple fissures. Either a triple antibiotic ointment or mupirocin can be prescribed. A topical steroid cream can be used to facilitate healing for cases in which the nipples that are very red and inflamed. Every treatment regimen must include the simultaneous treatment of the mother and baby. Oral nystatin is the most common treatment for the baby, followed by oral fluconazole.

A 27-year-old G2P1 woman at 36 weeks gestation is admitted for preeclampsia with severe features. Her blood pressure is 200/105. Diastolic blood pressures during her pregnancy have ranged from 50-60 mmHg. She has received two doses of IV hydralazine to lower her blood pressure. What diastolic blood pressure should you aim for in this patient? A. 50-55 mmHg B. 60-65 mmHg C. 70-75 mmHg D. 80-85 mmHg E. 90-95 mmHg

E. Treatment with an antihypertensive is indicated for blood pressures persistently greater than 160 systolic and 105 diastolic. First-line agents include hydralazine (a direct vasodilator) 5 mg IV followed by 5-10 mg doses IV at 20-minute intervals (maximum dose = 40 mg); or labetalol (combined alpha & beta-adrenergic antagonist) 10-20 mg IV followed by 20 mg, then 40 mg, then 80 mg IV every 10 minutes (maximum dose = 220 mg). The goal is not a normal blood pressure, but to reduce the diastolic blood pressure into a safe range of 90-100 mmHg to prevent maternal stroke or abruption, without compromising uterine perfusion.

A 38-year-old G2P0 woman at 28 weeks gestation has been diagnosed with preterm labor and is currently stable on nifedipine. Her cervical exam has remained unchanged at 2 cm dilated, 75% effaced and -2 station. Her vital signs are stable and fetal heart tracing is Category I. You recommend treatment with betamethasone (a steroid). Which of the following is associated with betamethasone therapy in the newborn? A. Enhancement of fetal growth B. Increased risk of infection C. Increased incidence of necrotizing enterocolitis D. Increased incidence of intracerebral hemorrhage E. Decreased incidence of intracerebral hemorrhage

E. Treatment with betamethasone from 24 to 34 weeks gestation has been shown to increase pulmonary maturity and reduce the incidence and severity of RDS (respiratory distress syndrome) in the newborn. It is also associated with decreased intracerebral hemorrhage and necrotizing enterocolitis in the newborn. It has not been associated with increased infection or enhanced growth.

A 25-year-old G1 is at 18 weeks gestation. A 2 cm subserosal fibroid was noted on the anterior fundal wall of her uterus at the time of her obstetric ultrasound at 17 weeks gestation. Which of the following treatment options for the uterine fibroid is most appropriate? A. Obtain a follow up ultrasound every six weeks to follow growth of the fibroid B. Laparoscopy now to remove the fibroid C. Perform delivery by Cesarean section at term D. Perform delivery by Cesarean section at term with removal of the fibroid after delivery of the baby and placenta E. No further treatment is necessary

E. Uterine fibroids are the most common solid pelvic tumors in women. On postmortem examination, fibroids can be detected in as high as 80% of women. Most uterine fibroids are asymptomatic and do not require any treatment. Pregnant patients with fibroids usually are asymptomatic and do not have any complications related to the fibroids. Fibroids may grow or become symptomatic in pregnancy due to hemorrhagic changes associated with rapid growth, known as red or carneous degeneration. However, this is uncommon for smaller fibroids. Uncommonly, fibroids can be located below the fetus, in the lower uterine segment, or cervix, causing a soft tissue dystocia, necessitating delivery by Cesarean section. In this case, it is not indicated given the location of the fibroid. Myomectomy (removal of the fibroid) during pregnancy is contraindicated. Myomectomy at the time of Cesarean section should be avoided, if possible, secondary to the risk for increased blood loss. It is not necessary to follow the growth of fibroids during pregnancy, except for the rare cases when the fibroid is causing symptoms (primarily pain) or appear to be located in a position likely to cause dystocia.

A 42-year-old G2P2 woman undergoes a hysterectomy for definitive treatment of her dysmenorrhea and large uterine fibroids. The uterus is sent to pathology. Which of the following would confirm the diagnosis of fibroids? A. Hyperplastic overgrowth of endometrial glands/stroma B. Decidual effect in the endometrium C. Endometrial glands/stroma and hemosiderin-laden macrophages D. Invasion of endometrial glands into the myometrium E. Well-circumscribed, non-encapsulated myometrium

E. Well-circumscribed, non-encapsulated myometrium confirms the diagnosis of fibroids. Hyperplastic overgrowth of endometrial glands/stroma is consistent with endometrial polyps. Decidual effect on the endometrium are seen during pregnancy. Invasion of endometrial glands into the myometrium is seen with adenomyosis.

A 30-year-old G1P1 woman is breastfeeding her baby and feels there is not enough milk. She is pumping in order to improve the supply of milk. You tell her that more frequent suckling would be better as it will stimulate which of the following hormones? A. Progesterone B. Insulin C. Cortisol D. Prolactin E. Oxytocin

E. While prolactin is responsible for milk production, oxytocin is responsible for milk ejection. Production of oxytocin is stimulated by suckling which works better than a breast pump for stimulating the secretion of milk. Cortisol and insulin act in concert with other hormones to stimulate the growth and development of the milk-secreting apparatus.

A 33-year-old G2P1 presents at 34 weeks gestation for consultation because ultrasound revealed a 3900 gm fetus with biometrics consistent with 39 weeks. Her prior pregnancy was complicated by gestational diabetes and a shoulder dystocia. Which of the following complications is this fetus at greatest risk? A. Birth trauma B. Hyperglycemia C. Hypobilirubinemia D. Hypothyroidism E. Congenital anomalies

The fetus with enhanced general growth or macrosomia is defined by a birth weight at or above the 90th percentile for gestational age. The condition can usually be ascribed to one of three etiologies: enhanced growth potential (50-60%); abnormal maternal glucose homeostasis (35-40%); or underestimation of fetal age (5%). Macrosomic newborns of diabetic mothers experience excessive rates of neonatal morbidity, including birth trauma such as shoulder dystocia and brachial plexus injury. These infants have significantly higher rates of severe hypoglycemia and neonatal jaundice. Neonatal acidosis occurs with poor glycemic control, thus increasing the incidence of fetal demise. While poorly controlled pre-existing diabetes is associated with an increased risk of congenital anomalies, gestational diabetes is not associated with increased risk of congenital anomalies.


Kaugnay na mga set ng pag-aaral

AP Psychology - Chapter 2 - I think

View Set

Medical Review: Part 2 Questions

View Set

Chapter 8 - Exploring Marine and Aviation Coverage

View Set

10 Real Estate Finance - Chapter 6: Predatory Lending and Mortgage Fraud

View Set

DECA Business Management Administration Test

View Set

PSY 302 Research Questions and Designs Module 5

View Set